Как Π²Ρ‹Ρ‡Π΅ΡΡ‚ΡŒ ΠΈΠ· ΠΌΠ°Ρ‚Ρ€ΠΈΡ†Ρ‹ ΠΌΠ°Ρ‚Ρ€ΠΈΡ†Ρƒ: опрСдСлСния, свойства ΠΈ ΠΏΡ€ΠΈΠΌΠ΅Ρ€Ρ‹ Ρ€Π΅ΡˆΠ΅Π½ΠΈΡ Π·Π°Π΄Π°Ρ‡

Π‘ΠΎΠ΄Π΅Ρ€ΠΆΠ°Π½ΠΈΠ΅

опрСдСлСния, свойства ΠΈ ΠΏΡ€ΠΈΠΌΠ΅Ρ€Ρ‹ Ρ€Π΅ΡˆΠ΅Π½ΠΈΡ Π·Π°Π΄Π°Ρ‡

Π‘ΠΎΠ΄Π΅Ρ€ΠΆΠ°Π½ΠΈΠ΅:

Π‘Π»ΠΎΠΆΠ΅Π½ΠΈΠ΅ ΠΈ Π²Ρ‹Ρ‡ΠΈΡ‚Π°Π½ΠΈΠ΅ ΠΌΠ°Ρ‚Ρ€ΠΈΡ†, Π΄ΠΎΠΏΡƒΡΠΊΠ°ΡŽΡ‚ΡΡ Ρ‚ΠΎΠ»ΡŒΠΊΠΎ для ΠΌΠ°Ρ‚Ρ€ΠΈΡ† ΠΎΠ΄ΠΈΠ½Π°ΠΊΠΎΠ²ΠΎΠ³ΠΎ Ρ€Π°Π·ΠΌΠ΅Ρ€Π°.

Π‘ΡƒΠΌΠΌΠ° ΠΌΠ°Ρ‚Ρ€ΠΈΡ†

ΠžΠΏΡ€Π΅Π΄Π΅Π»Π΅Π½ΠΈΠ΅

Π‘ΡƒΠΌΠΌΠΎΠΉ ΠΌΠ°Ρ‚Ρ€ΠΈΡ† $A$ ΠΈ $B$ ΠΎΠ΄Π½ΠΎΠ³ΠΎ Ρ€Π°Π·ΠΌΠ΅Ρ€Π° называСтся ΠΌΠ°Ρ‚Ρ€ΠΈΡ†Π° $C = A+B$ Ρ‚Π°ΠΊΠΎΠ³ΠΎ ΠΆΠ΅ Ρ€Π°Π·ΠΌΠ΅Ρ€Π°, получаСмая ΠΈΠ· исходных ΠΏΡƒΡ‚Π΅ΠΌ слоТСния ΡΠΎΠΎΡ‚Π²Π΅Ρ‚ΡΡ‚Π²ΡƒΡŽΡ‰ΠΈΡ… элСмСнтов:

$$ A_{m \times n}+B_{m \times n}=C_{m \times n} ; c_{i j}=a_{i j}+b_{i j}, i=\overline{1 ; m}, j=\overline{1 ; n} $$

Π—Π°ΠΌΠ΅Ρ‡Π°Π½ΠΈΠ΅

Π‘ΠΊΠ»Π°Π΄Ρ‹Π²Π°Ρ‚ΡŒ ΠΌΠΎΠΆΠ½ΠΎ Ρ‚ΠΎΠ»ΡŒΠΊΠΎ ΠΌΠ°Ρ‚Ρ€ΠΈΡ†Ρ‹ ΠΎΠ΄ΠΈΠ½Π°ΠΊΠΎΠ²ΠΎΠ³ΠΎ Ρ€Π°Π·ΠΌΠ΅Ρ€Π°.

ΠŸΡ€ΠΈΠΌΠ΅Ρ€

Π—Π°Π΄Π°Π½ΠΈΠ΅. Найти $A+B$, Ссли $ A=\left( \begin{array}{ll}{1} & {4} \\ {2} & {3}\end{array}\right) $ , $ B=\left( \begin{array}{ll}{4} & {4} \\ {5} & {2}\end{array}\right) $

РСшСниС. $ C=A+B=\left( \begin{array}{cc}{1} & {4} \\ {2} & {3}\end{array}\right)_{2 \times 2}+\left( \begin{array}{ll}{4} & {4} \\ {5} & {2}\end{array}\right)_{2 \times 2}= $

$ =\left( \begin{array}{cc}{1+4} & {4+4} \\ {2+5} & {3+2}\end{array}\right)=\left( \begin{array}{ll}{5} & {8} \\ {7} & {5}\end{array}\right) $

ΠžΡ‚Π²Π΅Ρ‚. $ A+B=\left( \begin{array}{ll}{5} & {8} \\ {7} & {5}\end{array}\right) $

Бвойства слоТСния ΠΈ вычитания ΠΌΠ°Ρ‚Ρ€ΠΈΡ†:

  1. Β  ΠΡΡΠΎΡ†ΠΈΠ°Ρ‚ΠΈΠ²Π½ΠΎΡΡ‚ΡŒ $ (A+B)+C=A+(B+C) $
  2. Β  $ A+\Theta=\Theta+A $, Π³Π΄Π΅ $\Theta$ – нулСвая ΠΌΠ°Ρ‚Ρ€ΠΈΡ†Π° ΡΠΎΠΎΡ‚Π²Π΅Ρ‚ΡΡ‚Π²ΡƒΡŽΡ‰Π΅Π³ΠΎ Ρ€Π°Π·ΠΌΠ΅Ρ€Π°.
  3. Β  $ A-A=\Theta $
  4. Β  ΠšΠΎΠΌΠΌΡƒΡ‚Π°Ρ‚ΠΈΠ²Π½ΠΎΡΡ‚ΡŒ $ A+B=B+A $

Π Π°Π·Π½ΠΎΡΡ‚ΡŒ ΠΌΠ°Ρ‚Ρ€ΠΈΡ†

Π Π°Π·Π½ΠΎΡΡ‚ΡŒ Π΄Π²ΡƒΡ… ΠΌΠ°Ρ‚Ρ€ΠΈΡ† ΠΎΠ΄ΠΈΠ½Π°ΠΊΠΎΠ²ΠΎΠ³ΠΎ Ρ€Π°Π·ΠΌΠ΅Ρ€Π° ΠΌΠΎΠΆΠ½ΠΎ ΠΎΠΏΡ€Π΅Π΄Π΅Π»ΠΈΡ‚ΡŒ Ρ‡Π΅Ρ€Π΅Π· ΠΎΠΏΠ΅Ρ€Π°Ρ†ΠΈΡŽ слоТСния ΠΌΠ°Ρ‚Ρ€ΠΈΡ† ΠΈ Ρ‡Π΅Ρ€Π΅Π· ΡƒΠΌΠ½ΠΎΠΆΠ΅Π½ΠΈΠ΅ ΠΌΠ°Ρ‚Ρ€ΠΈΡ†Ρ‹ Π½Π° число.

Π’Ρ‹Ρ‡ΠΈΡ‚Π°Π½ΠΈΠ΅ ΠΌΠ°Ρ‚Ρ€ΠΈΡ† вводится ΡΠ»Π΅Π΄ΡƒΡŽΡ‰ΠΈΠΌ ΠΎΠ±Ρ€Π°Π·ΠΎΠΌ: $ A-B=A+(-1) \cdot B $

Π’ΠΎ Π΅ΡΡ‚ΡŒ ΠΊ ΠΌΠ°Ρ‚Ρ€ΠΈΡ†Π΅ $A$ прибавляСтся ΠΌΠ°Ρ‚Ρ€ΠΈΡ†Π° $B$, умноТСнная Π½Π° (-1).

ΠžΠΏΡ€Π΅Π΄Π΅Π»Π΅Π½ΠΈΠ΅

Π Π°Π·Π½ΠΎΡΡ‚ΡŒΡŽ ΠΌΠ°Ρ‚Ρ€ΠΈΡ† $A$ ΠΈ $B$ ΠΎΠ΄Π½ΠΎΠ³ΠΎ ΠΈ Ρ‚ΠΎΠ³ΠΎ ΠΆΠ΅ Ρ€Π°Π·ΠΌΠ΅Ρ€Π° называСтся ΠΌΠ°Ρ‚Ρ€ΠΈΡ†Π° $C = A-B$ Ρ‚Π°ΠΊΠΎΠ³ΠΎ ΠΆΠ΅ Ρ€Π°Π·ΠΌΠ΅Ρ€Π°, получаСмая ΠΈΠ· исходных ΠΏΡƒΡ‚Π΅ΠΌ прибавлСния ΠΊ ΠΌΠ°Ρ‚Ρ€ΠΈΡ†Π΅ $A$ ΠΌΠ°Ρ‚Ρ€ΠΈΡ†Ρ‹ $B$, ΡƒΠΌΠ½ΠΎΠΆΠ΅Π½Π½ΠΎΠΉ Π½Π° (-1).

На ΠΏΡ€Π°ΠΊΡ‚ΠΈΠΊΠ΅ ΠΆΠ΅ ΠΎΡ‚ элСмСнтов ΠΌΠ°Ρ‚Ρ€ΠΈΡ†Ρ‹ $A$ попросту ΠΎΡ‚Π½ΠΈΠΌΠ°ΡŽΡ‚ ΡΠΎΠΎΡ‚Π²Π΅Ρ‚ΡΡ‚Π²ΡƒΡŽΡ‰ΠΈΠ΅ элСмСнты ΠΌΠ°Ρ‚Ρ€ΠΈΡ†Ρ‹ $B$ ΠΏΡ€ΠΈ условии, Ρ‡Ρ‚ΠΎ Π·Π°Π΄Π°Π½Π½Ρ‹Π΅ ΠΌΠ°Ρ‚Ρ€ΠΈΡ†Ρ‹ ΠΎΠ΄Π½ΠΎΠ³ΠΎ Ρ€Π°Π·ΠΌΠ΅Ρ€Π°.

Π—Π°ΠΌΠ΅Ρ‡Π°Π½ΠΈΠ΅

Π’Ρ‹Ρ‡ΠΈΡ‚Π°Ρ‚ΡŒ ΠΌΠΎΠΆΠ½ΠΎ Ρ‚ΠΎΠ»ΡŒΠΊΠΎ ΠΌΠ°Ρ‚Ρ€ΠΈΡ†Ρ‹ ΠΎΠ΄ΠΈΠ½Π°ΠΊΠΎΠ²ΠΎΠ³ΠΎ Ρ€Π°Π·ΠΌΠ΅Ρ€Π°.

Блишком слоТно?

Π‘Π»ΠΎΠΆΠ΅Π½ΠΈΠ΅ ΠΈ Π²Ρ‹Ρ‡ΠΈΡ‚Π°Π½ΠΈΠ΅ ΠΌΠ°Ρ‚Ρ€ΠΈΡ† Π½Π΅ ΠΏΠΎ Π·ΡƒΠ±Π°ΠΌ? Π’Π΅Π±Π΅ ΠΎΡ‚Π²Π΅Ρ‚ΠΈΡ‚ экспСрт Ρ‡Π΅Ρ€Π΅Π· 10 ΠΌΠΈΠ½ΡƒΡ‚!

ΠŸΡ€ΠΈΠΌΠ΅Ρ€

Π—Π°Π΄Π°Π½ΠΈΠ΅. Найти ΠΌΠ°Ρ‚Ρ€ΠΈΡ†Ρƒ $ C=A-3 B $, Ссли $ A=\left( \begin{array}{rr}{1} & {2} \\ {2} & {-1} \\ {3} & {0}\end{array}\right) $ , $ B=\left( \begin{array}{rr}{-1} & {1} \\ {1} & {2} \\ {0} & {0}\end{array}\right) $

РСшСниС. $ C=A-3 B=\left( \begin{array}{rr}{1} & {2} \\ {2} & {-1} \\ {3} & {0}\end{array}\right)-3 \cdot \left( \begin{array}{rr}{-1} & {1} \\ {1} & {2} \\ {0} & {0}\end{array}\right)= $

$ \left( \begin{array}{rr}{1} & {2} \\ {2} & {-1} \\ {3} & {0}\end{array}\right)-\left( \begin{array}{rr}{-3} & {3} \\ {3} & {6} \\ {0} & {0}\end{array}\right)=\left( \begin{array}{rr}{1-(-3)} & {2-3} \\ {2-3} & {-1-6} \\ {3-0} & {0-0}\end{array}\right)=\left( \begin{array}{rr}{4} & {-1} \\ {-1} & {-7} \\ {3} & {0}\end{array}\right) $

ΠžΡ‚Π²Π΅Ρ‚. $ C=\left( \begin{array}{rr}{4} & {-1} \\ {-1} & {-7} \\ {3} & {0}\end{array}\right) $

Π§ΠΈΡ‚Π°Ρ‚ΡŒ дальшС: ΡƒΠΌΠ½ΠΎΠΆΠ΅Π½ΠΈΠ΅ ΠΌΠ°Ρ‚Ρ€ΠΈΡ†.

ΠœΠ°Ρ‚Ρ€ΠΈΡ† слоТСниС – ЭнциклопСдия ΠΏΠΎ ΠΌΠ°ΡˆΠΈΠ½ΠΎΡΡ‚Ρ€ΠΎΠ΅Π½ΠΈΡŽ XXL

Как ΠΈ Π² случаС ΠΌΠ°Ρ‚Ρ€ΠΈΡ†, слоТСнию ΠΏΠΎΠ²ΠΎΡ€ΠΎΡ‚ΠΎΠ² Π² случаС ΠΈΡ… Π°ΠΊΡ‚ΠΈΠ²Π½ΠΎΠ³ΠΎ прСдставлСния ΠΎΡ‚Π²Π΅Ρ‡Π°Π΅Ρ‚ ΠΏΡ€ΠΎΠΈΠ·Π²Π΅Π΄Π΅Π½ΠΈΠ΅ ΠΊΠ²Π°Ρ‚Π΅Ρ€Π½ΠΈΠΎΠ½ΠΎΠ² ΡΠΎΡΡ‚Π°Π²Π»ΡΡŽΡ‰ΠΈΡ… ΠΏΠΎΠ²ΠΎΡ€ΠΎΡ‚ΠΎΠ² Π² ΠΎΠ±Ρ€Π°Ρ‚Π½ΠΎΠΌ порядкС. ΠŸΡ€ΠΈ этом всС ΠΊΠ²Π°Ρ‚Π΅Ρ€Π½ΠΈΠΎΠ½Ρ‹ Π·Π°Π΄Π°Π½Ρ‹ Π² исходном базисС ΠΊ (/ = 1, 2, 3).  [c.46]

Π’ ΠΊΠΈΠ½Π΅ΠΌΠ°Ρ‚ΠΈΠΊΠ΅ ΠΌΠ΅Ρ…Π°Π½ΠΈΠ·ΠΌΠΎΠ² ΠΎΠΏΠ΅Ρ€Π°Ρ†ΠΈΠΈ слоТСния ΠΌΠ°Ρ‚Ρ€ΠΈΡ† ΠΈ умноТСния ΠΈΡ… Π½Π° скаляр находят ΠΏΡ€ΠΈΠΌΠ΅Π½Π΅Π½ΠΈΠ΅ Π² дСйствиях Π½Π°Π΄ ΠΌΠ°Ρ‚Ρ€ΠΈΡ†Π°ΠΌΠΈ-столбцами.  [c.631]

ΠŸΡ€ΠΈ слоТСнии Π΄Π²ΡƒΡ… ΠΌΠ°Ρ‚Ρ€ΠΈΡ† ΠΎΠ΄ΠΈΠ½Π°ΠΊΠΎΠ²ΠΎΠ³ΠΎ Ρ€Π°Π·ΠΌΠ΅Ρ€Π° получаСтся ΠΌΠ°Ρ‚Ρ€ΠΈΡ†Π° Ρ‚ΠΎΠ³ΠΎ ΠΆΠ΅ Ρ€Π°Π·ΠΌΠ΅Ρ€Π°, ΠΊΠ°ΠΆΠ΄Ρ‹ΠΉ элСмСнт ΠΊΠΎΡ‚ΠΎΡ€ΠΎΠΉ Ρ€Π°Π²Π΅ΠΈ суммС ΡΠΎΠΎΡ‚Π²Π΅Ρ‚ΡΡ‚Π²ΡƒΡŽΡ‰ΠΈΡ… элСмСнтов слагаСмых ΠΌΠ°Ρ‚Ρ€ΠΈΡ†.  [c.104]


Над ΠΌΠ°Ρ‚Ρ€ΠΈΡ†Π°ΠΌΠΈ ΠΌΠΎΠΆΠ½ΠΎ Π²Ρ‹ΠΏΠΎΠ»Π½ΡΡ‚ΡŒ дСйствия транспонирования, слоТСния, умноТСния. ΠœΠ°Ρ‚Ρ€ΠΈΡ†Π° А, транспонированная ΠΏΠΎ ΠΎΡ‚Π½ΠΎΡˆΠ΅Π½ΠΈΡŽ ΠΊ ΠΌΠ°Ρ‚Ρ€ΠΈΡ†Π΅ А, образуСтся ΠΈΠ· ΠΌΠ°Ρ‚Ρ€ΠΈΡ†Ρ‹ А Π·Π°ΠΌΠ΅Π½ΠΎΠΉ ΠΊΠ°ΠΆΠ΄ΠΎΠΉ Π΅Π΅ строки Π½Π° столбСц Ρ‚ΠΎΠ³ΠΎ ΠΆΠ΅ Π½ΠΎΠΌΠ΅Ρ€Π°. НапримСр, ΠΏΡ€ΠΈ транспонировании ΠΌΠ°Ρ‚Ρ€ΠΈΡ†Ρ‹  [c.50]

ΠœΠ½ΠΎΠΆΠ΅ΡΡ‚Π²ΠΎ J n всСх Π»-ΠΌΠ΅Ρ€Π½Ρ‹Ρ… Π²Π΅ΠΊΡ‚ΠΎΡ€ΠΎΠ² Π½Π°Π·Ρ‹Π²Π°ΡŽΡ‚ Π»ΠΈΠ½Π΅ΠΉΠ½Ρ‹ΠΌ алгСбраичСским пространством, Ссли Π² Π½Π΅ΠΌ ΠΎΠΏΡ€Π΅Π΄Π΅Π»Π΅Π½Ρ‹ ΠΎΠΏΠ΅Ρ€Π°Ρ†ΠΈΠΈ слоТСния ΠΈ умноТСния Π½Π° скаляр Ρ‚ΠΎΡ‡Π½ΠΎ Ρ‚Π°ΠΊ ΠΆΠ΅, ΠΊΠ°ΠΊ для ΠΌΠ°Ρ‚Ρ€ΠΈΡ†. Число я называСтся Ρ€Π°Π·ΠΌΠ΅Ρ€Π½ΠΎΡΡ‚ΡŒΡŽ пространства Rn- Рассмотрим ΠΏΠΎΠΌΠΈΠΌΠΎ Π²Π΅ΠΊΡ‚ΠΎΡ€Π° Π° Π΄Ρ€ΡƒΠ³ΠΎΠΉ ΠΏ-ΠΌΠ΅Ρ€Π½Ρ‹ΠΉ Π²Π΅ΠΊΡ‚ΠΎΡ€  

[c.19]

ΠœΠ°Ρ‚Ρ€ΠΈΡ†Π° [/(], называСмая глобальной ΠΌΠ°Ρ‚Ρ€ΠΈΡ†Π΅ΠΉ ТСсткости ΠΈΠ»ΠΈ просто ΠΌΠ°Ρ‚Ρ€ΠΈΡ†Π΅ΠΉ ТСсткости систСмы, получаСтся слоТСниСм Π»ΠΎΠΊΠ°Π»ΡŒΠ½Ρ‹Ρ… ΠΌΠ°Ρ‚Ρ€ΠΈΡ† ТСсткости [Π› ] ΠΏΠΎ ΡΠ»Π΅Π΄ΡƒΡŽΡ‰Π΅ΠΌΡƒ ΠΏΡ€Π°Π²ΠΈΠ»Ρƒ сначала ΠΊ Π½ΡƒΠ»Π΅Π²ΠΎΠΉ ΠΌΠ°Ρ‚Ρ€ΠΈΡ†Π΅ размСрности NxN добавляСтся ΠΌΠ°Ρ‚Ρ€ΠΈΡ†Π°, Π² Π»Π΅Π²ΠΎΠΌ Π²Π΅Ρ€Ρ…Π½Π΅ΠΌ ΡƒΠ³Π»Ρƒ ΠΊΠΎΡ‚ΠΎΡ€ΠΎΠΉ стоит локальная ΠΌΠ°Ρ‚Ρ€ΠΈΡ†Π° ТСсткости 1-Π³ΠΎ элСмСнта, ΠΊ ΠΏΠΎΠ»ΡƒΡ‡ΠΈΠ²ΡˆΠ΅ΠΉΡΡ ΠΌΠ°Ρ‚Ρ€ΠΈΡ†Π΅ добавляСтся ΠΌΠ°Ρ‚Ρ€ΠΈΡ†Π° Ρ€Π°Π·ΠΌΠ΅Ρ€Π° /V Ρ… /V, Π½Π΅Π½ΡƒΠ»Π΅Π²Ρ‹Π΅ элСмСнты ΠΊΠΎΡ‚ΠΎΡ€ΠΎΠΉ располоТСны Π½Π° пСрСсСчСнии 2-Π³ΠΎ ΠΈ 3-Π³ΠΎ столбцов ΠΈ 2-ΠΉ ΠΈ 3-ΠΉ строк ΠΈ Ρ€Π°Π²Π½Ρ‹ ΡΠΎΠΎΡ‚Π²Π΅Ρ‚ΡΡ‚Π²ΡƒΡŽΡ‰ΠΈΠΌ элСмСнтам локальной ΠΌΠ°Ρ‚Ρ€ΠΈΡ†Ρ‹ ТСсткости для 2-Π³ΠΎ элСмСнта ΠΈ Ρ‚. Π΄. Π½Π° -ΠΌ шагС добавляСтся ΠΌΠ°Ρ‚Ρ€ΠΈΡ†Π°, Π½Π΅Π½ΡƒΠ»Π΅Π²Ρ‹Π΅ элСмСнты ΠΊΠΎΡ‚ΠΎΡ€ΠΎΠΉ располоТСны Π½Π° пСрСсСчСнии ΠΊ ΠΈ ΠΊ- строк ΠΈ ΠΊ Π½ k- – столбцов ΠΈ Ρ€Π°Π²Π½Ρ‹ ΡΠΎΠΎΡ‚Π²Π΅Ρ‚ΡΡ‚Π²ΡƒΡŽΡ‰ΠΈΠΌ элСмСнтам локальной ΠΌΠ°Ρ‚Ρ€ΠΈΡ†Ρ‹ ТСсткости k-ro элСмСнта.  

[c.134]

ΠŸΡ€ΠΈ вычислСнии ΠΌΠ°Ρ‚Ρ€ΠΈΡ†Ρ‹ плотности ΠΌΡ‹ Π±ΡƒΠ΄Π΅ΠΌ ΠΏΡ€Π΅Π΄ΠΏΠΎΠ»Π°Π³Π°Ρ‚ΡŒ, Ρ‡Ρ‚ΠΎ рассСянныС Π²ΠΎΠ»Π½Ρ‹ ΠΈΠΌΠ΅ΡŽΡ‚ бСспорядочныС Ρ„Π°Π·Ρ‹ ΠΈ ΠΏΡ€ΠΈ слоТСнии Π² срСднСм Π΄Π°ΡŽΡ‚ Π½ΡƒΠ»ΡŒ. Если ΠΈ Π³ ΠΈ Π³ Π»Π΅ΠΆΠ°Ρ‚ слСва ΠΎΡ‚ слоя, Ρ‚ΠΎ  [c.718]

ВсС ΠΎΡΡ‚Π°Π»ΡŒΠ½Ρ‹Π΅ нСизвСстныС ΠΎΠΏΡ€Π΅Π΄Π΅Π»ΡΡŽΡ‚ΡΡ лишь ΠΏΡƒΡ‚Π΅ΠΌ умноТСния ΠΈ слоТСния ΠΌΠ°Ρ‚Ρ€ΠΈΡ†. Π’Π΅ΡΡŒ процСсс вычислСний ΠΏΡ€ΠΈ этом Π»Π΅Π³ΠΊΠΎ программируСтся с использованиСм стандартных ΠΏΡ€ΠΎΠ³Ρ€Π°ΠΌΠΌ слоТСния, умноТСния ΠΈ обращСния ΠΌΠ°Ρ‚Ρ€ΠΈΡ†.  [c.97]

НСпосрСдствСнным ΠΏΠ΅Ρ€Π΅ΠΌΠ½ΠΎΠΆΠ΅Π½ΠΈΠ΅ΠΌ ΠΈ слоТСниСм ΠΌΠ°Ρ‚Ρ€ΠΈΡ† (71.29) Π½Π΅Ρ‚Ρ€ΡƒΠ΄Π½ΠΎ ΡƒΠ±Π΅Π΄ΠΈΡ‚ΡŒΡΡ, Ρ‡Ρ‚ΠΎ ΠΎΠ½ΠΈ ΡƒΠ΄ΠΎΠ²Π»Π΅Ρ‚Π²ΠΎΡ€ΡΡŽΡ‚ ΡΠΎΠΎΡ‚Π½ΠΎΡˆΠ΅Π½ΠΈΡΠΌ (71.28), понимая, Ρ‡Ρ‚ΠΎ Π² ΠΈΡ… ΠΏΡ€Π°Π²ΠΎΠΉ части стоит Сдиничная ΠΌΠ°Ρ‚Ρ€ΠΈΡ†Π°. НапримСр, для ΠΈΠΌΠ΅Π΅ΠΌ  

[c.387]

Описанная ΠΏΡ€ΠΎΡ†Π΅Π΄ΡƒΡ€Π° Π»Π΅ΠΆΠΈΡ‚ Π² основС Π°Π»Π³ΠΎΡ€ΠΈΡ‚ΠΌΠ° формирования глобальной ΠΌΠ°Ρ‚Ρ€ΠΈΡ†Ρ‹ ΠΈ глобального Π²Π΅ΠΊΡ‚ΠΎΡ€-столбца. Как Π±Ρ‹Π»ΠΎ ΡƒΠΆΠ΅ ΠΎΡ‚ΠΌΠ΅Ρ‡Π΅Π½ΠΎ Π²Ρ‹ΡˆΠ΅, ΠΎΠ½Π° рСализуСтся ΠΏΡƒΡ‚Π΅ΠΌ ΠΏΠΎΡΠ»Π΅Π΄ΠΎΠ²Π°Ρ‚Π΅Π»ΡŒΠ½ΠΎΠ³ΠΎ ΠΏΠ΅Ρ€Π΅Π±ΠΎΡ€Π° элСмСнтов ΡΠ»Π΅Π΄ΡƒΡŽΡ‰ΠΈΠΌ ΠΎΠ±Ρ€Π°Π·ΠΎΠΌ. БСрСтся ΠΏΠ΅Ρ€Π²Ρ‹ΠΉ элСмСнт, анализируСтся Π΅Π³ΠΎ строка Π² индСксной ΠΌΠ°Ρ‚Ρ€ΠΈΡ†Π΅ ΠΈ устанавливаСтся, Π² ΠΊΠ°ΠΊΠΈΠ΅ Ρ‚Ρ€ΠΈ уравнСния этот элСмСнт Π΄Π°Π΅Ρ‚ Π²ΠΊΠ»Π°Π΄ . Π”Π°Π»Π΅Π΅ Ρ€Π°ΡΡΡ‡ΠΈΡ‚Ρ‹Π²Π°ΡŽΡ‚ΡΡ Π΅Π³ΠΎ локальная ΠΌΠ°Ρ‚Ρ€ΠΈΡ†Π° ΠΈ Π²Π΅ΠΊΡ‚ΠΎΡ€-столбСц. ΠŸΡ€ΠΈ этом расчСтС ΠΈΡΠΏΠΎΠ»ΡŒΠ·ΡƒΠ΅Ρ‚ΡΡ информация ΠΎ Π½Π°Π»ΠΈΡ‡ΠΈΠΈ Ρƒ Π΄Π°Π½Π½ΠΎΠ³ΠΎ элСмСнта Π³Ρ€Π°Π½ΠΈΡ‡Π½Ρ‹Ρ… сторон, содСрТащаяся Π² Ρ‡Π΅Ρ‚Π²Π΅Ρ€Ρ‚ΠΎΠΌ столбцС индСксной ΠΌΠ°Ρ‚Ρ€ΠΈΡ†Ρ‹. ΠŸΡƒΡΡ‚ΡŒ Π»ΠΎΠΊΠ°Π»ΡŒΠ½Ρ‹ΠΌ Π½ΠΎΠΌΠ΅Ρ€Π°ΠΌ 1, 2, 3 ΡΠΎΠΎΡ‚Π²Π΅Ρ‚ΡΡ‚Π²ΡƒΡŽΡ‚ фактичСскиС Π½ΠΎΠΌΠ΅Ρ€Π° i, j, k. Π’ΠΎΠ³Π΄Π° пСрвая строка локальной ΠΌΠ°Ρ‚Ρ€ΠΈΡ†Ρ‹ ΠΈ ΠΏΠ΅Ρ€Π²Ρ‹ΠΉ коэффициСнт локального Π²Π΅ΠΊΡ‚ΠΎΡ€-столбца ΡƒΡ‡Π°ΡΡ‚Π²ΡƒΡŽΡ‚ Π² Ρ„ΠΎΡ€ΠΌΠΈΡ€ΠΎΠ²Π°Π½ΠΈΠΈ i-ΠΉ строки глобальной ΠΌΠ°Ρ‚Ρ€ΠΈΡ†Ρ‹ ΠΈ i-ro коэффициСнта глобального Π²Π΅ΠΊΡ‚ΠΎΡ€-столбца. ΠŸΡ€ΠΎΠΈΠ·Π²ΠΎΠ΄ΠΈΡ‚ΡΡ слоТСниС Π½Π°ΠΉΠ΄Π΅Π½Π½Ρ‹Ρ… Π»ΠΎΠΊΠ°Π»ΡŒΠ½Ρ‹Ρ… коэффициСнтов с ΠΈΠΌΠ΅ΡŽΡ‰ΠΈΠΌΠΈΡΡ значСниями Π³Π»ΠΎΠ±Π°Π»ΡŒΠ½Ρ‹Ρ… коэффициСнтов Π΄Ρ†, Gij, Π—Π°Ρ‚Π΅ΠΌ аналогичная ΠΏΡ€ΠΎΡ†Π΅Π΄ΡƒΡ€Π° повторяСтся для Π²Ρ‚ΠΎΡ€ΠΎΠΉ ΠΈ Ρ‚Ρ€Π΅Ρ‚ΡŒΠ΅ΠΉ строк локальной ΠΌΠ°Ρ‚Ρ€ΠΈΡ†Ρ‹ ΠΈ Π²Ρ‚ΠΎΡ€ΠΎΠ³ΠΎ ΠΈ Ρ‚Ρ€Π΅Ρ‚ΡŒΠ΅Π³ΠΎ коэффициСнтов локального столбца. Они ΡƒΡ‡Π°ΡΡ‚Π²ΡƒΡŽΡ‚ Π² Ρ„ΠΎΡ€ΠΌΠΈΡ€ΠΎΠ²Π°Π½ΠΈΠΈ строк глобальной ΠΌΠ°Ρ‚Ρ€ΠΈΡ†Ρ‹ ΠΈ коэффициСнтов глобального столбца с Π½ΠΎΠΌΠ΅Ρ€Π°ΠΌΠΈ / ΠΈ ΠΊ, ΠΊΠΎΡ‚ΠΎΡ€Ρ‹Π΅ ΡΠΎΠΎΡ‚Π²Π΅Ρ‚ΡΡ‚Π²ΡƒΡŽΡ‚ Π»ΠΎΠΊΠ°Π»ΡŒΠ½Ρ‹ΠΌ Π½ΠΎΠΌΠ΅Ρ€Π°ΠΌ 2 ΠΈ 3.

 [c.144]


Π‘Π»ΠΎΠΆΠ΅Π½ΠΈΠ΅ ΠΈ Π²Ρ‹Ρ‡ΠΈΡ‚Π°Π½ΠΈΠ΅ ΠΌΠ°Ρ‚Ρ€ΠΈΡ†. Π”Π²Π΅ ΠΌΠ°Ρ‚Ρ€ΠΈΡ†Ρ‹ ΠΌΠΎΠΆΠ½ΠΎ ΡΠ»ΠΎΠΆΠΈΡ‚ΡŒ (Π²Ρ‹Ρ‡Π΅ΡΡ‚ΡŒ). Для этого Π½Π΅ΠΎΠ±Ρ…ΠΎΠ΄ΠΈΠΌΠΎ, Ρ‡Ρ‚ΠΎΠ±Ρ‹ порядки ΠΌΠ°Ρ‚Ρ€ΠΈΡ† Π±Ρ‹Π»ΠΈ ΠΎΠ΄ΠΈΠ½Π°ΠΊΠΎΠ²Ρ‹, ΠΈ Ρ‚ΠΎΠ³Π΄Π° ΡΠ»ΠΎΠΆΠΈΡ‚ΡŒ (Π²Ρ‹Ρ‡Π΅ΡΡ‚ΡŒ) ΡΠΎΠΎΡ‚Π²Π΅Ρ‚ΡΡ‚Π²ΡƒΡŽΡ‰ΠΈΠ΅ элСмСнты ΠΌΠ°Ρ‚Ρ€ΠΈΡ†.  [c.179]

НСтрудно ΡƒΠ±Π΅Π΄ΠΈΡ‚ΡŒΡΡ, Ρ‡Ρ‚ΠΎ Π²Ρ‹ΠΏΠΎΠ»Π½Π΅Π½ΠΈΠ΅ ΠΎΠΏΠ΅Ρ€Π°Ρ†ΠΈΠΉ Π΄ΠΈΡ„Ρ„Π΅Ρ€Π΅Π½Ρ†ΠΈΡ€ΠΎΠ²Π°Π½ΠΈΠ΅ Π²Π΅ΠΊΡ‚ΠΎΡ€Π° [X], ΡƒΠΌΠ½ΠΎΠΆΠ΅Π½ΠΈΠ΅ ΠΌΠ°Ρ‚Ρ€ΠΈΡ†Ρ‹ [К] Π½Π° Π²Π΅ΠΊΡ‚ΠΎΡ€ [Π ] ΠΈ слоТСниС с Π²Π΅ΠΊΡ‚ΠΎΡ€ΠΎΠΌ [F] β€”ΠΏΡ€ΠΈΠ²ΠΎΠ΄ΠΈΡ‚ ΠΊ систСмС ΡƒΡ€Π°Π²Π½Π΅Π½ΠΈΠΉ Π²ΠΈΠ΄Π° (1). Однако матричная Ρ„ΠΎΡ€ΠΌΠ° записи Π±ΠΎΠ»Π΅Π΅ ΠΊΠΎΠΌΠΏΠ°ΠΊΡ‚Π½Π° ΠΉ ΡƒΠΏΡ€ΠΎΡ‰Π°Π΅Ρ‚ дСйствия ΠΏΠΎ ΠΏΡ€Π΅ΠΎΠ±Ρ€Π°Π·ΠΎΠ²Π°Π½ΠΈΡŽ систСмы ΡƒΡ€Π°Π²Π½Π΅Π½ΠΈΠΉ.  

[c.181]

ПолноС ΠΈΠ·ΠΎΠ±Ρ€Π°ΠΆΠ΅Π½ΠΈΠ΅ Ρ‚ΠΈΠΏΠ° Π‘ образуСтся ΠΏΡ€ΠΈ ΠΏΠ΅Ρ€Π΅ΠΌΠ΅Ρ‰Π΅Π½ΠΈΠΈ прСобразоватСля Π² Π½Π°ΠΏΡ€Π°Π²Π»Π΅Π½ΠΈΠΈ, пСрпСндикулярном ΠΊ Π½Π°ΠΏΡ€Π°Π²Π»Π΅Π½ΠΈΡŽ элСктронного сканирования. ΠŸΡ€ΠΈ этом сигналы ΠΊΠΎΠΎΡ€Π΄ΠΈ-, Π½Π°Ρ‚ строки Π²Ρ‹Ρ€Π°Π±Π°Ρ‚Ρ‹Π²Π°ΡŽΡ‚ΡΡ Π΄Π°Ρ‚Ρ‡ΠΈΠΊΠ°ΠΌΠΈ ΠΊΠΎΠΎΡ€Π΄ΠΈΠ½Π°Ρ‚, ΠΊΠ°ΠΊ Π² систСмС с Ρ€ΡƒΡ‡Π½Ρ‹ΠΌ (мСханичСским) сканированиСм. Π‘ΠΎΠ»Π΅Π΅ простоС Ρ€Π΅ΡˆΠ΅Π½ΠΈΠ΅ этой Π·Π°Π΄Π°Ρ‡ΠΈ ΠΌΠΎΠΆΠ΅Ρ‚ Π±Ρ‹Ρ‚ΡŒ ΠΏΠΎΠ»ΡƒΡ‡Π΅Π½ΠΎ с ΠΏΡ€ΠΈΠΌΠ΅Π½Π΅Π½ΠΈΠ΅ΠΌ Π΄Π²ΡƒΠΌΠ΅Ρ€Π½ΠΎΠ³ΠΎ элСктронного сканирования. ΠŸΡŒΠ΅Π·ΠΎΡΠ»Π΅ΠΌΠ΅Π½Ρ‚Ρ‹ Π΄Π²ΡƒΠΌΠ΅Ρ€Π½ΠΎΠΉ ΠΌΠ°Ρ‚Ρ€ΠΈΡ†Ρ‹ (Π½Π°ΠΏΡ€ΠΈΠΌΠ΅Ρ€, с числом элСмСнтов 8X8) Π²ΠΎΠ·Π±ΡƒΠΆΠ΄Π°ΡŽΡ‚ΡΡ с Π·Π°Π΄Π΅Ρ€ΠΆΠΊΠ°ΠΌΠΈ, ΠΎΠ±Π΅ΡΠΏΠ΅Ρ‡ΠΈΠ²Π°ΡŽΡ‰ΠΈΠΌΠΈ слоТСниС Π°ΠΌΠΏΠ»ΠΈΡ‚ΡƒΠ΄ акустичСских ΠΈΠΌΠΏΡƒΠ»ΡŒΡΠΎΠ² лишь Π½Π° ΠΎΠΏΡ€Π΅Π΄Π΅Π»Π΅Π½Π½Ρ‹Ρ… направлСниях Π² ΠΎΠ±ΡŠΠ΅ΠΊΡ‚Π΅ контроля. Аналогично Π² Ρ‚Ρ€Π°ΠΊΡ‚Π΅ ΠΏΡ€ΠΈΠ΅ΠΌΠ° принятыС ΠΏΡŒΠ΅Π·ΠΎΡΠ»Π΅ΠΌΠ΅Π½Ρ‚Π°ΠΌΠΈ сигналы ΠΏΡ€Π΅Π΄Π²Π°Ρ€ΠΈΡ‚Π΅Π»ΡŒΠ½ΠΎ Π·Π°Π΄Π΅Ρ€ΠΆΠΈΠ²Π°ΡŽΡ‚ΡΡ Ρ‚Π°ΠΊ, Ρ‡Ρ‚ΠΎ суммированиС Π°ΠΌΠΏΠ»ΠΈΡ‚ΡƒΠ΄ соотвСтствуСт Π½Π°ΠΏΡ€Π°Π²Π»Π΅Π½ΠΈΡŽ излучСния.  

[c.271]

ΠœΠ°Ρ‚Ρ€ΠΈΡ†Π° называСтся Π΅Π΄ΠΈΠ½ΠΈΡ‡Π½ΠΎΠΉ ΠΌΠ°Ρ‚Ρ€ΠΈΡ†Π΅ΠΉ. Удобство Π·Π°ΠΏΠΈΡΡ‹Π²Π°Ρ‚ΡŒ совокупности Π·Π½Π°Ρ‡Π΅Π½ΠΈΠΉ ΠΊΠ°ΠΊΠΈΡ…-Π»ΠΈΠ±ΠΎ Π²Π΅Π»ΠΈΡ‡ΠΈΠ½ Π² Ρ‚Π°ΠΊΠΎΠΌ Π²ΠΈΠ΄Π΅ Π·Π°ΠΊΠ»ΡŽΡ‡Π°Π΅Ρ‚ΡΡ Π² Ρ‚ΠΎΠΌ, Ρ‡Ρ‚ΠΎ Π½Π°Π΄ ΠΌΠ°Ρ‚Ρ€ΠΈΡ†Π°ΠΌΠΈ ΠΌΠΎΠΆΠ½ΠΎ ΠΏΡ€ΠΎΠΈΠ·Π²ΠΎΠ΄ΠΈΡ‚ΡŒ ΡΠΎΠΎΡ‚Π²Π΅Ρ‚ΡΡ‚Π²ΡƒΡŽΡ‰ΠΈΠ΅ дСйствия (слоТСниС, ΡƒΠΌΠ½ΠΎΠΆΠ΅Π½ΠΈΠ΅, см., Π½Π°ΠΏΡ€ΠΈΠΌΠ΅Ρ€, Ρ€]).  [c.93]

БСсконСчно ΠΌΠ°Π»Ρ‹Π΅ ΠΏΠΎΠ²ΠΎΡ€ΠΎΡ‚Ρ‹. ЦСлСсообразно ΠΏΠΎΠΏΡ‹Ρ‚Π°Ρ‚ΡŒΡΡ ΡƒΡΡ‚Π°Π½ΠΎΠ²ΠΈΡ‚ΡŒ соотвСтствиС ΠΌΠ΅ΠΆΠ΄Ρƒ Π²Π΅ΠΊΡ‚ΠΎΡ€Π°ΠΌΠΈ ΠΈ ΠΊΠΎΠ½Π΅Ρ‡Π½Ρ‹ΠΌΠΈ ΠΏΠΎΠ²ΠΎΡ€ΠΎΡ‚Π°ΠΌΠΈ, описываСмыми ΠΎΡ€Ρ‚ΠΎΠ³ΠΎΠ½Π°Π»ΡŒΠ½Ρ‹ΠΌΠΈ ΠΌΠ°Ρ‚Ρ€ΠΈΡ†Π°ΠΌΠΈ. Π’Π΅ΠΊΡ‚ΠΎΡ€, ΠΊΠΎΡ‚ΠΎΡ€Ρ‹ΠΉ ΠΌΡ‹ поставим Π² соотвСтствиС Π½Π΅ΠΊΠΎΡ‚ΠΎΡ€ΠΎΠΌΡƒ ΠΏΠΎΠ²ΠΎΡ€ΠΎΡ‚Ρƒ, Π΄ΠΎΠ»ΠΆΠ΅Π½, ΠΊΠΎΠ½Π΅Ρ‡Π½ΠΎ, ΠΈΠΌΠ΅Ρ‚ΡŒ ΠΎΠΏΡ€Π΅Π΄Π΅Π»Π΅Π½Π½ΠΎΠ΅ Π½Π°ΠΏΡ€Π°Π²Π»Π΅Π½ΠΈΠ΅ β€”Π½Π°ΠΏΡ€Π°Π²Π»Π΅Π½ΠΈΠ΅ оси вращСния ΠΈ ΠΎΠΏΡ€Π΅Π΄Π΅Π»Π΅Π½Π½ΡƒΡŽ Π²Π΅Π»ΠΈΡ‡ΠΈΠ½Ρƒ, Π½Π°ΠΏΡ€ΠΈΠΌΠ΅Ρ€ Ρ€Π°Π²Π½ΡƒΡŽ ΡƒΠ³Π»Ρƒ ΠΏΠΎΠ²ΠΎΡ€ΠΎΡ‚Π°. ΠœΡ‹ сСйчас ΡƒΠ²ΠΈΠ΄ΠΈΠΌ, Ρ‡Ρ‚ΠΎ ΡƒΡΠΏΠ΅ΡˆΠ½ΠΎ ΠΎΡΡƒΡ‰Π΅ΡΡ‚Π²ΠΈΡ‚ΡŒ Ρ‚Π°ΠΊΠΎΠ΅ соотвСтствиС оказываСтся Π½Π΅Π²ΠΎΠ·ΠΌΠΎΠΆΠ½Ρ‹ΠΌ. ΠŸΡ€Π΅Π΄ΠΏΠΎΠ»ΠΎΠΆΠΈΠΌ, Ρ‡Ρ‚ΠΎ А ΠΈ Π’ Π±ΡƒΠ΄ΡƒΡ‚ двумя Ρ‚Π°ΠΊΠΈΠΌΠΈ Π²Π΅ΠΊΡ‚ΠΎΡ€Π°ΠΌΠΈ , связанными с прСобразованиями А ΠΈ Π’. Π’ΠΎΠ³Π΄Π°, ΠΏΠΎΡΠΊΠΎΠ»ΡŒΠΊΡƒ это Π²Π΅ΠΊΡ‚ΠΎΡ€Ρ‹, ΠΎΠ½ΠΈ Π΄ΠΎΠ»ΠΆΠ½Ρ‹ ΠΎΠ±Π»Π°Π΄Π°Ρ‚ΡŒ свойством коммутативности ΠΏΡ€ΠΈ слоТСнии, Ρ‚. Π΅. для Π½ΠΈΡ… Π΄ΠΎΠ»ΠΆΠ½ΠΎ Π²Ρ‹ΠΏΠΎΠ»Π½ΡΡ‚ΡŒΡΡ равСнство  [c.142]

Но слоТСниС Π΄Π²ΡƒΡ… Π²Ρ€Π°Ρ‰Π΅Π½ΠΈΠΉ, Ρ‚. Π΅. ΠΏΠΎΡΠ»Π΅Π΄ΠΎΠ²Π°Ρ‚Π΅Π»ΡŒΠ½ΠΎΠ΅ Π²Ρ‹ΠΏΠΎΠ»Π½Π΅Π½ΠΈΠ΅ ΠΎΠ΄Π½ΠΎΠ³ΠΎ ΠΈΠ· Π½ΠΈΡ… Π·Π° Π΄Ρ€ΡƒΠ³ΠΈΠΌ, описываСтся, ΠΊΠ°ΠΊ ΠΌΡ‹ Π·Π½Π°Π΅ΠΌ, ΠΏΡ€ΠΎΠΈΠ·Π²Π΅Π΄Π΅Π½ΠΈΠ΅ΠΌ ΠΌΠ°Ρ‚Ρ€ΠΈΡ† АВ, ΠΈ это ΡƒΠΌΠ½ΠΎΠΆΠ΅Π½ΠΈΠ΅ Π½Π΅ ΠΊΠΎΠΌΠΌΡƒΡ‚Π°Ρ‚ΠΈΠ²Π½ΠΎ, Ρ‚. Π΅. АВ =ВА. Π‘Π»Π΅Π΄ΠΎΠ²Π°Ρ‚Π΅Π»ΡŒΠ½ΠΎ, Π²Π΅ΠΊΡ‚ΠΎΡ€Ρ‹ Π› ΠΈ Π” Π½Π΅ Π±ΡƒΠ΄ΡƒΡ‚ ΠΎΠ±Π»Π°Π΄Π°Ρ‚ΡŒ ΠΊΠΎΠΌΠΌΡƒΡ‚Π°Ρ‚ΠΈΠ²Π½ΠΎΡΡ‚ΡŒΡŽ слоТСния ΠΈ поэтому ΠΈΡ… нСльзя Π±ΡƒΠ΄Π΅Ρ‚ ΡΡ‡ΠΈΡ‚Π°Ρ‚ΡŒ Π²  [c.142]

ΠœΡ‹ ΠΈΠΌΠ΅Π΅ΠΌ здСсь ΠΈΠ»Π»ΡŽΡΡ‚Ρ€Π°Ρ†ΠΈΡŽ слоТСния Π²Ρ€Π°Ρ‰Π΅Π½ΠΈΠΉ согласно Ρ„ΠΎΡ€ΠΌΡƒΠ»Π΅ (9.13), ΠΈΠ±ΠΎ, транспонируя элСмСнты Π² (11.1)β€”(11.3), ΠΌΠΎΠΆΠ½ΠΎ ΠΏΠΎΠ»ΡƒΡ‡ΠΈΡ‚ΡŒ ΠΌΠ°Ρ‚Ρ€ΠΈΡ†Ρ‹ М, М2, Mg ΠΈ нСпосрСдствСнно ΡƒΠ±Π΅Π΄ΠΈΡ‚ΡŒΡΡ, Ρ‡Ρ‚ΠΎ ΠΌΠ°Ρ‚Ρ€ΠΈΡ†Π° М (11.5) прСдставима Π² Π²ΠΈΠ΄Π΅  [c.47]

Π’ Π°Π»Π³Π΅Π±Ρ€Π΅ ΠΌΠ°Ρ‚Ρ€ΠΈΡ† ΠΎΠΏΡ€Π΅Π΄Π΅Π»ΡΡŽΡ‚ΡΡ ΡΠ»Π΅Π΄ΡƒΡŽΡ‰ΠΈΠ΅ дСйствия Π½Π°Π΄ ΠΌΠ°Ρ‚Ρ€ΠΈΡ†Π°ΠΌΠΈ Π°) слоТСниС ΠΌΠ°Ρ‚Ρ€ΠΈΡ† Π±) ΡƒΠΌΠ½ΠΎΠΆΠ΅Π½ΠΈΠ΅ ΠΌΠ°Ρ‚Ρ€ΠΈΡ†Ρ‹ Π½Π° число Π²) ΡƒΠΌΠ½ΠΎΠΆΠ΅Π½ΠΈΠ΅ ΠΌΠ°Ρ‚Ρ€ΠΈΡ†. Π£ΠΊΠ°Π·Π°Π½Π½Ρ‹Π΅ дСйствия ΠΏΠΎΠ·Π²ΠΎΠ»ΡΡŽΡ‚ Π²Ρ‹Ρ‡ΠΈΡΠ»ΠΈΡ‚ΡŒ соотвСтствСнно сумму ΠΌΠ°Ρ‚Ρ€ΠΈΡ†, ΠΏΡ€ΠΎΠΈΠ·Π²Π΅Π΄Π΅Π½ΠΈΠ΅ ΠΌΠ°Ρ‚Ρ€ΠΈΡ†Ρ‹ Π½Π° число, ΠΏΡ€ΠΎΠΈΠ·Π²Π΅Π΄Π΅Π½ΠΈΠ΅ ΠΌΠ°Ρ‚Ρ€ΠΈΡ† ΠΈ, ΠΊΠ°ΠΊ слСдствиС, Ρ€Π°Π·Π½ΠΎΡΡ‚ΡŒ ΠΌΠ°Ρ‚Ρ€ΠΈΡ†.  [c.41]

ПослС построСния /-ΠΉ строки ΠΌΠ°Ρ‚Ρ€ΠΈΡ†Ρ‹ F выполняСтся цикличСский сдвиг элСмСнтов ΠΌΠ°Ρ‚Ρ€ΠΈΡ†Ρ‹-строки R Π½Π° ΠΎΠ΄ΠΈΠ½ Π²Π»Π΅Π²ΠΎ ΠΈΠ»ΠΈ Π²ΠΏΡ€Π°Π²ΠΎ для формирования Π½Π°ΠΏΡ€Π°Π²Π»Π΅Π½ΠΈΠΉ ΡˆΡ‚Ρ€ΠΈΡ…ΠΎΠ²ΠΊΠΈ, ΡΠΎΠΎΡ‚Π²Π΅Ρ‚ΡΡ‚Π²ΡƒΡŽΡ‰ΠΈΡ… ΡƒΠ³Π»Π°ΠΌ 45 ΠΈ 135Β°. Π—Π°Π²Π΅Ρ€ΡˆΠΈΠ² Ρ„ΠΎΡ€ΠΌΠΈΡ€ΠΎΠ²Π°Π½ΠΈΠ΅ ΠΌΠ°Ρ‚Ρ€ΠΈΡ†Ρ‹ F, с ΠΏΠΎΠΌΠΎΡ‰ΡŒΡŽ логичСского слоТСния ΠΏΠΎΠ»ΡƒΡ‡ΠΈΠΌ ΠΌΠ°Ρ‚Ρ€ΠΈΡ†Ρƒ ΡΠΎΠ΄Π΅Ρ€ΠΆΠ°Ρ‰ΡƒΡŽ описаниС Π·Π°ΡˆΡ‚Ρ€ΠΈΡ…ΠΎΠ²Π°Π½Π½ΠΎΠΉ области сСчСния  [c.123]

ЛогичСскоС слоТСниС ΠΌΠ°Ρ‚Ρ€ΠΈΡ† рСализуСтся ΠΏΡƒΡ‚Π΅ΠΌ ΠΏΠΎΠΏΠ°Ρ€Π½ΠΎΠ³ΠΎ логичСского слоТСния ΠΎΠ΄Π½ΠΎΠΈΠΌΠ΅Π½Π½Ρ‹Ρ… элСмСнтов исходных ΠΌΠ°Ρ‚Ρ€ΠΈΡ†  [c.123]

Π‘Π»ΠΎΠΆΠ΅Π½ΠΈΠ΅ ΠΈ Π²Ρ‹Ρ‡ΠΈΡ‚Π°Π½ΠΈΠ΅ ΠΌΠ°Ρ‚Ρ€ΠΈΡ†. Π­Ρ‚ΠΈ ΠΎΠΏΠ΅Ρ€Π°Ρ†ΠΈΠΈ ΠΈΠΌΠ΅ΡŽΡ‚ смысл лишь ΠΏΡ€ΠΈ ΠΈΡ… ΠΎΠ΄ΠΈΠ½Π°ΠΊΠΎΠ²ΠΎΠΉ структурС (ΠΎΠ΄ΠΈΠ½Π°ΠΊΠΎΠ²ΠΎΠΌ количСствС строк ΠΈ ΠΎΠ΄ΠΈΠ½Π°ΠΊΠΎΠ²ΠΎΠΌ количСствС столбцов) слагаСмых ΠΈΠ»ΠΈ Π²Ρ‹Ρ‡ΠΈΡ‚Π°Π΅ΠΌΡ‹Ρ… ΠΌΠ°Ρ‚Ρ€ΠΈΡ†. Π‘ΡƒΠΌΠΌΠΎΠΉ (ΠΈΠ»ΠΈ Ρ€Π°Π·Π½ΠΎΡΡ‚ΡŒΡŽ) Π΄Π²ΡƒΡ… (Ρ‚ Ρ… ΠΏ) ΠΌΠ°Ρ‚Ρ€ΠΈΡ† А =– Π°,-/ ΠΈ Π’ = ( fe(jj называСтся (Ρ‚ Ρ… ΠΏ) ΠΌΠ°Ρ‚Ρ€ΠΈΡ†Π° Π‘ = с Ρƒ (Π³ = 1, 2,. . ., Ρ‚, / = 1, 2,. . ., ΠΏ), элСмСнты ΠΊΠΎΡ‚ΠΎΡ€ΠΎΠΉ Ρ€Π°Π²Π½Ρ‹ суммС (ΠΈΠ»ΠΈ разности) ΡΠΎΠΎΡ‚Π²Π΅Ρ‚ΡΡ‚Π²ΡƒΡŽΡ‰ΠΈΡ… элСмСнтов слагаСмых (Π²Ρ‹Ρ‡ΠΈΡ‚Π°Π΅ΠΌΡ‹Ρ…) ΠΌΠ°Ρ‚Ρ€ΠΈΡ†, Ρ‚. Π΅.  [c.22]

Π‘Π»ΠΎΠΆΠ΅Π½ΠΈΠ΅ (Π° ΡΠ»Π΅Π΄ΠΎΠ²Π°Ρ‚Π΅Π»ΡŒΠ½ΠΎ, ΠΈ Π²Ρ‹Ρ‡ΠΈΡ‚Π°Π½ΠΈΠ΅) ΠΌΠ°Ρ‚Ρ€ΠΈΡ† ассоциативно ΠΈ ΠΊΠΎΠΌΠΌΡƒΡ‚Π°Ρ‚ΠΈΠ²Π½ΠΎ, Ρ‚. Π΅.  [c.22]

ΠžΠΏΠ΅Ρ€Π°Ρ†ΠΈΠΈ слоТСния ΠΈ вычитания ΠΌΠ°Ρ‚Ρ€ΠΈΡ† ΠΏΡ€ΠΈΠΌΠ΅Π½ΡΡŽΡ‚ΡΡ ΠΏΡ€ΠΈ пСрСносС систСм ΠΊΠΎΠΎΡ€Π΄ΠΈΠ½Π°Ρ‚ Π² процСссС Ρ€Π΅ΡˆΠ΅Π½ΠΈΡ Π·Π°Π΄Π°Ρ‡ Π°Π½Π°Π»ΠΈΠ·Π° ΠΌΠ΅Ρ…Π°Π½ΠΈΠ·ΠΌΠΎΠ² (см. Π³Π». 17, ΠΏ. 34).  [c.22]

Если ΡƒΡ‡Π΅ΡΡ‚ΡŒ, Ρ‡Ρ‚ΠΎ для пСрСмноТСния Π΄Π²ΡƒΡ… ΠΌΠ°Ρ‚Ρ€ΠΈΡ† ΠΏ-Ρ‚ΠΎ порядка Π½Π΅ΠΎΠ±Ρ…ΠΎΠ΄ΠΈΠΌΠΎ ΠΎΡΡƒΡ‰Π΅ΡΡ‚Π²ΠΈΡ‚ΡŒ Π» ΡƒΠΌΠ½ΠΎΠΆΠ΅Π½ΠΈΠΉ ΠΈ ΠΏ β€” 1) слоТСний, Ρ‚ΠΎ для пСрСмноТСния Π΄Π²ΡƒΡ… ΠΌΠ°Ρ‚Ρ€ΠΈΡ† 4-Π³ΠΎ порядка Π½ΡƒΠΆΠ½ΠΎ произвСсти 64 умноТСния ΠΈ 48 слоТСний ΠΈΠ»ΠΈ всСго 112 Π²Ρ‹Ρ‡ΠΈΡΠ»ΠΈΡ‚Π΅Π»ΡŒΠ½Ρ‹Ρ… дСйствий, Π° для ΠΌΠ°Ρ‚Ρ€ΠΈΡ† 3-Π³ΠΎ порядка соотвСтствСнно 27 + 18 = = 45 дСйствий.  [c.188]


Π’ Π½ΠΈΠΆΠ½Π΅ΠΉ Ρ‚Ρ€Π°Π²Π΅Ρ€Π·Π΅ прСсса имССтся ΠΏΡ€ΠΎΡ‘ΠΌ для пропуска Ρ‡Π΅Ρ€Π΅Π· прСсс Π±Π°Π»ΠΊΠΈ ΠΈΠ»ΠΈ Π΄Π²ΡƒΡ… ΡˆΠ²Π΅Π»Π»Π΅Ρ€ΠΎΠ², слоТСнных стСнками Π½ΠΈΠΆΠ½ΠΈΠΉ стол, Π½Π° ΠΊΠΎΡ‚ΠΎΡ€ΠΎΠΌ устанавливаСтся инструмСнт, состоит ΠΈΠ· Π΄Π²ΡƒΡ… частСй. ΠŸΠΎΠ΄ΡƒΡˆΠΊΠΈ с ΠΌΠ°Ρ‚Ρ€ΠΈΡ†Π°ΠΌΠΈ для пробивания отвСрстий Π² ΠΏΠΎΠ»ΠΊΠ°Ρ… ΡƒΡΡ‚Π°Π½Π°Π²Π»ΠΈΠ²Π°ΡŽΡ‚ΡΡ Π½Π° ΠΊΠΎΠ½Ρ†Π°Ρ… ΠΊΠΎΠ½ΡΠΎΠ»ΡŒΠ½Ρ‹Ρ… выступов стола.  [c.483]

Рассмотрим, ΠΊΠ°ΠΊ влияСт ΡƒΡ‡Π΅Ρ‚ диспСрсий Π½Π° Ρ€Π΅Π·ΡƒΠ»ΡŒΡ‚Π°Ρ‚Ρ‹ расчСта. Π€ΠΎΡ€ΠΌΠ°Π»ΡŒΠ½ΠΎ ΡƒΡ‡Π΅Ρ‚ диспСрсий выраТаСтся Π² слоТСнии ΠΌΠ°Ρ‚Ρ€ΠΈΡ†Ρ‹ [Π°] [Π°] с диагональной ΠΌΠ°Ρ‚Ρ€ΠΈΡ†Π΅ΠΉ [ )], ΠΈΠΌΠ΅ΡŽΡ‰Π΅ΠΉ ΠΏΠΎΠ»ΠΎΠΆΠΈΡ‚Π΅Π»ΡŒΠ½Ρ‹Π΅ Ρ‡Π»Π΅Π½Ρ‹. Π­Ρ‚ΠΎ способствуСт ΡƒΠ»ΡƒΡ‡ΡˆΠ΅Π½ΠΈΡŽ обусловлСнности ΠΌΠ°Ρ‚Ρ€ΠΈΡ†Ρ‹ [Π°] [Π°], Ρ‡Ρ‚ΠΎ особСнно Π²Π°ΠΆΠ½ΠΎ Π² случаях, ΠΊΠΎΠ³Π΄Π° послСдняя ΠΏΠ»ΠΎΡ…ΠΎ  [c.58]

Π‘Π»ΠΎΠΆΠ΅Π½ΠΈΠ΅ ΠΌΠ°Ρ‚Ρ€ΠΈΡ†. Π‘ΡƒΠΌΠΌΠΎΠΉ ΠΌΠ°Ρ‚Ρ€ΠΈΡ†  [c.95]

DN β€” присвоСниС Π·Π½Π°Ρ‡Π΅Π½ΠΈΠΉ Π΅Π΄ΠΈΠ½ΠΈΡ†Ρ‹ Π΄ΠΈΠ°Π³ΠΎΠ½Π°Π»ΡŒΠ½Ρ‹ΠΌ элСмСнтам Π΄Π²ΡƒΠΌΠ΅Ρ€Π½ΠΎΠ³ΠΎ массива А+Π’ β€” слоТСниС ΠΌΠ°Ρ‚Ρ€ΠΈΡ† А ΠΈ Π’  [c.163]

ΠΠ΅ΠΎΠ±Ρ…ΠΎΠ΄ΠΈΠΌΠΎΡΡ‚ΡŒ примСнСния динамичСского ΠΌΠ΅Ρ‚ΠΎΠ΄Π° сущСствСнно услоТняСт Ρ€Π΅ΡˆΠ΅Π½ΠΈΠ΅ нСконсСрвативных Π·Π°Π΄Π°Ρ‡ устойчивости. Π—Π΄Π΅ΡΡŒ трСбуСтся вСсьма эффСктивный ΠΌΠ΅Ρ‚ΠΎΠ΄ опрСдСлСния частот собствСнных ΠΊΠΎΠ»Π΅Π±Π°Π½ΠΈΠΉ. Π‘Ρ€Π΅Π΄ΠΈ Π΄Ρ€ΡƒΠ³ΠΈΡ… ΠΌΠ΅Ρ‚ΠΎΠ΄ΠΎΠ² Π² этом ΠΎΡ‚Π½ΠΎΡˆΠ΅Π½ΠΈΠΈ Π²ΡŒΡ‰Π΅Π»ΡΠ΅Ρ‚ΡΡ ΠœΠ“Π­. Он позволяСт ΠΏΠΎΠ»ΡƒΡ‡Π°Ρ‚ΡŒ Ρ‚ΠΎΡ‡Π½Ρ‹ΠΉ спСктр частот (устраняСт нСдостаток МКЭ), Π° Π² трансцСндСнтном частотном ΡƒΡ€Π°Π²Π½Π΅Π½ΠΈΠΈ ΠΎΡ‚ΡΡƒΡ‚ΡΡ‚Π²ΡƒΡŽΡ‚ Ρ‚ΠΎΡ‡ΠΊΠΈ Ρ€Π°Π·Ρ€Ρ‹Π²Π° 2-Π³ΠΎ Ρ€ΠΎΠ΄Π° (устраняСт нСдостаток ΠΌΠ΅Ρ‚ΠΎΠ΄Π° ΠΏΠ΅Ρ€Π΅ΠΌΠ΅Ρ‰Π΅Π½ΠΈΠΉ). Π”ΠΎΠΏΠΎΠ»Π½ΠΈΡ‚Π΅Π»ΡŒΠ½Ρ‹ΠΌΠΈ ΠΏΠΎΠ»ΠΎΠΆΠΈΡ‚Π΅Π»ΡŒΠ½Ρ‹ΠΌΠΈ Ρ„Π°ΠΊΡ‚ΠΎΡ€Π°ΠΌΠΈ ΡΠ²Π»ΡΡŽΡ‚ΡΡ простая Π»ΠΎΠ³ΠΈΠΊΠ° формирования динамичСской ΠΌΠ°Ρ‚Ρ€ΠΈΡ†Ρ‹ устойчивости, отсутствиС ΠΎΠΏΠ΅Ρ€Π°Ρ†ΠΈΠΉ умноТСния, обращСния ΠΈ слоТСния ΠΌΠ°Ρ‚Ρ€ΠΈΡ†, Ρ…ΠΎΡ€ΠΎΡˆΠ°Ρ ΡƒΡΡ‚ΠΎΠΉΡ‡ΠΈΠ²ΠΎΡΡ‚ΡŒ числСнных ΠΎΠΏΠ΅Ρ€Π°Ρ†ΠΈΠΉ ΠΏΡ€ΠΈ вычислСнии опрСдСлитСля ΠΈ Ρ‚.ΠΏ.  [c.196]

ΠšΠ²Π°Ρ‚Π΅Ρ€Π½ΠΈΠΎΠ½Π½ΠΎΠ΅ слоТСниС ΠΏΠΎΠ²ΠΎΡ€ΠΎΡ‚ΠΎΠ². Как ΠΈ Π² случаС ΠΌΠ°Ρ‚Ρ€ΠΈΡ†, слоТСнию ΠΏΠΎΠ²ΠΎΡ€ΠΎΡ‚ΠΎΠ² ΠΎΡ‚Π²Π΅Ρ‡Π°Π΅Ρ‚ ΠΏΡ€ΠΎΠΈΠ·Π²Π΅Π΄Π΅Π½ΠΈΠ΅ ΠΊΠ²Π°Ρ‚Π΅Ρ€Π½ΠΈΠΎΠ½ΠΎΠ², ΠΏΡ€ΠΈ этом активная ΠΈ пассивная Ρ‚ΠΎΡ‡ΠΊΠΈ зрСния Π½Π° прСобразования ΠΈΠΌΠ΅ΡŽΡ‚ сущСствСнныС отличия.  [c.46]

Π‘Π»ΠΎΠΆΠ΅Π½ΠΈΠ΅. ΠœΠ°Ρ‚Ρ€ΠΈΡ†Ρ‹ А ΠΏ Π’ ΠΌΠΎΠ³ΡƒΡ‚ Π±Ρ‹Ρ‚ΡŒ слоТСны, Ссли ΠΎΠ½ΠΈ ΠΈΠΌΠ΅ΡŽΡ‚ ΠΎΠ΄ΠΈ-Π½Π°ΠΊΠΎΠΈΠΎΠ΅ число строк ΠΈ столбцов.  [c.631]

Π’Π°ΠΊΠΎΠΌ ΠΎΠ±Ρ€Π°Π·ΠΎΠΌ, для выполнСния Π°Π»Π³ΠΎΡ€ΠΈΡ‚ΠΌΠ° (55) Ρ‚Ρ€Π΅Π±ΡƒΡŽΡ‚ΡΡ Π΄Π²Π° прямых ΠΈ ΠΎΠ΄Π½ΠΎ ΠΎΠ±Ρ€Π°Ρ‚Π½ΠΎΠ΅ ΠΏΡ€Π΅ΠΎΠ±Ρ€Π°Π·ΠΎΠ²Π°Π½ΠΈΠ΅ Π€/Ρ€ΡŒΠ΅, Π° Ρ‚Π°ΠΊΠΆΠ΅ прямоС ΡƒΠΌΠ½ΠΎΠΆΠ΅Π½ΠΈΠ΅ ΠΌΠ°Ρ‚Ρ€ΠΈΡ†Ρ‹ Π½Π° ΠΌΠ°Ρ‚Ρ€ΠΈΡ†Ρƒ. Если Π² качСствС Π΄ΠΈΠΆΡ€Π΅Ρ‚Π½ΠΎΠ³ΠΎ прСобразования Π€ΡƒΡ€ΡŒΠ΅ ΠΈΡΠΏΠΎΠ»ΡŒΠ·ΠΎΠ²Π°Ρ‚ΡŒ Π°Π»Π³ΠΎΡ€ΠΈΡ‚ΠΌ Π‘ΠŸΠ€, число ΠΎΠΏΠ΅Ρ€Π° Π΄Π°ΠΉ слоТСния составит 2N og2 , Π° число ΠΎΠΏΠ΅Ρ€Π°Ρ†ΠΈΠΉ умноТСния -.  [c.63]

Показано, Ρ‡Ρ‚ΠΎ Π² случаС ΠΏΡ€ΠΎΠΈΠ·Π²ΠΎΠ»ΡŒΠ½Ρ‹Ρ… распрСдСлСний Π½Π°Ρ€Π°Π±ΠΎΡ‚ΠΎΠΊ ΠΈ Π²Ρ€Π΅ΠΌΠ΅Π½ΠΈ восстановлСния Π² ΠΌΠ°Ρ‚Ρ€ΠΈΡ†Ρƒ Π“Π±) вводятся ооотвСтствувщиС вСроятности ΠΏΠ΅Ρ€Π΅Ρ…ΠΎΠ΄ΠΎΠ², Π° вмСсто слоТСния интСнсивностСй осущСствляСтся ΡƒΠΌΠ½ΠΎΠΆΠ΅Π½ΠΈΠ΅ вСроятностСй нСзависимых событий.  [c.15]

Π‘Π»ΠΎΠΆΠ΅Π½ΠΈΠ΅ Π΄Π²ΡƒΡ… ΠΌΠ°Ρ‚Ρ€ΠΈΡ† Π½Π΅ являСтся Ρ‚Π°ΠΊΠΎΠΉ Π²Π°ΠΆΠ½ΠΎΠΉ ΠΎΠΏΠ΅Ρ€Π°Ρ†ΠΈΠ΅ΠΉ, ΠΊΠ°ΠΊ ΠΈΡ… ΡƒΠΌΠ½ΠΎΠΆΠ΅Π½ΠΈΠ΅, ΠΎΠ΄Π½Π°ΠΊΠΎ ΠΎΠ½ΠΎ встрСчаСтся достаточно часто. Под суммойА + Π’ понимаСтся такая ΠΌΠ°Ρ‚Ρ€ΠΈΡ†Π° Π‘, элСмСнты ΠΊΠΎΡ‚ΠΎΡ€ΠΎΠΉ ΠΏΠΎΠ»ΡƒΡ‡Π°ΡŽΡ‚ΡΡ посрСдством слоТСния ΡΠΎΠΎΡ‚Π²Π΅Ρ‚ΡΡ‚Π²ΡƒΡŽΡ‰ΠΈΡ… элСмСнтов А ΠΈ Π’. Π’Π°ΠΊΠΈΠΌ ΠΎΠ±Ρ€Π°Π·ΠΎΠΌ, ΠΌΠΎΠΆΠ½ΠΎ Π½Π°ΠΏΠΈΡΠ°Ρ‚ΡŒ  [c.119]

ΠœΠΎΠΆΠ΅Ρ‚ ΠΏΠΎΠΊΠ°Π·Π°Ρ‚ΡŒΡΡ, Ρ‡Ρ‚ΠΎ ΡΠΊΠΎΡ€ΠΎΡΡ‚ΡŒ, Π±ΠΎΠ»ΡŒΡˆΡƒΡŽ скорости свСта с, ΠΌΠΎΠΆΠ½ΠΎ ΠΏΠΎΠ»ΡƒΡ‡ΠΈΡ‚ΡŒ с ΠΏΠΎΠΌΠΎΡ‰ΡŒΡŽ Π΄Π²ΡƒΡ… ΠΏΠΎΡΠ»Π΅Π΄ΠΎΠ²Π°Ρ‚Π΅Π»ΡŒΠ½Ρ‹Ρ… ΠΏΡ€Π΅ΠΎΠ±Ρ€Π°Π·ΠΎΠ²Π°Π½ΠΈΠΉ Π›ΠΎΡ€Π΅Π½Ρ†Π°. ΠŸΡƒΡΡ‚ΡŒ, Π½Π°ΠΏΡ€ΠΈΠΌΠ΅Ρ€, вторая систСма двиТСтся ΠΎΡ‚Π½ΠΎΡΠΈΡ‚Π΅Π»ΡŒΠ½ΠΎ ΠΏΠ΅Ρ€Π²ΠΎΠΉ со ΡΠΊΠΎΡ€ΠΎΡΡ‚ΡŒΡŽ Vi > с/2, Π° Ρ‚Ρ€Π΅Ρ‚ΡŒΡ систСма двиТСтся ΠΎΡ‚Π½ΠΎΡΠΈΡ‚Π΅Π»ΡŒΠ½ΠΎ Π²Ρ‚ΠΎΡ€ΠΎΠΉ со ΡΠΊΠΎΡ€ΠΎΡΡ‚ΡŒΡŽ 02, Ρ‚Π°ΠΊΠΆΠ΅ большСй, Ρ‡Π΅ΠΌ с/2 (Π² Ρ‚ΠΎΠΌ ΠΆΠ΅ Π½Π°ΠΏΡ€Π°Π²Π»Π΅Π½ΠΈΠΈ). МоТно ΠΏΠΎΠ΄ΡƒΠΌΠ°Ρ‚ΡŒ, Ρ‡Ρ‚ΠΎ ΡΠΊΠΎΡ€ΠΎΡΡ‚ΡŒ Ρ‚Ρ€Π΅Ρ‚ΡŒΠ΅ΠΉ систСмы ΠΎΡ‚Π½ΠΎΡΠΈΡ‚Π΅Π»ΡŒΠ½ΠΎ ΠΏΠ΅Ρ€Π²ΠΎΠΉ Π±ΡƒΠ΄Π΅Ρ‚ Ρ‚ΠΎΠ³Π΄Π° большС Ρ‡Π΅ΠΌ с. Однако это Π½Π΅ Ρ‚Π°ΠΊ, ΠΈΠ±ΠΎ эта ΡΠΊΠΎΡ€ΠΎΡΡ‚ΡŒ Π½Π΅ Ρ€Π°Π²Π½Π° просто V -f Ua-Π§Ρ‚ΠΎΠ±Ρ‹ ΡƒΠ±Π΅Π΄ΠΈΡ‚ΡŒΡΡ Π² этом, достаточно Π½Π°ΠΉΡ‚ΠΈ ΠΏΡ€Π΅ΠΎΠ±Ρ€Π°Π·ΠΎΠ²Π°Π½ΠΈΠ΅ Π›ΠΎ-)Π΅Π½Ρ†Π°, ΠΎΠΏΠΈΡΡ‹Π²Π°ΡŽΡ‰Π΅Π΅ ΠΏΠ΅Ρ€Π΅Ρ…ΠΎΠ΄ ΠΎΡ‚ ΠΏΠ΅Ρ€Π²ΠΎΠΉ систСмы ΠΊ Ρ‚Ρ€Π΅Ρ‚ΡŒΠ΅ΠΉ. 1СрСмноТая для этого ΠΌΠ°Ρ‚Ρ€ΠΈΡ†Ρ‹ рассматриваСмых ΠΏΡ€Π΅ΠΎΠ±Ρ€Π°Π·ΠΎΠ²Π°Π½ΠΈΠΉ, ΠΌΡ‹ Π½Π°ΠΉΠ΄Π΅ΠΌ ΠΏΠΎΠ»Π½ΠΎΠ΅ ΠΏΡ€Π΅ΠΎΠ±Ρ€Π°Π·ΠΎΠ²Π°Π½ΠΈΠ΅ ΠΈ ΡƒΠ²ΠΈΠ΄ΠΈΠΌ, Ρ‡Ρ‚ΠΎ ΠΎΠ½ΠΎ соотвСтствуСт скорости ΠΈΠ·, опрСдСляСмой Ρ‚Π°ΠΊ Π½Π°Π·Ρ‹Π²Π°Π΅ΠΌΡ‹ΠΌ Π·Π°ΠΊΠΎΠ½ΠΎΠΌ Π­ΠΉΠ½ΡˆΡ‚Π΅ΠΉΠ½Π° для слоТСния скоростСй. Богласно этому Π·Π°ΠΊΠΎΠ½Ρƒ  [c.217]

ОписаниС мСханичСских свойств ΠΊΠΎΠΌΠΏΠΎΠ·ΠΈΡ‚Π½Ρ‹Ρ… ΠΌΠ°Ρ‚Π΅Ρ€ΠΈΠ°Π»ΠΎΠ², ΠΊΠΎΡ‚ΠΎΡ€Ρ‹Π΅ ΠΌΠΎΠ³ΡƒΡ‚ ΠΎΠ±Π»Π°Π΄Π°Ρ‚ΡŒ вСсьма высокой ΠΏΡ€ΠΎΡ‡Π½ΠΎΡΡ‚ΡŒΡŽ (особСнно статичСской ΠΈ ΡƒΠ΄Π°Ρ€Π½ΠΎΠΉ), ΠΌΠΎΠΆΠ½ΠΎ ΠΏΡ€ΠΎΠΈΠ·Π²ΠΎΠ΄ΠΈΡ‚ΡŒ двумя путями. Π’ ΠΏΠ΅Ρ€Π²ΠΎΠΌ случаС ΠΊΠΎΠΌΠΏΠΎΠ·ΠΈΡ‚Π½Ρ‹Π΅ ΠΌΠ°Ρ‚Π΅Ρ€ΠΈΠ°Π»Ρ‹ Ρ€Π°ΡΡΠΌΠ°Ρ‚Ρ€ΠΈΠ²Π°ΡŽΡ‚ΡΡ ΠΊΠ°ΠΊ ΠΊΠ²Π°Π·ΠΈΠΎΠ΄Π½ΠΎ-Ρ€ΠΎΠ΄Π½Ρ‹Π΅ (Π³ΠΎΠΌΠΎΠ³Π΅Π½Π½Ρ‹Π΅), ΠΎΠ±Π»Π°Π΄Π°ΡŽΡ‰ΠΈΠ΅ Π² случаС объСмного диспСрсного армирования ΠΈΠ·ΠΎΡ‚Ρ€ΠΎΠΏΠΈΠ΅ΠΉ Π΄Π΅Ρ„ΠΎΡ€ΠΌΠ°Ρ†ΠΈΠΎΠ½Π½Ρ‹Ρ… ΠΈ прочностных свойств, Π° Π² случаС армирования Π²ΠΎΠ»ΠΎΠΊΠ½Π°ΠΌΠΈ, плоскими сСтками ΠΈΠ»ΠΈ тканями β€” ΠΎΠΏΡ€Π΅Π΄Π΅Π»Π΅Π½Π½ΠΎΠ³ΠΎ Ρ‚ΠΈΠΏΠ° Π°Π½ΠΈΠ·ΠΎΡ‚Ρ€ΠΎΠΏΠΈΠ΅ΠΉ. ΠžΠ±Ρ‹Ρ‡Π½ΠΎ ΠΏΡ€ΠΈΠΌΠ΅Π½ΡΡŽΡ‚ ΠΌΠΎΠ΄Π΅Π»ΠΈ ΠΎΡ€Ρ‚ΠΎΡ‚Ρ€ΠΎΠΏΠ½ΠΎΠ³ΠΎ ΠΈΠ»ΠΈ Ρ‚Ρ€Π°Π½ΡΠ²Π΅Ρ€ΡΠ°Π»ΡŒΠ½ΠΎ-ΠΈΠ·ΠΎΡ‚Ρ€ΠΎΠΏΠ½ΠΎΠ³ΠΎ Ρ‚Π΅Π»Π°. ΠŸΡ€ΠΈ Ρ‚Π°ΠΊΠΎΠΌ ΠΏΠΎΠ΄Ρ…ΠΎΠ΄Π΅ Ρ€Π΅Ρ‡ΡŒ ΠΈΠ΄Π΅Ρ‚ ΠΎ мСханичСских характСристиках, осрСд-Π½Π΅Π½Π½Ρ‹Ρ… Π² достаточно Π±ΠΎΠ»ΡŒΡˆΠΈΡ… ΠΎΠ±ΡŠΠ΅ΠΌΠ°Ρ…, содСрТащих ΠΌΠ½ΠΎΠ³ΠΎ ΠΎΠ΄Π½ΠΎΡ‚ΠΈΠΏΠ½Ρ‹Ρ… Π°Ρ€ΠΌΠΈΡ€ΡƒΡŽΡ‰ΠΈΡ… элСмСнтов. Π”Ρ€ΡƒΠ³ΠΎΠΉ, нСсравнСнно Π±ΠΎΠ»Π΅Π΅ слоТный, Π½ΠΎ ΠΈ Π±ΠΎΠ»Π΅Π΅ ΠΈΠ½Ρ„ΠΎΡ€ΠΌΠ°Ρ‚ΠΈΠ²Π½Ρ‹ΠΉ ΠΏΡƒΡ‚ΡŒ состоит Π² Ρ€Π°Π·Π΄Π΅Π»ΡŒΠ½ΠΎΠΌ рассмотрСнии мСханичСских свойств ΠΊΠ°ΠΆΠ΄ΠΎΠΉ Ρ„Π°Π·Ρ‹ с ΠΏΠΎΡΠ»Π΅Π΄ΡƒΡŽΡ‰ΠΈΠΌ тСорСтичСским ΠΏΡ€ΠΎΠ³Π½ΠΎΠ·ΠΈΡ€ΠΎΠ²Π°Π½ΠΈΠ΅ΠΌ свойств всСго ΠΊΠΎΠΌΠΏΠΎΠ·ΠΈΡ‚Π° Π² Ρ†Π΅Π»ΠΎΠΌ. ΠŸΡ€ΠΈ этом приходится Ρ€Π°ΡΡΠΌΠ°Ρ‚Ρ€ΠΈΠ²Π°Ρ‚ΡŒ фактичСски Π΅Ρ‰Π΅ ΠΎΠ΄Π½Ρƒ Π΄ΠΎΠΏΠΎΠ»Π½ΠΈΡ‚Π΅Π»ΡŒΠ½ΡƒΡŽ Ρ„Π°Π·Ρƒ Π·ΠΎΠ½Ρ‹ сопряТСния основных Ρ„Π°Π·, Π½Π°ΠΏΡ€ΠΈΠΌΠ΅Ρ€, ΠΌΠ°Ρ‚Ρ€ΠΈΡ†Ρ‹ с Π°Ρ€ΠΌΠΈΡ€ΡƒΡŽΡ‰ΠΈΠΌΠΈ Π²ΠΎΠ»ΠΎΠΊΠ½Π°ΠΌΠΈ. ΠœΠ΅Ρ…Π°Π½ΠΈΠ·ΠΌ ΠΏΠΎΠ²Ρ€Π΅ΠΆΠ΄Π΅Π½ΠΈΠΉ, Ρ€Π°Π·Π²ΠΈΠ²Π°ΡŽΡ‰ΠΈΡ…ΡΡ Π½Π° Π³Ρ€Π°Π½ΠΈΡ†Π°Ρ… Ρ„Π°Π·, ΠΎΠ±Ρ‹Ρ‡Π½ΠΎ вСсьма слоТСн ΠΈ опрСдСляСтся ΠΏΠΎΠΌΠΈΠΌΠΎ свойств основных ΠΊΠΎΠΌΠΏΠΎΠ½Π΅Π½Ρ‚ΠΎΠ² Π³Π΅Ρ‚Π΅Ρ€ΠΎΠ³Π΅Π½Π½ΠΎΠΉ систСмы Π΅Ρ‰Π΅ рядом Π΄ΠΎΠΏΠΎΠ»Π½ΠΈΡ‚Π΅Π»ΡŒΠ½Ρ‹Ρ… Ρ„Π°ΠΊΡ‚ΠΎΡ€ΠΎΠ², Ρ‚Π°ΠΊΠΈΡ… ΠΊΠ°ΠΊ адгСзия Ρ„Π°Π·, тСхнологичСскиС ΠΈ Ρ‚Π΅ΠΌΠΏΠ΅Ρ€Π°Ρ‚ΡƒΡ€Π½Ρ‹Π΅ мСстныС напряТСния, ΠΎΠ±Ρ‹Ρ‡Π½ΠΎ Π²ΠΎΠ·Π½ΠΈΠΊΠ°ΡŽΡ‰ΠΈΠ΅ Π²Π±Π»ΠΈΠ·ΠΈ Π³Ρ€Π°Π½ΠΈΡ†, Π½Π°Π»ΠΈΡ‡ΠΈΠ΅ Π΄Π΅Ρ„Π΅ΠΊΡ‚ΠΎΠ² ΠΈ Π΄Ρ€. Π“Ρ€Π°Π½ΠΈΡ†Ρ‹ Ρ„Π°Π· ΠΊΠ°ΠΊ Π·ΠΎΠ½Ρ‹ ΠΊΠΎΠ½Ρ†Π΅Π½Ρ‚Ρ€Π°Ρ†ΠΈΠΉ напряТСний ΠΈΠ³Ρ€Π°ΡŽΡ‚ особСнно Π²Π°ΠΆΠ½ΡƒΡŽ Ρ€ΠΎΠ»ΡŒ Π² Ρ€Π°Π·Π²ΠΈΡ‚ΠΈΠΈ ΠΌΠ½ΠΎΠ³ΠΎ- ΠΈ ΠΌΠ°Π»ΠΎΡ†ΠΈΠΊΠ»ΠΎΠ²Ρ‹Ρ… усталостных ΠΏΠΎΠ²Ρ€Π΅ΠΆΠ΄Π΅Π½ΠΈΠΉ ΠΊΠΎΠΌΠΏΠΎΠ·ΠΈΡ‚ΠΎΠ².  [c.37]

ΠŸΡ€ΠΈ нСобходимости ΠΏΠΎΠ΄Ρ‡Π΅Ρ€ΠΊΠ½ΡƒΡ‚ΡŒ состав мноТСств D я Z Π΄Π°Π»Π΅Π΅ Π±ΡƒΠ΄ΡƒΡ‚ ΠΈΡΠΏΠΎΠ»ΡŒΠ·ΠΎΠ²Π°Ρ‚ΡŒΡΡ ΠΈ Ρ‚Π°ΠΊΠΈΠ΅ обозначСния a D, Z] = a[, 2,…,d Z] = a D , 2,…,Z]== = a d I, 2, z] ΠΈ T. П. Π‘ ΠΈΡ… ΠΏΠΎΠΌΠΎΡ‰ΡŒΡŽ ΠΌΠΎΠΆΠ½ΠΎ ΡƒΠ΄ΠΎΠ±Π½ΠΎ Π·Π°ΠΏΠΈΡΡ‹Π²Π°Ρ‚ΡŒ ΠΎΡ‚Π΄Π΅Π»ΡŒΠ½Ρ‹Π΅ составныС части (ΠΏΠΎΠ΄ΠΌΠ°Ρ‚Ρ€ΠΈΡ†Ρ‹) ΠΌΠ°Ρ‚Ρ€ΠΈΡ†Ρ‹ ΠΈΠ»ΠΈ Π½ΠΎΠ²Ρ‹Π΅ ΠΌΠ°Ρ‚Ρ€ΠΈΡ†Ρ‹, ΠΎΠ±Ρ€Π°Π·ΠΎΠ²Π°Π½Π½Ρ‹Π΅ Π½Π° основС a[D, Z], НапримСр, Π° , 2, kβ€”l, k+l, d z] = a[D/ k), Z]β€”ΠΌΠ°Ρ‚Ρ€ΠΈΡ†Π°, получСнная ΠΈΠ· a[D,Z] Π² Ρ€Π΅Π·ΡƒΠ»ΡŒΡ‚Π°Ρ‚Π΅ вычСркивания ΠΉ-ΠΉ строки, Π° d D β€” k, Z ], Π³Π΄Π΅ Z β€” мноТСство Ρ‡Π΅Ρ‚Π½Ρ‹Ρ… чисСл (Z zZ)β€”ΠΌΠ°Ρ‚Ρ€ΠΈΡ†Π°, Π² ΠΊΠΎΡ‚ΠΎΡ€ΠΎΠΉ ΠΊ Ρ‚ΠΎΠΌΡƒ ΠΆΠ΅ Π΅Ρ‰Π΅ Π²Ρ‹Ρ‡Π΅Ρ€ΠΊΠ½ΡƒΡ‚Ρ‹ всС столбцы с Π½Π΅Ρ‡Π΅Ρ‚Π½Ρ‹ΠΌΠΈ Π½ΠΎΠΌΠ΅Ρ€Π°ΠΌΠΈ. Π’ частности, a[k, Z] β€” k-я строка ΠΌΠ°Ρ‚Ρ€ΠΈΡ†Ρ‹ a[D, Z], Π° a[D, /] β€” Π΅Π΅ /-Π™ столбСц. Π—Π°ΠΏΠΈΡΡŒ a[D 1, 2 + 3 + 5, 4] ΠΎΠ±ΠΎΠ·Π½Π°Ρ‡Π°Π΅Ρ‚ ΠΌΠ°Ρ‚Ρ€ΠΈΡ†Ρƒ, Π² ΠΊΠΎΡ‚ΠΎΡ€ΠΎΠΉ Π²Ρ‚ΠΎΡ€ΠΎΠΉ, Ρ‚Ρ€Π΅Ρ‚ΠΈΠΉ ΠΈ пятый столбцы слоТСны поэлСмСнтно ΠΈ ΠΏΠΎΠ»ΡƒΡ‡Π΅Π½Π½Ρ‹ΠΉ суммарный столбСц ΠΏΠΎΠΌΠ΅Ρ‰Π΅Π½ Π½Π° Π²Ρ‚ΠΎΡ€ΠΎΠ΅ мСсто. ΠŸΡƒΡΡ‚ΡŒ Zi сг Z ΠΈ Z2 с= Zβ€” подмноТСства мноТСства Z, ΠΏΡ€ΠΈΡ‡Π΅ΠΌ Z, 0 2=0 Z/”, Z β€” обозначСния столбцов, ΠΏΠΎΠ»ΡƒΡ‡Π΅Π½Π½Ρ‹Ρ… ΠΊΠ°ΠΊ Ρ€Π΅Π·ΡƒΠ»ΡŒΡ‚Π°Ρ‚ слоТСния столбцов с Π½ΠΎΠΌΠ΅Ρ€Π°ΠΌΠΈ ΠΈΠ· мноТСств Zi ΠΈ Z2. Π’ΠΎΠ³Π΄Π° ΠΌΠ°Ρ‚Ρ€ΠΈΡ†Π° Π°[0] Zt, Z β€”(Z1UZ2), Zt  [c.88]

Π’Π°ΠΊΠΈΠΌ ΠΎΠ±Ρ€Π°Π·ΠΎΠΌ, Π½Π° всСх стадиях опрСдСлСния скоростСй ΠΈ ΠΌΠΎΠΌΠ΅Π½Ρ‚ΠΎΠ² ΠΈΡΠΏΠΎΠ»ΡŒΠ·ΡƒΠ΅Ρ‚ΡΡ ΠΎΠ΄ΠΈΠ½ ΠΈ Ρ‚ΠΎΡ‚ ΠΆΠ΅ Π°Π»Π³ΠΎΡ€ΠΈΡ‚ΠΌ, ΠΏΠΎΠ·Π²ΠΎΠ»ΡΡŽΡ‰ΠΈΠΉ Π»Π΅Π³ΠΊΠΎ Π°Π²Ρ‚ΠΎΠΌΠ°Ρ‚ΠΈΠ·ΠΈΡ€ΠΎΠ²Π°Ρ‚ΡŒ вСсь процСсс вычислСния. Π•Π³ΠΎ основной нСдостаток состоит Π² Ρ‚ΠΎΠΌ, ΠΊΠ°ΠΊ ΡƒΠΆΠ΅ ΠΎΡ‚ΠΌΠ΅Ρ‡Π°Π»ΠΎΡΡŒ Π²Ρ‹ΡˆΠ΅, Ρ‡Ρ‚ΠΎ ΠΎΠ½ ΠΏΡ€ΠΎΠΈΠ·Π²ΠΎΠ΄ΠΈΡ‚ ΠΌΠ½ΠΎΠ³ΠΎ Π»ΠΈΡˆΠ½ΠΈΡ… дСйствий, связанных с ΡƒΠΌΠ½ΠΎΠΆΠ΅Π½ΠΈΠ΅ΠΌ ΠΈ слоТСниСм Π½ΡƒΠ»Π΅ΠΉ ΠΏΡ€ΠΈ вычислСнии ΠΎΠΏΡ€Π΅Π΄Π΅Π»ΠΈΡ‚Π΅Π»Π΅ΠΉ Ρ€Π΅Π΄-ΠΊΠΎΠ·Π°ΠΏΠΎΠ»Π½Π΅Π½Π½Ρ‹Ρ… ΠΌΠ°Ρ‚Ρ€ΠΈΡ†. ΠŸΡ€ΠΈΠΌΠ΅Π½Π΅Π½ΠΈΠ΅ Π½Π°ΠΏΡ€Π°Π²Π»Π΅Π½Π½Ρ‹Ρ… Π³Ρ€Π°Ρ„ΠΎΠ² ΠΈ ΡΠΎΠΎΡ‚Π²Π΅Ρ‚ΡΡ‚Π²ΡƒΡŽΡ‰Π΅Π³ΠΎ матСматичСского Π°ΠΏΠΏΠ°Ρ€Π°Ρ‚Π° [2, 21] Π΄Π°Π΅Ρ‚ Π²ΠΎΠ·ΠΌΠΎΠΆΠ½ΠΎΡΡ‚ΡŒ ΠΈΠ·Π±Π°Π²ΠΈΡ‚ΡŒΡΡ ΠΎΡ‚ этого нСдостатка ΠΈ Ρ‚Π΅ΠΌ са.ΠΌΡ‹ΠΌ Π·Π½Π°Ρ‡ΠΈΡ‚Π΅Π»ΡŒΠ½ΠΎ ΡΠΎΠΊΡ€Π°Ρ‚ΠΈΡ‚ΡŒ машинноС врСмя Ρ€Π΅ΡˆΠ΅Π½ΠΈΡ Π·Π°Π΄Π°Ρ‡ΠΈ.  [c.98]


ΠŸΠΎΡ€ΡΠ΄ΠΎΠΊ ΠΌΠ°Ρ‚Ρ€ΠΈΡ† ΠΏΡ€ΠΈ транспонировании Π² ΠΎΠ±Ρ‰Π΅ΠΌ случаС измСняСтся. Π‘Π»ΠΎΠΆΠ΅Π½ΠΈΠ΅ ΠΈ Π²Ρ‹Ρ‡ΠΈΡ‚Π°Π½ΠΈΠ΅ ΠΌΠ°Ρ‚Ρ€ΠΈΠΈ  [c.18]

К ΠΌΠ°Ρ‚Ρ€ΠΈΡ†Π°ΠΌ ΠΎΠ΄Π½ΠΎΠ³ΠΎ ΠΈ Ρ‚ΠΎΠ³ΠΎ ΠΆΠ΅ порядка ΠΏΡ€ΠΈΠΌΠ΅Π½ΠΈΠΌΡ‹ ΠΎΠΏΠ΅Ρ€Π°Ρ†ΠΈΠΈ слоТСния ΠΈ вычитания. Π Π΅Π·ΡƒΠ»ΡŒΡ‚Π°Ρ‚ΠΎΠΌ слоТСния (вычитания) Π΄Π²ΡƒΡ… ΠΌΠ°Ρ‚Ρ€ΠΈΠΈ А ΠΈ Π’ порядка n m являСтся новая ΠΌΠ°Ρ‚Ρ€ΠΈΡ†Π° Π‘ Ρ‚ΠΎΠ³ΠΎ ΠΆΠ΅ порядка, ΠΊΠ°ΠΆΠ΄Ρ‹ΠΉ элСмСнт ΠΊΠΎΡ‚ΠΎΡ€ΠΎΠΉ jj = ajj bjj, Π³Π΄Π΅ i – Π½ΠΎΠΌΠ΅Ρ€ строки, j – Π½ΠΎΠΌΠ΅Ρ€ столбца. ΠŸΡƒΡΡ‚ΡŒ,  [c.18]

Число критичСских сил ΠΏΠΎ МКЭ Ρ€Π°Π²Π½ΠΎ стСпСни кинСматичСской нСопрСдСлимости стСрТнСвой систСмы, Π° ΠΏΡ€ΠΈ Ρ„ΠΎΡ€ΠΌΠΈΡ€ΠΎΠ²Π°Π½ΠΈΠΈ Π²Π΅ΠΊΠΎΠ²ΠΎΠ³ΠΎ уравнСния ΠΈΡΠΏΠΎΠ»ΡŒΠ·ΡƒΡŽΡ‚ΡΡ ΠΎΠΏΠ΅Ρ€Π°Ρ†ΠΈΠΈ слоТСния, умноТСния ΠΈ траспонирования ΠΌΠ°Ρ‚Ρ€ΠΈΡ†.  [c.179]


Π‘Π»ΠΎΠΆΠ΅Π½ΠΈΠ΅ ΠΈ Π²Ρ‹Ρ‡ΠΈΡ‚Π°Π½ΠΈΠ΅ ΠΌΠ°Ρ‚Ρ€ΠΈΡ†, ΠΎΠ½Π»Π°ΠΉΠ½ ΠΊΠ°Π»ΡŒΠΊΡƒΠ»ΡΡ‚ΠΎΡ€

Наш ΠΎΠ½Π»Π°ΠΉΠ½ ΠΊΠ°Π»ΡŒΠΊΡƒΠ»ΡΡ‚ΠΎΡ€ позволяСт Π½Π°ΠΉΡ‚ΠΈ сумму ΠΈ Ρ€Π°Π·Π½ΠΎΡΡ‚ΡŒ Π΄Π²ΡƒΡ… ΠΌΠ°Ρ‚Ρ€ΠΈΡ† всСго Π² ΠΏΠ°Ρ€Ρƒ ΠΊΠ»ΠΈΠΊΠΎΠ². Для Ρ‚ΠΎΠ³ΠΎ Ρ‡Ρ‚ΠΎΠ±Ρ‹ ΡΠ»ΠΎΠΆΠΈΡ‚ΡŒ ΠΈΠ»ΠΈ Π²Ρ‹Ρ‡Π΅ΡΡ‚ΡŒ Π΄Π²Π΅ ΠΌΠ°Ρ‚Ρ€ΠΈΡ†Ρ‹ Π²Ρ‹Π±Π΅Ρ€ΠΈΡ‚Π΅ Ρ€Π°Π·ΠΌΠ΅Ρ€ ΠΌΠ°Ρ‚Ρ€ΠΈΡ†, Π²Π²Π΅Π΄ΠΈΡ‚Π΅ Π½Π΅ΠΎΠ±Ρ…ΠΎΠ΄ΠΈΠΌΡ‹Π΅ значСния Π² поля Π²Π²ΠΎΠ΄Π° ΠΈ Π½Π°ΠΆΠΌΠΈΡ‚Π΅ ΠΊΠ½ΠΎΠΏΠΊΡƒ Β«Π’Ρ‹Ρ‡ΠΈΡΠ»ΠΈΡ‚ΡŒΒ», ΠΊΠ°Π»ΡŒΠΊΡƒΠ»ΡΡ‚ΠΎΡ€ выдаст ΠΎΡ‚Π²Π΅Ρ‚ ΠΈ ΠΏΠΎΠ΄Ρ€ΠΎΠ±Π½ΠΎΠ΅ Ρ€Π΅ΡˆΠ΅Π½ΠΈΠ΅! ΠšΠ°ΠΆΠ΄Ρ‹ΠΉ шаг Π±ΡƒΠ΄Π΅Ρ‚ Π΄Π΅Ρ‚Π°Π»ΡŒΠ½ΠΎ расписан, это ΠΏΠΎΠΌΠΎΠΆΠ΅Ρ‚ Π²Π°ΠΌ ΠΏΠΎΠ½ΡΡ‚ΡŒ, ΠΊΠ°ΠΊ Π±Ρ‹Π» ΠΏΠΎΠ»ΡƒΡ‡Π΅Π½ ΠΎΡ‚Π²Π΅Ρ‚.

Π—Π°ΠΏΠΎΠ»Π½ΠΈΡ‚Π΅ элСмСнты ΠΌΠ°Ρ‚Ρ€ΠΈΡ† Β 

ΠŸΠ΅Ρ€Π²Π°Ρ ΠΌΠ°Ρ‚Ρ€ΠΈΡ†Π°:

Вторая ΠΌΠ°Ρ‚Ρ€ΠΈΡ†Π°:

РСшили сСгодня: Ρ€Π°Π·, всСго Ρ€Π°Π·
ΠŸΠΎΠ½Ρ€Π°Π²ΠΈΠ»ΡΡ сайт? РасскаТи Π΄Ρ€ΡƒΠ·ΡŒΡΠΌ!

Как Π½Π°ΠΉΡ‚ΠΈ сумму ΠΈ Ρ€Π°Π·Π½ΠΎΡΡ‚ΡŒ ΠΌΠ°Ρ‚Ρ€ΠΈΡ† ΠΎΠ½Π»Π°ΠΉΠ½

Π‘Π»Π΅Π΄ΡƒΠ΅Ρ‚ Π·Π°ΠΌΠ΅Ρ‚ΠΈΡ‚ΡŒ, Ρ‡Ρ‚ΠΎ Π΄Π°Π½Π½ΠΎΠΉ ΠΎΠΏΠ΅Ρ€Π°Ρ†ΠΈΠΈ ΠΏΠΎΠ΄Π΄Π°ΡŽΡ‚ΡΡ Ρ‚ΠΎΠ»ΡŒΠΊΠΎ ΠΌΠ°Ρ‚Ρ€ΠΈΡ†Ρ‹ ΠΎΠ΄ΠΈΠ½Π°ΠΊΠΎΠ²ΠΎΠ³ΠΎ Ρ€Π°Π·ΠΌΠ΅Ρ€Π°. ΠŸΡ€ΠΈ слоТСнии Π΄Π²ΡƒΡ… ΠΌΠ°Ρ‚Ρ€ΠΈΡ† ΠΏΠΎΠΏΠ°Ρ€Π½ΠΎ ΡΡƒΠΌΠΌΠΈΡ€ΡƒΡŽΡ‚ΡΡ всС ΠΈΡ… элСмСнты, Π° ΠΏΡ€ΠΈ Π²Ρ‹Ρ‡ΠΈΡ‚Π°Π½ΠΈΠΈ ΠΌΡ‹, соотвСтствСнно, ΠΈΠΌΠ΅Π΅ΠΌ Π΄Π΅Π»ΠΎ с ΠΈΡ… ΠΏΠΎΠΏΠ°Ρ€Π½ΠΎΠΉ Ρ€Π°Π·Π½ΠΎΡΡ‚ΡŒΡŽ. ΠŸΠΎΠ»ΡƒΡ‡ΠΈΠ² Π΄Π΅Ρ‚Π°Π»ΡŒΠ½ΠΎΠ΅ ΠΈ пошаговоС Ρ€Π΅ΡˆΠ΅Π½ΠΈΠ΅, Π²Ρ‹ смоТСтС Π»ΡƒΡ‡ΡˆΠ΅ Ρ€Π°Π·ΠΎΠ±Ρ€Π°Ρ‚ΡŒΡΡ с процСссом нахоТдСния суммы ΠΈ разности ΠΌΠ°Ρ‚Ρ€ΠΈΡ†.

Π˜Ρ‚Π°ΠΊ, ΠΏΠ΅Ρ€Π΅Π΄ Π²Π°ΠΌΠΈ Π΄Π²Π΅ ΠΌΠ°Ρ‚Ρ€ΠΈΡ†Ρ‹, ΠΈ Π²Π°ΠΌ Π½Π΅ΠΎΠ±Ρ…ΠΎΠ΄ΠΈΠΌΠΎ ΡƒΠ·Π½Π°Ρ‚ΡŒ ΠΈΡ… сумму, Π»ΠΈΠ±ΠΎ ΠΆΠ΅ Ρ€Π°Π·Π½ΠΎΡΡ‚ΡŒ. И Ρ‚ΠΎ, ΠΈ Π΄Ρ€ΡƒΠ³ΠΎΠ΅ Π²Ρ‹ смоТСтС Π»Π΅Π³ΠΊΠΎ ΠΈ ΠΎΠΏΠ΅Ρ€Π°Ρ‚ΠΈΠ²Π½ΠΎ ΡΠ΄Π΅Π»Π°Ρ‚ΡŒ, Ссли Π²ΠΎΡΠΏΠΎΠ»ΡŒΠ·ΡƒΠ΅Ρ‚Π΅ΡΡŒ нашим ΠΎΠ½Π»Π°ΠΉΠ½ ΠΊΠ°Π»ΡŒΠΊΡƒΠ»ΡΡ‚ΠΎΡ€ΠΎΠΌ. Он Π±ΡƒΠ΄Π΅Ρ‚ ΠΎΡ‡Π΅Π½ΡŒ Π²Π°ΠΌ ΠΏΠΎΠ»Π΅Π·Π΅Π½, Ссли Π²Ρ‹ ΠΆΠ΅Π»Π°Π΅Ρ‚Π΅ Ρ€Π°Π·ΠΎΠ±Ρ€Π°Ρ‚ΡŒΡΡ Π² Π°Π»Π³ΠΎΡ€ΠΈΡ‚ΠΌΠ΅ Π΄Π°Π½Π½Ρ‹Ρ… ΠΎΠΏΠ΅Ρ€Π°Ρ†ΠΈΠΉ. ВСория Π½Π΅ всСгда способна Π΄Π°Ρ‚ΡŒ Ρ‡Ρ‘Ρ‚ΠΊΠΈΠΉ ΠΎΡ‚Π²Π΅Ρ‚ Π½Π° всС вопросы, ΠΊΡƒΠ΄Π° Π»ΡƒΡ‡ΡˆΠ΅ с этой Π·Π°Π΄Π°Ρ‡Π΅ΠΉ ΡΠΏΡ€Π°Π²Π»ΡΡŽΡ‚ΡΡ практичСскиС расчёты. Π˜ΡΠΏΠΎΠ»ΡŒΠ·ΡƒΡ ΠΎΠ½Π»Π°ΠΉΠ½ ΠΊΠ°Π»ΡŒΠΊΡƒΠ»ΡΡ‚ΠΎΡ€, Π²Ρ‹ ΠΏΠΎΠ»ΡƒΡ‡ΠΈΡ‚Π΅ ΠΏΠΎΠ΄Ρ€ΠΎΠ±Π½ΡƒΡŽ схСму, ΠΏΠΎ ΠΊΠΎΡ‚ΠΎΡ€ΠΎΠΉ происходит Π²Ρ‹Ρ‡ΠΈΡ‚Π°Π½ΠΈΠ΅ ΠΈΠ»ΠΈ слоТСниС ΠΌΠ°Ρ‚Ρ€ΠΈΡ†. К Ρ‚ΠΎΠΌΡƒ ΠΆΠ΅, Π²Ρ‹ ΠΌΠΎΠΆΠ΅Ρ‚Π΅ сначала ΠΏΠΎΠΏΡ€ΠΎΠ±ΠΎΠ²Π°Ρ‚ΡŒ ΠΏΡ€ΠΎΡΡ‡ΠΈΡ‚Π°Ρ‚ΡŒ всё ΡΠ°ΠΌΠΎΡΡ‚ΠΎΡΡ‚Π΅Π»ΡŒΠ½ΠΎ, Π° Π·Π°Ρ‚Π΅ΠΌ ΠΏΠ΅Ρ€Π΅ΠΏΡ€ΠΎΠ²Π΅Ρ€ΠΈΡ‚ΡŒ сСбя здСсь.

Π”Π°Π½Π½Ρ‹ΠΉ ΠΎΠ½Π»Π°ΠΉΠ½ ΠΊΠ°Π»ΡŒΠΊΡƒΠ»ΡΡ‚ΠΎΡ€ ΠΈΠΌΠ΅Π΅Ρ‚ ΠΏΡ€Π΅Π΄Π΅Π»ΡŒΠ½ΠΎ ΠΏΡ€ΠΎΡΡ‚ΡƒΡŽ ΠΈΠ½ΡΡ‚Ρ€ΡƒΠΊΡ†ΠΈΡŽ. Π£ΠΊΠ°Π·Π°Ρ‚ΡŒ Ρ€Π°Π·ΠΌΠ΅Ρ€Ρ‹ ΠΊΠ°ΠΆΠ΄ΠΎΠΉ ΠΈΠ· ΠΌΠ°Ρ‚Ρ€ΠΈΡ† Π²Ρ‹ смоТСтС, наТимая Π½Π° ΠΈΠΊΠΎΠ½ΠΊΠΈ Β«+Β» ΠΈΠ»ΠΈ Β«-Β» слСва ΠΎΡ‚ ΠΌΠ°Ρ‚Ρ€ΠΈΡ† ΠΈ ΠΏΠΎΠ΄ Π½ΠΈΠΌΠΈ. Π”Π°Π»Π΅Π΅ Π²Π°ΠΌ потрСбуСтся ввСсти всС элСмСнты. И Π·Π°Ρ‚Π΅ΠΌ, Π½Π°ΠΆΠ°Π² ΠΊΠ½ΠΎΠΏΠΊΡƒ Β«Π’Ρ‹Ρ‡ΠΈΡΠ»ΠΈΡ‚ΡŒΒ», Π²Ρ‹ смоТСтС быстро ΠΏΠΎΠ»ΡƒΡ‡ΠΈΡ‚ΡŒ искомоС Π·Π½Π°Ρ‡Π΅Π½ΠΈΠ΅ вмСстС с Ρ€Π°Π·Π²Ρ‘Ρ€Π½ΡƒΡ‚Ρ‹ΠΌ Π°Π»Π³ΠΎΡ€ΠΈΡ‚ΠΌΠΎΠΌ вычислСний.

ΠœΠ°Ρ‚Ρ€ΠΈΡ†Ρ‹ слоТСниС ΠΈ Π²Ρ‹Ρ‡ΠΈΡ‚Π°Π½ΠΈΠ΅ – Π‘ΠΏΡ€Π°Π²ΠΎΡ‡Π½ΠΈΠΊ Ρ…ΠΈΠΌΠΈΠΊΠ° 21

    БлоТСниС (Π²Ρ‹Ρ‡ΠΈΡ‚Π°Π½ΠΈΠ΅) ΠΌΠ°Ρ‚Ρ€ΠΈΡ†. Π”Π²Π΅ ΠΌΠ°Ρ‚Ρ€ΠΈΡ†Ρ‹ А ΠΈ И ΠΌΠΎΠΆΠ½ΠΎ ΡΠ»ΠΎΠΆΠΈΡ‚ΡŒ, Ссли ΠΎΠΏΠΈ ΠΈΠΌΠ΅ΡŽΡ‚ ΠΎΠ΄ΠΈΠ½Π°ΠΊΠΎΠ²ΠΎΠ΅ число строк ΠΈ столбцов. Π‘ΡƒΠΌΠΌΠΎΠΉ ΠΌΠ°Ρ‚Ρ€ΠΈΡ† А ΠΈ Π’ размСрности Ρ‚ X ΠΏ являСтся ΠΌΠ°Ρ‚Ρ€ΠΈΡ†Π° Π‘ Ρ‚ΠΎΠΉ ΠΆΠ΅ размСрности, ΠΊΠ°ΠΆΠ΄Ρ‹ΠΉ элСмСнт ΠΊΠΎΡ‚ΠΎΡ€ΠΎΠΉ Ρ€Π°Π²Π΅Π½ суммС ΡΠΎΠΎΡ‚Π²Π΅Ρ‚ΡΡ‚Π²ΡƒΡŽΡ‰ΠΈΡ… элСмСнтов ΠΌΠ°Ρ‚Ρ€ΠΈΡ†Ρ‹ А VI Π’. [c.232]

    БлоТСниС (Π²Ρ‹Ρ‡ΠΈΡ‚Π°Π½ΠΈΠ΅) строк Π² ΠΌΠ°Ρ‚Ρ€ΠΈΡ†Π΅ А ΠΏΡ€ΠΈΠ²ΠΎΠ΄ΠΈΡ‚ ΠΊ Π½ΠΎΠ²ΠΎΠΉ Π»ΠΈΠ½Π΅ΠΉΠ½ΠΎΠΉ ΠΊΠΎΠΌΠ±ΠΈΠ½Π°Ρ†ΠΈΠΈ ΡƒΠ·Π»ΠΎΠ², Π° слоТСниС строквВ – ΠΊ Π½ΠΎΠ²ΠΎΠΉ систСмС ΠΊΠΎΠ½Ρ‚ΡƒΡ€ΠΎΠ². ΠŸΡ€ΠΈΠ±Π°Π²ΠΈΠΌ, ΠΊ ΠΏΡ€ΠΈΠΌΠ΅Ρ€Ρƒ, Π² ΠΌΠ°Ρ‚Ρ€ΠΈΡ†Π΅ соСдинСний (4.12) ΠΏΠ΅Ρ€Π²ΡƒΡŽ строку ΠΊΠΎ Π²Ρ‚ΠΎΡ€ΠΎΠΉ. ΠŸΠΎΠ»ΡƒΡ‡ΠΈΠΌ Π½ΠΎΠ²ΡƒΡŽ ΠΌΠ°Ρ‚Ρ€ΠΈΡ†Ρƒ ΠΈ схСму соСдинСний (рис. 4.4, Π°), Π² ΠΊΠΎΡ‚ΠΎΡ€ΠΎΠΉ ΡƒΠ·Π΅Π» 1 + 2 соотвСтствуСт области, ΠΎΠ³Ρ€Π°Π½ΠΈΡ‡Π΅Π½Π½ΠΎΠΉ ΠΏΡƒΠ½ΠΊΡ‚ΠΈΡ€Π½ΠΎΠΉ ΠΊΡ€ΠΈΠ²ΠΎΠΉ, Ρ‚.Π΅. [c.53]


β€‚β€‚β€‚β€‚ΠšΠ²Π°Π΄Ρ€Π°Ρ‚Π½Ρ‹Π΅ ΠΌΠ°Ρ‚Ρ€ΠΈΡ†Ρ‹ ΠΎΠ±Π»Π°Π΄Π°ΡŽΡ‚ ΡΠ»Π΅Π΄ΡƒΡŽΡ‰ΠΈΠΌ Π²Π°ΠΆΠ½Ρ‹ΠΌ свойством, Π²Ρ‹Ρ‚Π΅ΠΊΠ°ΡŽΡ‰ΠΈΠΌ нСпосрСдствСнно ΠΈΠ· Π·Π°ΠΊΠΎΠ½ΠΎΠ² ΠΈΡ… слоТСния ΠΈ вычитания любая квадратная ΠΌΠ°Ρ‚Ρ€ΠΈΡ†Π° Π› ΠΌΠΎΠΆΠ΅Ρ‚ Π±Ρ‹Ρ‚ΡŒ прСдставлСна Π² Π²ΠΈΠ΄Π΅ симмСтричной ΠΈ антисиммСтричной ΠΌΠ°Ρ‚Ρ€ΠΈΡ†. Π’Π°ΠΊ, Ссли [c.43]

    Аналогично ΠΊΠ°ΠΆΠ΄Ρ‹ΠΉ элСмСнт ΠΌΠ°Ρ‚Ρ€ΠΈΡ†Ρ‹ разности Π΅ΡΡ‚ΡŒ Ρ€Π°Π·Π½ΠΎΡΡ‚ΡŒ ΡΠΎΠΎΡ‚Π²Π΅Ρ‚ΡΡ‚Π²ΡƒΡŽΡ‰ΠΈΡ… элСмСнтов ΠΌΠ°Ρ‚Ρ€ΠΈΡ†Ρ‹-ΡƒΠΌΠ΅Π½ΡŒΡˆΠ°Π΅ΠΌΠΎΠ³ΠΎ ΠΈ ΠΌΠ°Ρ‚Ρ€ΠΈΡ†Ρ‹-Π²Ρ‹Ρ‡ΠΈΡ‚Π°Π΅ΠΌΠΎΠ³ΠΎ Π‘Ρ† = Ац β€” Bij. ΠŸΡ€ΠΎΡ†Π΅Π΄ΡƒΡ€Π° вычитания ΠΌΠ°Ρ‚Ρ€ΠΈΡ† Π°Π½Π°Π»ΠΎΠ³ΠΈΡ‡Π½Π° ΠΏΡ€ΠΎΡ†Π΅Π΄ΡƒΡ€Π΅ слоТСния. [c.233]

β€‚β€‚β€‚β€‚Π˜Π½Ρ‚Π΅Π»Π»Π΅ΠΊΡ‚ΡƒΠ°Π»ΡŒΠ½Ρ‹Π΅ систСмы аналитичСских ΠΏΡ€Π΅ΠΎΠ±Ρ€Π°Π·ΠΎΠ²Π°Π½ΠΈΠΉ (БАП). Π’ матСматичСском обСспСчСнии Π­Π’Πœ Π² послСдниС Π³ΠΎΠ΄Ρ‹ всС Ρ‡Π°Ρ‰Π΅ ΠΏΡ€ΠΈΡΡƒΡ‚ΡΡ‚Π²ΡƒΡŽΡ‚ систСмы аналитичСских ΠΏΡ€Π΅ΠΎΠ±Ρ€Π°Π·ΠΎΠ²Π°Π½ΠΈΠΉ (БАП). Они ΠΏΡ€Π΅Π΄Π½Π°Π·Π½Π°Ρ‡Π΅Π½Ρ‹ для облСгчСния программирования ΠΏ Ρ€Π΅ΡˆΠ΅Π½ΠΈΡ Π·Π°Π΄Π°Ρ‡, связанных с ΠΏΡ€Π΅ΠΎΠ±Ρ€Π°Π·ΠΎΠ²Π°Π½ΠΈΠ΅ΠΌ матСматичСских Π²Ρ‹Ρ€Π°ΠΆΠ΅Π½ΠΈΠΉ. АвтоматизированноС Π²Ρ‹ΠΏΠΎΠ»Π½Π΅Π½ΠΈΠ΅ аналитичСских ΠΏΡ€Π΅ΠΎΠ±Ρ€Π°Π·ΠΎΠ²Π°Π½ΠΈΠΉ ΠΏΡ€ΠΈ ΠΏΠΎΠΌΠΎΡ‰ΠΈ Π­Π’Πœ стало Π²ΠΎΠ·ΠΌΠΎΠΆΠ½Ρ‹ΠΌ благодаря Ρ€Π°Π·Π²ΠΈΡ‚ΠΈΡŽ ΠΌΠ΅Ρ‚ΠΎΠ΄ΠΎΠ² ΠΎΠ±Ρ€Π°Π±ΠΎΡ‚ΠΊΠΈ символьной ΠΈΠ½Ρ„ΠΎΡ€ΠΌΠ°Ρ†ΠΈΠΈ ΠΈ искусствСнного ΠΈΠ½Ρ‚Π΅Π»Π»Π΅ΠΊΡ‚Π° ΡΠΎΠΎΡ‚Π²Π΅Ρ‚ΡΡ‚Π²ΡƒΡŽΡ‰ΠΈΡ… языков программирования ΠΌΠ΅Ρ‚ΠΎΠ΄ΠΎΠ² трансляции ΠΈ ΠΎΡ€Π³Π°Π½ΠΈΠ·Π°Ρ†ΠΈΠΈ памяти Ρ€Π°Π·Ρ€Π°Π±ΠΎΡ‚ΠΊΠ΅ вычислСнных Π°Π»Π³ΠΎΡ€ΠΈΡ‚ΠΌΠΎΠ² [62] ΠΈ Ρ‚. ΠΏ. Под аналитичСским ΠΏΡ€Π΅ΠΎΠ±Ρ€Π°Π·ΠΎΠ²Π°Π½ΠΈΠ΅ΠΌ ΠΏΠΎΠ½ΠΈΠΌΠ°Π΅ΠΌ Ρ„ΠΎΡ€ΠΌΠ°Π»ΡŒΠ½ΠΎΠ΅ ΠΏΡ€Π΅ΠΎΠ±Ρ€Π°Π·ΠΎΠ²Π°Π½ΠΈΠ΅ матСматичСского выраТСния, Π·Π°Π΄Π°Π½Π½ΠΎΠ³ΠΎ Π² символьном Π²ΠΈΠ΄Π΅, ΠΏΠΎ ΠΎΠΏΡ€Π΅Π΄Π΅Π»Π΅Π½Π½Ρ‹ΠΌ ΠΏΡ€Π°Π²ΠΈΠ»Π°ΠΌ. НаиболСС часто Π²ΡΡ‚Ρ€Π΅Ρ‡Π°ΡŽΡ‰ΠΈΠΌΠΈΡΡ опСрациями аналитичСского прСобразования ΡΠ²Π»ΡΡŽΡ‚ΡΡ Π΄ΠΈΡ„Ρ„Π΅Ρ€Π΅Π½Ρ†ΠΈΡ€ΠΎΠ²Π°Π½ΠΈΠ΅ ΠΈ ΠΈΠ½Ρ‚Π΅Π³Ρ€ΠΈΡ€ΠΎΠ²Π°Π½ΠΈΠ΅ Ρ„ΡƒΠ½ΠΊΡ†ΠΈΠΎΠ½Π°Π»ΡŒΠ½Ρ‹Ρ… Π²Ρ‹Ρ€Π°ΠΆΠ΅Π½ΠΈΠΉ подстановка вмСсто ΠΏΠ΅Ρ€Π΅ΠΌΠ΅Π½Π½Ρ‹Ρ… констант ΠΈ Π²Ρ‹Ρ€Π°ΠΆΠ΅Π½ΠΈΠΉ ΡƒΠΏΡ€ΠΎΡ‰Π΅Π½ΠΈΠ΅ Π²Ρ‹Ρ€Π°ΠΆΠ΅Π½ΠΈΠΉ (свСртка констант, ΠΏΡ€ΠΈΠ²Π΅Π΄Π΅Π½ΠΈΠ΅ ΠΏΠΎΠ΄ΠΎΠ±Π½Ρ‹Ρ… Ρ‡Π»Π΅Π½ΠΎΠ² Π² ΠΌΠ½ΠΎΠ³ΠΎΡ‡Π»Π΅Π½Π°Ρ… ΠΈ Ρ‚. ΠΏ.) Ρ€Π°Π·Ρ€Π΅ΡˆΠ΅Π½ΠΈΠ΅ ΡƒΡ€Π°Π²Π½Π΅Π½ΠΈΠΉ ΠΎΡ‚Π½ΠΎΡΠΈΡ‚Π΅Π»ΡŒΠ½ΠΎ Π·Π°Π΄Π°Π½Π½Ρ‹Ρ… ΠΏΠ΅Ρ€Π΅ΠΌΠ΅Π½Π½Ρ‹Ρ… дСйствия Π½Π°Π΄ ΠΌΠ°Ρ‚Ρ€ΠΈΡ†Π°ΠΌΠΈ, элСмСнтами ΠΊΠΎΡ‚ΠΎΡ€Ρ‹Ρ… ΡΠ²Π»ΡΡŽΡ‚ΡΡ ΡΠΈΠΌΠ²ΠΎΠ»ΡŒΠ½Ρ‹Π΅ выраТСния Π²Ρ‹Π½ΠΎΠ»Π½Π΅Π½ΠΈΠ΅ алгСбраичСских дСйствий (слоТСниС, Π²Ρ‹Ρ‡ΠΈΡ‚Π°Π½ΠΈΠ΅, ΡƒΠΌΠ½ΠΎΠΆΠ΅Π½ΠΈΠ΅, Π΄Π΅Π»Π΅Π½ΠΈΠ΅) Π½Π°Π΄ арифмСтичСскими выраТСниями ΠΈ Ρ‚. ΠΏ. [c.248]

    Для слоТСния (вычитания) ΠΌΠ°Ρ‚Ρ€ΠΈΡ† Π½Π΅ΠΎΠ±Ρ…ΠΎΠ΄ΠΈΠΌΠΎ ΡΠ»ΠΎΠΆΠΈΡ‚ΡŒ (Π²Ρ‹Ρ‡Π΅ΡΡ‚ΡŒ) ΠΈΡ… элСмСнты, ΠΈΠΌΠ΅ΡŽΡ‰ΠΈΠ΅ ΠΎΠ΄ΠΈΠ½Π°ΠΊΠΎΠ²Ρ‹Π΅ индСксы. Π‘ΠΊΠ»Π°Π΄Ρ‹Π²Π°Ρ‚ΡŒ ΠΌΠΎΠΆΠ½ΠΎ ΠΌΠ°Ρ‚Ρ€ΠΈΡ†Ρ‹, содСрТащиС ΠΎΠ΄ΠΈΠ½Π°ΠΊΠΎΠ²ΠΎΠ΅ число строк ΠΈ ΠΊΠΎΠ»ΠΎΠ½ΠΎΠΊ  [c.167]

    БлоТСниС, Π²Ρ‹Ρ‡ΠΈΡ‚Π°Π½ΠΈΠ΅, ΠΏΡ€ΠΎΠΈΠ·Π²Π΅Π΄Π΅Π½ΠΈΠ΅ матриц [c.383]

β€‚β€‚β€‚β€‚Π˜Π· ΠΎΠΏΠ΅Ρ€Π°Ρ†ΠΈΠΉ слоТСния, вычитания ΠΈ умноТСния ΠΌΠ°Ρ‚Ρ€ΠΈΡ† послСдняя самая слоТная ΠΈ Π½Π°ΠΈΠ±ΠΎΠ»Π΅Π΅ часто ΠΈΡΠΏΠΎΠ»ΡŒΠ·ΡƒΠ΅ΠΌΠ°Ρ. Π’ дальнСйшСм ΠΌΠ°Ρ‚Ρ€ΠΈΡ†Ρ‹ Π±ΡƒΠ΄Π΅ΠΌ ΠΎΠ±ΠΎΠ·Π½Π°Ρ‡Π°Ρ‚ΡŒ прописными Π±ΡƒΠΊΠ²Π°ΠΌΠΈ.  [c.148]

    БлоТСниС, Π²Ρ‹Ρ‡ΠΈΡ‚Π°Π½ΠΈΠ΅ ΠΈ равСнство ΠΌΠ°Ρ‚Ρ€ΠΈΡ† ΠΌΠΎΠΆΠ½ΠΎ ΠΎΠΏΡ€Π΅Π΄Π΅Π»ΠΈΡ‚ΡŒ Ρ‚ΠΎΠ»ΡŒΠΊΠΎ Ρ‚ΠΎΠ³Π΄Π°, ΠΊΠΎΠ³Π΄Π° ΠΌΠ°Ρ‚Ρ€ΠΈΡ†Ρ‹ ΠΈΠΌΠ΅ΡŽΡ‚ ΠΎΠ΄ΠΈΠ½Π°ΠΊΠΎΠ²ΡƒΡŽ Ρ„ΠΎΡ€ΠΌΡƒ ΠΈ Ρ€Π°Π·ΠΌΠ΅Ρ€. Π’ этом случаС [c.65]

    ЭлСмСнтами мноТСства Π– ΠΎΠ±Ρ‹Ρ‡Π½ΠΎ слуТат вСщСствСнныС ΠΈΠ»ΠΈ комплСксныС числа, Ρ†Π΅Π»Ρ‹Π΅ числа, ΠΏΠΎΠ»ΠΈΠ½ΠΎΠΌΡ‹ Π½Π΅ΠΊΠΎΡ‚ΠΎΡ€ΠΎΠΉ ΠΏΠ΅Ρ€Π΅ΠΌΠ΅Π½Π½ΠΎΠΉ X, Ρ„ΡƒΠ½ΠΊΡ†ΠΈΠΈ ΠΎΠ΄Π½ΠΎΠΉ ΠΈΠ»ΠΈ Π½Π΅ΡΠΊΠΎΠ»ΡŒΠΊΠΈΡ… ΠΏΠ΅Ρ€Π΅ΠΌΠ΅Π½Π½Ρ‹Ρ… ΠΈ Ρ‚. ΠΏ. Если эти элСмСнты ΠΎΠ±Ρ€Π°Π·ΡƒΡŽΡ‚ ΠΏΠΎΠ»Π΅, Ρ‚. Π΅. для ΠΈΡ… совокупности ΠΎΠΏΡ€Π΅Π΄Π΅Π»Π΅Π½Ρ‹ Π²Ρ‹ΠΏΠΎΠ»Π½ΡΡŽΡ‰ΠΈΠ΅ΡΡ ΠΎΠ΄Π½ΠΎΠ·Π½Π°Ρ‡Π½ΠΎ ΠΎΠΏΠ΅Ρ€Π°Ρ†ΠΈΠΈ слоТСния, вычитания, умноТСния ΠΈ дСлСния, Ρ‚ΠΎ говорят, Ρ‡Ρ‚ΠΎ ΠΌΠ°Ρ‚Ρ€ΠΈΡ†Π° А Π·Π°Π΄Π°Π½Π° Π½Π°Π΄ ΠΏΠΎΠ»Π΅ΠΌ. Π”Π°Π»Π΅Π΅ ΠΌΡ‹ Π±ΡƒΠ΄Π΅ΠΌ ΠΈΠΌΠ΅Ρ‚ΡŒ Π΄Π΅Π»ΠΎ Π² основном с ΠΌΠ°Ρ‚Ρ€ΠΈΡ†Π°ΠΌΠΈ, Π·Π°Π΄Π°Π½Π½Ρ‹ΠΌΠΈ Π½Π°Π΄ ΠΏΠΎΠ»Π΅ΠΌ Π΄Π΅ΠΉΡΡ‚Π²ΠΈΡ‚Π΅Π»ΡŒΠ½Ρ‹Ρ… ΠΈΠ»ΠΈ вСщСствСнных чисСл. [c.9]


β€‚β€‚β€‚β€‚ΠžΠΏΠ΅Ρ€Π°Ρ†ΠΈΠΈ слоТСния ΠΈ вычитания ΠΌΠ°Ρ‚Ρ€ΠΈΡ† понятны ΠΈΠ· ΡΠ»Π΅Π΄ΡƒΡŽΡ‰ΠΈΡ… ΠΏΡ€ΠΈΠΌΠ΅Ρ€ΠΎΠ²  [c.695]

    Если Π¬Ρ† = Π¬Ρ†, Ρ‚ΠΎ ΠΌΠ°Ρ‚Ρ€ΠΈΡ†Ρƒ Π½Π°Π·Ρ‹Π²Π°ΡŽΡ‚ симмСтричной. А-5Π°. Π‘Π»ΠΎΠΆΠ΅Π½ΠΈΠ΅ ΠΈ Π²Ρ‹Ρ‡ΠΈΡ‚Π°Π½ΠΈΠ΅ ΠΌΠ°Ρ‚Ρ€ΠΈΡ†. ΠžΠΏΠ΅Ρ€Π°Ρ†ΠΈΡŽ Π’=А+В [c.432]

    БлоТСниС ΠΈ Π²Ρ‹Ρ‡ΠΈΡ‚Π°Π½ΠΈΠ΅ ΠΌΠ°Ρ‚Ρ€ΠΈΡ† ΠΎΠ΄Π½ΠΎΠ³ΠΎ Ρ€Π°Π·ΠΌΠ΅Ρ€Π°  [c. 7]

    Бумма ΠΈ Ρ€Π°Π·Π½ΠΎΡΡ‚ΡŒ. Π‘ΡƒΠΌΠΌΠΎΠΉ ΠΈΠ»ΠΈ Ρ€Π°Π·Π½ΠΎΡΡ‚ΡŒΡŽ Π΄ ΠΌΠ°Ρ‚Ρ€ΠΈΡ† А ΠΈ Π’ ΠΎΠ΄ΠΈΠ½Π°ΠΊΠΎΠ²Ρ‹Ρ… Ρ€Π°Π·ΠΌΠ΅Ρ€ΠΎΠ² называСтся ΠΌΠ°Ρ‚Ρ€ΠΈΡ†Π° Π‘, элСмСнтами ΠΊΠΎΡ‚ΠΎΡ€ΠΎΠΉ слуТат суммы ΠΈΠ»ΠΈ разности ΡΠΎΠΎΡ‚Π²Π΅Ρ‚ΡΡ‚Π²ΡƒΡŽΡ‰ΠΈΡ… элСмСнтов А ΠΈ Π’, Ρ‚.Π΅. Π‘=А+Π’, Ссли ΠΎΡ‚Π½ΠΎΡˆΠ΅Π½ΠΈΠΈ дСйствий слоТСния ΠΈ вычитания ΠΌΠ°Ρ‚Ρ€ΠΈΡ† справСдливы ΠΏΠ΅Ρ€Π΅ΠΌΠ΅ΡΡ‚ΠΈΡ‚Π΅Π»ΡŒΠ½Ρ‹ΠΉ ΠΈ ΡΠΎΡ‡Π΅Ρ‚Π°Ρ‚Π΅Π»ΡŒΠ½Ρ‹ΠΉ Π·Π°ΠΊΠΎΠ½Ρ‹  [c.216]

β€‚β€‚β€‚β€‚ΠŸΡ€ΠΎΠ³Ρ€Π°ΠΌΠΌΠ° Π½Π° стр. 290 Ρ€Π΅Π°Π»ΠΈΠ·ΡƒΠ΅Ρ‚ ΠΌΠ΅Ρ‚ΠΎΠ΄ ΡƒΠ½ΠΈΡ‚Π°Ρ€Π½Ρ‹Ρ… ΠΏΡ€Π΅ΠΎΠ±Ρ€Π°Π·ΠΎΠ²Π°Π½ΠΈΠΉ для нахоТдСния собствСнных Π·Π½Π°Ρ‡Π΅Π½ΠΈΠΉ Π΄Π΅ΠΉΡΡ‚Π²ΠΈΡ‚Π΅Π»ΡŒΠ½Ρ‹Ρ… нСсиммСтричСских ΠΌΠ°Ρ‚Ρ€ΠΈΡ†. Π’Ρ‹Ρ‡ΠΈΡΠ»ΠΈΡ‚Π΅Π»ΡŒΠ½Π°Ρ Ρ‡Π°ΡΡ‚ΡŒ ΠΏΡ€ΠΎΠ³Ρ€Π°ΠΌΠΌΡ‹ ΠΎΡ„ΠΎΡ€ΠΌΠ»Π΅Π½Π° Π² Π²ΠΈΠ΄Π΅ ΠΏΡ€ΠΎΡ†Π΅Π΄ΡƒΡ€Ρ‹ UNITIM, Π²Ρ…ΠΎΠ΄Π½Ρ‹ΠΌΠΈ ΠΏΠ°Ρ€Π°ΠΌΠ΅Ρ‚Ρ€Π°ΠΌΠΈ ΠΊΠΎΡ‚ΠΎΡ€ΠΎΠΉ ΡΠ²Π»ΡΡŽΡ‚ΡΡ порядок ΠΌΠ°Ρ‚Ρ€ΠΈΡ†Ρ‹ Π , ΠΌΠ°Ρ‚Ρ€ΠΈΡ†Π° U, Ρ‚ΠΎΡ‡Π½ΠΎΡΡ‚ΡŒ расчСта EPS. Π’Ρ‹Ρ…ΠΎΠ΄Π½Ρ‹ΠΌ ΠΏΠ°Ρ€Π°ΠΌΠ΅Ρ‚Ρ€ΠΎΠΌ ΠΏΡ€ΠΎΡ†Π΅Π΄ΡƒΡ€Ρ‹ являСтся ΠΌΠ°Ρ‚Ρ€ΠΈΡ†Π° L размСрности Π  X 2, строки ΠΊΠΎΡ‚ΠΎΡ€ΠΎΠΉ содСрТат Π΄Π΅ΠΉΡΡ‚Π²ΠΈΡ‚Π΅Π»ΡŒΠ½Ρ‹Π΅ ΠΈ ΠΌΠ½ΠΈΠΌΡ‹Π΅ части Π½Π°ΠΉΠ΄Π΅Π½Π½Ρ‹Ρ… собствСнных Π·Π½Π°Ρ‡Π΅Π½ΠΈΠΉ исходной ΠΌΠ°Ρ‚Ρ€ΠΈΡ†Ρ‹. Π’ ΠΏΡ€ΠΎΡ†Π΅Π΄ΡƒΡ€Π΅ UNI TIM ΠΈΡΠΏΠΎΠ»ΡŒΠ·ΡƒΡŽΡ‚ΡΡ Π΄Π²Π΅ ΠΏΡ€ΠΎΡ†Π΅Π΄ΡƒΡ€Ρ‹ SDM ΠΈ БОМР, пСрвая ΠΈΠ· ΠΊΠΎΡ‚ΠΎΡ€Ρ‹Ρ… Ρ€Π΅Π°Π»ΠΈΠ·ΡƒΠ΅Ρ‚ слоТСниС ΠΈ Π²Ρ‹Ρ‡ΠΈΡ‚Π°Π½ΠΈΠ΅ ΠΌΠ°Ρ‚Ρ€ΠΈΡ†, Π° вторая β€” ΠΏΡ€Π΅ΠΎΠ±Ρ€Π°Π·ΠΎΠ²Π°Π½ΠΈΠ΅ комплСксных чисСл ΠΈΠ· алгСбраичСской Π² Ρ‚Ρ€ΠΈΠ³ΠΎΠ½ΠΎΠΌΠ΅Ρ‚Ρ€ΠΈΡ‡Π΅ΡΠΊΡƒΡŽ Ρ„ΠΎΡ€ΠΌΡƒ ΠΈ ΠΎΠ±Ρ€Π°Ρ‚Π½ΠΎ.  [c.295]


Π£ΠΌΠ½ΠΎΠΆΠ΅Π½ΠΈΠ΅ ΠΌΠ°Ρ‚Ρ€ΠΈΡ†Ρ‹ Π½Π° скаляр. Π‘Π»ΠΎΠΆΠ΅Π½ΠΈΠ΅ ΠΈ Π²Ρ‹Ρ‡ΠΈΡ‚Π°Π½ΠΈΠ΅ ΠΌΠ°Ρ‚Ρ€ΠΈΡ†. Π£ΠΌΠ½ΠΎΠΆΠ΅Π½ΠΈΠ΅ ΠΌΠ°Ρ‚Ρ€ΠΈΡ†. ВранспонированиС ΠΌΠ°Ρ‚Ρ€ΠΈΡ†

Β Β Β Β Β Β Β Β Β Β Β Β Β Β Β Β Β Β Β Β Β Β Β Β Β Β Β Β Β Β Β Β Β Β Β Β Β Β Β Β  Π‘ΠΎΠ΄Π΅Ρ€ΠΆΠ°Π½ΠΈΠ΅

Π—Π°Π΄Π°Π½ΠΈΠ΅ ΠΊ курсовой Ρ€Π°Π±ΠΎΡ‚Π΅ ………………………….2

ВСорСтичСская Ρ‡Π°ΡΡ‚ΡŒ :

Β  1. Π£ΠΌΠ½ΠΎΠΆΠ΅Π½ΠΈΠ΅ ΠΌΠ°Ρ‚Ρ€ΠΈΡ†Ρ‹ Π½Π° скаляр ……………………3

Β  2. Π‘Π»ΠΎΠΆΠ΅Π½ΠΈΠ΅ ΠΈ Π²Ρ‹Ρ‡ΠΈΡ‚Π°Π½ΠΈΠ΅ ΠΌΠ°Ρ‚Ρ€ΠΈΡ† ……………………3

Β  3. Π£ΠΌΠ½ΠΎΠΆΠ΅Π½ΠΈΠ΅ ΠΌΠ°Ρ‚Ρ€ΠΈΡ† ……………………………..4

Β  4. ВранспонированиС ΠΌΠ°Ρ‚Ρ€ΠΈΡ† ……………………….5

Β  5.Поиск максимального (минимального) элСмСнта ΠΌΠ°Ρ‚Ρ€ΠΈΡ†Ρ‹ .6

Β  6. Π€ΠΎΡ€ΠΌΠΈΡ€ΠΎΠ²Π°Π½ΠΈΠ΅ Π²Π΅ΠΊΡ‚ΠΎΡ€Π° ΠΈΠ· элСмСнтов ΠΌΠ°Ρ‚Ρ€ΠΈΡ†Ρ‹ ……….7

Π‘Π»ΠΎΠΊ-схСма Π°Π»Π³ΠΎΡ€ΠΈΡ‚ΠΌΠ° Ρ€Π΅ΡˆΠ΅Π½ΠΈΡ Π·Π°Π΄Π°Ρ‡ΠΈ …………………9

ΠŸΡ€ΠΎΠ³Ρ€Π°ΠΌΠΌΠ° Ρ€Π΅ΡˆΠ΅Π½ΠΈΡ Π·Π°Π΄Π°Ρ‡ΠΈ ………………………….13

Π Π΅Π·ΡƒΠ»ΡŒΡ‚Π°Ρ‚Ρ‹ вычислСния ΠΏΠΎ ΠΏΡ€ΠΎΠ³Ρ€Π°ΠΌΠΌΠ΅ . ………………..16

ΠšΡ€Π°Ρ‚ΠΊΠΈΠ΅ Π²Ρ‹Π²ΠΎΠ΄Ρ‹ ΠΏΠΎ Ρ€Π΅ΡˆΠ΅Π½ΠΈΡŽ Π·Π°Π΄Π°Ρ‡ΠΈ …………………..17

Бписок ΠΈΡΠΏΠΎΠ»ΡŒΠ·ΡƒΠ΅ΠΌΠΎΠΉ Π»ΠΈΡ‚Π΅Ρ€Π°Ρ‚ΡƒΡ€Ρ‹ …………………….18

Π—Π°Π΄Π°Π½ΠΈΠ΅ ΠΊ курсовой Ρ€Π°Π±ΠΎΡ‚Π΅

Π‘ΠΎΡΡ‚Π°Π²ΠΈΡ‚ΡŒ ΠΏΡ€ΠΎΠ³Ρ€Π°ΠΌΠΌΡƒ вычислСния ΠΈ прСобразования ΠΌΠ°Ρ‚Ρ€ΠΈΡ†Ρ‹ D=F(A,B,C),Β  Π³Π΄Π΅ F(A,B,C) -ΠΌΠ°Ρ‚Ρ€ΠΈΡ‡Π½ΠΎΠ΅ Π²Ρ‹Ρ€Π°ΠΆΠ΅Π½ΠΈΠ΅ А,Π’,Π‘-исходныС ΠΌΠ°Ρ‚Ρ€ΠΈΡ†Ρ‹ Ρ€Π°Π·ΠΌΠ΅Ρ€ ΠΈ Π·Π½Π°Ρ‡Π΅Π½ΠΈΠ΅ элСмнтов ΠΊΠΎΡ‚ΠΎΡ€Ρ‹Ρ… Π²Ρ‹Π±ΠΈΡ€Π°ΡŽΡ‚ΡΡ ΠΏΡ€ΠΎΠΈΠ·Π²Π°Π»ΡŒΠ½ΠΎ.

Π’Ρ‹Ρ€Π°ΠΆΠ΅Π½ΠΈΠ΅ для расчёта ΠΌΠ°Ρ‚Ρ€ΠΈΡ†Ρ‹ D ΠΈ дСйствия ΠΏΠΎ Π΅Ρ‘ ΠΏΡ€Π΅ΠΎΠ±Ρ€Π°Π·ΠΎΠ²Π°Π½ΠΈΡŽ Π²Ρ‹Π±ΠΈΡ€Π°ΡŽΡ‚ΡΡ ΠΈΠ· Ρ‚Π°Π±Π»ΠΈΡ†Ρ‹ β„– 1 ΠΈ β„–2 Π² соотвСтствии с Π½ΠΎΠΌΠ΅Ρ€ΠΎΠΌ Π²Π°Ρ€ΠΈΠ°Π½Ρ‚Π° задания.

Π’ ΠΏΡ€ΠΎΠ³Ρ€Π°ΠΌΠΌΠ΅ ΠΏΡ€Π΅Π΄ΡƒΡΠΌΠΎΡ‚Ρ€Π΅Ρ‚ΡŒ Π²Ρ‹Π²ΠΎΠ΄ Π½Π° ΠΏΠ΅Ρ‡Π°Ρ‚ΡŒ всСх исходных, ΠΏΡ€ΠΎΠΌΠ΅ΠΆΡƒΡ‚ΠΎΡ‡Π½Ρ‹Ρ…Β  ΠΈ Ρ€Π°Π·ΡƒΠ»ΡŒΡ‚ΠΈΡ€ΡƒΡŽΡ‰ΠΈΡ… ΠΌΠ°Ρ‚Ρ€ΠΈΡ†, Π° Ρ‚Π°ΠΊΠΆΠ΅ Ρ€Π΅Π·ΡƒΠ»ΡŒΡ‚Π°Ρ‚ΠΎΠ² прСобразования 

Π’Π°Π±Π»ΠΈΡ†Π° β„–1

D= (2*A*BT-Π‘) TΒ 

Π’Π°Π±Π»ΠΈΡ†Π° β„–2

ΠžΠΏΡ€Π΅Π΄Π΅Π»ΠΈΡ‚ΡŒ количСство элСмСнтов, ΡƒΠ΄ΠΎΠ²Π»Π΅Ρ‚Π²ΠΎΡ€ΡΡŽΡ‰ΠΈΡ… условиям:

1) Pij>Q1Β Β Β Β Β Β Β Β  2)Pij<Q2Β Β Β Β Β Β Β Β Β Β  3)Q2<=Pij<=Q1.

ЗначСния Q1 ΠΈ Q2 Π²Ρ‹Π±Ρ€Π°Ρ‚ΡŒ ΠΏΡ€ΠΎΠΈΠ·Π²ΠΎΠ»ΡŒΠ½ΠΎ

1. Π£ΠΌΠ½ΠΎΠΆΠ΅Π½ΠΈΠ΅ ΠΌΠ°Ρ‚Ρ€ΠΈΡ†Ρ‹ Π½Π° скаляр

Π§Ρ‚ΠΎΠ±Ρ‹ ΡƒΠΌΠ½ΠΎΠΆΠΈΡ‚ΡŒ ΠΌΠ°Ρ‚Ρ€ΠΈΡ†Ρƒ Π½Π° скаляр, Π½Π΅ΠΎΠ±Ρ…ΠΎΠ΄ΠΈΠΌΠΎ ΠΊΠ°ΠΆΠ΄Ρ‹ΠΉ элСмСнт этой ΠΌΠ°Ρ‚Ρ€ΠΈΡ†Ρ‹ ΡƒΠΌΠ½ΠΎΠΆΠΈΡ‚ΡŒ Π½Π° скаляр. НапримСр, Ссли трСбуСтся Π²Ρ‹Ρ‡ΠΈΡΠ»ΠΈΡ‚ΡŒ А=d*Π’, Π³Π΄Π΅ А ΠΈ Π’ ΠΌΠ°Ρ‚Ρ€ΠΈΡ†Ρ‹, d – скаляр, Ρ‚ΠΎ ΠΊΠ°ΠΆΠ΄Ρ‹ΠΉ элСмСнт ΠΌΠ°Ρ‚Ρ€ΠΈΡ†Ρ‹ А опрСдСляСтся ΠΏΠΎ Ρ„ΠΎΡ€ΠΌΡƒΠ»Π΅ Π°ij = Π²*Π²ij; i = 1, 2,…, n;Β Β  j = 1, 2,…, m,Β Β Β Β Β Β Β Β Β Β Β Β Β Β Β Β Β Β  (1)

Π³Π΄Π΅ n, m – Ρ€Π°Π·ΠΌΠ΅Ρ€Ρ‹ ΠΌΠ°Ρ‚Ρ€ΠΈΡ†.

Алгоритмы умноТСния ΠΌΠ°Ρ‚Ρ€ΠΈΡ†Ρ‹ Π½Π° скаляр прСдставлСн Π½Π° рис. 1. Π—Π΄Π΅ΡΡŒ Ρ†Π΅Π½Ρ‚Ρ€Π°Π»ΡŒΠ½Ρ‹ΠΉ ΠΎΠΏΠ΅Ρ€Π°Ρ‚ΠΎΡ€ вычислСния ΠΎΡ‡Π΅Ρ€Π΅Π΄Π½ΠΎΠ³ΠΎ элСмСнта ΠΌΠ°Ρ‚Ρ€ΠΈΡ†Ρ‹-произвСдСния (Π±Π»ΠΎΠΊ 4) являСтся Ρ‚Π΅Π»ΠΎΠΌ Π΄Π²ΠΎΠΉΠ½ΠΎΠ³ΠΎ Ρ†ΠΈΠΊΠ»Π° ΠΏΠΎ строкам (i) ΠΈ столбцам (j).

Рис. 1

Π’Ρ…ΠΎΠ΄Π½Ρ‹Π΅ ΠΏΠ°Ρ€Π°ΠΌΠ΅Ρ‚Ρ€Ρ‹ ΠΌΠ°Ρ‚Ρ€ΠΈΡ†Π° Π’(n,m) скаляр dΒ Β Β Β Β Β Β Β Β Β Β Β Β Β Β Β Β Β Β Β Β 

Β Β Β Β Β Β Β Β Β Β Β Β Β Β Β Β Β Β Β Β Β Β Β Β Β Β Β Β Β Β Β Β Β Β Β Β Β Β Β Β Β Β Β Β  1

Π’Π₯ΠžΠ”Β 

Β 

Β Β Β Β Β Β Β Β Β Β Β Β Β Β Β Β Β Β Β Β Β Β Β Β Β Β Β Β Β Β Β Β Β Β Β Β Β Β Β Β Β Β Β Β  2Β Β Β 

Β Β Β Β Β Β Β Β Β Β Β Β Β Β Β Β Β Β Β Β Β Β Β Β Β Β Β Β Β Β Β Β Β Β Β Β Β Β Β Β Β Β Β Β Β Β Β Β  i=I,(1),n

Β Β Β Β Β Β Β Β Β Β Β Β 

Β Β Β Β Β Β Β Β Β Β Β Β Β Β Β Β Β Β Β Β Β Β Β Β Β Β Β Β Β Β Β Β Β Β Β Β Β Β Β Β Β Β Β Β Β  3

Β Β Β Β Β Β Β Β Β Β Β Β Β Β Β Β Β Β Β Β Β Β Β Β Β Β Β Β Β Β Β Β Β Β Β Β Β Β Β Β Β Β Β Β Β Β Β Β Β  j=I,(1),m

Β 

Β Β Β Β Β Β Β Β Β Β Β Β Β Β Β Β Β Β Β Β Β Β Β Β Β Β Β Β Β Β Β Β Β Β Β Β Β Β Β Β Β Β Β Β Β  4

Aij =d*Bij

Β 

Β Β Β Β Β Β Β Β Β Β Β Β Β Β Β Β Β Β Β Β Β Β Β Β Β Β Β Β Β Β 

Β 

Β Β Β Β Β Β Β Β Β Β Β Β Β Β Β Β Β Β Β Β Β Β Β Β Β Β Β Β Β Β Β Β Β Β Β Β Β Β Β Β Β Β Β Β Β Β  5

Β Β Β Β Β  Β Β Β Β Β Β Β Β Β Β Β Β Β Β Β Β Β Β Β Β Β Β Β Β Β Β Β Β Β Β Β Β Β Β Β Β Β Β Β Β Β Β Π’Π«Π₯ΠžΠ”Β Β 

2. Π‘Π»ΠΎΠΆΠ΅Π½ΠΈΠ΅ ΠΈ Π²Ρ‹Ρ‡ΠΈΡ‚Π°Π½ΠΈΠ΅ ΠΌΠ°Ρ‚Ρ€ΠΈΡ†

Π§Ρ‚ΠΎΠ±Ρ‹ ΡΠ»ΠΎΠΆΠΈΡ‚ΡŒ (Π²Ρ‹Ρ‡Π΅ΡΡ‚ΡŒ) Π΄Π²Π΅ ΠΌΠ°Ρ‚Ρ€ΠΈΡ†Ρ‹, Π½Π΅ΠΎΠ±Ρ…ΠΎΠ΄ΠΈΠΌΠΎ ΡΠ»ΠΎΠΆΠΈΡ‚ΡŒ (Π²Ρ‹Ρ‡Π΅ΡΡ‚ΡŒ) ΡΠΎΠΎΡ‚Π²Π΅Ρ‚ΡΡ‚Π²ΡƒΡŽΡ‰ΠΈΠ΅ элСмСнты этих ΠΌΠ°Ρ‚Ρ€ΠΈΡ†. НапримСр, Ссли трСбуСтся Π²Ρ‹Ρ‡ΠΈΡΠ»ΠΈΡ‚ΡŒ Π‘ = А+Π’, Π³Π΄Π΅ А ΠΈ Π’ – ΠΌΠ°Ρ‚Ρ€ΠΈΡ†Ρ‹, Ρ‚ΠΎ ΠΊΠ°ΠΆΠ΄Ρ‹ΠΉ элСмСнт ΠΌΠ°Ρ‚Ρ€ΠΈΡ†Ρ‹ Π‘ опрСдСляСтся ΠΏΠΎ Ρ„ΠΎΡ€ΠΌΡƒΠ»Π΅ сijΒ  = a ij = bij Β ; i = 1, 2,…, n ;Β Β  j = 1, 2,…, m,Β Β Β Β Β Β Β Β Β Β Β Β Β  (2)

Π³Π΄Π΅ n, m – Ρ€Π°Π·ΠΌΠ΅Ρ€Ρ‹ ΠΌΠ°Ρ‚Ρ€ΠΈΡ†.

Из выраТСния (2) слСдуСт, Ρ‡Ρ‚ΠΎ ΡΠΊΠ»Π°Π΄Ρ‹Π²Π°Ρ‚ΡŒ (Π²Ρ‹Ρ‡ΠΈΡ‚Π°Ρ‚ΡŒ) ΠΌΠΎΠΆΠ½ΠΎ Ρ‚ΠΎΠ»ΡŒΠΊΠΎ ΠΌΠ°Ρ‚Ρ€ΠΈΡ†Ρ‹ ΠΎΠ΄ΠΈΠ½Π°ΠΊΠΎΠ²ΠΎΠ³ΠΎ Ρ€Π°Π·ΠΌΠ΅Ρ€Π°. Алгоритм слоТСния (вычитания) ΠΌΠ°Ρ‚Ρ€ΠΈΡ† Π±ΡƒΠ΄Π΅Ρ‚ Ρ‚Π°ΠΊΠΈΠΌ ΠΆΠ΅, ΠΊΠ°ΠΊ Π½Π° рис. 1, Ссли Π² Π±Π»ΠΎΠΊΠ΅ 4 вмСсто Ρ„ΠΎΡ€ΠΌΡƒΠ»Ρ‹ (1) Π·Π°ΠΏΠΈΡΠ°Ρ‚ΡŒ Ρ„ΠΎΡ€ΠΌΡƒΠ»Ρƒ (2). ΠΏΡ€ΠΈ этом соотвСтствСнно измСнятся ΠΈ Π²Ρ‹Ρ…ΠΎΠ΄Π½Ρ‹Π΅ ΠΏΠ°Ρ€Π°ΠΌΠ΅Ρ‚Ρ€Ρ‹.

3. Π£ΠΌΠ½ΠΎΠΆΠ΅Π½ΠΈΠ΅ ΠΌΠ°Ρ‚Ρ€ΠΈΡ†

ΠŸΡ€ΠΈ ΠΏΠ΅Ρ€Π΅ΠΌΠ½ΠΎΠΆΠ΅Π½ΠΈΠΈ ΠΌΠ°Ρ‚Ρ€ΠΈΡ† ΠΊΠ°ΠΆΠ΄Ρ‹ΠΉ элСмСнт ΠΌΠ°Ρ‚Ρ€ΠΈΡ†Ρ‹-произвСдСния опрСдСляСтся ΠΊΠ°ΠΊ сумма ΠΏΡ€ΠΎΠΈΠ·Π²Π΅Π΄Π΅Π½ΠΈΠΉ элСмСнтов строки ΠΌΠ°Ρ‚Ρ€ΠΈΡ†Ρ‹-мноТитСля [1]. НапримСр, Ссли трСбуСтся Π²Ρ‹Ρ‡ΠΈΡΠ»ΠΈΡ‚ΡŒ C = A*B, Π³Π΄Π΅ A ΠΈ B – ΠΌΠ°Ρ‚Ρ€ΠΈΡ†Ρ‹, Ρ‚ΠΎ ΠΊΠ°ΠΆΠ΄Ρ‹ΠΉ элСмСнт ΠΌΠ°Ρ‚Ρ€ΠΈΡ†Ρ‹ Π‘ опрСдСляСтся ΠΏΠΎ Ρ„ΠΎΡ€ΠΌΡƒΠ»Π΅

m

Π‘ij =Β Β Β Β Β Β  aik * bkj ,Β  i = 1, 2,. .., n ;Β Β  j = 1, 2,…, m,Β Β Β  (3)

Β Β Β Β Β Β Β Β Β Β Β Β Β Β Β Β Β Β Β  k=1Β Β Β  Π³Π΄Π΅ n – число строк ΠΌΠ°Ρ‚Ρ€ΠΈΡ†Ρ‹ А;

i – число столбцов ΠΌΠ°Ρ‚Ρ€ΠΈΡ†Ρ‹ B;

m – число столбцов ΠΌΠ°Ρ‚Ρ€ΠΈΡ†Ρ‹ А ΠΈΠ»ΠΈ число строк ΠΌΠ°Ρ‚Ρ€ΠΈΡ†Ρ‹ Π’. Из (3) слСдуСт, Ρ‡Ρ‚ΠΎ ΠΏΠ΅Ρ€Π΅ΠΌΠ½ΠΎΠΆΠ°Ρ‚ΡŒ ΠΌΠΎΠΆΠ½ΠΎ Ρ‚ΠΎΠ»ΡŒΠΊΠΎ Ρ‚Π°ΠΊΠΈΠ΅ ΠΌΠ°Ρ‚Ρ€ΠΈΡ†Ρ‹, Ρƒ ΠΊΠΎΡ‚ΠΎΡ€Ρ‹Ρ… число столбцов ΠΌΠ°Ρ‚Ρ€ΠΈΡ†Ρ‹-ΠΌΠ½ΠΎΠΆΠΈΠΌΠΎΠ³ΠΎ Ρ€Π°Π²Π½ΠΎ числу строк ΠΌΠ°Ρ‚Ρ€ΠΈΡ†Ρ‹-мноТитСля, Ρ‚.Π΅.

Β aijΒ Β Β Β Β  nmΒ  *Β Β Β Β  bijΒ Β Β Β Β Β Β  nlΒ  =Β Β Β Β  cijΒ Β Β Β Β Β Β Β  nlΒ  .

Π‘Π»ΠΎΠΊ-схСма Π°Π»Π³ΠΎΡ€ΠΈΡ‚ΠΌΠ° пСрСмноТСния ΠΌΠ°Ρ‚Ρ€ΠΈΡ† прСдставлСна Π½Π° рис. 2. Π—Π΄Π΅ΡΡŒ ΠΊΠ°ΠΆΠ΄Ρ‹ΠΉ элСмСнт ΠΌΠ°Ρ‚Ρ€ΠΈΡ†Ρ‹-произвСдСния вычисляСтся Π² Π±Π»ΠΎΠΊΠ°Ρ… 4-7 ΠΊΠ°ΠΊ сумма ΠΏΡ€ΠΎΠΈΠ·Π²Π΅Π΄Π΅Π½ΠΈΠΉ. Π’ Π±Π»ΠΎΠΊΠ΅ 4 сумма сбрасываСтся Π² ноль, Π² Π±Π»ΠΎΠΊΠ΅ 6 производится нСпосрСдствСнноС суммированиС ΠΏΡ€ΠΎΠΈΠ·Π²Π΅Π΄Π΅Π½ΠΈΠΉ, Π² Π±Π»ΠΎΠΊΠ΅ 7 накоплСнная сумма записываСтся Π² Π²Ρ‹Ρ…ΠΎΠ΄Π½ΠΎΠΉ массив.

Рис. 2

Π’Ρ…ΠΎΠ΄Π½Ρ‹Π΅ ΠΏΠ°Ρ€Π°ΠΌΠ΅Ρ‚Ρ€Ρ‹ ΠΌΠ°Ρ‚Ρ€ΠΈΡ†Π° А(n,m), Π’(n,l)Β Β Β Β Β Β Β Β Β 

Β Β Β Β Β Β Β Β Β Β Β Β Β Β Β Β Β Β Β Β Β Β Β Β Β Β Β Β Β Β Β Β Β Β Β Β Β Β Β Β Β Β Β Β Β Β Β Β Β Β Β Β Β Β Β Β Β Β Β Β Β Β Β  1

Π’Π₯ΠžΠ”

Β 

Β Β Β Β Β Β Β Β Β Β Β Β Β Β Β Β Β Β Β Β Β Β Β Β Β Β Β Β Β Β Β Β Β Β Β Β Β Β Β Β Β Β Β Β Β Β Β Β Β Β Β Β Β Β Β Β Β Β Β Β Β Β Β Β  2

Β Β Β Β Β Β Β Β Β Β Β Β Β Β Β Β Β Β Β Β Β Β Β Β Β Β Β Β Β Β Β Β Β Β Β Β Β Β Β Β Β Β Β Β Β Β Β Β Β Β Β Β Β Β Β Β Β Β Β Β Β Β Β Β Β Β Β Β Β  i=I,(1),nΒ Β Β Β Β 

Β 

Β Β Β Β Β Β Β Β Β Β Β Β Β Β Β Β Β Β Β Β Β Β Β Β Β Β Β Β Β Β Β Β Β Β Β Β Β Β Β Β Β Β Β Β Β Β Β Β Β Β Β Β Β Β Β Β Β Β Β Β Β Β Β Β  3

Β Β Β Β Β Β Β Β Β Β Β Β Β Β Β Β Β Β Β Β Β Β Β Β Β Β Β Β Β Β Β Β Β Β Β Β Β Β Β Β Β Β Β  Β Β Β Β Β Β Β Β Β Β Β Β Β Β  Β Β Β Β Β Β Β Β Β Β j=I,(1),mΒ Β Β Β Β Β Β Β Β Β 

Β 

Β Β Β Β Β Β Β Β Β Β Β Β Β Β Β Β Β Β Β Β Β Β Β Β Β Β Β Β Β Β Β Β Β Β Β Β Β Β Β Β Β Β Β Β Β Β Β Β Β Β Β Β Β Β Β Β Β Β  Β Β Β Β Β  4

S=0Β Β 

Β Β Β Β Β Β Β  Β Β Β Β Β Β Β Β Β Β Β Β Β Β Β Β Β Β Β Β Β Β Β Β Β Β Β Β Β Β Β Β Β Β Β Β Β Β Β Β Β Β Β Β Β Β Β Β Β Β  Β Β Β Β Β Β 5

ΠžΠ±Ρ€Π°Ρ‚Π½Π°Ρ ΠΌΠ°Ρ‚Ρ€ΠΈΡ†Π°

Π­Ρ‚Π° Ρ‚Π΅ΠΌΠ° являСтся ΠΎΠ΄Π½ΠΎΠΉ ΠΈΠ· самых нСнавистных срСди студСнтов. Π₯ΡƒΠΆΠ΅, Π½Π°Π²Π΅Ρ€Π½ΠΎΠ΅, Ρ‚ΠΎΠ»ΡŒΠΊΠΎ ΠΎΠΏΡ€Π΅Π΄Π΅Π»ΠΈΡ‚Π΅Π»ΠΈ.

Ѐишка Π² Ρ‚ΠΎΠΌ, Ρ‡Ρ‚ΠΎ само понятиС ΠΎΠ±Ρ€Π°Ρ‚Π½ΠΎΠ³ΠΎ элСмСнта (ΠΈ я сСйчас Π½Π΅ Ρ‚ΠΎΠ»ΡŒΠΊΠΎ ΠΎ ΠΌΠ°Ρ‚Ρ€ΠΈΡ†Π°Ρ…) отсылаСт нас ΠΊ ΠΎΠΏΠ΅Ρ€Π°Ρ†ΠΈΠΈ умноТСния. Π”Π°ΠΆΠ΅ Π² школьной ΠΏΡ€ΠΎΠ³Ρ€Π°ΠΌΠΌΠ΅ ΡƒΠΌΠ½ΠΎΠΆΠ΅Π½ΠΈΠ΅ считаСтся слоТной ΠΎΠΏΠ΅Ρ€Π°Ρ†ΠΈΠ΅ΠΉ, Π° ΡƒΠΆ ΡƒΠΌΠ½ΠΎΠΆΠ΅Π½ΠΈΠ΅ ΠΌΠ°Ρ‚Ρ€ΠΈΡ† β€” Π²ΠΎΠΎΠ±Ρ‰Π΅ ΠΎΡ‚Π΄Π΅Π»ΡŒΠ½Π°Ρ Ρ‚Π΅ΠΌΠ°, ΠΊΠΎΡ‚ΠΎΡ€ΠΎΠΉ Ρƒ мСня посвящён Ρ†Π΅Π»Ρ‹ΠΉ ΠΏΠ°Ρ€Π°Π³Ρ€Π°Ρ„ ΠΈ Π²ΠΈΠ΄Π΅ΠΎΡƒΡ€ΠΎΠΊ.

БСгодня ΠΌΡ‹ Π½Π΅ Π±ΡƒΠ΄Π΅ΠΌ Π²Π΄Π°Π²Π°Ρ‚ΡŒΡΡ Π² подробности ΠΌΠ°Ρ‚Ρ€ΠΈΡ‡Π½Ρ‹Ρ… вычислСний. ΠŸΡ€ΠΎΡΡ‚ΠΎ вспомним: ΠΊΠ°ΠΊ ΠΎΠ±ΠΎΠ·Π½Π°Ρ‡Π°ΡŽΡ‚ΡΡ ΠΌΠ°Ρ‚Ρ€ΠΈΡ†Ρ‹, ΠΊΠ°ΠΊ ΠΎΠ½ΠΈ ΡƒΠΌΠ½ΠΎΠΆΠ°ΡŽΡ‚ΡΡ ΠΈ Ρ‡Ρ‚ΠΎ ΠΈΠ· этого слСдуСт.

ΠŸΠΎΠ²Ρ‚ΠΎΡ€Π΅Π½ΠΈΠ΅: ΡƒΠΌΠ½ΠΎΠΆΠ΅Π½ΠΈΠ΅ ΠΌΠ°Ρ‚Ρ€ΠΈΡ†

ΠŸΡ€Π΅ΠΆΠ΄Π΅ всСго договоримся ΠΎΠ± обозначСниях. ΠœΠ°Ρ‚Ρ€ΠΈΡ†Π΅ΠΉ $A$ Ρ€Π°Π·ΠΌΠ΅Ρ€Π° $\left[ m\times n \right]$ называСтся просто Ρ‚Π°Π±Π»ΠΈΡ†Π° ΠΈΠ· чисСл, Π² ΠΊΠΎΡ‚ΠΎΡ€ΠΎΠΉ Ρ€ΠΎΠ²Π½ΠΎ $m$ строк ΠΈ $n$ столбцов:

\[A=\left[ m\times n \right]=\underbrace{\left[ \begin{matrix} {{a}_{11}} & {{a}_{12}} & … & {{a}_{1n}} \\ {{a}_{21}} & {{a}_{22}} & . .. & {{a}_{2n}} \\ … & … & … & … \\ {{a}_{m1}} & {{a}_{m2}} & … & {{a}_{mn}} \\\end{matrix} \right]}_{n}\]

Π§Ρ‚ΠΎΠ±Ρ‹ случайно Π½Π΅ ΠΏΠ΅Ρ€Π΅ΠΏΡƒΡ‚Π°Ρ‚ΡŒ строки ΠΈ столбцы мСстами (ΠΏΠΎΠ²Π΅Ρ€ΡŒΡ‚Π΅, Π½Π° экзамСнС ΠΌΠΎΠΆΠ½ΠΎ ΠΈ Π΅Π΄ΠΈΠ½ΠΈΡ†Ρƒ с Π΄Π²ΠΎΠΉΠΊΠΎΠΉ ΠΏΠ΅Ρ€Π΅ΠΏΡƒΡ‚Π°Ρ‚ΡŒ β€” Ρ‡Ρ‚ΠΎ ΡƒΠΆ Π³ΠΎΠ²ΠΎΡ€ΠΈΡ‚ΡŒ ΠΏΡ€ΠΎ ΠΊΠ°ΠΊΠΈΠ΅-Ρ‚ΠΎ Ρ‚Π°ΠΌ строки), просто взглянитС Π½Π° ΠΊΠ°Ρ€Ρ‚ΠΈΠ½ΠΊΡƒ:

ΠžΠΏΡ€Π΅Π΄Π΅Π»Π΅Π½ΠΈΠ΅ индСксов для ΠΊΠ»Π΅Ρ‚ΠΎΠΊ ΠΌΠ°Ρ‚Ρ€ΠΈΡ†Ρ‹

Π§Ρ‚ΠΎ происходит? Если Ρ€Π°Π·ΠΌΠ΅ΡΡ‚ΠΈΡ‚ΡŒ ΡΡ‚Π°Π½Π΄Π°Ρ€Ρ‚Π½ΡƒΡŽ систСму ΠΊΠΎΠΎΡ€Π΄ΠΈΠ½Π°Ρ‚ $OXY$ Π² Π»Π΅Π²ΠΎΠΌ Π²Π΅Ρ€Ρ…Π½Π΅ΠΌ ΡƒΠ³Π»Ρƒ ΠΈ Π½Π°ΠΏΡ€Π°Π²ΠΈΡ‚ΡŒ оси Ρ‚Π°ΠΊ, Ρ‡Ρ‚ΠΎΠ±Ρ‹ ΠΎΠ½ΠΈ ΠΎΡ…Π²Π°Ρ‚Ρ‹Π²Π°Π»ΠΈ всю ΠΌΠ°Ρ‚Ρ€ΠΈΡ†Ρƒ, Ρ‚ΠΎ ΠΊΠ°ΠΆΠ΄ΠΎΠΉ ΠΊΠ»Π΅Ρ‚ΠΊΠ΅ этой ΠΌΠ°Ρ‚Ρ€ΠΈΡ†Ρ‹ ΠΌΠΎΠΆΠ½ΠΎ ΠΎΠ΄Π½ΠΎΠ·Π½Π°Ρ‡Π½ΠΎ ΡΠΎΠΏΠΎΡΡ‚Π°Π²ΠΈΡ‚ΡŒ ΠΊΠΎΠΎΡ€Π΄ΠΈΠ½Π°Ρ‚Ρ‹ $\left( x;y \right)$ β€” это ΠΈ Π±ΡƒΠ΄Π΅Ρ‚ Π½ΠΎΠΌΠ΅Ρ€ строки ΠΈ Π½ΠΎΠΌΠ΅Ρ€ столбца.

ΠŸΠΎΡ‡Π΅ΠΌΡƒ систСма ΠΊΠΎΠΎΡ€Π΄ΠΈΠ½Π°Ρ‚ Ρ€Π°Π·ΠΌΠ΅Ρ‰Π΅Π½Π° ΠΈΠΌΠ΅Π½Π½ΠΎ Π² Π»Π΅Π²ΠΎΠΌ Π²Π΅Ρ€Ρ…Π½Π΅ΠΌ ΡƒΠ³Π»Ρƒ? Π”Π° ΠΏΠΎΡ‚ΠΎΠΌΡƒ Ρ‡Ρ‚ΠΎ ΠΈΠΌΠ΅Π½Π½ΠΎ ΠΎΡ‚Ρ‚ΡƒΠ΄Π° ΠΌΡ‹ Π½Π°Ρ‡ΠΈΠ½Π°Π΅ΠΌ Ρ‡ΠΈΡ‚Π°Ρ‚ΡŒ Π»ΡŽΠ±Ρ‹Π΅ тСксты. Π­Ρ‚ΠΎ ΠΎΡ‡Π΅Π½ΡŒ просто Π·Π°ΠΏΠΎΠΌΠ½ΠΈΡ‚ΡŒ.

А ΠΏΠΎΡ‡Π΅ΠΌΡƒ ось $x$ Π½Π°ΠΏΡ€Π°Π²Π»Π΅Π½Π° ΠΈΠΌΠ΅Π½Π½ΠΎ Π²Π½ΠΈΠ·, Π° Π½Π΅ Π²ΠΏΡ€Π°Π²ΠΎ? ΠžΠΏΡΡ‚ΡŒ всё просто: Π²ΠΎΠ·ΡŒΠΌΠΈΡ‚Π΅ ΡΡ‚Π°Π½Π΄Π°Ρ€Ρ‚Π½ΡƒΡŽ систСму ΠΊΠΎΠΎΡ€Π΄ΠΈΠ½Π°Ρ‚ (ось $x$ ΠΈΠ΄Ρ‘Ρ‚ Π²ΠΏΡ€Π°Π²ΠΎ, ось $y$ β€” Π²Π²Π΅Ρ€Ρ…) ΠΈ ΠΏΠΎΠ²Π΅Ρ€Π½ΠΈΡ‚Π΅ Π΅Ρ‘ Ρ‚Π°ΠΊ, Ρ‡Ρ‚ΠΎΠ±Ρ‹ ΠΎΠ½Π° ΠΎΡ…Π²Π°Ρ‚Ρ‹Π²Π°Π»Π° ΠΌΠ°Ρ‚Ρ€ΠΈΡ†Ρƒ. Π­Ρ‚ΠΎ ΠΏΠΎΠ²ΠΎΡ€ΠΎΡ‚ Π½Π° 90 градусов ΠΏΠΎ часовой стрСлкС β€” Π΅Π³ΠΎ Ρ€Π΅Π·ΡƒΠ»ΡŒΡ‚Π°Ρ‚ ΠΌΡ‹ ΠΈ Π²ΠΈΠ΄ΠΈΠΌ Π½Π° ΠΊΠ°Ρ€Ρ‚ΠΈΠ½ΠΊΠ΅.

Π’ ΠΎΠ±Ρ‰Π΅ΠΌ, ΠΊΠ°ΠΊ ΠΎΠΏΡ€Π΅Π΄Π΅Π»ΡΡ‚ΡŒ индСксы Ρƒ элСмСнтов ΠΌΠ°Ρ‚Ρ€ΠΈΡ†Ρ‹, ΠΌΡ‹ Ρ€Π°Π·ΠΎΠ±Ρ€Π°Π»ΠΈΡΡŒ. Π’Π΅ΠΏΠ΅Ρ€ΡŒ Π΄Π°Π²Π°ΠΉΡ‚Π΅ разбСрёмся с ΡƒΠΌΠ½ΠΎΠΆΠ΅Π½ΠΈΠ΅ΠΌ.

ΠžΠΏΡ€Π΅Π΄Π΅Π»Π΅Π½ΠΈΠ΅. ΠœΠ°Ρ‚Ρ€ΠΈΡ†Ρ‹ $A=\left[ m\times n \right]$ ΠΈ $B=\left[ n\times k \right]$, ΠΊΠΎΠ³Π΄Π° количСство столбцов Π² ΠΏΠ΅Ρ€Π²ΠΎΠΉ совпадаСт с количСством строк Π²ΠΎ Π²Ρ‚ΠΎΡ€ΠΎΠΉ, Π½Π°Π·Ρ‹Π²Π°ΡŽΡ‚ΡΡ согласованными.

ИмСнно Π² Ρ‚Π°ΠΊΠΎΠΌ порядкС. МоТно ΡΡƒΠΌΠ½ΠΈΡ‡Π°Ρ‚ΡŒ ΠΈ ΡΠΊΠ°Π·Π°Ρ‚ΡŒ, ΠΌΠΎΠ», ΠΌΠ°Ρ‚Ρ€ΠΈΡ†Ρ‹ $A$ ΠΈ $B$ ΠΎΠ±Ρ€Π°Π·ΡƒΡŽΡ‚ ΡƒΠΏΠΎΡ€ΡΠ΄ΠΎΡ‡Π΅Π½Π½ΡƒΡŽ ΠΏΠ°Ρ€Ρƒ $\left( A;B \right)$: Ссли ΠΎΠ½ΠΈ согласованы Π² Ρ‚Π°ΠΊΠΎΠΌ порядкС, Ρ‚ΠΎ ΡΠΎΠ²Π΅Ρ€ΡˆΠ΅Π½Π½ΠΎ Π½Π΅ΠΎΠ±ΡΠ·Π°Ρ‚Π΅Π»ΡŒΠ½ΠΎ, Ρ‡Ρ‚ΠΎ $B$ ΠΈ $A$, Ρ‚.Π΅. ΠΏΠ°Ρ€Π° $\left( B;A \right)$ β€” Ρ‚ΠΎΠΆΠ΅ согласована.

Π£ΠΌΠ½ΠΎΠΆΠ°Ρ‚ΡŒ ΠΌΠΎΠΆΠ½ΠΎ Ρ‚ΠΎΠ»ΡŒΠΊΠΎ согласованныС ΠΌΠ°Ρ‚Ρ€ΠΈΡ†Ρ‹.

ΠžΠΏΡ€Π΅Π΄Π΅Π»Π΅Π½ΠΈΠ΅. ΠŸΡ€ΠΎΠΈΠ·Π²Π΅Π΄Π΅Π½ΠΈΠ΅ согласованных ΠΌΠ°Ρ‚Ρ€ΠΈΡ† $A=\left[ m\times n \right]$ ΠΈ $B=\left[ n\times k \right]$ β€” это новая ΠΌΠ°Ρ‚Ρ€ΠΈΡ†Π° $C=\left[ m\times k \right]$, элСмСнты ΠΊΠΎΡ‚ΠΎΡ€ΠΎΠΉ ${{c}_{ij}}$ ΡΡ‡ΠΈΡ‚Π°ΡŽΡ‚ΡΡ ΠΏΠΎ Ρ„ΠΎΡ€ΠΌΡƒΠ»Π΅:

\[{{c}_{ij}}=\sum\limits_{k=1}^{n}{{{a}_{ik}}}\cdot {{b}_{kj}}\]

Π”Ρ€ΡƒΠ³ΠΈΠΌΠΈ словами: Ρ‡Ρ‚ΠΎΠ±Ρ‹ ΠΏΠΎΠ»ΡƒΡ‡ΠΈΡ‚ΡŒ элСмСнт ${{c}_{ij}}$ ΠΌΠ°Ρ‚Ρ€ΠΈΡ†Ρ‹ $C=A\cdot B$, Π½ΡƒΠΆΠ½ΠΎ Π²Π·ΡΡ‚ΡŒ $i$-строку ΠΏΠ΅Ρ€Π²ΠΎΠΉ ΠΌΠ°Ρ‚Ρ€ΠΈΡ†Ρ‹, $j$-ΠΉ столбСц Π²Ρ‚ΠΎΡ€ΠΎΠΉ ΠΌΠ°Ρ‚Ρ€ΠΈΡ†Ρ‹, Π° Π·Π°Ρ‚Π΅ΠΌ ΠΏΠΎΠΏΠ°Ρ€Π½ΠΎ ΠΏΠ΅Ρ€Π΅ΠΌΠ½ΠΎΠΆΠΈΡ‚ΡŒ элСмСнты ΠΈΠ· этой строки ΠΈ столбца. Π Π΅Π·ΡƒΠ»ΡŒΡ‚Π°Ρ‚Ρ‹ ΡΠ»ΠΎΠΆΠΈΡ‚ΡŒ.

Π”Π°, Π²ΠΎΡ‚ Ρ‚Π°ΠΊΠΎΠ΅ суровоС ΠΎΠΏΡ€Π΅Π΄Π΅Π»Π΅Π½ΠΈΠ΅. Из Π½Π΅Π³ΠΎ сразу слСдуСт нСсколько Ρ„Π°ΠΊΡ‚ΠΎΠ²:

  1. Π£ΠΌΠ½ΠΎΠΆΠ΅Π½ΠΈΠ΅ ΠΌΠ°Ρ‚Ρ€ΠΈΡ†, Π²ΠΎΠΎΠ±Ρ‰Π΅ говоря, Π½Π΅ΠΊΠΎΠΌΠΌΡƒΡ‚Π°Ρ‚ΠΈΠ²Π½ΠΎ: $A\cdot B\ne B\cdot A$;
  2. Однако ΡƒΠΌΠ½ΠΎΠΆΠ΅Π½ΠΈΠ΅ ассоциативно: $\left( A\cdot B \right)\cdot C=A\cdot \left( B\cdot C \right)$;
  3. И Π΄Π°ΠΆΠ΅ дистрибутивно: $\left( A+B \right)\cdot C=A\cdot C+B\cdot C$;
  4. И Π΅Ρ‰Ρ‘ Ρ€Π°Π· дистрибутивно: $A\cdot \left( B+C \right)=A\cdot B+A\cdot C$.

Π”ΠΈΡΡ‚Ρ€ΠΈΠ±ΡƒΡ‚ΠΈΠ²Π½ΠΎΡΡ‚ΡŒ умноТСния ΠΏΡ€ΠΈΡˆΠ»ΠΎΡΡŒ ΠΎΡ‚Π΄Π΅Π»ΡŒΠ½ΠΎ ΠΎΠΏΠΈΡΡ‹Π²Π°Ρ‚ΡŒ для Π»Π΅Π²ΠΎΠ³ΠΎ ΠΈ ΠΏΡ€Π°Π²ΠΎΠ³ΠΎ мноТитСля-суммы ΠΊΠ°ΠΊ Ρ€Π°Π· ΠΈΠ·-Π·Π° нСкоммутативности ΠΎΠΏΠ΅Ρ€Π°Ρ†ΠΈΠΈ умноТСния.

Если всё ΠΆΠ΅ получаСтся Ρ‚Π°ΠΊ, Ρ‡Ρ‚ΠΎ $A\cdot B=B\cdot A$, Ρ‚Π°ΠΊΠΈΠ΅ ΠΌΠ°Ρ‚Ρ€ΠΈΡ†Ρ‹ Π½Π°Π·Ρ‹Π²Π°ΡŽΡ‚ΡΡ пСрСстановочными.

Π‘Ρ€Π΅Π΄ΠΈ всСх ΠΌΠ°Ρ‚Ρ€ΠΈΡ†, ΠΊΠΎΡ‚ΠΎΡ€Ρ‹Π΅ Ρ‚Π°ΠΌ Π½Π° Ρ‡Ρ‚ΠΎ-Ρ‚ΠΎ ΡƒΠΌΠ½ΠΎΠΆΠ°ΡŽΡ‚ΡΡ, Π΅ΡΡ‚ΡŒ особыС β€” Ρ‚Π΅, ΠΊΠΎΡ‚ΠΎΡ€Ρ‹Π΅ ΠΏΡ€ΠΈ ΡƒΠΌΠ½ΠΎΠΆΠ΅Π½ΠΈΠΈ Π½Π° Π»ΡŽΠ±ΡƒΡŽ ΠΌΠ°Ρ‚Ρ€ΠΈΡ†Ρƒ $A$ снова Π΄Π°ΡŽΡ‚ $A$:

ΠžΠΏΡ€Π΅Π΄Π΅Π»Π΅Π½ΠΈΠ΅. ΠœΠ°Ρ‚Ρ€ΠΈΡ†Π° $E$ называСтся Π΅Π΄ΠΈΠ½ΠΈΡ‡Π½ΠΎΠΉ, Ссли $A\cdot E=A$ ΠΈΠ»ΠΈ $E\cdot A=A$. Π’ случаС с ΠΊΠ²Π°Π΄Ρ€Π°Ρ‚Π½ΠΎΠΉ ΠΌΠ°Ρ‚Ρ€ΠΈΡ†Π΅ΠΉ $A$ ΠΌΠΎΠΆΠ΅ΠΌ Π·Π°ΠΏΠΈΡΠ°Ρ‚ΡŒ:

\[A\cdot E=E\cdot A=A\]

Единичная ΠΌΠ°Ρ‚Ρ€ΠΈΡ†Π° β€” частый Π³ΠΎΡΡ‚ΡŒ ΠΏΡ€ΠΈ Ρ€Π΅ΡˆΠ΅Π½ΠΈΠΈ ΠΌΠ°Ρ‚Ρ€ΠΈΡ‡Π½Ρ‹Ρ… ΡƒΡ€Π°Π²Π½Π΅Π½ΠΈΠΉ. {-1}}$. Π’ΠΎΠ³Π΄Π° эта обратная ΠΌΠ°Ρ‚Ρ€ΠΈΡ†Π° β€” СдинствСнная.

Π”ΠΎΠΊΠ°Π·Π°Ρ‚Π΅Π»ΡŒΡΡ‚Π²ΠΎ. ΠŸΠΎΠΉΠ΄Ρ‘ΠΌ ΠΎΡ‚ ΠΏΡ€ΠΎΡ‚ΠΈΠ²Π½ΠΎΠ³ΠΎ: ΠΏΡƒΡΡ‚ΡŒ Ρƒ ΠΌΠ°Ρ‚Ρ€ΠΈΡ†Ρ‹ $A$ Π΅ΡΡ‚ΡŒ хотя Π±Ρ‹ Π΄Π²Π° экзСмпляра ΠΎΠ±Ρ€Π°Ρ‚Π½Ρ‹Ρ… β€”$B$ ΠΈ $C$. Π’ΠΎΠ³Π΄Π°, согласно ΠΎΠΏΡ€Π΅Π΄Π΅Π»Π΅Π½ΠΈΡŽ, Π²Π΅Ρ€Π½Ρ‹ ΡΠ»Π΅Π΄ΡƒΡŽΡ‰ΠΈΠ΅ равСнства:

\[\begin{align} & A\cdot B=B\cdot A=E; \\ & A\cdot C=C\cdot A=E. \\ \end{align}\]

Из Π»Π΅ΠΌΠΌΡ‹ 1 ΠΌΡ‹ Π·Π°ΠΊΠ»ΡŽΡ‡Π°Π΅ΠΌ, Ρ‡Ρ‚ΠΎ всС Ρ‡Π΅Ρ‚Ρ‹Ρ€Π΅ ΠΌΠ°Ρ‚Ρ€ΠΈΡ†Ρ‹ β€” $A$, $B$, $C$ ΠΈ $E$ β€” ΡΠ²Π»ΡΡŽΡ‚ΡΡ ΠΊΠ²Π°Π΄Ρ€Π°Ρ‚Π½Ρ‹ΠΌΠΈ ΠΎΠ΄ΠΈΠ½Π°ΠΊΠΎΠ²ΠΎΠ³ΠΎ порядка: $\left[ n\times n \right]$. Π‘Π»Π΅Π΄ΠΎΠ²Π°Ρ‚Π΅Π»ΡŒΠ½ΠΎ, ΠΎΠΏΡ€Π΅Π΄Π΅Π»Π΅Π½ΠΎ ΠΏΡ€ΠΎΠΈΠ·Π²Π΅Π΄Π΅Π½ΠΈΠ΅:

\[B\cdot A\cdot C\]

ΠŸΠΎΡΠΊΠΎΠ»ΡŒΠΊΡƒ ΡƒΠΌΠ½ΠΎΠΆΠ΅Π½ΠΈΠ΅ ΠΌΠ°Ρ‚Ρ€ΠΈΡ† ассоциативно (Π½ΠΎ Π½Π΅ ΠΊΠΎΠΌΠΌΡƒΡ‚Π°Ρ‚ΠΈΠ²Π½ΠΎ!), ΠΌΡ‹ ΠΌΠΎΠΆΠ΅ΠΌ Π·Π°ΠΏΠΈΡΠ°Ρ‚ΡŒ:

\[\begin{align} & B\cdot A\cdot C=\left( B\cdot A \right)\cdot C=E\cdot C=C; \\ & B\cdot A\cdot C=B\cdot \left( A\cdot C \right)=B\cdot E=B; \\ & B\cdot A\cdot C=C=B\Rightarrow B=C. \\ \end{align}\]

ΠŸΠΎΠ»ΡƒΡ‡ΠΈΠ»ΠΈ СдинствСнно Π²ΠΎΠ·ΠΌΠΎΠΆΠ½Ρ‹ΠΉ Π²Π°Ρ€ΠΈΠ°Π½Ρ‚: Π΄Π²Π° экзСмпляра ΠΎΠ±Ρ€Π°Ρ‚Π½ΠΎΠΉ ΠΌΠ°Ρ‚Ρ€ΠΈΡ†Ρ‹ Ρ€Π°Π²Π½Ρ‹. Π›Π΅ΠΌΠΌΠ° Π΄ΠΎΠΊΠ°Π·Π°Π½Π°.

ΠŸΡ€ΠΈΠ²Π΅Π΄Ρ‘Π½Π½Ρ‹Π΅ рассуТдСния ΠΏΠΎΡ‡Ρ‚ΠΈ дословно ΠΏΠΎΠ²Ρ‚ΠΎΡ€ΡΡŽΡ‚ Π΄ΠΎΠΊΠ°Π·Π°Ρ‚Π΅Π»ΡŒΡΡ‚Π²ΠΎ Π΅Π΄ΠΈΠ½ΡΡ‚Π²Π΅Π½Π½ΠΎΡΡ‚ΡŒ ΠΎΠ±Ρ€Π°Ρ‚Π½ΠΎΠ³ΠΎ элСмСнта для всСх Π΄Π΅ΠΉΡΡ‚Π²ΠΈΡ‚Π΅Π»ΡŒΠ½Ρ‹Ρ… чисСл $b\ne 0$. {-1}} \right|\ne 0.\]

Π’ΠΎΡ‚ ΠΈ получаСтся, Ρ‡Ρ‚ΠΎ $\left| A \right|\ne 0$. Π›Π΅ΠΌΠΌΠ° Π΄ΠΎΠΊΠ°Π·Π°Π½Π°.

На самом Π΄Π΅Π»Π΅ это Ρ‚Ρ€Π΅Π±ΠΎΠ²Π°Π½ΠΈΠ΅ Π²ΠΏΠΎΠ»Π½Π΅ Π»ΠΎΠ³ΠΈΡ‡Π½ΠΎ. БСйчас ΠΌΡ‹ Ρ€Π°Π·Π±Π΅Ρ€Ρ‘ΠΌ Π°Π»Π³ΠΎΡ€ΠΈΡ‚ΠΌ нахоТдСния ΠΎΠ±Ρ€Π°Ρ‚Π½ΠΎΠΉ ΠΌΠ°Ρ‚Ρ€ΠΈΡ†Ρ‹ β€” ΠΈ станСт ΡΠΎΠ²Π΅Ρ€ΡˆΠ΅Π½Π½ΠΎ ясно, ΠΏΠΎΡ‡Π΅ΠΌΡƒ ΠΏΡ€ΠΈ Π½ΡƒΠ»Π΅Π²ΠΎΠΌ ΠΎΠΏΡ€Π΅Π΄Π΅Π»ΠΈΡ‚Π΅Π»Π΅ Π½ΠΈΠΊΠ°ΠΊΠΎΠΉ ΠΎΠ±Ρ€Π°Ρ‚Π½ΠΎΠΉ ΠΌΠ°Ρ‚Ρ€ΠΈΡ†Ρ‹ Π² ΠΏΡ€ΠΈΠ½Ρ†ΠΈΠΏΠ΅ Π½Π΅ ΠΌΠΎΠΆΠ΅Ρ‚ ΡΡƒΡ‰Π΅ΡΡ‚Π²ΠΎΠ²Π°Ρ‚ΡŒ.

Но для Π½Π°Ρ‡Π°Π»Π° сформулируСм Β«Π²ΡΠΏΠΎΠΌΠΎΠ³Π°Ρ‚Π΅Π»ΡŒΠ½ΠΎΠ΅Β» ΠΎΠΏΡ€Π΅Π΄Π΅Π»Π΅Π½ΠΈΠ΅:

ΠžΠΏΡ€Π΅Π΄Π΅Π»Π΅Π½ΠΈΠ΅. ВыроТдСнная ΠΌΠ°Ρ‚Ρ€ΠΈΡ†Π° β€” это квадратная ΠΌΠ°Ρ‚Ρ€ΠΈΡ†Π° Ρ€Π°Π·ΠΌΠ΅Ρ€Π° $\left[ n\times n \right]$, Ρ‡Π΅ΠΉ ΠΎΠΏΡ€Π΅Π΄Π΅Π»ΠΈΡ‚Π΅Π»ΡŒ Ρ€Π°Π²Π΅Π½ Π½ΡƒΠ»ΡŽ.

Π’Π°ΠΊΠΈΠΌ ΠΎΠ±Ρ€Π°Π·ΠΎΠΌ, ΠΌΡ‹ ΠΌΠΎΠΆΠ΅ΠΌ ΡƒΡ‚Π²Π΅Ρ€ΠΆΠ΄Π°Ρ‚ΡŒ, Ρ‡Ρ‚ΠΎ всякая обратимая ΠΌΠ°Ρ‚Ρ€ΠΈΡ†Π° являСтся Π½Π΅Π²Ρ‹Ρ€ΠΎΠΆΠ΄Π΅Π½Π½ΠΎΠΉ.

Как Π½Π°ΠΉΡ‚ΠΈ ΠΎΠ±Ρ€Π°Ρ‚Π½ΡƒΡŽ ΠΌΠ°Ρ‚Ρ€ΠΈΡ†Ρƒ

БСйчас ΠΌΡ‹ рассмотрим ΡƒΠ½ΠΈΠ²Π΅Ρ€ΡΠ°Π»ΡŒΠ½Ρ‹ΠΉ Π°Π»Π³ΠΎΡ€ΠΈΡ‚ΠΌ нахоТдСния ΠΎΠ±Ρ€Π°Ρ‚Π½Ρ‹Ρ… ΠΌΠ°Ρ‚Ρ€ΠΈΡ†. Π’ΠΎΠΎΠ±Ρ‰Π΅, сущСствуСт Π΄Π²Π° общСпринятых Π°Π»Π³ΠΎΡ€ΠΈΡ‚ΠΌΠ°, ΠΈ Π²Ρ‚ΠΎΡ€ΠΎΠΉ ΠΌΡ‹ Ρ‚ΠΎΠΆΠ΅ сСгодня рассмотрим.

Π’ΠΎΡ‚, ΠΊΠΎΡ‚ΠΎΡ€Ρ‹ΠΉ Π±ΡƒΠ΄Π΅Ρ‚ рассмотрСн сСйчас, ΠΎΡ‡Π΅Π½ΡŒ эффСктивСн для ΠΌΠ°Ρ‚Ρ€ΠΈΡ† Ρ€Π°Π·ΠΌΠ΅Ρ€Π° $\left[ 2\times 2 \right]$ ΠΈ β€” частично β€” Ρ€Π°Π·ΠΌΠ΅Ρ€Π° $\left[ 3\times 3 \right]$. {t}}$, Π³Π΄Π΅ $t$ β€” это (Π²ΠΎΡ‚ сСйчас Π²Π½ΠΈΠΌΠ°Π½ΠΈΠ΅!) сумма Π½ΠΎΠΌΠ΅Ρ€ΠΎΠ² всСх Π²Ρ‹Π±Ρ€Π°Π½Π½Ρ‹Ρ… строчСк ΠΈ столбцов. Π­Ρ‚ΠΎ ΠΈ Π±ΡƒΠ΄Π΅Ρ‚ алгСбраичСскоС Π΄ΠΎΠΏΠΎΠ»Π½Π΅Π½ΠΈΠ΅.

ВзглянитС Π½Π° Ρ‚Ρ€Π΅Ρ‚ΠΈΠΉ шаг: Ρ‚Π°ΠΌ Π²ΠΎΠΎΠ±Ρ‰Π΅-Ρ‚ΠΎ сумма $2k$ слагаСмых! Π”Ρ€ΡƒΠ³ΠΎΠ΅ Π΄Π΅Π»ΠΎ, Ρ‡Ρ‚ΠΎ для $k=1$ ΠΌΡ‹ ΠΏΠΎΠ»ΡƒΡ‡ΠΈΠΌ лишь 2 слагаСмых β€” это ΠΈ Π±ΡƒΠ΄ΡƒΡ‚ Ρ‚Π΅ самыС $i+j$ β€” Β«ΠΊΠΎΠΎΡ€Π΄ΠΈΠ½Π°Ρ‚Ρ‹Β» элСмСнта ${{a}_{ij}}$, для ΠΊΠΎΡ‚ΠΎΡ€ΠΎΠ³ΠΎ ΠΌΡ‹ ΠΈΡ‰Π΅ΠΌ алгСбраичСскоС Π΄ΠΎΠΏΠΎΠ»Π½Π΅Π½ΠΈΠ΅.

Π’Π°ΠΊΠΈΠΌ ΠΎΠ±Ρ€Π°Π·ΠΎΠΌ сСгодня ΠΌΡ‹ ΠΈΡΠΏΠΎΠ»ΡŒΠ·ΡƒΠ΅ΠΌ слСгка ΡƒΠΏΡ€ΠΎΡ‰Ρ‘Π½Π½ΠΎΠ΅ ΠΎΠΏΡ€Π΅Π΄Π΅Π»Π΅Π½ΠΈΠ΅. Но ΠΊΠ°ΠΊ ΠΌΡ‹ ΡƒΠ²ΠΈΠ΄ΠΈΠΌ Π² дальнСйшСм, Π΅Π³ΠΎ окаТСтся Π±ΠΎΠ»Π΅Π΅ Ρ‡Π΅ΠΌ достаточно. ΠšΡƒΠ΄Π° Π²Π°ΠΆΠ½Π΅Π΅ ΡΠ»Π΅Π΄ΡƒΡŽΡ‰Π°Ρ ΡˆΡ‚ΡƒΠΊΠ°:

ΠžΠΏΡ€Π΅Π΄Π΅Π»Π΅Π½ΠΈΠ΅. Боюзная ΠΌΠ°Ρ‚Ρ€ΠΈΡ†Π° $S$ ΠΊ ΠΊΠ²Π°Π΄Ρ€Π°Ρ‚Π½ΠΎΠΉ ΠΌΠ°Ρ‚Ρ€ΠΈΡ†Π΅ $A=\left[ n\times n \right]$ β€” это новая ΠΌΠ°Ρ‚Ρ€ΠΈΡ†Π° Ρ€Π°Π·ΠΌΠ΅Ρ€Π° $\left[ n\times n \right]$, которая получаСтся ΠΈΠ· $A$ Π·Π°ΠΌΠ΅Π½ΠΎΠΉ ${{a}_{ij}}$ алгСбраичСскими дополнСниями ${{A}_{ij}}$:

\[A=\left[ \begin{matrix} {{a}_{11}} & {{a}_{12}} & … & {{a}_{1n}} \\ {{a}_{21}} & {{a}_{22}} & … & {{a}_{2n}} \\ … & … & … & … \\ {{a}_{n1}} & {{a}_{n2}} & . {T}}=\left[ \begin{array}{*{35}{r}} 2 & -1 \\ -5 & 3 \\\end{array} \right]\]

Ну Π²ΠΎΡ‚ ΠΈ всё. Π—Π°Π΄Π°Ρ‡Π° Ρ€Π΅ΡˆΠ΅Π½Π°.

ΠžΡ‚Π²Π΅Ρ‚. $\left[ \begin{array}{*{35}{r}} 2 & -1 \\ -5 & 3 \\\end{array} \right]$

Π—Π°Π΄Π°Ρ‡Π°. НайдитС ΠΎΠ±Ρ€Π°Ρ‚Π½ΡƒΡŽ ΠΌΠ°Ρ‚Ρ€ΠΈΡ†Ρƒ:

\[\left[ \begin{array}{*{35}{r}} 1 & -1 & 2 \\ 0 & 2 & -1 \\ 1 & 0 & 1 \\\end{array} \right]\]

РСшСниС. ΠžΠΏΡΡ‚ΡŒ считаСм ΠΎΠΏΡ€Π΅Π΄Π΅Π»ΠΈΡ‚Π΅Π»ΡŒ:

\[\begin{align} & \left| \begin{array}{*{35}{r}} 1 & -1 & 2 \\ 0 & 2 & -1 \\ 1 & 0 & 1 \\\end{array} \right|=\begin{matrix} \left( 1\cdot 2\cdot 1+\left( -1 \right)\cdot \left( -1 \right)\cdot 1+2\cdot 0\cdot 0 \right)- \\ -\left( 2\cdot 2\cdot 1+\left( -1 \right)\cdot 0\cdot 1+1\cdot \left( -1 \right)\cdot 0 \right) \\\end{matrix}= \\ & =\left( 2+1+0 \right)-\left( 4+0+0 \right)=-1\ne 0. \\ \end{align}\]

ΠžΠΏΡ€Π΅Π΄Π΅Π»ΠΈΡ‚Π΅Π»ΡŒ ΠΎΡ‚Π»ΠΈΡ‡Π΅Π½ ΠΎΡ‚ нуля β€” ΠΌΠ°Ρ‚Ρ€ΠΈΡ†Π° ΠΎΠ±Ρ€Π°Ρ‚ΠΈΠΌΠ°. А Π²ΠΎΡ‚ сСйчас Π±ΡƒΠ΄Π΅Ρ‚ самая ΠΆΠ΅ΡΡ‚ΡŒ: Π½Π°Π΄ΠΎ ΠΏΠΎΡΡ‡ΠΈΡ‚Π°Ρ‚ΡŒ Π°ΠΆ 9 (Π΄Π΅Π²ΡΡ‚ΡŒ, ΠΌΠ°Ρ‚ΡŒ ΠΈΡ…!) алгСбраичСских Π΄ΠΎΠΏΠΎΠ»Π½Π΅Π½ΠΈΠΉ. {-1}}=\frac{1}{-1}\cdot \left[ \begin{matrix} 2 & -1 & -2 \\ 1 & -1 & -1 \\ -3 & 1 & 2 \\\end{matrix} \right]=\left[ \begin{array}{*{35}{r}}-2 & -1 & 3 \\ 1 & 1 & -1 \\ 2 & 1 & -2 \\\end{array} \right]\]

Ну ΠΈ всё. Π’ΠΎΡ‚ ΠΈ ΠΎΡ‚Π²Π΅Ρ‚.

ΠžΡ‚Π²Π΅Ρ‚. $\left[ \begin{array}{*{35}{r}} -2 & -1 & 3 \\ 1 & 1 & -1 \\ 2 & 1 & -2 \\\end{array} \right]$

Как Π²ΠΈΠ΄ΠΈΡ‚Π΅, Π² ΠΊΠΎΠ½Ρ†Π΅ ΠΊΠ°ΠΆΠ΄ΠΎΠ³ΠΎ ΠΏΡ€ΠΈΠΌΠ΅Ρ€Π° ΠΌΡ‹ выполняли ΠΏΡ€ΠΎΠ²Π΅Ρ€ΠΊΡƒ. Π’ связи с этим Π²Π°ΠΆΠ½ΠΎΠ΅ Π·Π°ΠΌΠ΅Ρ‡Π°Π½ΠΈΠ΅:

НС Π»Π΅Π½ΠΈΡ‚Π΅ΡΡŒ Π²Ρ‹ΠΏΠΎΠ»Π½ΡΡ‚ΡŒ ΠΏΡ€ΠΎΠ²Π΅Ρ€ΠΊΡƒ. Π£ΠΌΠ½ΠΎΠΆΡŒΡ‚Π΅ ΠΈΡΡ…ΠΎΠ΄Π½ΡƒΡŽ ΠΌΠ°Ρ‚Ρ€ΠΈΡ†Ρƒ Π½Π° Π½Π°ΠΉΠ΄Π΅Π½Π½ΡƒΡŽ ΠΎΠ±Ρ€Π°Ρ‚Π½ΡƒΡŽ β€” Π΄ΠΎΠ»ΠΆΠ½Π° ΠΏΠΎΠ»ΡƒΡ‡ΠΈΡ‚ΡŒΡΡ $E$.

Π’Ρ‹ΠΏΠΎΠ»Π½ΠΈΡ‚ΡŒ эту ΠΏΡ€ΠΎΠ²Π΅Ρ€ΠΊΡƒ Π½Π°ΠΌΠ½ΠΎΠ³ΠΎ ΠΏΡ€ΠΎΡ‰Π΅ ΠΈ быстрСС, Ρ‡Π΅ΠΌ ΠΈΡΠΊΠ°Ρ‚ΡŒ ΠΎΡˆΠΈΠ±ΠΊΡƒ Π² Π΄Π°Π»ΡŒΠ½Π΅ΠΉΡˆΠΈΡ… вычислСниях, ΠΊΠΎΠ³Π΄Π°, Π½Π°ΠΏΡ€ΠΈΠΌΠ΅Ρ€, Π²Ρ‹ Ρ€Π΅ΡˆΠ°Π΅Ρ‚Π΅ ΠΌΠ°Ρ‚Ρ€ΠΈΡ‡Π½ΠΎΠ΅ ΡƒΡ€Π°Π²Π½Π΅Π½ΠΈΠ΅.

ΠΠ»ΡŒΡ‚Π΅Ρ€Π½Π°Ρ‚ΠΈΠ²Π½Ρ‹ΠΉ способ

Как я ΠΈ Π³ΠΎΠ²ΠΎΡ€ΠΈΠ», Ρ‚Π΅ΠΎΡ€Π΅ΠΌΠ° ΠΎΠ± ΠΎΠ±Ρ€Π°Ρ‚Π½ΠΎΠΉ ΠΌΠ°Ρ‚Ρ€ΠΈΡ†Π΅ прСкрасно Ρ€Π°Π±ΠΎΡ‚Π°Π΅Ρ‚ для Ρ€Π°Π·ΠΌΠ΅Ρ€ΠΎΠ² $\left[ 2\times 2 \right]$ ΠΈ $\left[ 3\times 3 \right]$ (Π² послСднСм случаС β€” ΡƒΠΆΠ΅ Π½Π΅ Ρ‚Π°ΠΊ ΡƒΠΆ ΠΈ «прСкрасно»), Π° Π²ΠΎΡ‚ для ΠΌΠ°Ρ‚Ρ€ΠΈΡ† Π±ΠΎΠ»ΡŒΡˆΠΈΡ… Ρ€Π°Π·ΠΌΠ΅Ρ€ΠΎΠ² начинаСтся прям ΠΏΠ΅Ρ‡Π°Π»ΡŒ.

Но Π½Π΅ ΠΏΠ΅Ρ€Π΅ΠΆΠΈΠ²Π°ΠΉΡ‚Π΅: Π΅ΡΡ‚ΡŒ Π°Π»ΡŒΡ‚Π΅Ρ€Π½Π°Ρ‚ΠΈΠ²Π½Ρ‹ΠΉ Π°Π»Π³ΠΎΡ€ΠΈΡ‚ΠΌ, с ΠΏΠΎΠΌΠΎΡ‰ΡŒΡŽ ΠΊΠΎΡ‚ΠΎΡ€ΠΎΠ³ΠΎ ΠΌΠΎΠΆΠ½ΠΎ Π½Π΅Π²ΠΎΠ·ΠΌΡƒΡ‚ΠΈΠΌΠΎ Π½Π°ΠΉΡ‚ΠΈ ΠΎΠ±Ρ€Π°Ρ‚Π½ΡƒΡŽ Ρ…ΠΎΡ‚ΡŒ для ΠΌΠ°Ρ‚Ρ€ΠΈΡ†Ρ‹ $\left[ 10\times 10 \right]$. Но, ΠΊΠ°ΠΊ это часто Π±Ρ‹Π²Π°Π΅Ρ‚, для рассмотрСния этого Π°Π»Π³ΠΎΡ€ΠΈΡ‚ΠΌΠ° Π½Π°ΠΌ потрСбуСтся нСбольшая тСорСтичСская вводная.

Π­Π»Π΅ΠΌΠ΅Π½Ρ‚Π°Ρ€Π½Ρ‹Π΅ прСобразования

Π‘Ρ€Π΅Π΄ΠΈ всСвозмоТных ΠΏΡ€Π΅ΠΎΠ±Ρ€Π°Π·ΠΎΠ²Π°Π½ΠΈΠΉ ΠΌΠ°Ρ‚Ρ€ΠΈΡ†Ρ‹ Π΅ΡΡ‚ΡŒ нСсколько особых β€” ΠΈΡ… Π½Π°Π·Ρ‹Π²Π°ΡŽΡ‚ элСмСнтарными. Π’Π°ΠΊΠΈΡ… ΠΏΡ€Π΅ΠΎΠ±Ρ€Π°Π·ΠΎΠ²Π°Π½ΠΈΠΉ Ρ€ΠΎΠ²Π½ΠΎ Ρ‚Ρ€ΠΈ:

  1. Π£ΠΌΠ½ΠΎΠΆΠ΅Π½ΠΈΠ΅. МоТно Π²Π·ΡΡ‚ΡŒ $i$-ю строку (столбСц) ΠΈ ΡƒΠΌΠ½ΠΎΠΆΠΈΡ‚ΡŒ Π΅Ρ‘ Π½Π° любоС число $k\ne 0$;
  2. Π‘Π»ΠΎΠΆΠ΅Π½ΠΈΠ΅. ΠŸΡ€ΠΈΠ±Π°Π²ΠΈΡ‚ΡŒ ΠΊ $i$-ΠΉ строкС (столбцу) Π»ΡŽΠ±ΡƒΡŽ Π΄Ρ€ΡƒΠ³ΡƒΡŽ $j$-ю строку (столбСц), ΡƒΠΌΠ½ΠΎΠΆΠ΅Π½Π½ΡƒΡŽ Π½Π° любоС число $k\ne 0$ (ΠΌΠΎΠΆΠ½ΠΎ, ΠΊΠΎΠ½Π΅Ρ‡Π½ΠΎ, ΠΈ $k=0$, Π½ΠΎ ΠΊΠ°ΠΊΠΎΠΉ Π² этом смысл? НичСго Π½Π΅ измСнится ΠΆΠ΅).
  3. ΠŸΠ΅Ρ€Π΅ΡΡ‚Π°Π½ΠΎΠ²ΠΊΠ°. Π’Π·ΡΡ‚ΡŒ $i$-ю ΠΈ $j$-ю строки (столбцы) ΠΈ ΠΏΠΎΠΌΠ΅Π½ΡΡ‚ΡŒ мСстами.

ΠŸΠΎΡ‡Π΅ΠΌΡƒ эти прСобразования Π½Π°Π·Ρ‹Π²Π°ΡŽΡ‚ΡΡ элСмСнтарными (для Π±ΠΎΠ»ΡŒΡˆΠΈΡ… ΠΌΠ°Ρ‚Ρ€ΠΈΡ† ΠΎΠ½ΠΈ выглядят Π½Π΅ Ρ‚Π°ΠΊΠΈΠΌΠΈ ΡƒΠΆ элСмСнтарными) ΠΈ ΠΏΠΎΡ‡Π΅ΠΌΡƒ ΠΈΡ… Ρ‚ΠΎΠ»ΡŒΠΊΠΎ Ρ‚Ρ€ΠΈ β€” эти вопросы выходят Π·Π° Ρ€Π°ΠΌΠΊΠΈ сСгодняшнСго ΡƒΡ€ΠΎΠΊΠ°. ΠŸΠΎΡΡ‚ΠΎΠΌΡƒ Π½Π΅ Π±ΡƒΠ΄Π΅ΠΌ Π²Π΄Π°Π²Π°Ρ‚ΡŒΡΡ Π² подробности.

Π’Π°ΠΆΠ½ΠΎ Π΄Ρ€ΡƒΠ³ΠΎΠ΅: всС эти извращСния Π½Π°ΠΌ прСдстоит Π²Ρ‹ΠΏΠΎΠ»Π½ΡΡ‚ΡŒ Π½Π°Π΄ присоСдинённой ΠΌΠ°Ρ‚Ρ€ΠΈΡ†Π΅ΠΉ. Π”Π°, Π΄Π°: Π²Ρ‹ Π½Π΅ ΠΎΡΠ»Ρ‹ΡˆΠ°Π»ΠΈΡΡŒ. БСйчас Π±ΡƒΠ΄Π΅Ρ‚ Π΅Ρ‰Ρ‘ ΠΎΠ΄Π½ΠΎ ΠΎΠΏΡ€Π΅Π΄Π΅Π»Π΅Π½ΠΈΠ΅ β€” послСднСС Π² сСгодняшнСм ΡƒΡ€ΠΎΠΊΠ΅.

ΠŸΡ€ΠΈΡΠΎΠ΅Π΄ΠΈΠ½Ρ‘Π½Π½Π°Ρ ΠΌΠ°Ρ‚Ρ€ΠΈΡ†Π°

НавСрняка Π² школС Π²Ρ‹ Ρ€Π΅ΡˆΠ°Π»ΠΈ систСмы ΡƒΡ€Π°Π²Π½Π΅Π½ΠΈΠΉ ΠΌΠ΅Ρ‚ΠΎΠ΄ΠΎΠΌ слоТСния. Ну, Ρ‚Π°ΠΌ, Π²Ρ‹Ρ‡Π΅ΡΡ‚ΡŒ ΠΈΠ· ΠΎΠ΄Π½ΠΎΠΉ строки Π΄Ρ€ΡƒΠ³ΡƒΡŽ, ΡƒΠΌΠ½ΠΎΠΆΠΈΡ‚ΡŒ ΠΊΠ°ΠΊΡƒΡŽ-Ρ‚ΠΎ строку Π½Π° число β€” Π²ΠΎΡ‚ это Π²ΠΎΡ‚ всё.

Π’Π°ΠΊ Π²ΠΎΡ‚: сСйчас Π±ΡƒΠ΄Π΅Ρ‚ всё Ρ‚ΠΎ ΠΆΠ΅, Π½ΠΎ ΡƒΠΆΠ΅ Β«ΠΏΠΎ-взрослому». Π“ΠΎΡ‚ΠΎΠ²Ρ‹?

ΠžΠΏΡ€Π΅Π΄Π΅Π»Π΅Π½ΠΈΠ΅. ΠŸΡƒΡΡ‚ΡŒ Π΄Π°Π½Π° ΠΌΠ°Ρ‚Ρ€ΠΈΡ†Π° $A=\left[ n\times n \right]$ ΠΈ Сдиничная ΠΌΠ°Ρ‚Ρ€ΠΈΡ†Π° $E$ Ρ‚Π°ΠΊΠΎΠ³ΠΎ ΠΆΠ΅ Ρ€Π°Π·ΠΌΠ΅Ρ€Π° $n$. Π’ΠΎΠ³Π΄Π° присоСдинённая ΠΌΠ°Ρ‚Ρ€ΠΈΡ†Π° $\left[ A\left| E \right. \right]$ β€” это новая ΠΌΠ°Ρ‚Ρ€ΠΈΡ†Π° Ρ€Π°Π·ΠΌΠ΅Ρ€Π° $\left[ n\times 2n \right]$, которая выглядит Ρ‚Π°ΠΊ:

\[\left[ A\left| E \right. \right]=\left[ \begin{array}{rrrr|rrrr}{{a}_{11}} & {{a}_{12}} & … & {{a}_{1n}} & 1 & 0 & … & 0 \\{{a}_{21}} & {{a}_{22}} & … & {{a}_{2n}} & 0 & 1 & . {-1}}\]

Π’ΠΎΡ‚ Ρ‚Π°ΠΊ всё просто! ΠšΠΎΡ€ΠΎΡ‡Π΅ говоря, Π°Π»Π³ΠΎΡ€ΠΈΡ‚ΠΌ нахоТдСния ΠΎΠ±Ρ€Π°Ρ‚Π½ΠΎΠΉ ΠΌΠ°Ρ‚Ρ€ΠΈΡ†Ρ‹ выглядит Ρ‚Π°ΠΊ:

  1. Π—Π°ΠΏΠΈΡΠ°Ρ‚ΡŒ ΠΏΡ€ΠΈΡΠΎΠ΅Π΄ΠΈΠ½Ρ‘Π½Π½ΡƒΡŽ ΠΌΠ°Ρ‚Ρ€ΠΈΡ†Ρƒ $\left[ A\left| E \right. \right]$;
  2. Π’Ρ‹ΠΏΠΎΠ»Π½ΡΡ‚ΡŒ элСмСнтарныС прСобразования строк Π΄ΠΎ Ρ‚Π΅Ρ… ΠΏΠΎΡ€, ΠΏΠΎΠΊΠ° ΠΏΡ€Π°Π²Π° вмСсто $A$ Π½Π΅ появится $E$;
  3. РазумССтся, слСва Ρ‚ΠΎΠΆΠ΅ Ρ‡Ρ‚ΠΎ-Ρ‚ΠΎ появится β€” нСкая ΠΌΠ°Ρ‚Ρ€ΠΈΡ†Π° $B$. Π­Ρ‚ΠΎ ΠΈ Π±ΡƒΠ΄Π΅Ρ‚ обратная;
  4. PROFIT!:)

ΠšΠΎΠ½Π΅Ρ‡Π½ΠΎ, ΡΠΊΠ°Π·Π°Ρ‚ΡŒ Π½Π°ΠΌΠ½ΠΎΠ³ΠΎ ΠΏΡ€ΠΎΡ‰Π΅, Ρ‡Π΅ΠΌ ΡΠ΄Π΅Π»Π°Ρ‚ΡŒ. ΠŸΠΎΡΡ‚ΠΎΠΌΡƒ Π΄Π°Π²Π°ΠΉΡ‚Π΅ рассмотрим ΠΏΠ°Ρ€ΠΎΡ‡ΠΊΡƒ ΠΏΡ€ΠΈΠΌΠ΅Ρ€ΠΎΠ²: для Ρ€Π°Π·ΠΌΠ΅Ρ€ΠΎΠ² $\left[ 3\times 3 \right]$ ΠΈ $\left[ 4\times 4 \right]$.

Π—Π°Π΄Π°Ρ‡Π°. НайдитС ΠΎΠ±Ρ€Π°Ρ‚Π½ΡƒΡŽ ΠΌΠ°Ρ‚Ρ€ΠΈΡ†Ρƒ:

\[\left[ \begin{array}{*{35}{r}} 1 & 5 & 1 \\ 3 & 2 & 1 \\ 6 & -2 & 1 \\\end{array} \right]\]

РСшСниС. БоставляСм ΠΏΡ€ΠΈΡΠΎΠ΅Π΄ΠΈΠ½Ρ‘Π½Π½ΡƒΡŽ ΠΌΠ°Ρ‚Ρ€ΠΈΡ†Ρƒ:

\[\left[ \begin{array}{rrr|rrr} 1 & 5 & 1 & 1 & 0 & 0 \\ 3 & 2 & 1 & 0 & 1 & 0 \\ 6 & -2 & 1 & 0 & 0 & 1 \\\end{array} \right]\]

ΠŸΠΎΡΠΊΠΎΠ»ΡŒΠΊΡƒ послСдний столбСц исходной ΠΌΠ°Ρ‚Ρ€ΠΈΡ†Ρ‹ Π·Π°ΠΏΠΎΠ»Π½Π΅Π½ Π΅Π΄ΠΈΠ½ΠΈΡ†Π°ΠΌΠΈ, Π²Ρ‹Ρ‡Ρ‚Π΅ΠΌ ΠΏΠ΅Ρ€Π²ΡƒΡŽ строку ΠΈΠ· ΠΎΡΡ‚Π°Π»ΡŒΠ½Ρ‹Ρ…:

\[\begin{align} & \left[ \begin{array}{rrr|rrr} 1 & 5 & 1 & 1 & 0 & 0 \\ 3 & 2 & 1 & 0 & 1 & 0 \\ 6 & -2 & 1 & 0 & 0 & 1 \\\end{array} \right]\begin{matrix} \downarrow \\ -1 \\ -1 \\\end{matrix}\to \\ & \to \left[ \begin{array}{rrr|rrr} 1 & 5 & 1 & 1 & 0 & 0 \\ 2 & -3 & 0 & -1 & 1 & 0 \\ 5 & -7 & 0 & -1 & 0 & 1 \\\end{array} \right] \\ \end{align}\]

Π‘ΠΎΠ»ΡŒΡˆΠ΅ Π΅Π΄ΠΈΠ½ΠΈΡ† Π½Π΅Ρ‚, ΠΊΡ€ΠΎΠΌΠ΅ ΠΏΠ΅Ρ€Π²ΠΎΠΉ строки. Но Π΅Ρ‘ ΠΌΡ‹ Π½Π΅ Ρ‚Ρ€ΠΎΠ³Π°Π΅ΠΌ, ΠΈΠ½Π°Ρ‡Π΅ Π² Ρ‚Ρ€Π΅Ρ‚ΡŒΠ΅ΠΌ столбцС Π½Π°Ρ‡Π½ΡƒΡ‚ Β«Ρ€Π°Π·ΠΌΠ½ΠΎΠΆΠ°Ρ‚ΡŒΡΡΒ» Ρ‚ΠΎΠ»ΡŒΠΊΠΎ Ρ‡Ρ‚ΠΎ ΡƒΠ±Ρ€Π°Π½Π½Ρ‹Π΅ Π΅Π΄ΠΈΠ½ΠΈΡ†Ρ‹.

Π—Π°Ρ‚ΠΎ ΠΌΠΎΠΆΠ΅ΠΌ Π²Ρ‹Ρ‡Π΅ΡΡ‚ΡŒ Π²Ρ‚ΠΎΡ€ΡƒΡŽ строку Π΄Π²Π°ΠΆΠ΄Ρ‹ ΠΈΠ· послСднСй β€” ΠΏΠΎΠ»ΡƒΡ‡ΠΈΠΌ Π΅Π΄ΠΈΠ½ΠΈΡ†Ρƒ Π² Π»Π΅Π²ΠΎΠΌ Π½ΠΈΠΆΠ½Π΅ΠΌ ΡƒΠ³Π»Ρƒ:

\[\begin{align} & \left[ \begin{array}{rrr|rrr} 1 & 5 & 1 & 1 & 0 & 0 \\ 2 & -3 & 0 & -1 & 1 & 0 \\ 5 & -7 & 0 & -1 & 0 & 1 \\\end{array} \right]\begin{matrix} \ \\ \downarrow \\ -2 \\\end{matrix}\to \\ & \left[ \begin{array}{rrr|rrr} 1 & 5 & 1 & 1 & 0 & 0 \\ 2 & -3 & 0 & -1 & 1 & 0 \\ 1 & -1 & 0 & 1 & -2 & 1 \\\end{array} \right] \\ \end{align}\]

Π’Π΅ΠΏΠ΅Ρ€ΡŒ ΠΌΠΎΠΆΠ½ΠΎ Π²Ρ‹Ρ‡Π΅ΡΡ‚ΡŒ послСднюю строку ΠΈΠ· ΠΏΠ΅Ρ€Π²ΠΎΠΉ ΠΈ Π΄Π²Π°ΠΆΠ΄Ρ‹ ΠΈΠ· Π²Ρ‚ΠΎΡ€ΠΎΠΉ β€” Ρ‚Π°ΠΊΠΈΠΌ ΠΎΠ±Ρ€Π°Π·ΠΎΠΌ ΠΌΡ‹ Β«Π·Π°Π½ΡƒΠ»ΠΈΠΌΒ» ΠΏΠ΅Ρ€Π²Ρ‹ΠΉ столбСц:

\[\begin{align} & \left[ \begin{array}{rrr|rrr} 1 & 5 & 1 & 1 & 0 & 0 \\ 2 & -3 & 0 & -1 & 1 & 0 \\ 1 & -1 & 0 & 1 & -2 & 1 \\\end{array} \right]\begin{matrix} -1 \\ -2 \\ \uparrow \\\end{matrix}\to \\ & \to \left[ \begin{array}{rrr|rrr} 0 & 6 & 1 & 0 & 2 & -1 \\ 0 & -1 & 0 & -3 & 5 & -2 \\ 1 & -1 & 0 & 1 & -2 & 1 \\\end{array} \right] \\ \end{align}\]

Π£ΠΌΠ½ΠΎΠΆΠΈΠΌ Π²Ρ‚ΠΎΡ€ΡƒΡŽ строку Π½Π° βˆ’1, Π° Π·Π°Ρ‚Π΅ΠΌ Π²Ρ‹Ρ‡Ρ‚Π΅ΠΌ Π΅Ρ‘ 6 Ρ€Π°Π· ΠΈΠ· ΠΏΠ΅Ρ€Π²ΠΎΠΉ ΠΈ ΠΏΡ€ΠΈΠ±Π°Π²ΠΈΠΌ 1 Ρ€Π°Π· ΠΊ послСднСй:

\[\begin{align} & \left[ \begin{array}{rrr|rrr} 0 & 6 & 1 & 0 & 2 & -1 \\ 0 & -1 & 0 & -3 & 5 & -2 \\ 1 & -1 & 0 & 1 & -2 & 1 \\\end{array} \right]\begin{matrix} \ \\ \left| \cdot \left( -1 \right) \right. \\ \ \\\end{matrix}\to \\ & \to \left[ \begin{array}{rrr|rrr} 0 & 6 & 1 & 0 & 2 & -1 \\ 0 & 1 & 0 & 3 & -5 & 2 \\ 1 & -1 & 0 & 1 & -2 & 1 \\\end{array} \right]\begin{matrix} -6 \\ \updownarrow \\ +1 \\\end{matrix}\to \\ & \to \left[ \begin{array}{rrr|rrr} 0 & 0 & 1 & -18 & 32 & -13 \\ 0 & 1 & 0 & 3 & -5 & 2 \\ 1 & 0 & 0 & 4 & -7 & 3 \\\end{array} \right] \\ \end{align}\]

ΠžΡΡ‚Π°Π»ΠΎΡΡŒ лишь ΠΏΠΎΠΌΠ΅Π½ΡΡ‚ΡŒ мСстами строки 1 ΠΈ 3:

\[\left[ \begin{array}{rrr|rrr} 1 & 0 & 0 & 4 & -7 & 3 \\ 0 & 1 & 0 & 3 & -5 & 2 \\ 0 & 0 & 1 & -18 & 32 & -13 \\\end{array} \right]\]

Π“ΠΎΡ‚ΠΎΠ²ΠΎ! Π‘ΠΏΡ€Π°Π²Π° β€” искомая обратная ΠΌΠ°Ρ‚Ρ€ΠΈΡ†Π°.

ΠžΡ‚Π²Π΅Ρ‚. $\left[ \begin{array}{*{35}{r}}4 & -7 & 3 \\ 3 & -5 & 2 \\ -18 & 32 & -13 \\\end{array} \right]$

Π—Π°Π΄Π°Ρ‡Π°. НайдитС ΠΎΠ±Ρ€Π°Ρ‚Π½ΡƒΡŽ ΠΌΠ°Ρ‚Ρ€ΠΈΡ†Ρƒ:

\[\left[ \begin{matrix} 1 & 4 & 2 & 3 \\ 1 & -2 & 1 & -2 \\ 1 & -1 & 1 & 1 \\ 0 & -10 & -2 & -5 \\\end{matrix} \right]\]

РСшСниС. Π‘Π½ΠΎΠ²Π° составляСм ΠΏΡ€ΠΈΡΠΎΠ΅Π΄ΠΈΠ½Ρ‘Π½Π½ΡƒΡŽ:

\[\left[ \begin{array}{rrrr|rrrr} 1 & 4 & 2 & 3 & 1 & 0 & 0 & 0 \\ 1 & -2 & 1 & -2 & 0 & 1 & 0 & 0 \\ 1 & -1 & 1 & 1 & 0 & 0 & 1 & 0 \\ 0 & -10 & -2 & -5 & 0 & 0 & 0 & 1 \\\end{array} \right]\]

НСмного ΠΏΠΎΠ·Π°Π»ΠΈΠΌΠ°Π΅ΠΌ, попСчалимся ΠΎΡ‚ Ρ‚ΠΎΠ³ΠΎ, сколько сСйчас придётся ΡΡ‡ΠΈΡ‚Π°Ρ‚ΡŒ… ΠΈ Π½Π°Ρ‡Π½Ρ‘ΠΌ ΡΡ‡ΠΈΡ‚Π°Ρ‚ΡŒ. Для Π½Π°Ρ‡Π°Π»Π° Β«ΠΎΠ±Π½ΡƒΠ»ΠΈΠΌΒ» ΠΏΠ΅Ρ€Π²Ρ‹ΠΉ столбСц, вычитая строку 1 ΠΈΠ· строк 2 ΠΈ 3:

\[\begin{align} & \left[ \begin{array}{rrrr|rrrr} 1 & 4 & 2 & 3 & 1 & 0 & 0 & 0 \\ 1 & -2 & 1 & -2 & 0 & 1 & 0 & 0 \\ 1 & -1 & 1 & 1 & 0 & 0 & 1 & 0 \\ 0 & -10 & -2 & -5 & 0 & 0 & 0 & 1 \\\end{array} \right]\begin{matrix} \downarrow \\ -1 \\ -1 \\ \ \\\end{matrix}\to \\ & \to \left[ \begin{array}{rrrr|rrrr} 1 & 4 & 2 & 3 & 1 & 0 & 0 & 0 \\ 0 & -6 & -1 & -5 & -1 & 1 & 0 & 0 \\ 0 & -5 & -1 & -2 & -1 & 0 & 1 & 0 \\ 0 & -10 & -2 & -5 & 0 & 0 & 0 & 1 \\\end{array} \right] \\ \end{align}\]

НаблюдаСм слишком ΠΌΠ½ΠΎΠ³ΠΎ «минусов» Π² строках 2β€”4. Π£ΠΌΠ½ΠΎΠΆΠΈΠΌ всС Ρ‚Ρ€ΠΈ строки Π½Π° βˆ’1, Π° Π·Π°Ρ‚Π΅ΠΌ Β«Π²Ρ‹ΠΆΠΆΠ΅ΠΌΒ» Ρ‚Ρ€Π΅Ρ‚ΠΈΠΉ столбСц, вычитая строку 3 ΠΈΠ· ΠΎΡΡ‚Π°Π»ΡŒΠ½Ρ‹Ρ…:

\[\begin{align} & \left[ \begin{array}{rrrr|rrrr} 1 & 4 & 2 & 3 & 1 & 0 & 0 & 0 \\ 0 & -6 & -1 & -5 & -1 & 1 & 0 & 0 \\ 0 & -5 & -1 & -2 & -1 & 0 & 1 & 0 \\ 0 & -10 & -2 & -5 & 0 & 0 & 0 & 1 \\\end{array} \right]\begin{matrix} \ \\ \left| \cdot \left( -1 \right) \right. \\ \left| \cdot \left( -1 \right) \right. \\ \left| \cdot \left( -1 \right) \right. \\\end{matrix}\to \\ & \to \left[ \begin{array}{rrrr|rrrr} 1 & 4 & 2 & 3 & 1 & 0 & 0 & 0 \\ 0 & 6 & 1 & 5 & 1 & -1 & 0 & 0 \\ 0 & 5 & 1 & 2 & 1 & 0 & -1 & 0 \\ 0 & 10 & 2 & 5 & 0 & 0 & 0 & -1 \\\end{array} \right]\begin{matrix} -2 \\ -1 \\ \updownarrow \\ -2 \\\end{matrix}\to \\ & \to \left[ \begin{array}{rrrr|rrrr} 1 & -6 & 0 & -1 & -1 & 0 & 2 & 0 \\ 0 & 1 & 0 & 3 & 0 & -1 & 1 & 0 \\ 0 & 5 & 1 & 2 & 1 & 0 & -1 & 0 \\ 0 & 0 & 0 & 1 & -2 & 0 & 2 & -1 \\\end{array} \right] \\ \end{align}\]

Π’Π΅ΠΏΠ΅Ρ€ΡŒ самоС врСмя Β«ΠΏΠΎΠ΄ΠΆΠ°Ρ€ΠΈΡ‚ΡŒΒ» послСдний столбСц исходной ΠΌΠ°Ρ‚Ρ€ΠΈΡ†Ρ‹: Π²Ρ‹Ρ‡ΠΈΡ‚Π°Π΅ΠΌ строку 4 ΠΈΠ· ΠΎΡΡ‚Π°Π»ΡŒΠ½Ρ‹Ρ…:

\[\begin{align} & \left[ \begin{array}{rrrr|rrrr} 1 & -6 & 0 & -1 & -1 & 0 & 2 & 0 \\ 0 & 1 & 0 & 3 & 0 & -1 & 1 & 0 \\ 0 & 5 & 1 & 2 & 1 & 0 & -1 & 0 \\ 0 & 0 & 0 & 1 & -2 & 0 & 2 & -1 \\\end{array} \right]\begin{matrix} +1 \\ -3 \\ -2 \\ \uparrow \\\end{matrix}\to \\ & \to \left[ \begin{array}{rrrr|rrrr} 1 & -6 & 0 & 0 & -3 & 0 & 4 & -1 \\ 0 & 1 & 0 & 0 & 6 & -1 & -5 & 3 \\ 0 & 5 & 1 & 0 & 5 & 0 & -5 & 2 \\ 0 & 0 & 0 & 1 & -2 & 0 & 2 & -1 \\\end{array} \right] \\ \end{align}\]

Π€ΠΈΠ½Π°Π»ΡŒΠ½Ρ‹ΠΉ бросок: Β«Π²Ρ‹ΠΆΠΈΠ³Π°Π΅ΠΌΒ» Π²Ρ‚ΠΎΡ€ΠΎΠΉ столбСц, вычитая строку 2 ΠΈΠ· строки 1 ΠΈ 3:

\[\begin{align} & \left[ \begin{array}{rrrr|rrrr} 1 & -6 & 0 & 0 & -3 & 0 & 4 & -1 \\ 0 & 1 & 0 & 0 & 6 & -1 & -5 & 3 \\ 0 & 5 & 1 & 0 & 5 & 0 & -5 & 2 \\ 0 & 0 & 0 & 1 & -2 & 0 & 2 & -1 \\\end{array} \right]\begin{matrix} 6 \\ \updownarrow \\ -5 \\ \ \\\end{matrix}\to \\ & \to \left[ \begin{array}{rrrr|rrrr} 1 & 0 & 0 & 0 & 33 & -6 & -26 & -17 \\ 0 & 1 & 0 & 0 & 6 & -1 & -5 & 3 \\ 0 & 0 & 1 & 0 & -25 & 5 & 20 & -13 \\ 0 & 0 & 0 & 1 & -2 & 0 & 2 & -1 \\\end{array} \right] \\ \end{align}\]

И снова слСва Сдиничная ΠΌΠ°Ρ‚Ρ€ΠΈΡ†Π°, Π·Π½Π°Ρ‡ΠΈΡ‚ справа β€” обратная.:)

ΠžΡ‚Π²Π΅Ρ‚. $\left[ \begin{matrix} 33 & -6 & -26 & 17 \\ 6 & -1 & -5 & 3 \\ -25 & 5 & 20 & -13 \\ -2 & 0 & 2 & -1 \\\end{matrix} \right]$

Ну Π²ΠΎΡ‚ ΠΈ всё. ΠŸΡ€ΠΎΠ²Π΅Ρ€ΠΊΡƒ сдСлайтС сами β€” ΠΌΠ½Π΅ Π² Π»ΠΎΠΌ.:)

Π‘ΠΌΠΎΡ‚Ρ€ΠΈΡ‚Π΅ Ρ‚Π°ΠΊΠΆΠ΅:

  1. ΠžΠΏΡ€Π΅Π΄Π΅Π»ΠΈΡ‚Π΅Π»ΡŒ
  2. Π£Π³ΠΎΠ» ΠΌΠ΅ΠΆΠ΄Ρƒ двумя прямыми
  3. ВСст ΠΊ ΡƒΡ€ΠΎΠΊΡƒ Β«Π‘Π»ΠΎΠΆΠ½Ρ‹Π΅ выраТСния с дробями» (Π»Π΅Π³ΠΊΠΈΠΉ)
  4. ΠŸΡ€Π°Π²ΠΈΠ»Π° ΠΊΠΎΠΌΠ±ΠΈΠ½Π°Ρ‚ΠΎΡ€ΠΈΠΊΠΈ Π² Π·Π°Π΄Π°Ρ‡Π΅ B6
  5. ΠŸΡ€ΠΈΠΌΠ΅Ρ€ Ρ€Π΅ΡˆΠ΅Π½ΠΈΡ Π·Π°Π΄Π°Ρ‡ΠΈ 15
  6. ВригономСтрия Π² Π·Π°Π΄Π°Ρ‡Π΅ B15: Ρ€Π΅ΡˆΠ°Π΅ΠΌ Π±Π΅Π· ΠΏΡ€ΠΎΠΈΠ·Π²ΠΎΠ΄Π½Ρ‹Ρ…

TensorFlow – ΠœΠ°Ρ‚Π΅ΠΌΠ°Ρ‚ΠΈΡ‡Π΅ΡΠΊΠΈΠ΅ основы

Π’Π°ΠΆΠ½ΠΎ ΠΏΠΎΠ½ΡΡ‚ΡŒ матСматичСскиС понятия, Π½Π΅ΠΎΠ±Ρ…ΠΎΠ΄ΠΈΠΌΡ‹Π΅ для TensorFlow, ΠΏΡ€Π΅ΠΆΠ΄Π΅ Ρ‡Π΅ΠΌ ΡΠΎΠ·Π΄Π°Π²Π°Ρ‚ΡŒ Π±Π°Π·ΠΎΠ²ΠΎΠ΅ ΠΏΡ€ΠΈΠ»ΠΎΠΆΠ΅Π½ΠΈΠ΅ Π² TensorFlow.Β ΠœΠ°Ρ‚Π΅ΠΌΠ°Ρ‚ΠΈΠΊΠ° считаСтся сСрдцСм любого Π°Π»Π³ΠΎΡ€ΠΈΡ‚ΠΌΠ° машинного обучСния. ИмСнно с ΠΏΠΎΠΌΠΎΡ‰ΡŒΡŽ основных понятий ΠΌΠ°Ρ‚Π΅ΠΌΠ°Ρ‚ΠΈΠΊΠΈ опрСдСляСтся Ρ€Π΅ΡˆΠ΅Π½ΠΈΠ΅ для ΠΊΠΎΠ½ΠΊΡ€Π΅Ρ‚Π½ΠΎΠ³ΠΎ Π°Π»Π³ΠΎΡ€ΠΈΡ‚ΠΌΠ° машинного обучСния.

Β 

Π’Π΅ΠΊΡ‚ΠΎΡ€

Массив чисСл, ΠΊΠΎΡ‚ΠΎΡ€Ρ‹ΠΉ являСтся Π»ΠΈΠ±ΠΎ Π½Π΅ΠΏΡ€Π΅Ρ€Ρ‹Π²Π½Ρ‹ΠΌ, Π»ΠΈΠ±ΠΎ дискрСтным, опрСдСляСтся ΠΊΠ°ΠΊ Π²Π΅ΠΊΡ‚ΠΎΡ€. Алгоритмы машинного обучСния Ρ€Π°Π±ΠΎΡ‚Π°ΡŽΡ‚ с Π²Π΅ΠΊΡ‚ΠΎΡ€Π°ΠΌΠΈ фиксированной Π΄Π»ΠΈΠ½Ρ‹ для Π»ΡƒΡ‡ΡˆΠ΅Π³ΠΎ гСнСрирования Π²Ρ‹Ρ…ΠΎΠ΄Π½Ρ‹Ρ… Π΄Π°Π½Π½Ρ‹Ρ….

Алгоритмы машинного обучСния Ρ€Π°Π±ΠΎΡ‚Π°ΡŽΡ‚ с ΠΌΠ½ΠΎΠ³ΠΎΠΌΠ΅Ρ€Π½Ρ‹ΠΌΠΈ Π΄Π°Π½Π½Ρ‹ΠΌΠΈ, поэтому Π²Π΅ΠΊΡ‚ΠΎΡ€Ρ‹ ΠΈΠ³Ρ€Π°ΡŽΡ‚ Ρ€Π΅ΡˆΠ°ΡŽΡ‰ΡƒΡŽ Ρ€ΠΎΠ»ΡŒ.

ГрафичСскоС прСдставлСниС Π²Π΅ΠΊΡ‚ΠΎΡ€Π½ΠΎΠΉ ΠΌΠΎΠ΄Π΅Π»ΠΈ ΠΏΠΎΠΊΠ°Π·Π°Π½ΠΎ Π½ΠΈΠΆΠ΅:

Β 

Бкаляр

Бкаляр ΠΌΠΎΠΆΠ΅Ρ‚ Π±Ρ‹Ρ‚ΡŒ ΠΎΠΏΡ€Π΅Π΄Π΅Π»Π΅Π½ ΠΊΠ°ΠΊ ΠΎΠ΄Π½ΠΎΠΌΠ΅Ρ€Π½Ρ‹ΠΉ Π²Π΅ΠΊΡ‚ΠΎΡ€. Бкаляры – это Ρ‚Π΅, ΠΊΠΎΡ‚ΠΎΡ€Ρ‹Π΅ Π²ΠΊΠ»ΡŽΡ‡Π°ΡŽΡ‚ Ρ‚ΠΎΠ»ΡŒΠΊΠΎ Π²Π΅Π»ΠΈΡ‡ΠΈΠ½Ρƒ ΠΈ отсутствиС направлСния.Β Π‘ΠΎ скалярами нас интСрСсуСт Ρ‚ΠΎΠ»ΡŒΠΊΠΎ Π²Π΅Π»ΠΈΡ‡ΠΈΠ½Π°.

ΠŸΡ€ΠΈΠΌΠ΅Ρ€Ρ‹ скаляров Π²ΠΊΠ»ΡŽΡ‡Π°ΡŽΡ‚ Π² сСбя ΠΏΠ°Ρ€Π°ΠΌΠ΅Ρ‚Ρ€Ρ‹ вСса ΠΈ роста Π΄Π΅Ρ‚Π΅ΠΉ.

Β 

ΠœΠ°Ρ‚Ρ€ΠΈΡ†Π°

ΠœΠ°Ρ‚Ρ€ΠΈΡ†Π° ΠΌΠΎΠΆΠ΅Ρ‚ Π±Ρ‹Ρ‚ΡŒ ΠΎΠΏΡ€Π΅Π΄Π΅Π»Π΅Π½Π° ΠΊΠ°ΠΊ ΠΌΠ½ΠΎΠ³ΠΎΠΌΠ΅Ρ€Π½Ρ‹Π΅ массивы, ΠΊΠΎΡ‚ΠΎΡ€Ρ‹Π΅ располоТСны Π² Ρ„ΠΎΡ€ΠΌΠ°Ρ‚Π΅ строк ΠΈ столбцов.Β Π Π°Π·ΠΌΠ΅Ρ€ ΠΌΠ°Ρ‚Ρ€ΠΈΡ†Ρ‹ опрСдСляСтся Π΄Π»ΠΈΠ½ΠΎΠΉ строки ΠΈ Π΄Π»ΠΈΠ½ΠΎΠΉ столбца. На ΡΠ»Π΅Π΄ΡƒΡŽΡ‰Π΅ΠΌ рисункС ΠΏΠΎΠΊΠ°Π·Π°Π½ΠΎ прСдставлСниС любой ΡƒΠΊΠ°Π·Π°Π½Π½ΠΎΠΉ ΠΌΠ°Ρ‚Ρ€ΠΈΡ†Ρ‹.

Β 

Рассмотрим ΠΌΠ°Ρ‚Ρ€ΠΈΡ†Ρƒ с Β«mΒ» строками ΠΈ Β«nΒ» столбцами, ΠΊΠ°ΠΊ упомянуто Π²Ρ‹ΡˆΠ΅, прСдставлСниС ΠΌΠ°Ρ‚Ρ€ΠΈΡ†Ρ‹ Π±ΡƒΠ΄Π΅Ρ‚ ΠΎΠΏΡ€Π΅Π΄Π΅Π»Π΅Π½ΠΎ ΠΊΠ°ΠΊ Β«m * n matrixΒ», ΠΊΠΎΡ‚ΠΎΡ€ΠΎΠ΅ Ρ‚Π°ΠΊΠΆΠ΅ ΠΎΠΏΡ€Π΅Π΄Π΅Π»ΠΈΠ»ΠΎ Π΄Π»ΠΈΠ½Ρƒ ΠΌΠ°Ρ‚Ρ€ΠΈΡ†Ρ‹.

Β 

ΠœΠ°Ρ‚Π΅ΠΌΠ°Ρ‚ΠΈΡ‡Π΅ΡΠΊΠΈΠ΅ вычислСния

Π’ этом Ρ€Π°Π·Π΄Π΅Π»Π΅ ΠΌΡ‹ ΡƒΠ·Π½Π°Π΅ΠΌ ΠΎ Ρ€Π°Π·Π»ΠΈΡ‡Π½Ρ‹Ρ… матСматичСских вычислСниях Π² TensorFlow.

Β 

Π”ΠΎΠ±Π°Π²Π»Π΅Π½ΠΈΠ΅ ΠΌΠ°Ρ‚Ρ€ΠΈΡ†

Π”ΠΎΠ±Π°Π²Π»Π΅Π½ΠΈΠ΅ Π΄Π²ΡƒΡ… ΠΈΠ»ΠΈ Π±ΠΎΠ»Π΅Π΅ ΠΌΠ°Ρ‚Ρ€ΠΈΡ† Π²ΠΎΠ·ΠΌΠΎΠΆΠ½ΠΎ, Ссли ΠΌΠ°Ρ‚Ρ€ΠΈΡ†Ρ‹ ΠΈΠΌΠ΅ΡŽΡ‚ ΠΎΠ΄ΠΈΠ½Π°ΠΊΠΎΠ²ΠΎΠ΅ ΠΈΠ·ΠΌΠ΅Ρ€Π΅Π½ΠΈΠ΅.Β Π”ΠΎΠ±Π°Π²Π»Π΅Π½ΠΈΠ΅ ΠΏΠΎΠ΄Ρ€Π°Π·ΡƒΠΌΠ΅Π²Π°Π΅Ρ‚ Π΄ΠΎΠ±Π°Π²Π»Π΅Π½ΠΈΠ΅ ΠΊΠ°ΠΆΠ΄ΠΎΠ³ΠΎ элСмСнта Π² соотвСтствии с Π·Π°Π΄Π°Π½Π½ΠΎΠΉ ΠΏΠΎΠ·ΠΈΡ†ΠΈΠ΅ΠΉ.

Рассмотрим ΡΠ»Π΅Π΄ΡƒΡŽΡ‰ΠΈΠΉ ΠΏΡ€ΠΈΠΌΠ΅Ρ€, Ρ‡Ρ‚ΠΎΠ±Ρ‹ ΠΏΠΎΠ½ΡΡ‚ΡŒ, ΠΊΠ°ΠΊ Ρ€Π°Π±ΠΎΡ‚Π°Π΅Ρ‚ слоТСниС ΠΌΠ°Ρ‚Ρ€ΠΈΡ†:

Π’Ρ‹Ρ‡ΠΈΡ‚Π°Π½ΠΈΠ΅ ΠΌΠ°Ρ‚Ρ€ΠΈΡ†

Π’Ρ‹Ρ‡ΠΈΡ‚Π°Π½ΠΈΠ΅ ΠΌΠ°Ρ‚Ρ€ΠΈΡ† Ρ€Π°Π±ΠΎΡ‚Π°Π΅Ρ‚ Π°Π½Π°Π»ΠΎΠ³ΠΈΡ‡Π½ΠΎ добавлСнию Π΄Π²ΡƒΡ… ΠΌΠ°Ρ‚Ρ€ΠΈΡ†.Β ΠŸΠΎΠ»ΡŒΠ·ΠΎΠ²Π°Ρ‚Π΅Π»ΡŒ ΠΌΠΎΠΆΠ΅Ρ‚ Π²Ρ‹Ρ‡Π΅ΡΡ‚ΡŒ Π΄Π²Π΅ ΠΌΠ°Ρ‚Ρ€ΠΈΡ†Ρ‹ ΠΏΡ€ΠΈ условии, Ρ‡Ρ‚ΠΎ Ρ€Π°Π·ΠΌΠ΅Ρ€Ρ‹ Ρ€Π°Π²Π½Ρ‹.

Β 

Π£ΠΌΠ½ΠΎΠΆΠ΅Π½ΠΈΠ΅ ΠΌΠ°Ρ‚Ρ€ΠΈΡ†

Для Ρ‚ΠΎΠ³ΠΎ Ρ‡Ρ‚ΠΎΠ±Ρ‹ Π΄Π²Π΅ ΠΌΠ°Ρ‚Ρ€ΠΈΡ†Ρ‹ A m * n ΠΈ B p * q Π±Ρ‹Π»ΠΈ ΡƒΠΌΠ½ΠΎΠΆΠ°Π΅ΠΌΡ‹ΠΌΠΈ,Β n Π΄ΠΎΠ»ΠΆΠ½ΠΎ Π±Ρ‹Ρ‚ΡŒ Ρ€Π°Π²Π½ΠΎΒ p.Β ΠŸΠΎΠ»ΡƒΡ‡Π΅Π½Π½Π°Ρ ΠΌΠ°Ρ‚Ρ€ΠΈΡ†Π°:

ВранспонированиС ΠΌΠ°Ρ‚Ρ€ΠΈΡ†Ρ‹

ВранспонированиС ΠΌΠ°Ρ‚Ρ€ΠΈΡ†Ρ‹ A, m * n ΠΎΠ±Ρ‹Ρ‡Π½ΠΎ прСдставляСтся AT (транспонированиС) n * m ΠΈ получаСтся ΠΏΡƒΡ‚Π΅ΠΌ транспонирования Π²Π΅ΠΊΡ‚ΠΎΡ€ΠΎΠ² столбцов Π² качСствС Π²Π΅ΠΊΡ‚ΠΎΡ€ΠΎΠ² строк.

Π’ΠΎΡ‡Π΅Ρ‡Π½ΠΎΠ΅ ΠΏΡ€ΠΎΠΈΠ·Π²Π΅Π΄Π΅Π½ΠΈΠ΅ Π²Π΅ΠΊΡ‚ΠΎΡ€ΠΎΠ²

Π›ΡŽΠ±ΠΎΠΉ Π²Π΅ΠΊΡ‚ΠΎΡ€ размСрности n ΠΌΠΎΠΆΠ½ΠΎ ΠΏΡ€Π΅Π΄ΡΡ‚Π°Π²ΠΈΡ‚ΡŒ Π² Π²ΠΈΠ΄Π΅ ΠΌΠ°Ρ‚Ρ€ΠΈΡ†Ρ‹ v = R ^ n * 1.

Π’ΠΎΡ‡Π΅Ρ‡Π½ΠΎΠ΅ ΠΏΡ€ΠΎΠΈΠ·Π²Π΅Π΄Π΅Π½ΠΈΠ΅ Π΄Π²ΡƒΡ… Π²Π΅ΠΊΡ‚ΠΎΡ€ΠΎΠ² являСтся суммой ΠΏΡ€ΠΎΠΈΠ·Π²Π΅Π΄Π΅Π½ΠΈΠΉ ΡΠΎΠΎΡ‚Π²Π΅Ρ‚ΡΡ‚Π²ΡƒΡŽΡ‰ΠΈΡ… ΠΊΠΎΠΌΠΏΠΎΠ½Π΅Π½Ρ‚ΠΎΠ² – ΠšΠΎΠΌΠΏΠΎΠ½Π΅Π½Ρ‚ΠΎΠ² вдоль ΠΎΠ΄Π½ΠΎΠ³ΠΎ измСрСния ΠΈ ΠΌΠΎΠΆΠ΅Ρ‚ Π±Ρ‹Ρ‚ΡŒ Π²Ρ‹Ρ€Π°ΠΆΠ΅Π½ΠΎ ΠΊΠ°ΠΊ

ΠŸΡ€ΠΈΠΌΠ΅Ρ€ Ρ‚ΠΎΡ‡Π΅Ρ‡Π½ΠΎΠ³ΠΎ произвСдСния Π²Π΅ΠΊΡ‚ΠΎΡ€ΠΎΠ² ΠΏΡ€ΠΈΠ²Π΅Π΄Π΅Π½ Π½ΠΈΠΆΠ΅:

Если Π²Ρ‹ нашли ΠΎΡˆΠΈΠ±ΠΊΡƒ, поТалуйста, Π²Ρ‹Π΄Π΅Π»ΠΈΡ‚Π΅ Ρ„Ρ€Π°Π³ΠΌΠ΅Π½Ρ‚ тСкста ΠΈ Π½Π°ΠΆΠΌΠΈΡ‚Π΅ Ctrl+Enter.

Как ΡΠΊΠ»Π°Π΄Ρ‹Π²Π°Ρ‚ΡŒ ΠΈ Π²Ρ‹Ρ‡ΠΈΡ‚Π°Ρ‚ΡŒ ΠΌΠ°Ρ‚Ρ€ΠΈΡ†Ρ‹

Π‘Π»ΠΎΠΆΠ΅Π½ΠΈΠ΅ ΠΈ Π²Ρ‹Ρ‡ΠΈΡ‚Π°Π½ΠΈΠ΅ ΠΌΠ°Ρ‚Ρ€ΠΈΡ†

На этом этапС нашСго курса Π»ΠΈΠ½Π΅ΠΉΠ½ΠΎΠΉ Π°Π»Π³Π΅Π±Ρ€Ρ‹ ΠΌΡ‹ ΠΌΠ½ΠΎΠ³ΠΎ Ρ€Π°Π±ΠΎΡ‚Π°Π»ΠΈ с ΠΌΠ°Ρ‚Ρ€ΠΈΡ†Π°ΠΌΠΈ, ΠΌΡ‹ выполняли ΠΎΠΏΠ΅Ρ€Π°Ρ†ΠΈΠΈ со строками для Ρ€Π΅ΡˆΠ΅Π½ΠΈΡ систСм Π»ΠΈΠ½Π΅ΠΉΠ½Ρ‹Ρ… ΡƒΡ€Π°Π²Π½Π΅Π½ΠΈΠΉ, прСдставляя Π»ΠΈΠ½Π΅ΠΉΠ½ΡƒΡŽ систСму Π² Π²ΠΈΠ΄Π΅ ΠΌΠ°Ρ‚Ρ€ΠΈΡ†Ρ‹ ΠΈ ΠΈΡΠΏΠΎΠ»ΡŒΠ·ΡƒΡ ΠΈΡ… Π² Π½Π°ΡˆΠΈΡ… интСрСсах посрСдством сокращСния строк ΠΈ ΡΡˆΠ΅Π»ΠΎΠ½ΠΈΡ€ΠΎΠ²Π°Π½Π½Ρ‹Ρ… Ρ„ΠΎΡ€ΠΌ. ΠœΡ‹ Ρ‚Π°ΠΊΠΆΠ΅ Π½Π°ΡƒΡ‡ΠΈΠ»ΠΈΡΡŒ ΠΏΠΎΠ½ΠΈΠΌΠ°Ρ‚ΡŒ Π½Π΅ Ρ‚ΠΎΠ»ΡŒΠΊΠΎ обозначСния ΠΌΠ°Ρ‚Ρ€ΠΈΡ†, Π½ΠΎ ΠΈ ΠΈΡ… элСмСнты, ΠΈΡ… Ρ€Π°Π·ΠΌΠ΅Ρ€Ρ‹, ΠΈ Ρ‚Π΅ΠΏΠ΅Ρ€ΡŒ ΠΏΠΎΡ€Π° ΠΏΠΎΠ΄ΡƒΠΌΠ°Ρ‚ΡŒ ΠΎ Π½ΠΈΡ… Π² Ρ†Π΅Π»ΠΎΠΌ ΠΈ ΡƒΠ·Π½Π°Ρ‚ΡŒ, ΠΊΠ°ΠΊΠΈΠ΅ ΠΎΠΏΠ΅Ρ€Π°Ρ†ΠΈΠΈ ΠΌΠΎΠ³ΡƒΡ‚ Π±Ρ‹Ρ‚ΡŒ Π²Ρ‹ΠΏΠΎΠ»Π½Π΅Π½Ρ‹ с Π½ΠΈΠΌΠΈ (ΠΎΠ±Ρ€Π°Ρ‚ΠΈΡ‚Π΅ Π²Π½ΠΈΠΌΠ°Π½ΠΈΠ΅: «с Π½ΠΈΠΌΠΈΒ», НЕ Β«Π² ΠΈΡ…”).

Π’ этом ΡƒΡ€ΠΎΠΊΠ΅ ΠΌΡ‹ познакомимся с ΠΏΠ΅Ρ€Π²Ρ‹ΠΌΠΈ двумя ΠΈΠ· Ρ‡Π΅Ρ‚Ρ‹Ρ€Π΅Ρ… основных ΠΎΠΏΠ΅Ρ€Π°Ρ†ΠΈΠΉ, ΠΊΠΎΡ‚ΠΎΡ€Ρ‹Π΅ ΠΌΠΎΠΆΠ½ΠΎ Π²Ρ‹ΠΏΠΎΠ»Π½ΠΈΡ‚ΡŒ с ΠΏΠ°Ρ€ΠΎΠΉ (ΠΈΠ»ΠΈ Π±ΠΎΠ»Π΅Π΅) ΠΌΠ°Ρ‚Ρ€ΠΈΡ†: слоТСниС, Π²Ρ‹Ρ‡ΠΈΡ‚Π°Π½ΠΈΠ΅, скалярноС ΡƒΠΌΠ½ΠΎΠΆΠ΅Π½ΠΈΠ΅ ΠΈ ΡƒΠΌΠ½ΠΎΠΆΠ΅Π½ΠΈΠ΅. ΠœΡ‹ Π·Π½Π°Π΅ΠΌ эти Π±Π°Π·ΠΎΠ²Ρ‹Π΅ ΠΎΠΏΠ΅Ρ€Π°Ρ†ΠΈΠΈ ΠΈΠ· Π°Ρ€ΠΈΡ„ΠΌΠ΅Ρ‚ΠΈΠΊΠΈ, поэтому Π½Π°ΠΌ просто Π½ΡƒΠΆΠ½ΠΎ ΡƒΠ·Π½Π°Ρ‚ΡŒ, ΠΊΠ°ΠΊ ΠΎΠ½ΠΈ Ρ€Π°Π±ΠΎΡ‚Π°ΡŽΡ‚ со всСй структурой ΠΌΠ°Ρ‚Ρ€ΠΈΡ† ΠΈ с Ρ€Π΅Π·ΡƒΠ»ΡŒΡ‚Π°Ρ‚Π°ΠΌΠΈ, ΠΊΠΎΡ‚ΠΎΡ€Ρ‹Π΅ ΠΎΠ½ΠΈ производят. ΠžΠ±Ρ€Π°Ρ‚ΠΈΡ‚Π΅ Π²Π½ΠΈΠΌΠ°Π½ΠΈΠ΅, Ρ‡Ρ‚ΠΎ ΠΌΡ‹ Π½Π΅ упомянули Π΄Π΅Π»Π΅Π½ΠΈΠ΅ ΠΌΠ°Ρ‚Ρ€ΠΈΡ†, ΠΏΠΎΡ‚ΠΎΠΌΡƒ Ρ‡Ρ‚ΠΎ дСлСния ΠΌΠ΅ΠΆΠ΄Ρƒ ΠΌΠ°Ρ‚Ρ€ΠΈΡ†Π°ΠΌΠΈ Π½Π΅ сущСствуСт (ΠΈΠ»ΠΈ, ΠΏΠΎ ΠΊΡ€Π°ΠΉΠ½Π΅ΠΉ ΠΌΠ΅Ρ€Π΅, процСсс Π΅Ρ‰Π΅ Π½Π΅ ΠΎΠΏΡ€Π΅Π΄Π΅Π»Π΅Π½), СдинствСнноС Π΄Π΅Π»Π΅Π½ΠΈΠ΅, ΠΊΠΎΡ‚ΠΎΡ€ΠΎΠ΅ ΠΌΡ‹ ΠΌΠΎΠΆΠ΅ΠΌ ΡΠ΄Π΅Π»Π°Ρ‚ΡŒ, – это Ρ€Π°Π·Π΄Π΅Π»ΠΈΡ‚ΡŒ элСмСнты Π² ΠΌΠ°Ρ‚Ρ€ΠΈΡ†Π΅ Π½Π° скаляр, Π½ΠΎ Ρ€Π°Π·Π΄Π΅Π»ΠΈΡ‚ΡŒ ΠΌΠ°Ρ‚Ρ€ΠΈΡ†Ρƒ Π½Π° Π΄Ρ€ΡƒΠ³ΡƒΡŽ ΠΌΠ°Ρ‚Ρ€ΠΈΡ†Ρƒ Π½Π΅Π²ΠΎΠ·ΠΌΠΎΠΆΠ½ΠΎ.

Π‘Π»ΠΎΠΆΠ΅Π½ΠΈΠ΅ ΠΈ Π²Ρ‹Ρ‡ΠΈΡ‚Π°Π½ΠΈΠ΅ ΠΌΠ°Ρ‚Ρ€ΠΈΡ†

ΠœΡ‹ Π½Π°Ρ‡ΠΈΠ½Π°Π΅ΠΌ с Ρ€Π°Π±ΠΎΡ‚Ρ‹ с двумя основными опСрациями, извСстными Π½Π°ΠΌ с Ρ‚Π΅Ρ… ΠΏΠΎΡ€, ΠΊΠ°ΠΊ ΠΌΡ‹ Π²ΠΏΠ΅Ρ€Π²Ρ‹Π΅ познакомились с ΠΌΠ°Ρ‚Π΅ΠΌΠ°Ρ‚ΠΈΠΊΠΎΠΉ Π² β€‹β€‹Π½Π°Ρ‡Π°Π»ΡŒΠ½ΠΎΠΉ школС: слоТСниС ΠΈ Π²Ρ‹Ρ‡ΠΈΡ‚Π°Π½ΠΈΠ΅ (ΠΈΠ»ΠΈ Ρ‡Ρ‚ΠΎ-Ρ‚ΠΎ «плюс» ΠΈ «минус»). Π§Ρ‚ΠΎΠ±Ρ‹ ΠΏΡ€ΠΎΡ€Π°Π±ΠΎΡ‚Π°Ρ‚ΡŒ эти ΠΎΠΏΠ΅Ρ€Π°Ρ†ΠΈΠΈ Π² этом ΡƒΡ€ΠΎΠΊΠ΅, Π΄Π°Π²Π°ΠΉΡ‚Π΅ ΠΎΠΏΡ€Π΅Π΄Π΅Π»ΠΈΠΌ Π½Π°Π±ΠΎΡ€ ΠΈΠ· Π΄Π²ΡƒΡ… ΠΌΠ°Ρ‚Ρ€ΠΈΡ† для слоТСния ΠΈ вычитания. Π˜Ρ‚Π°ΠΊ, опрСдСляСм ΠΌΠ°Ρ‚Ρ€ΠΈΡ†Ρ‹ A ΠΈ B ΡΠ»Π΅Π΄ΡƒΡŽΡ‰ΠΈΠΌ ΠΎΠ±Ρ€Π°Π·ΠΎΠΌ:

Π£Ρ€Π°Π²Π½Π΅Π½ΠΈΠ΅ 1: ΠΌΠ°Ρ‚Ρ€ΠΈΡ†Ρ‹ A ΠΈ B

Π’Π΅ΠΏΠ΅Ρ€ΡŒ с Π½ΠΈΠΌΠΈ ΠΌΡ‹ настроСны для Ρ€Π°Π±ΠΎΡ‚Ρ‹ Ρ‡Π΅Ρ€Π΅Π· ΠΎΠΏΠ΅Ρ€Π°Ρ†ΠΈΠΈ слоТСния ΠΈ вычитания ΠΌΠ°Ρ‚Ρ€ΠΈΡ†.К ΠΎΠΏΠ΅Ρ€Π°Ρ†ΠΈΠΈ Π±ΡƒΠ΄ΡƒΡ‚ ΠΏΡ€ΠΈΠΌΠ΅Π½ΡΡ‚ΡŒΡΡ нСсколько ΠΏΡ€Π°Π²ΠΈΠ», Π½ΠΎ ΠΌΡ‹ ΠΏΠΎΠ³ΠΎΠ²ΠΎΡ€ΠΈΠΌ ΠΎ Π½ΠΈΡ… Π² ΡΠ»Π΅Π΄ΡƒΡŽΡ‰Π΅ΠΌ Ρ€Π°Π·Π΄Π΅Π»Π΅, Π° ΠΏΠΎΠΊΠ° Π΄Π°Π²Π°ΠΉΡ‚Π΅ быстро пройдСмся ΠΏΠΎ процСссу, Π° Π·Π°Ρ‚Π΅ΠΌ ΠΌΡ‹ смоТСм Π²Π΅Ρ€Π½ΡƒΡ‚ΡŒΡΡ ΠΈ ΠΏΡ€ΠΎΠ²Π΅Ρ€ΠΈΡ‚ΡŒ, насколько ΠΎΠΏΠ΅Ρ€Π°Ρ†ΠΈΠΈ Π±Ρ‹Π»ΠΈ Π²ΠΎΠ·ΠΌΠΎΠΆΠ½Ρ‹. НачнСм с добавлСния A ΠΈ B:

Π£Ρ€Π°Π²Π½Π΅Π½ΠΈΠ΅ 2: слоТСниС ΠΌΠ°Ρ‚Ρ€ΠΈΡ† A ΠΈ B

ΠœΡ‹ Π·Π°ΠΊΠΎΠ΄ΠΈΡ€ΠΎΠ²Π°Π»ΠΈ ΠΊΠ°ΠΆΠ΄Ρ‹ΠΉ элСмСнт Π² ΠΌΠ°Ρ‚Ρ€ΠΈΡ†Π°Ρ… Ρ†Π²Π΅Ρ‚ΠΎΠΌ, Ρ‡Ρ‚ΠΎΠ±Ρ‹ Π²Ρ‹ ΠΌΠΎΠ³Π»ΠΈ Π²ΠΈΠ΄Π΅Ρ‚ΡŒ, Ρ‡Ρ‚ΠΎ Π±Ρ‹Π»ΠΎ сдСлано для ΠΊΠ°ΠΆΠ΄ΠΎΠ³ΠΎ элСмСнта. ΠžΠ±Ρ€Π°Ρ‚ΠΈΡ‚Π΅ Π²Π½ΠΈΠΌΠ°Π½ΠΈΠ΅ Π½Π° Ρ‚ΠΎ, ΠΊΠ°ΠΊ слоТСниС Π΄Π²ΡƒΡ… ΠΌΠ°Ρ‚Ρ€ΠΈΡ† дСйствуСт Π² случаС ΠΎΡ‚Π΄Π΅Π»ΡŒΠ½Ρ‹Ρ… элСмСнтов, трСбуя, Ρ‡Ρ‚ΠΎΠ±Ρ‹ Ρƒ вас Π±Ρ‹Π»ΠΈ Ρ‚ΠΎΡ‡Π½ΠΎ Ρ‚Π°ΠΊΠΈΠ΅ ΠΆΠ΅ элСмСнты Π² ΠΌΠ°Ρ‚Ρ€ΠΈΡ†Π°Ρ…, ΡƒΡ‡Π°ΡΡ‚Π²ΡƒΡŽΡ‰ΠΈΡ… Π² ΠΎΠΏΠ΅Ρ€Π°Ρ†ΠΈΠΈ, ΠΈ Ρ‡Ρ‚ΠΎΠ±Ρ‹ ΠΎΠ½ΠΈ Ρ€Π°ΡΠΏΡ€Π΅Π΄Π΅Π»ΡΠ»ΠΈΡΡŒ ΠΎΠ΄ΠΈΠ½Π°ΠΊΠΎΠ²Ρ‹ΠΌ ΠΎΠ±Ρ€Π°Π·ΠΎΠΌ.

ΠœΡ‹ ΠΏΠΎΠ²Ρ‚ΠΎΡ€Π½ΠΎ рассмотрим эту Ρ‚Π΅ΠΌΡƒ Π² ΡΠ»Π΅Π΄ΡƒΡŽΡ‰Π΅ΠΌ Ρ€Π°Π·Π΄Π΅Π»Π΅, Π½ΠΎ ΠΏΠΎΠΊΠ° Π²Ρ‹ Π΄ΠΎΠ»ΠΆΠ½Ρ‹ ΠΏΠΎΠΌΠ½ΠΈΡ‚ΡŒ, Ρ‡Ρ‚ΠΎ ΠΏΡ€ΠΈ Π΄ΠΎΠ±Π°Π²Π»Π΅Π½ΠΈΠΈ ΠΌΠ°Ρ‚Ρ€ΠΈΡ† Π²Ρ‹ Π±ΡƒΠ΄Π΅Ρ‚Π΅ Π΄ΠΎΠ±Π°Π²Π»ΡΡ‚ΡŒ элСмСнты ΠΏΠ΅Ρ€Π²ΠΎΠΉ ΠΌΠ°Ρ‚Ρ€ΠΈΡ†Ρ‹ ΠΊ ΡΠΎΠΎΡ‚Π²Π΅Ρ‚ΡΡ‚Π²ΡƒΡŽΡ‰Π΅ΠΉ (Ρ‚ΠΎΠΉ, которая находится Π² Β«Ρ‚ΠΎΠΌ ΠΆΠ΅ мСстС», Ρ‚ΠΎΠΉ ΠΆΠ΅ строкС ΠΈ столбцС число) Π²ΠΎ Π²Ρ‚ΠΎΡ€ΠΎΠΉ ΠΌΠ°Ρ‚Ρ€ΠΈΡ†Π΅.

Π’Π΅ΠΏΠ΅Ρ€ΡŒ Π΄Π°Π²Π°ΠΉΡ‚Π΅ посмотрим Π½Π° ΠΌΠ°Ρ‚Ρ€ΠΈΡ‡Π½ΠΎΠ΅ Π²Ρ‹Ρ‡ΠΈΡ‚Π°Π½ΠΈΠ΅:

Π£Ρ€Π°Π²Π½Π΅Π½ΠΈΠ΅ 3: Π²Ρ‹Ρ‡ΠΈΡ‚Π°Π½ΠΈΠ΅ ΠΌΠ°Ρ‚Ρ€ΠΈΡ†Ρ‹ B ΠΈΠ· A

Глядя Π½Π° ΠΏΡ€ΠΈΠ²Π΅Π΄Π΅Π½Π½ΠΎΠ΅ Π²Ρ‹ΡˆΠ΅ Π²Ρ‹Ρ‡ΠΈΡ‚Π°Π½ΠΈΠ΅, ΠΌΡ‹ ΠΌΠΎΠΆΠ΅ΠΌ Π»Π΅Π³ΠΊΠΎ Π·Π°ΠΊΠ»ΡŽΡ‡ΠΈΡ‚ΡŒ, Ρ‡Ρ‚ΠΎ этот процСсс ΠΎΡ‡Π΅Π½ΡŒ ΠΏΠΎΡ…ΠΎΠΆ Π½Π° процСсс слоТСния, ΠΏΠΎΡΠΊΠΎΠ»ΡŒΠΊΡƒ Π²Ρ‹Ρ‡ΠΈΡ‚Π°Π½ΠΈΠ΅ вычисляСтся ΠΏΠΎ ΠΎΡ‚Π΄Π΅Π»ΡŒΠ½ΠΎΡΡ‚ΠΈ для ΠΊΠ°ΠΆΠ΄ΠΎΠ³ΠΎ ΠΈΠ· элСмСнтов ΠΌΠ°Ρ‚Ρ€ΠΈΡ†, Ρ‚Π°ΠΊΠΈΠΌ ΠΎΠ±Ρ€Π°Π·ΠΎΠΌ давая Ρ€Π΅Π·ΡƒΠ»ΡŒΡ‚Π°Ρ‚ для ΠΊΠ°ΠΆΠ΄ΠΎΠ³ΠΎ ΡΠΎΠΎΡ‚Π²Π΅Ρ‚ΡΡ‚Π²ΡƒΡŽΡ‰Π΅Π³ΠΎ пространства элСмСнтов ΠΈ Π² Ρ€Π΅Π·ΡƒΠ»ΡŒΡ‚Π°Ρ‚Π΅ получаСтся ΠΌΠ°Ρ‚Ρ€ΠΈΡ†Π° с Ρ‚Π°ΠΊΠΈΠΌ ΠΆΠ΅ количСством элСмСнтов Π² Ρ‚ΠΎΠΉ ΠΆΠ΅ ΠΎΡ€Π³Π°Π½ΠΈΠ·Π°Ρ†ΠΈΠΈ.

ΠŸΡ€ΠΎΡ‰Π΅ говоря, слоТСниС ΠΈ Π²Ρ‹Ρ‡ΠΈΡ‚Π°Π½ΠΈΠ΅ ΠΌΠ°Ρ‚Ρ€ΠΈΡ† Ρ€Π°Π±ΠΎΡ‚Π°ΡŽΡ‚ ΠΎΠ΄ΠΈΠ½Π°ΠΊΠΎΠ²ΠΎ, СдинствСнная Ρ€Π°Π·Π½ΠΈΡ†Π° ΠΌΠ΅ΠΆΠ΄Ρƒ Π½ΠΈΠΌΠΈ – это арифмСтичСский Π·Π½Π°ΠΊ, ΠΊΠΎΡ‚ΠΎΡ€Ρ‹ΠΉ Π²Ρ‹ ΠΈΡΠΏΠΎΠ»ΡŒΠ·ΡƒΠ΅Ρ‚Π΅ для Ρ€Π°Π±ΠΎΡ‚Ρ‹: Π·Π½Π°ΠΊ плюс для слоТСния ΠΈ Π·Π½Π°ΠΊ минус для вычитания.

ΠšΡ€Π°ΠΉΠ½Π΅ Π²Π°ΠΆΠ½ΠΎ, Ρ‡Ρ‚ΠΎΠ±Ρ‹ Π²Ρ‹ поняли этот Π²Ρ‹Π²ΠΎΠ΄, ΠΏΠΎΡΠΊΠΎΠ»ΡŒΠΊΡƒ это ΠΌΠΎΠΆΠ΅Ρ‚ ΠΏΠΎΠΌΠΎΡ‡ΡŒ Π²Π°ΠΌ Π½Π΅ Π·Π°ΠΏΡƒΡ‚Π°Ρ‚ΡŒΡΡ ΠΏΡ€ΠΈ Ρ€Π°Π±ΠΎΡ‚Π΅ с Ρ€Π°Π·Π»ΠΈΡ‡Π½ΠΎΠΉ Π»ΠΈΡ‚Π΅Ρ€Π°Ρ‚ΡƒΡ€ΠΎΠΉ ΠΏΠΎ Π΄Π°Π½Π½ΠΎΠΉ Ρ‚Π΅ΠΌΠ΅. Π”Π΅Π»ΠΎ Π² Ρ‚ΠΎΠΌ, Ρ‡Ρ‚ΠΎ Π½Π΅ΠΊΠΎΡ‚ΠΎΡ€Ρ‹Π΅ ΠΌΠ°Ρ‚Π΅ΠΌΠ°Ρ‚ΠΈΠΊΠΈ, ΠΊΠ½ΠΈΠ³ΠΈ ΠΈΠ»ΠΈ ΡƒΡ‡Π΅Π±Π½Ρ‹Π΅ ΠΌΠ°Ρ‚Π΅Ρ€ΠΈΠ°Π»Ρ‹ Π² Ρ†Π΅Π»ΠΎΠΌ ΠΌΠΎΠ³ΡƒΡ‚ ΠΎΡ‚Π½ΠΎΡΠΈΡ‚ΡŒΡΡ ΠΊ опСрациям слоТСния ΠΈ вычитания ΠΊΠ°ΠΊ ΠΊ «слоТСнию» Π² Ρ†Π΅Π»ΠΎΠΌ.Π­Ρ‚Π° конкрСтная ссылка исходит ΠΈΠ· Ρ‚ΠΎΠ³ΠΎ Ρ„Π°ΠΊΡ‚Π°, Ρ‡Ρ‚ΠΎ Π²Ρ‹Ρ‡ΠΈΡ‚Π°Π½ΠΈΠ΅ эквивалСнтно слоТСнию чисСл, Π² ΠΊΠΎΡ‚ΠΎΡ€Ρ‹Ρ… хотя Π±Ρ‹ ΠΎΠ΄Π½ΠΎ ΠΈΠ· Π½ΠΈΡ… ΠΎΡ‚Ρ€ΠΈΡ†Π°Ρ‚Π΅Π»ΡŒΠ½ΠΎ. Если Ρƒ вас Π΅ΡΡ‚ΡŒ ΠΊΠ°ΠΊΠΈΠ΅-Π»ΠΈΠ±ΠΎ вопросы ΠΏΠΎ этому ΠΏΠΎΠ²ΠΎΠ΄Ρƒ, ΠΏΠΎΠΌΠ½ΠΈΡ‚Π΅, Ρ‡Ρ‚ΠΎ Π²Ρ‹Ρ‡ΠΈΡ‚Π°Π½ΠΈΠ΅ Π² Π°Ρ€ΠΈΡ„ΠΌΠ΅Ρ‚ΠΈΠΊΠ΅ Ρ‚Π°ΠΊΠΆΠ΅ называСтся Β«Ρ€Π°Π·Π½ΠΈΡ†Π΅ΠΉΒ», ΠΈ всС ΠΏΠΎΡ‚ΠΎΠΌΡƒ, Ρ‡Ρ‚ΠΎ ΠΊΠΎΠ³Π΄Π° Π²Ρ‹ складываСтС числа с Ρ€Π°Π·Π½Ρ‹ΠΌΠΈ Π·Π½Π°ΠΊΠ°ΠΌΠΈ, Ρ€Π΅Π·ΡƒΠ»ΡŒΡ‚Π°Ρ‚ΠΎΠΌ становится Ρ€Π°Π·Π½ΠΈΡ†Π° ΠΌΠ΅ΠΆΠ΄Ρƒ Π½ΠΈΠΌΠΈ. ΠœΡ‹ рассмотрСли эту Ρ‚Π΅ΠΌΡƒ Π² нашСм ΡƒΡ€ΠΎΠΊΠ΅ ΠΏΠΎ Π²Ρ‹Ρ‡ΠΈΡ‚Π°Π½ΠΈΡŽ Ρ†Π΅Π»Ρ‹Ρ… чисСл, ΠΊΠΎΡ‚ΠΎΡ€Ρ‹ΠΉ, хотя ΠΈ Π½Π΅ являСтся Ρ‡Π°ΡΡ‚ΡŒΡŽ этого курса Π»ΠΈΠ½Π΅ΠΉΠ½ΠΎΠΉ Π°Π»Π³Π΅Π±Ρ€Ρ‹, Π²Ρ‹ ΠΌΠΎΠΆΠ΅Ρ‚Π΅ ΠΏΡ€ΠΎΠ²Π΅Ρ€ΠΈΡ‚ΡŒ Π² любоС врСмя Π½Π° Π²Π΅Π±-сайтС StudyPug.Π’Ρ‹ Ρ‚Π°ΠΊΠΆΠ΅ ΠΌΠΎΠΆΠ΅Ρ‚Π΅ ΠΏΡ€ΠΎΠ²Π΅Ρ€ΠΈΡ‚ΡŒ ΡΠ»Π΅Π΄ΡƒΡŽΡ‰ΡƒΡŽ ΡΡ‚Π°Ρ‚ΡŒΡŽ ΠΎ слоТСнии ΠΈ Π²Ρ‹Ρ‡ΠΈΡ‚Π°Π½ΠΈΠΈ Ρ†Π΅Π»Ρ‹Ρ… чисСл, которая содСрТит Π΄ΠΎΠΏΠΎΠ»Π½ΠΈΡ‚Π΅Π»ΡŒΠ½Ρ‹Π΅ упраТнСния ΠΈ Π½Π΅ΠΊΠΎΡ‚ΠΎΡ€Ρ‹Π΅ графичСскиС изобраТСния.

Π’ΠΊΡ€Π°Ρ‚Ρ†Π΅: Как ΡΠΊΠ»Π°Π΄Ρ‹Π²Π°Ρ‚ΡŒ ΠΌΠ°Ρ‚Ρ€ΠΈΡ†Ρ‹? ΠœΡ‹ добавляСм ΠΊΠ°ΠΆΠ΄Ρ‹ΠΉ ΡΠΎΠΎΡ‚Π²Π΅Ρ‚ΡΡ‚Π²ΡƒΡŽΡ‰ΠΈΠΉ элСмСнт Π² задСйствованныС ΠΌΠ°Ρ‚Ρ€ΠΈΡ†Ρ‹, Ρ‡Ρ‚ΠΎΠ±Ρ‹ ΡΠΎΠ·Π΄Π°Ρ‚ΡŒ Π½ΠΎΠ²ΡƒΡŽ ΠΌΠ°Ρ‚Ρ€ΠΈΡ†Ρƒ, Π² ΠΊΠΎΡ‚ΠΎΡ€ΠΎΠΉ Ρ‚Π°ΠΊΠΈΠ΅ элСмСнты Π±ΡƒΠ΄ΡƒΡ‚ Π·Π°Π½ΠΈΠΌΠ°Ρ‚ΡŒ Ρ‚ΠΎ ΠΆΠ΅ мСсто, Ρ‡Ρ‚ΠΎ ΠΈ ΠΈΡ… ΠΏΡ€Π΅Π΄ΡˆΠ΅ΡΡ‚Π²Π΅Π½Π½ΠΈΠΊΠΈ. Как Π²Ρ‹Ρ‡Π΅ΡΡ‚ΡŒ ΠΌΠ°Ρ‚Ρ€ΠΈΡ†Ρ‹? Π’ΠΎΡ‡Π½ΠΎ Ρ‚Π°ΠΊ ΠΆΠ΅, ΠΊΠ°ΠΊ ΠΌΡ‹ вычисляСм слоТСниС, Π·Π° ΠΈΡΠΊΠ»ΡŽΡ‡Π΅Π½ΠΈΠ΅ΠΌ Ρ‚ΠΎΠ³ΠΎ, Ρ‡Ρ‚ΠΎ ΠΌΡ‹ ΠΈΡΠΏΠΎΠ»ΡŒΠ·ΡƒΠ΅ΠΌ Π·Π½Π°ΠΊ минус для Ρ€Π°Π±ΠΎΡ‚Ρ‹ вмСсто Π·Π½Π°ΠΊΠ° плюс.

ΠŸΡ€Π°Π²ΠΈΠ»Π° слоТСния ΠΈ вычитания ΠΌΠ°Ρ‚Ρ€ΠΈΡ†

ΠœΠ°Ρ‚Ρ€ΠΈΡ‡Π½ΠΎΠ΅ слоТСниС ΠΈ Π²Ρ‹Ρ‡ΠΈΡ‚Π°Π½ΠΈΠ΅ – это Π΄Π²Π΅ ΠΎΠΏΠ΅Ρ€Π°Ρ†ΠΈΠΈ, ΠΊΠΎΡ‚ΠΎΡ€Ρ‹Π΅, хотя ΠΈ нСслоТны ΠΈ просты для Ρ€Π΅ΡˆΠ΅Π½ΠΈΡ (ΠΊΠ°ΠΊ ΠΏΠΎΠΊΠ°Π·Π°Π½ΠΎ Π²Ρ‹ΡˆΠ΅), Ρƒ вас всС ΠΆΠ΅ Π΅ΡΡ‚ΡŒ нСсколько ΠΏΡ€Π°Π²ΠΈΠ», ΠΊΠΎΡ‚ΠΎΡ€Ρ‹ΠΌ Π½ΡƒΠΆΠ½ΠΎ ΡΠ»Π΅Π΄ΠΎΠ²Π°Ρ‚ΡŒ, Ρ‡Ρ‚ΠΎΠ±Ρ‹ Ρ€Π°Π±ΠΎΡ‚Π°Ρ‚ΡŒ.

НапримСр, ΠΊΠΎΠ³Π΄Π° ΠΌΡ‹ добавляли ΠΌΠ°Ρ‚Ρ€ΠΈΡ†Ρƒ A ΠΊ ΠΌΠ°Ρ‚Ρ€ΠΈΡ†Π΅ B Π²Ρ‹ΡˆΠ΅, сначала ΠΎΠ±Ρ€Π°Ρ‚ΠΈΡ‚Π΅ Π²Π½ΠΈΠΌΠ°Π½ΠΈΠ΅, Ρ‡Ρ‚ΠΎ A ΠΈ B – ΠΎΡ‡Π΅Π½ΡŒ ΠΏΠΎΡ…ΠΎΠΆΠΈΠ΅ ΠΌΠ°Ρ‚Ρ€ΠΈΡ†Ρ‹, ΠΎΠ½ΠΈ ΠΈΠΌΠ΅ΡŽΡ‚ ΠΎΠ΄ΠΈΠ½Π°ΠΊΠΎΠ²ΠΎΠ΅ количСство строк ΠΈ столбцов каТдая ΠΈ опСрация слоТСния примСняСтся ΠΊ ΠΊΠ°ΠΆΠ΄ΠΎΠΌΡƒ Β«ΡΠΎΠΎΡ‚Π²Π΅Ρ‚ΡΡ‚Π²ΡƒΡŽΡ‰Π΅ΠΌΡƒ Ρ‡Π»Π΅Π½ΡƒΒ» Π² Π½ΠΈΡ…. Π­Ρ‚ΠΎ ΠΏΡ€ΠΈΠ²ΠΎΠ΄ΠΈΡ‚ ΠΊ основному ΠΏΡ€Π°Π²ΠΈΠ»Ρƒ слоТСния ΠΈ вычитания ΠΌΠ°Ρ‚Ρ€ΠΈΡ†, ΠΊΠΎΡ‚ΠΎΡ€ΠΎΠ΅ соотвСтствуСт Ρ€Π°Π·ΠΌΠ΅Ρ€Π°ΠΌ ΠΌΠ°Ρ‚Ρ€ΠΈΡ†, с ΠΊΠΎΡ‚ΠΎΡ€Ρ‹ΠΌΠΈ Ρ€Π°Π±ΠΎΡ‚Π°ΡŽΡ‚ Π΄Ρ€ΡƒΠ³ с Π΄Ρ€ΡƒΠ³ΠΎΠΌ.

Π’Π°ΠΊΠΆΠ΅ ΠΎΠ±Ρ€Π°Ρ‚ΠΈΡ‚Π΅ Π²Π½ΠΈΠΌΠ°Π½ΠΈΠ΅, Ρ‡Ρ‚ΠΎ Π²Ρ‹Ρ‡ΠΈΡ‚Π°Π½ΠΈΠ΅ ΠΌΠ°Ρ‚Ρ€ΠΈΡ† происходит Π² ΠΎΡ‡Π΅Π½ΡŒ ΠΏΠΎΡ…ΠΎΠΆΠ΅ΠΌ Ρ€Π΅ΠΆΠΈΠΌΠ΅, Π½Π° самом Π΄Π΅Π»Π΅ СдинствСнноС ΠΈΠ·ΠΌΠ΅Π½Π΅Π½ΠΈΠ΅ состоит Π² Ρ‚ΠΎΠΌ, Ρ‡Ρ‚ΠΎ вмСсто Π·Π½Π°ΠΊΠ° слоТСния ΠΌΠ΅ΠΆΠ΄Ρƒ ΠΌΠ°Ρ‚Ρ€ΠΈΡ†Π°ΠΌΠΈ ΠΈ ΡΠΎΠΎΡ‚Π²Π΅Ρ‚ΡΡ‚Π²ΡƒΡŽΡ‰ΠΈΠΌΠΈ элСмСнтами Π² ΠΊΠ°ΠΆΠ΄ΠΎΠΌ ΠΌΠ°Ρ‚Ρ€ΠΈΡ‡Π½ΠΎΠΌ пространствС Ρƒ вас Π΅ΡΡ‚ΡŒ Π·Π½Π°ΠΊ вычитания.

Π‘ ΡƒΡ‡Π΅Ρ‚ΠΎΠΌ всСго этого ΠΏΡ€Π°Π²ΠΈΠ»Π° слоТСния ΠΈ вычитания ΠΌΠ°Ρ‚Ρ€ΠΈΡ† ΡΠ»Π΅Π΄ΡƒΡŽΡ‰ΠΈΠ΅:

  1. Π’Ρ‹ ΠΌΠΎΠΆΠ΅Ρ‚Π΅ ΡΠΊΠ»Π°Π΄Ρ‹Π²Π°Ρ‚ΡŒ ΠΈΠ»ΠΈ Π²Ρ‹Ρ‡ΠΈΡ‚Π°Ρ‚ΡŒ Ρ‚ΠΎΠ»ΡŒΠΊΠΎ ΠΌΠ°Ρ‚Ρ€ΠΈΡ†Ρ‹ с ΠΎΠ΄ΠΈΠ½Π°ΠΊΠΎΠ²Ρ‹ΠΌΠΈ Ρ€Π°Π·ΠΌΠ΅Ρ€Π°ΠΌΠΈ.
    1. Если Π·Π°Π΄ΡƒΠΌΠ°Ρ‚ΡŒΡΡ, это происходит ΠΈΠ·-Π·Π° Ρ‚ΠΎΠ³ΠΎ, Ρ‡Ρ‚ΠΎ Π΄ΠΎΠ±Π°Π²Π»Π΅Π½ΠΈΠ΅ ΠΌΠ°Ρ‚Ρ€ΠΈΡ† с Ρ€Π°Π·Π½Ρ‹ΠΌΠΈ Ρ€Π°Π·ΠΌΠ΅Ρ€Π°ΠΌΠΈ создаСт ΠΏΡ€ΠΎΠ±Π»Π΅ΠΌΡƒ, ΠΏΠΎΡ‚ΠΎΠΌΡƒ Ρ‡Ρ‚ΠΎ Π½Π΅ всС элСмСнты Π² ΠΊΠ°ΠΆΠ΄ΠΎΠΉ ΠΌΠ°Ρ‚Ρ€ΠΈΡ†Π΅ Π±ΡƒΠ΄ΡƒΡ‚ ΠΈΠΌΠ΅Ρ‚ΡŒ ΡΠΎΠΎΡ‚Π²Π΅Ρ‚ΡΡ‚Π²ΡƒΡŽΡ‰ΠΈΠΉ элСмСнт для Ρ€Π°Π±ΠΎΡ‚Ρ‹, ΠΈ, ΡΠ»Π΅Π΄ΠΎΠ²Π°Ρ‚Π΅Π»ΡŒΠ½ΠΎ, ΠΎΠΏΠ΅Ρ€Π°Ρ†ΠΈΡŽ Π½Π΅Π²ΠΎΠ·ΠΌΠΎΠΆΠ½ΠΎ Π·Π°Π²Π΅Ρ€ΡˆΠΈΡ‚ΡŒ.
  2. Π‘Π»ΠΎΠΆΠ΅Π½ΠΈΠ΅ ΠΌΠ°Ρ‚Ρ€ΠΈΡ† являСтся ΠΊΠΎΠΌΠΌΡƒΡ‚Π°Ρ‚ΠΈΠ²Π½Ρ‹ΠΌ, Π½ΠΎ Π²Ρ‹Ρ‡ΠΈΡ‚Π°Π½ΠΈΠ΅ – Π½Π΅Ρ‚, Ссли сначала Π²Ρ‹ Π½Π΅ ΠΏΡ€ΠΈΠΌΠ΅Π½ΠΈΡ‚Π΅ Π·Π½Π°ΠΊ минуса ΠΊ ΠΌΠ°Ρ‚Ρ€ΠΈΡ†Π΅ справа, Ρ‚Π°ΠΊΠΈΠΌ ΠΎΠ±Ρ€Π°Π·ΠΎΠΌ сначала прСобразуя ΠΎΠΏΠ΅Ρ€Π°Ρ†ΠΈΡŽ Π² слоТСниС.
    1. Бвойство коммутативности ΠΎΠ·Π½Π°Ρ‡Π°Π΅Ρ‚, Ρ‡Ρ‚ΠΎ ΠΏΡ€ΠΈ Π΄ΠΎΠ±Π°Π²Π»Π΅Π½ΠΈΠΈ Π΄Π²ΡƒΡ… ΠΌΠ°Ρ‚Ρ€ΠΈΡ† Π²Ρ‹ ΠΌΠΎΠΆΠ΅Ρ‚Π΅ ΠΈΠ·ΠΌΠ΅Π½ΠΈΡ‚ΡŒ порядок ΠΈΡ… добавлСния, ΠΈ это Π½Π΅ повлияСт Π½Π° Ρ€Π΅Π·ΡƒΠ»ΡŒΡ‚Π°Ρ‚.
    2. Π­Ρ‚ΠΎ Π½Π΅ Ρ‚Π°ΠΊ просто ΠΏΡ€ΠΈ Π²Ρ‹Ρ‡ΠΈΡ‚Π°Π½ΠΈΠΈ. Π’ ΡƒΡ€Π°Π²Π½Π΅Π½ΠΈΠΈ 3 Ρƒ нас Π΅ΡΡ‚ΡŒ ΠΏΡ€ΠΈΠΌΠ΅Ρ€ вычитания ΠΌΠ°Ρ‚Ρ€ΠΈΡ†, ΠΎΠ±Ρ€Π°Ρ‚ΠΈΡ‚Π΅ Π²Π½ΠΈΠΌΠ°Π½ΠΈΠ΅, Ρ‡Ρ‚ΠΎ Ссли Π²Ρ‹ просто ΠΈΠ·ΠΌΠ΅Π½ΠΈΡ‚Π΅ порядок ΠΌΠ°Ρ‚Ρ€ΠΈΡ†, Π½Π΅ пСрСмСщая Π·Π½Π°ΠΊ минус, Ρ€Π΅Π·ΡƒΠ»ΡŒΡ‚Π°Ρ‚ Π½Π΅ Π±ΡƒΠ΄Π΅Ρ‚ Ρ‚Π°ΠΊΠΈΠΌ ΠΆΠ΅, Ρ‚ΠΎΠ³Π΄Π° ΠΊΠ°ΠΊ Π²Ρ‹Ρ‡ΠΈΡ‚Π°Π½ΠΈΠ΅ ΠΌΠ°Ρ‚Ρ€ΠΈΡ† ΠΌΠΎΠΆΠ΅Ρ‚ Π±Ρ‹Ρ‚ΡŒ ΠΊΠΎΠΌΠΌΡƒΡ‚Π°Ρ‚ΠΈΠ²Π½Ρ‹ΠΌ? Π’Ρ‹ Π΄ΠΎΠ»ΠΆΠ½Ρ‹ ΠΏΡ€ΠΈΠΌΠ΅Π½ΠΈΡ‚ΡŒ ΠΎΡ‚Ρ€ΠΈΡ†Π°Ρ‚Π΅Π»ΡŒΠ½Ρ‹ΠΉ Π·Π½Π°ΠΊ ΠΊ ΠΌΠ°Ρ‚Ρ€ΠΈΡ†Π΅, которая вычитаСтся ΠΏΠ΅Ρ€Π²ΠΎΠΉ! ΠŸΡ€ΠΎΠ²Π΅Ρ€ΡŒΡ‚Π΅ процСсс шаг Π·Π° шагом Π½ΠΈΠΆΠ΅: Π£Ρ€Π°Π²Π½Π΅Π½ΠΈΠ΅ 4: УсловиС, Ρ‡Ρ‚ΠΎΠ±Ρ‹ ΡΠ΄Π΅Π»Π°Ρ‚ΡŒ Π²Ρ‹Ρ‡ΠΈΡ‚Π°Π½ΠΈΠ΅ ΠΌΠ°Ρ‚Ρ€ΠΈΡ†Ρ‹ ΠΊΠΎΠΌΠΌΡƒΡ‚Π°Ρ‚ΠΈΠ²Π½Ρ‹ΠΌ = сначала ΠΏΡ€Π΅ΠΎΠ±Ρ€Π°Π·ΠΎΠ²Π°Ρ‚ΡŒ Π΅Π³ΠΎ Π² слоТСниС! ΠžΠ±Ρ€Π°Ρ‚ΠΈΡ‚Π΅ Π²Π½ΠΈΠΌΠ°Π½ΠΈΠ΅, ΠΊΠ°ΠΊ для Ρ‚ΠΎΠ³ΠΎ, Ρ‡Ρ‚ΠΎΠ±Ρ‹ ΠΈΠΌΠ΅Ρ‚ΡŒ Π²ΠΎΠ·ΠΌΠΎΠΆΠ½ΠΎΡΡ‚ΡŒ ΠΈΠ·ΠΌΠ΅Π½ΡΡ‚ΡŒ порядок ΠΌΠ°Ρ‚Ρ€ΠΈΡ†, Π½Π΅ влияя Π½Π° Ρ€Π΅Π·ΡƒΠ»ΡŒΡ‚Π°Ρ‚, ΠΌΡ‹ Π΄ΠΎΠ»ΠΆΠ½Ρ‹ ΠΏΡ€Π΅ΠΎΠ±Ρ€Π°Π·ΠΎΠ²Π°Ρ‚ΡŒ Π²Ρ‹Ρ‡ΠΈΡ‚Π°Π½ΠΈΠ΅ Π² слоТСниС, Ρ‡Ρ‚ΠΎ просто ΠΎΠ·Π½Π°Ρ‡Π°Π΅Ρ‚, Ρ‡Ρ‚ΠΎ Π²Ρ‹ Π΄ΠΎΠ»ΠΆΠ½Ρ‹ сначала ΠΏΡ€ΠΈΠΌΠ΅Π½ΠΈΡ‚ΡŒ ΠΎΡ‚Ρ€ΠΈΡ†Π°Ρ‚Π΅Π»ΡŒΠ½Ρ‹ΠΉ Π·Π½Π°ΠΊ ΠΊΠΎ Π²Ρ‚ΠΎΡ€ΠΎΠΉ ΠΌΠ°Ρ‚Ρ€ΠΈΡ†Π΅, Ρ‚Π΅ΠΌ самым сдСлав всС ΠΎΡ‚Ρ€ΠΈΡ†Π°Ρ‚Π΅Π»ΡŒΠ½Ρ‹Π΅ числа Π΅Π³ΠΎ элСмСнтов (ΠΊΠΎΡ‚ΠΎΡ€Ρ‹Π΅ ΠΏΡ€ΠΎΠΈΠ·Π²Π΅Π΄ΡƒΡ‚ ΠΎΠΏΠ΅Ρ€Π°Ρ†ΠΈΡŽ вычитания, Π½Π΅ бСспокоясь ΠΎ порядкС элСмСнтов).Π­Ρ‚ΠΎ СдинствСнный способ ΠΈΠ·ΠΌΠ΅Π½ΠΈΡ‚ΡŒ порядок ΠΌΠ°Ρ‚Ρ€ΠΈΡ† ΠΏΡ€ΠΈ Π²Ρ‹Ρ‡ΠΈΡ‚Π°Π½ΠΈΠΈ, поэтому ΠΏΠΎΠΌΠ½ΠΈΡ‚Π΅ ΠΎΠ± этом процСссС ΠΏΡ€ΠΈ Ρ€Π°Π±ΠΎΡ‚Π΅ с Π΅Ρ‰Π΅ Π±ΠΎΠ»Π΅Π΅ слоТными ΠΏΡ€ΠΎΠ±Π»Π΅ΠΌΠ°ΠΌΠΈ: Π½Π°ΠΏΡ€ΠΈΠΌΠ΅Ρ€, Π² случаях, ΠΊΠΎΠ³Π΄Π° Ρƒ вас ΠΌΠΎΠΆΠ΅Ρ‚ Π±Ρ‹Ρ‚ΡŒ Π±ΠΎΠ»Π΅Π΅ Π΄Π²ΡƒΡ… ΠΌΠ°Ρ‚Ρ€ΠΈΡ† ΠΈ Π²Ρ‹ ΡΠ²ΡΠ·Ρ‹Π²Π°Π΅Ρ‚Π΅ΡΡŒ Π² Π³Ρ€ΡƒΠΏΠΏΠ°Ρ…. ΠŸΠΎΠ΄Ρ€ΠΎΠ±Π½Π΅Π΅ ΠΎΠ± ассоциации Π² ΡΠ»Π΅Π΄ΡƒΡŽΡ‰Π΅ΠΌ ΠΏΡƒΠ½ΠΊΡ‚Π΅.
  3. ΠœΠ°Ρ‚Ρ€ΠΈΡ‡Π½ΠΎΠ΅ слоТСниС ΠΈ Π²Ρ‹Ρ‡ΠΈΡ‚Π°Π½ΠΈΠ΅ ассоциативны.
    1. Бвойство ассоциативности ΠΈΡΠΏΠΎΠ»ΡŒΠ·ΡƒΠ΅Ρ‚ΡΡ Π² Π·Π°Π΄Π°Ρ‡Π°Ρ…, Π²ΠΊΠ»ΡŽΡ‡Π°ΡŽΡ‰ΠΈΡ… Π±ΠΎΠ»Π΅Π΅ Π΄Π²ΡƒΡ… ΠΌΠ°Ρ‚Ρ€ΠΈΡ†, ΠΊΠΎΡ‚ΠΎΡ€Ρ‹Π΅ ΡΠΊΠ»Π°Π΄Ρ‹Π²Π°ΡŽΡ‚ΡΡ ΠΈΠ»ΠΈ Π²Ρ‹Ρ‡ΠΈΡ‚Π°ΡŽΡ‚ΡΡ Π΄Ρ€ΡƒΠ³ ΠΈΠ· Π΄Ρ€ΡƒΠ³Π°.Π­Ρ‚ΠΎ свойство позволяСт Π²Π°ΠΌ сначала Ρ€Π°Π±ΠΎΡ‚Π°Ρ‚ΡŒ с ΠΎΠΏΠ΅Ρ€Π°Ρ†ΠΈΠ΅ΠΉ ΠΌΠ΅ΠΆΠ΄Ρƒ двумя ΠΌΠ°Ρ‚Ρ€ΠΈΡ†Π°ΠΌΠΈ, Π° Π·Π°Ρ‚Π΅ΠΌ ΠΈΡΠΏΠΎΠ»ΡŒΠ·ΠΎΠ²Π°Ρ‚ΡŒ ΠΌΠ°Ρ‚Ρ€ΠΈΡ†Ρƒ, ΠΏΠΎΠ»ΡƒΡ‡Π΅Π½Π½ΡƒΡŽ Π² Ρ€Π΅Π·ΡƒΠ»ΡŒΡ‚Π°Ρ‚Π΅ Ρ‚Π°ΠΊΠΎΠΉ ΠΎΠΏΠ΅Ρ€Π°Ρ†ΠΈΠΈ, для выполнСния Π²Ρ‚ΠΎΡ€ΠΎΠΉ ΠΎΠΏΠ΅Ρ€Π°Ρ†ΠΈΠΈ ΠΏΠΎΠ΄ Ρ€ΡƒΠΊΠΎΠΉ. ΠŸΠΎΠ²Ρ‚ΠΎΡ€ΡΠΉΡ‚Π΅ Ρ‚Π°ΠΊΠΎΠΉ процСсс Π΄ΠΎ Ρ‚Π΅Ρ… ΠΏΠΎΡ€, ΠΏΠΎΠΊΠ° Π½Π΅ Π²Ρ‹ΠΏΠΎΠ»Π½ΠΈΡ‚Π΅ всС Π½Π΅ΠΎΠ±Ρ…ΠΎΠ΄ΠΈΠΌΡ‹Π΅ ΠΎΠΏΠ΅Ρ€Π°Ρ†ΠΈΠΈ с участиСм всСх ΠΌΠ°Ρ‚Ρ€ΠΈΡ† Π² Π·Π°Π΄Π°Ρ‡Π΅.
  4. ΠŸΡ€ΠΈ слоТСнии ΠΈΠ»ΠΈ Π²Ρ‹Ρ‡ΠΈΡ‚Π°Π½ΠΈΠΈ ΠΌΠ°Ρ‚Ρ€ΠΈΡ† Ρ€Π΅Π·ΡƒΠ»ΡŒΡ‚ΠΈΡ€ΡƒΡŽΡ‰Π°Ρ ΠΌΠ°Ρ‚Ρ€ΠΈΡ†Π° Π±ΡƒΠ΄Π΅Ρ‚ ΠΈΠΌΠ΅Ρ‚ΡŒ Ρ‚Π΅ ΠΆΠ΅ Ρ€Π°Π·ΠΌΠ΅Ρ€Ρ‹, Ρ‡Ρ‚ΠΎ ΠΈ ΠΌΠ°Ρ‚Ρ€ΠΈΡ†Π°, которая Π΅Π΅ создала.
    1. Π­Ρ‚ΠΎ ΠΈΠΌΠ΅Π΅Ρ‚ смысл, ΠΏΠΎΡΠΊΠΎΠ»ΡŒΠΊΡƒ ΠΊΠ°ΠΆΠ΄Ρ‹ΠΉ элСмСнт Π² ΠΏΠ΅Ρ€Π²ΠΎΠΉ ΠΌΠ°Ρ‚Ρ€ΠΈΡ†Π΅ Π±ΡƒΠ΄Π΅Ρ‚ Ρ€Π°Π±ΠΎΡ‚Π°Ρ‚ΡŒ с ΡΠΎΠΎΡ‚Π²Π΅Ρ‚ΡΡ‚Π²ΡƒΡŽΡ‰ΠΈΠΌ элСмСнтом Π²Ρ‚ΠΎΡ€ΠΎΠΉ ΠΌΠ°Ρ‚Ρ€ΠΈΡ†Ρ‹, Ρ‚Π°ΠΊΠΈΠΌ ΠΎΠ±Ρ€Π°Π·ΠΎΠΌ производя Ρ€Π΅Π·ΡƒΠ»ΡŒΡ‚Π°Ρ‚ для ΠΊΠ°ΠΆΠ΄ΠΎΠ³ΠΎ пространства элСмСнтов.Π—Π°Ρ‚Π΅ΠΌ Ρ‚Π°ΠΊΠΈΠ΅ Ρ€Π΅Π·ΡƒΠ»ΡŒΡ‚Π°Ρ‚Ρ‹ станут элСмСнтами Π½ΠΎΠ²ΠΎΠΉ ΠΌΠ°Ρ‚Ρ€ΠΈΡ†Ρ‹.

ΠŸΡ€ΠΈΠΌΠ΅Ρ€Ρ‹ слоТСния ΠΈ вычитания

Π”Π°Π»Π΅Π΅ ΠΌΡ‹ ΠΏΠΎΠΊΠ°ΠΆΠ΅ΠΌ ΠΏΡ€ΠΈΠΌΠ΅Ρ€Ρ‹ слоТСния ΠΈ вычитания ΠΌΠ°Ρ‚Ρ€ΠΈΡ†. Π’ ΠΏΠ΅Ρ€Π²ΠΎΠΌ ΠΏΡ€ΠΈΠΌΠ΅Ρ€Π΅ упраТнСния ΠΌΡ‹ сосрСдоточимся Π½Π° дСмонстрации условий, Π² ΠΊΠΎΡ‚ΠΎΡ€Ρ‹Ρ… Ρ‚Π°ΠΊΠΈΠ΅ ΠΎΠΏΠ΅Ρ€Π°Ρ†ΠΈΠΈ Π²ΠΎΠ·ΠΌΠΎΠΆΠ½Ρ‹. Π—Π°Ρ‚Π΅ΠΌ ΠΌΡ‹ Ρ€Π΅ΡˆΠ°Π΅ΠΌ нСсколько ΠΏΡ€ΠΈΠΌΠ΅Ρ€ΠΎΠ² ΠΌΠ°Ρ‚Ρ€ΠΈΡ‡Π½Ρ‹Ρ… Π·Π°Π΄Π°Ρ‡ с ΠΏΠΎΡˆΠ°Π³ΠΎΠ²Ρ‹ΠΌ объяснСниСм ΠΎΠΏΠ΅Ρ€Π°Ρ†ΠΈΠΉ. ΠœΡ‹ Π·Π°ΠΊΠ°Π½Ρ‡ΠΈΠ²Π°Π΅ΠΌ этот Ρ€Π°Π·Π΄Π΅Π» Ρ€Π΅ΡˆΠ΅Π½ΠΈΠ΅ΠΌ ΠΌΠ°Ρ‚Ρ€ΠΈΡ‡Π½Ρ‹Ρ… ΡƒΡ€Π°Π²Π½Π΅Π½ΠΈΠΉ, ΠΊΠΎΡ‚ΠΎΡ€Ρ‹Π΅ Π²ΠΊΠ»ΡŽΡ‡Π°ΡŽΡ‚ Π»ΠΈΠ±ΠΎ слоТСниС, Π»ΠΈΠ±ΠΎ Π²Ρ‹Ρ‡ΠΈΡ‚Π°Π½ΠΈΠ΅ ΠΌΠ°Ρ‚Ρ€ΠΈΡ†.

ΠŸΡ€ΠΈΠΌΠ΅Ρ€ 1

МоТСм Π»ΠΈ ΠΌΡ‹ ΡΠ»ΠΎΠΆΠΈΡ‚ΡŒ ΠΈΠ»ΠΈ Π²Ρ‹Ρ‡Π΅ΡΡ‚ΡŒ ΡΠ»Π΅Π΄ΡƒΡŽΡ‰ΠΈΠ΅ ΠΌΠ°Ρ‚Ρ€ΠΈΡ†Ρ‹?

  • Π‘Π»ΡƒΡ‡Π°ΠΉ 1: Π£Ρ€Π°Π²Π½Π΅Π½ΠΈΠ΅ 5: ΠœΠΎΠΆΠ΅Ρ‚Π΅ Π»ΠΈ Π²Ρ‹ ΡΠ»ΠΎΠΆΠΈΡ‚ΡŒ эти Π΄Π²Π΅ ΠΌΠ°Ρ‚Ρ€ΠΈΡ†Ρ‹ вмСстС? (Π”Π΅Π»ΠΎ 1) ΠžΠ±Ρ€Π°Ρ‚ΠΈΡ‚Π΅ Π²Π½ΠΈΠΌΠ°Π½ΠΈΠ΅, Ρ‡Ρ‚ΠΎ Ρ€Π°Π·ΠΌΠ΅Ρ€Ρ‹ ΠΌΠ°Ρ‚Ρ€ΠΈΡ†, ΡƒΡ‡Π°ΡΡ‚Π²ΡƒΡŽΡ‰ΠΈΡ… Π² слоТСнии, Π½Π΅ ΡΠΎΠ²ΠΏΠ°Π΄Π°ΡŽΡ‚. ΠŸΠ΅Ρ€Π²Π°Ρ ΠΌΠ°Ρ‚Ρ€ΠΈΡ†Π° ΠΈΠΌΠ΅Π΅Ρ‚ Ρ‚Ρ€ΠΈ строки ΠΈ Π΄Π²Π° столбца, Ρ‚Π°ΠΊΠΈΠΌ ΠΎΠ±Ρ€Π°Π·ΠΎΠΌ, это ΠΌΠ°Ρ‚Ρ€ΠΈΡ†Π° 3×2, Π° вторая содСрТит Π΄Π²Π΅ строки ΠΈ Ρ‚Ρ€ΠΈ столбца, Ρ‡Ρ‚ΠΎ Π΄Π΅Π»Π°Π΅Ρ‚ Π΅Π΅ Ρ€Π°Π·ΠΌΠ΅Ρ€ Ρ€Π°Π²Π½Ρ‹ΠΌ 2×3. НаличиС Ρ‚Π°ΠΊΠΈΡ… Ρ€Π°Π·Π»ΠΈΡ‡ΠΈΠΉ ΠΎΠ·Π½Π°Ρ‡Π°Π΅Ρ‚, Ρ‡Ρ‚ΠΎ, хотя ΠΎΠ½ΠΈ ΠΈΠΌΠ΅ΡŽΡ‚ ΠΎΠ΄ΠΈΠ½Π°ΠΊΠΎΠ²ΠΎΠ΅ количСство элСмСнтов, ΠΈΡ… ΠΏΠΎΠ·ΠΈΡ†ΠΈΠΈ Π½Π΅ ΡΠΎΠΎΡ‚Π²Π΅Ρ‚ΡΡ‚Π²ΡƒΡŽΡ‚ Π΄Ρ€ΡƒΠ³ Π΄Ρ€ΡƒΠ³Ρƒ, ΠΈ поэтому ΠΏΠΎ ΠΏΡ€Π°Π²ΠΈΠ»Π°ΠΌ слоТСния ΠΌΠ°Ρ‚Ρ€ΠΈΡ† слоТСниС Π½Π΅Π²ΠΎΠ·ΠΌΠΎΠΆΠ½ΠΎ.
  • Π‘Π»ΡƒΡ‡Π°ΠΉ 2: Π£Ρ€Π°Π²Π½Π΅Π½ΠΈΠ΅ 6: ΠœΠΎΠΆΠ΅Ρ‚Π΅ Π»ΠΈ Π²Ρ‹ ΡΠ»ΠΎΠΆΠΈΡ‚ΡŒ эти Π΄Π²Π΅ ΠΌΠ°Ρ‚Ρ€ΠΈΡ†Ρ‹ вмСстС? (случай 2) Учитывая, Ρ‡Ρ‚ΠΎ Π² этом случаС ΠΌΡ‹ Π±ΡƒΠ΄Π΅ΠΌ Π΄ΠΎΠ±Π°Π²Π»ΡΡ‚ΡŒ ΠΌΠ°Ρ‚Ρ€ΠΈΡ†Ρ‹ 2×2 (явно; ΠΎΠ±Π΅ ΠΌΠ°Ρ‚Ρ€ΠΈΡ†Ρ‹ ΠΈΠΌΠ΅ΡŽΡ‚ ΠΎΠ΄ΠΈΠ½Π°ΠΊΠΎΠ²ΡƒΡŽ Ρ€Π°Π·ΠΌΠ΅Ρ€Π½ΠΎΡΡ‚ΡŒ 2×2), Ρ‚ΠΎΠ³Π΄Π° Π΄Π°, Π΄ΠΎΠ±Π°Π²Π»Π΅Π½ΠΈΠ΅ этих Π΄Π²ΡƒΡ… ΠΌΠ°Ρ‚Ρ€ΠΈΡ† Π²ΠΎΠ·ΠΌΠΎΠΆΠ½ΠΎ ΠΏΡƒΡ‚Π΅ΠΌ добавлСния ΠΊΠ°ΠΆΠ΄ΠΎΠ³ΠΎ ΠΈΠ· элСмСнтов Π² ΠΏΠ΅Ρ€Π²ΠΎΠΉ ΠΌΠ°Ρ‚Ρ€ΠΈΡ†Π΅ с ΡΠΎΠΎΡ‚Π²Π΅Ρ‚ΡΡ‚Π²ΡƒΡŽΡ‰ΠΈΠΌ Π΅ΠΌΡƒ элСмСнтом ΠΈΠ· вторая ΠΌΠ°Ρ‚Ρ€ΠΈΡ†Π°. Π Π΅Π·ΡƒΠ»ΡŒΡ‚ΠΈΡ€ΡƒΡŽΡ‰Π°Ρ ΠΌΠ°Ρ‚Ρ€ΠΈΡ†Π° Ρ‚Π°ΠΊΠΆΠ΅ Π±ΡƒΠ΄Π΅Ρ‚ ΠΈΠΌΠ΅Ρ‚ΡŒ Ρ€Π°Π·ΠΌΠ΅Ρ€Ρ‹ 2×2.
  • Π‘Π»ΡƒΡ‡Π°ΠΉ 3: Π£Ρ€Π°Π²Π½Π΅Π½ΠΈΠ΅ 7: ΠœΠΎΠΆΠ΅Ρ‚Π΅ Π»ΠΈ Π²Ρ‹ ΡΠ»ΠΎΠΆΠΈΡ‚ΡŒ эти Π΄Π²Π΅ ΠΌΠ°Ρ‚Ρ€ΠΈΡ†Ρ‹ вмСстС? (случай 3) Как Π²Ρ‹, Π²ΠΎΠ·ΠΌΠΎΠΆΠ½ΠΎ, ΡƒΠΆΠ΅ ΠΏΠΎΠ΄ΡƒΠΌΠ°Π»ΠΈ, Π΄ΠΎΠ±Π°Π²Π»Π΅Π½ΠΈΠ΅ Π΄Π²ΡƒΡ… ΠΌΠ°Ρ‚Ρ€ΠΈΡ†, ΠΏΠΎΠ΄ΠΎΠ±Π½Ρ‹Ρ… ΠΏΡ€ΠΈΠ²Π΅Π΄Π΅Π½Π½Ρ‹ΠΌ Π²Ρ‹ΡˆΠ΅, Π½Π΅Π²ΠΎΠ·ΠΌΠΎΠΆΠ½ΠΎ.Π Π°Π·ΠΌΠ΅Ρ€Ρ‹ этих ΠΌΠ°Ρ‚Ρ€ΠΈΡ† Π½Π΅ ΡΠΎΠ²ΠΏΠ°Π΄Π°ΡŽΡ‚ (ΠΎΠ΄Π½Π° – ΠΌΠ°Ρ‚Ρ€ΠΈΡ†Π° 3×3, Π° другая – 2×2), поэтому это Π΄ΠΎΠ±Π°Π²Π»Π΅Π½ΠΈΠ΅ Π½Π΅Π²ΠΎΠ·ΠΌΠΎΠΆΠ½ΠΎ.

ΠŸΡ€ΠΈΠΌΠ΅Ρ€ 2

Π‘ΠΊΠ»Π°Π΄Ρ‹Π²Π°Π΅ΠΌ ΡΠ»Π΅Π΄ΡƒΡŽΡ‰ΠΈΠ΅ Π΄Π²Π΅ ΠΌΠ°Ρ‚Ρ€ΠΈΡ†Ρ‹:

Π£Ρ€Π°Π²Π½Π΅Π½ΠΈΠ΅ 8: слоТСниС Π΄Π²ΡƒΡ… ΠΌΠ°Ρ‚Ρ€ΠΈΡ†

ΠŸΡ€ΠΈ Π΄ΠΎΠ±Π°Π²Π»Π΅Π½ΠΈΠΈ Π΄Π²ΡƒΡ… ΠΌΠ°Ρ‚Ρ€ΠΈΡ† ΠΊΠ°ΠΆΠ΄Ρ‹ΠΉ элСмСнт добавляСтся ΠΊ ΡΠΎΠΎΡ‚Π²Π΅Ρ‚ΡΡ‚Π²ΡƒΡŽΡ‰Π΅ΠΌΡƒ элСмСнту Π² Π΄Ρ€ΡƒΠ³ΠΎΠΉ ΠΌΠ°Ρ‚Ρ€ΠΈΡ†Π΅. ΠŸΡ€ΠΎΡ†Π΅ΡΡ прост ΠΈ ΠΏΠΎΠΊΠ°Π·Π°Π½ Π½ΠΈΠΆΠ΅:

Π£Ρ€Π°Π²Π½Π΅Π½ΠΈΠ΅ 9: РСшСниС для слоТСния Π΄Π²ΡƒΡ… ΠΌΠ°Ρ‚Ρ€ΠΈΡ†

Π’Ρ‹ΠΏΠΎΠ»Π½ΠΈΡ‚Π΅ Ρ‚Π΅ ΠΆΠ΅ дСйствия Π² ΡΠ»Π΅Π΄ΡƒΡŽΡ‰ΠΈΡ… Π΄Π²ΡƒΡ… упраТнСниях.

ΠŸΡ€ΠΈΠΌΠ΅Ρ€ 3

Π‘ΠΊΠ»Π°Π΄Ρ‹Π²Π°Π΅ΠΌ ΡΠ»Π΅Π΄ΡƒΡŽΡ‰ΠΈΠ΅ Π΄Π²Π΅ ΠΌΠ°Ρ‚Ρ€ΠΈΡ†Ρ‹:

Π£Ρ€Π°Π²Π½Π΅Π½ΠΈΠ΅ 10: слоТСниС Π΄Π²ΡƒΡ… ΠΌΠ°Ρ‚Ρ€ΠΈΡ†

Π˜Ρ‚Π°ΠΊ, ΠΌΡ‹ добавляСм ΠΊΠ°ΠΆΠ΄Ρ‹ΠΉ элСмСнт Π² ΠΌΠ°Ρ‚Ρ€ΠΈΡ†Π°Ρ… ΠΊ ΡΠΎΠΎΡ‚Π²Π΅Ρ‚ΡΡ‚Π²ΡƒΡŽΡ‰Π΅ΠΌΡƒ элСмСнту Π² Π΄Ρ€ΡƒΠ³ΠΎΠΉ ΠΌΠ°Ρ‚Ρ€ΠΈΡ†Π΅. ΠžΠ±Ρ€Π°Ρ‚ΠΈΡ‚Π΅ Π²Π½ΠΈΠΌΠ°Π½ΠΈΠ΅, Ρ‡Ρ‚ΠΎ, ΠΊΠ°ΠΊ ΡƒΠΏΠΎΠΌΠΈΠ½Π°Π»ΠΎΡΡŒ Ρ€Π°Π½Π΅Π΅, ΠΌΠ°Ρ‚Ρ€ΠΈΡ†Ρ‹ Π΄ΠΎΠ»ΠΆΠ½Ρ‹ ΠΈΠΌΠ΅Ρ‚ΡŒ ΠΎΠ΄ΠΈΠ½Π°ΠΊΠΎΠ²Ρ‹Π΅ Ρ€Π°Π·ΠΌΠ΅Ρ€Ρ‹, Ρ‡Ρ‚ΠΎΠ±Ρ‹ ΠΏΡ€ΠΎΠΈΠ·ΠΎΡˆΠ»ΠΎ поэлСмСнтноС соотвСтствиС:

Π£Ρ€Π°Π²Π½Π΅Π½ΠΈΠ΅ 11: РСшСниС для слоТСния Π΄Π²ΡƒΡ… ΠΌΠ°Ρ‚Ρ€ΠΈΡ†

ΠŸΡ€ΠΈΠΌΠ΅Ρ€ 4

Π‘ΠΊΠ»Π°Π΄Ρ‹Π²Π°Π΅ΠΌ ΡΠ»Π΅Π΄ΡƒΡŽΡ‰ΠΈΠ΅ Π΄Π²Π΅ ΠΌΠ°Ρ‚Ρ€ΠΈΡ†Ρ‹:

Π£Ρ€Π°Π²Π½Π΅Π½ΠΈΠ΅ 12: слоТСниС Ρ‚Ρ€Π΅Ρ… ΠΌΠ°Ρ‚Ρ€ΠΈΡ†

ΠŸΡ€ΠΈ Π΄ΠΎΠ±Π°Π²Π»Π΅Π½ΠΈΠΈ Π±ΠΎΠ»Π΅Π΅ Π΄Π²ΡƒΡ… ΠΌΠ°Ρ‚Ρ€ΠΈΡ† Π²Ρ‹ ΠΌΠΎΠΆΠ΅Ρ‚Π΅ ΡΠ²ΡΠ·Π°Ρ‚ΡŒ Π΄Π²Π΅, Π²Ρ‹ΠΏΠΎΠ»Π½ΠΈΠ² сначала ΠΈΡ… слоТСниС, Π° Π·Π°Ρ‚Π΅ΠΌ Π΄ΠΎΠ±Π°Π²ΠΈΠ² ΠΏΠΎΠ»ΡƒΡ‡Π΅Π½Π½ΡƒΡŽ ΠΌΠ°Ρ‚Ρ€ΠΈΡ†Ρƒ ΠΊ ΠΎΡΡ‚Π°Π²ΡˆΠ΅ΠΉΡΡ ΠΌΠ°Ρ‚Ρ€ΠΈΡ†Π΅.Π­Ρ‚ΠΎ Ρ‚ΠΎ, Ρ‡Ρ‚ΠΎ ΠΌΡ‹ Π½Π°Π·Ρ‹Π²Π°Π΅ΠΌ ассоциативным свойством слоТСния ΠΌΠ°Ρ‚Ρ€ΠΈΡ†, это ΠΈ Π΄Ρ€ΡƒΠ³ΠΈΠ΅ свойства слоТСния ΠΌΠ°Ρ‚Ρ€ΠΈΡ† Π±ΡƒΠ΄ΡƒΡ‚ ΠΎΠ±ΡŠΡΡΠ½Π΅Π½Ρ‹ Π² ΠΏΠΎΡΠ»Π΅Π΄ΡƒΡŽΡ‰ΠΈΡ… ΡƒΡ€ΠΎΠΊΠ°Ρ…, Π° ΠΏΠΎΠΊΠ° ΠΏΡ€ΠΎΠ²Π΅Ρ€ΡŒΡ‚Π΅ процСсс Ρ€Π΅ΡˆΠ΅Π½ΠΈΡ этой ΠΏΡ€ΠΎΠ±Π»Π΅ΠΌΡ‹ Π½ΠΈΠΆΠ΅:

Π£Ρ€Π°Π²Π½Π΅Π½ΠΈΠ΅ 13: РСшСниС для слоТСния Ρ‚Ρ€Π΅Ρ… ΠΌΠ°Ρ‚Ρ€ΠΈΡ†

Π˜Ρ‚Π°ΠΊ, ΠΊΠ°ΠΊ Π²Ρ‹ ΠΌΠΎΠΆΠ΅Ρ‚Π΅ Π²ΠΈΠ΄Π΅Ρ‚ΡŒ, ΠΏΠ΅Ρ€Π²Ρ‹ΠΌ шагом Π±Ρ‹Π»ΠΎ Π΄ΠΎΠ±Π°Π²Π»Π΅Π½ΠΈΠ΅ ΠΌΠ°Ρ‚Ρ€ΠΈΡ† ΠΎΠ΄ΠΈΠ½ ΠΈ Π΄Π²Π°, Ρ‡Ρ‚ΠΎ ΠΏΡ€ΠΈΠ²Π΅Π»ΠΎ ΠΊ Π½ΠΎΠ²ΠΎΠΉ ΠΌΠ°Ρ‚Ρ€ΠΈΡ†Π΅, которая ΠΏΠΎΠ·ΠΆΠ΅ добавляСтся ΠΊ Ρ‚Ρ€Π΅Ρ‚ΡŒΠ΅ΠΉ. Если Π΄ΠΎΠ±Π°Π²Π»Π΅Π½ΠΈΠ΅ ΠΌΠ°Ρ‚Ρ€ΠΈΡ† содСрТит большС Ρ‚Ρ€Π΅Ρ… ΠΌΠ°Ρ‚Ρ€ΠΈΡ†, Ρ‚ΠΎΡ‚ ΠΆΠ΅ процСсс связывания ΠΌΠΎΠΆΠ½ΠΎ ΠΏΠΎΠ²Ρ‚ΠΎΡ€ΡΡ‚ΡŒ ΡΡ‚ΠΎΠ»ΡŒΠΊΠΎ Ρ€Π°Π·, сколько Π½Π΅ΠΎΠ±Ρ…ΠΎΠ΄ΠΈΠΌΠΎ, Ρ‡Ρ‚ΠΎΠ±Ρ‹ ΡƒΠΏΡ€ΠΎΡΡ‚ΠΈΡ‚ΡŒ Π·Π°Π΄Π°Ρ‡Ρƒ, ΠΏΠΎΠΊΠ° Π²Ρ‹ Π½Π΅ ΠΏΠΎΠ»ΡƒΡ‡ΠΈΡ‚Π΅ ΠΏΡ€ΠΎΡΡ‚ΡƒΡŽ ΠΎΠΏΠ΅Ρ€Π°Ρ†ΠΈΡŽ с двумя ΠΌΠ°Ρ‚Ρ€ΠΈΡ†Π°ΠΌΠΈ.

ΠŸΡ€ΠΈΠΌΠ΅Ρ€ 5

Π’Ρ‹Ρ‡Ρ‚ΠΈΡ‚Π΅ ΡΠ»Π΅Π΄ΡƒΡŽΡ‰ΠΈΠ΅ ΠΌΠ°Ρ‚Ρ€ΠΈΡ†Ρ‹:

Π£Ρ€Π°Π²Π½Π΅Π½ΠΈΠ΅ 14: Π’Ρ‹Ρ‡ΠΈΡ‚Π°Π½ΠΈΠ΅ Π΄Π²ΡƒΡ… ΠΌΠ°Ρ‚Ρ€ΠΈΡ†

Π•Ρ‰Π΅ Ρ€Π°Π· ΠΎΠ±Ρ€Π°Ρ‚ΠΈΡ‚Π΅ Π²Π½ΠΈΠΌΠ°Π½ΠΈΠ΅, Ρ‡Ρ‚ΠΎ ΠΌΠ°Ρ‚Ρ€ΠΈΡ†Ρ‹, ΡƒΡ‡Π°ΡΡ‚Π²ΡƒΡŽΡ‰ΠΈΠ΅ Π² Π²Ρ‹Ρ‡ΠΈΡ‚Π°Π½ΠΈΠΈ, Π΄ΠΎΠ»ΠΆΠ½Ρ‹ ΠΈΠΌΠ΅Ρ‚ΡŒ ΠΎΠ΄ΠΈΠ½Π°ΠΊΠΎΠ²Ρ‹Π΅ Ρ€Π°Π·ΠΌΠ΅Ρ€Ρ‹, Ρ‡Ρ‚ΠΎΠ±Ρ‹ соблюдалось поэлСмСнтноС соотвСтствиС.

Π£Ρ€Π°Π²Π½Π΅Π½ΠΈΠ΅ 15: РСшСниС для вычитания Π΄Π²ΡƒΡ… ΠΌΠ°Ρ‚Ρ€ΠΈΡ†

ΠŸΡ€ΠΈΠΌΠ΅Ρ€ 6

Π’Ρ‹Ρ‡Ρ‚ΠΈΡ‚Π΅ ΡΠ»Π΅Π΄ΡƒΡŽΡ‰ΠΈΠ΅ ΠΌΠ°Ρ‚Ρ€ΠΈΡ†Ρ‹:

Π£Ρ€Π°Π²Π½Π΅Π½ΠΈΠ΅ 16: Π’Ρ‹Ρ‡ΠΈΡ‚Π°Π½ΠΈΠ΅ Π΄Π²ΡƒΡ… ΠΌΠ°Ρ‚Ρ€ΠΈΡ†

НСваТно, насколько Π²Π΅Π»ΠΈΠΊΠΈ ΠΌΠ°Ρ‚Ρ€ΠΈΡ†Ρ‹, Ссли ΠΎΠ½ΠΈ ΠΈΠΌΠ΅ΡŽΡ‚ ΠΎΠ΄ΠΈΠ½Π°ΠΊΠΎΠ²Ρ‹Π΅ Ρ€Π°Π·ΠΌΠ΅Ρ€Ρ‹, Π²Ρ‹ ΠΌΠΎΠΆΠ΅Ρ‚Π΅ Π²Ρ‹ΠΏΠΎΠ»Π½ΠΈΡ‚ΡŒ Π²Ρ‹Ρ‡ΠΈΡ‚Π°Π½ΠΈΠ΅.

Π£Ρ€Π°Π²Π½Π΅Π½ΠΈΠ΅ 17: РСшСниС для вычитания Π΄Π²ΡƒΡ… ΠΌΠ°Ρ‚Ρ€ΠΈΡ†

ΠŸΡ€ΠΈΠΌΠ΅Ρ€ 7

Π’Ρ‹Ρ‡Ρ‚ΠΈΡ‚Π΅ ΡΠ»Π΅Π΄ΡƒΡŽΡ‰ΠΈΠ΅ ΠΌΠ°Ρ‚Ρ€ΠΈΡ†Ρ‹:

Π£Ρ€Π°Π²Π½Π΅Π½ΠΈΠ΅ 18: Π’Ρ‹Ρ‡ΠΈΡ‚Π°Π½ΠΈΠ΅ Ρ‚Ρ€Π΅Ρ… ΠΌΠ°Ρ‚Ρ€ΠΈΡ†

ΠžΠ±Ρ€Π°Ρ‚ΠΈΡ‚Π΅ Π²Π½ΠΈΠΌΠ°Π½ΠΈΠ΅, Ρ‡Ρ‚ΠΎ для вычитания ΠΌΠ°Ρ‚Ρ€ΠΈΡ† с Π±ΠΎΠ»Π΅Π΅ Ρ‡Π΅ΠΌ двумя ΠΌΠ°Ρ‚Ρ€ΠΈΡ†Π°ΠΌΠΈ ΠΌΡ‹ ΠΌΠΎΠΆΠ΅ΠΌ снова ΠΈΡΠΏΠΎΠ»ΡŒΠ·ΠΎΠ²Π°Ρ‚ΡŒ свойство ассоциативности. ΠŸΠΎΠΌΠ½ΠΈΡ‚Π΅, Ρ‡Ρ‚ΠΎ Π²Ρ‹Ρ‡ΠΈΡ‚Π°Π½ΠΈΠ΅ ΠΌΠ°Ρ‚Ρ€ΠΈΡ† Π½Π΅ ΠΊΠΎΠΌΠΌΡƒΡ‚Π°Ρ‚ΠΈΠ²Π½ΠΎ (Π²Ρ‹ Π½Π΅ ΠΌΠΎΠΆΠ΅Ρ‚Π΅ ΠΈΠ·ΠΌΠ΅Π½ΠΈΡ‚ΡŒ порядок ΠΌΠ°Ρ‚Ρ€ΠΈΡ† Π² ΠΎΠΏΠ΅Ρ€Π°Ρ†ΠΈΠΈ ΠΈ ΠΏΠΎΠ»ΡƒΡ‡ΠΈΡ‚ΡŒ Ρ‚ΠΎΡ‚ ΠΆΠ΅ Ρ€Π΅Π·ΡƒΠ»ΡŒΡ‚Π°Ρ‚).

Π£Ρ€Π°Π²Π½Π΅Π½ΠΈΠ΅ 19: РСшСниС вычитания Ρ‚Ρ€Π΅Ρ… ΠΌΠ°Ρ‚Ρ€ΠΈΡ†

ΠŸΡ€ΠΈΠΌΠ΅Ρ€ 8

Π Π΅ΡˆΠΈΡ‚Π΅ ΡΠ»Π΅Π΄ΡƒΡŽΡ‰Π΅Π΅ ΡƒΡ€Π°Π²Π½Π΅Π½ΠΈΠ΅, содСрТащСС Π΄ΠΎΠ±Π°Π²Π»Π΅Π½ΠΈΠ΅ постоянной ΠΌΠ°Ρ‚Ρ€ΠΈΡ†Ρ‹ ΠΊ ΠΏΠ΅Ρ€Π΅ΠΌΠ΅Π½Π½ΠΎΠΉ.

Π£Ρ€Π°Π²Π½Π΅Π½ΠΈΠ΅ 20: НайдитС A Π² ΠΌΠ°Ρ‚Ρ€ΠΈΡ‡Π½ΠΎΠΌ ΡƒΡ€Π°Π²Π½Π΅Π½ΠΈΠΈ (Ρ‡Π°ΡΡ‚ΡŒ 1)

ΠœΡ‹ Π½Π°Ρ‡ΠΈΠ½Π°Π΅ΠΌ с вычитания ΠΌΠ°Ρ‚Ρ€ΠΈΡ†Ρ‹ Π² Π»Π΅Π²ΠΎΠΉ части ΠΊ ΠΎΠ±Π΅ΠΈΠΌ частям уравнСния, Ρ‡Ρ‚ΠΎΠ±Ρ‹ ΠΌΡ‹ ΠΌΠΎΠ³Π»ΠΈ Π»Π΅Π³ΠΊΠΎ Π½Π°ΠΉΡ‚ΠΈ A, Π²Ρ‹ΠΏΠΎΠ»Π½ΠΈΠ² Π²Ρ‹Ρ‡ΠΈΡ‚Π°Π½ΠΈΠ΅ ΠΌΠ°Ρ‚Ρ€ΠΈΡ†Ρ‹:

Π£Ρ€Π°Π²Π½Π΅Π½ΠΈΠ΅ 21: НайдитС A Π² ΠΌΠ°Ρ‚Ρ€ΠΈΡ‡Π½ΠΎΠΌ ΡƒΡ€Π°Π²Π½Π΅Π½ΠΈΠΈ (Ρ‡Π°ΡΡ‚ΡŒ 2)

ΠŸΡ€ΠΈΠΌΠ΅Ρ€ 9

Π Π΅ΡˆΠΈΡ‚Π΅ ΡΠ»Π΅Π΄ΡƒΡŽΡ‰ΠΈΠ΅ ΠΌΠ°Ρ‚Ρ€ΠΈΡ‡Π½Ρ‹Π΅ уравнСния

Π£Ρ€Π°Π²Π½Π΅Π½ΠΈΠ΅ 22: НайдитС X Π² ΠΌΠ°Ρ‚Ρ€ΠΈΡ‡Π½ΠΎΠΌ ΡƒΡ€Π°Π²Π½Π΅Π½ΠΈΠΈ (Ρ‡Π°ΡΡ‚ΡŒ 1)

Π’ этом случаС самый простой способ Π½Π°ΠΉΡ‚ΠΈ X – ΠΏΠ΅Ρ€Π΅Π΄Π°Ρ‚ΡŒ Π΅Π³ΠΎ Π² ΠΏΡ€Π°Π²ΡƒΡŽ Ρ‡Π°ΡΡ‚ΡŒ, Π΄ΠΎΠ±Π°Π²ΠΈΠ² Π΅Π³ΠΎ ΠΊ ΠΎΠ±Π΅ΠΈΠΌ частям уравнСния, Ρ‡Ρ‚ΠΎΠ±Ρ‹ ΠΎΠ½ΠΎ стало ΠΏΠΎΠ»ΠΎΠΆΠΈΡ‚Π΅Π»ΡŒΠ½Ρ‹ΠΌ.ΠœΡ‹ Ρ‚Π°ΠΊΠΆΠ΅ Π²Ρ‹Ρ‡ΠΈΡ‚Π°Π΅ΠΌ ΠΌΠ°Ρ‚Ρ€ΠΈΡ†Ρƒ, Π½Π°ΠΉΠ΄Π΅Π½Π½ΡƒΡŽ Π² ΠΏΡ€Π°Π²ΠΎΠΉ части уравнСния, ΠΈΠ· ΠΎΠ±Π΅ΠΈΡ… частСй, ΠΈ поэтому ΠΌΡ‹ ΠΏΠΎΠ»ΡƒΡ‡Π°Π΅ΠΌ Π²Ρ‹Ρ‡ΠΈΡ‚Π°Π½ΠΈΠ΅ ΠΌΠ°Ρ‚Ρ€ΠΈΡ†Ρ‹, ΠΊΠΎΡ‚ΠΎΡ€ΠΎΠ΅ опрСдСляСт X.

Π£Ρ€Π°Π²Π½Π΅Π½ΠΈΠ΅ 23: НайдитС X Π² ΠΌΠ°Ρ‚Ρ€ΠΈΡ‡Π½ΠΎΠΌ ΡƒΡ€Π°Π²Π½Π΅Π½ΠΈΠΈ (Ρ‡Π°ΡΡ‚ΡŒ 2)

РСшая Π²Ρ‹Ρ‡ΠΈΡ‚Π°Π½ΠΈΠ΅ ΠΌΠ°Ρ‚Ρ€ΠΈΡ†, Π½Π°Ρ…ΠΎΠ΄ΠΈΠΌ X.

ΠŸΡ€ΠΈΠΌΠ΅Ρ€ 10

НайдитС значСния элСмСнтов a, b, c, d, e ΠΈ f, Ρ€Π΅ΡˆΠΈΠ² ΡΠ»Π΅Π΄ΡƒΡŽΡ‰Π΅Π΅ ΠΌΠ°Ρ‚Ρ€ΠΈΡ‡Π½ΠΎΠ΅ ΡƒΡ€Π°Π²Π½Π΅Π½ΠΈΠ΅ Π½Π°ΠΈΠ±ΠΎΠ»Π΅Π΅ эффСктивным способом.

Π£Ρ€Π°Π²Π½Π΅Π½ΠΈΠ΅ 24: НахоТдСниС Π·Π½Π°Ρ‡Π΅Π½ΠΈΠΉ элСмСнтов Π² самой Π»Π΅Π²ΠΎΠΉ ΠΌΠ°Ρ‚Ρ€ΠΈΡ†Π΅

Π’ этой Π·Π°Π΄Π°Ρ‡Π΅ ΠΌΡ‹ ΠΌΠΎΠΆΠ΅ΠΌ поэлСмСнтно ΠΏΡ€Π΅ΠΎΠ±Ρ€Π°Π·ΠΎΠ²Ρ‹Π²Π°Ρ‚ΡŒ рСгистр элСмСнта, ΡƒΡ€Π°Π²Π½Π΅Π½ΠΈΠ΅, ΠΊΠΎΡ‚ΠΎΡ€ΠΎΠ΅ ΠΏΠΎΠ·Π²ΠΎΠ»ΠΈΡ‚ Π½Π°ΠΌ ΠΏΠΎΠ»ΡƒΡ‡ΠΈΡ‚ΡŒ Π·Π½Π°Ρ‡Π΅Π½ΠΈΠ΅ ΠΊΠ°ΠΆΠ΄ΠΎΠ³ΠΎ ΠΈΠ· элСмСнтов Π² ΠΏΠ΅Ρ€Π²ΠΎΠΉ ΠΌΠ°Ρ‚Ρ€ΠΈΡ†Π΅.

Π”Ρ€ΡƒΠ³ΠΈΠΌΠΈ словами, Π²Ρ‹ Π·Π½Π°Π΅Ρ‚Π΅, Ρ‡Ρ‚ΠΎ элСмСнт ΠΈΠ· ΠΏΠ΅Ρ€Π²ΠΎΠΉ строки ΠΈ ΠΏΠ΅Ρ€Π²ΠΎΠ³ΠΎ столбца (Π²Π²Π΅Ρ€Ρ…Ρƒ слСва) Π² ΠΏΠ΅Ρ€Π²ΠΎΠΉ ΠΌΠ°Ρ‚Ρ€ΠΈΡ†Π΅ Π±ΡƒΠ΄Π΅Ρ‚ Π΄ΠΎΠ±Π°Π²Π»Π΅Π½ ΠΊ элСмСнту Π² ΠΏΠ΅Ρ€Π²ΠΎΠΉ строкС ΠΈ ΠΏΠ΅Ρ€Π²ΠΎΠΌ столбцС Π²Ρ‚ΠΎΡ€ΠΎΠΉ ΠΌΠ°Ρ‚Ρ€ΠΈΡ†Ρ‹, Ρ‡Ρ‚ΠΎΠ±Ρ‹ ΠΏΠΎΠ»ΡƒΡ‡ΠΈΡ‚ΡŒ элСмСнт Π² этом пространствС. Π² ΠΏΠΎΠ»ΡƒΡ‡ΠΈΠ²ΡˆΠ΅ΠΉΡΡ ΠΌΠ°Ρ‚Ρ€ΠΈΡ†Π΅ справа. Π”Ρ€ΡƒΠ³ΠΈΠΌΠΈ словами, ΠΌΡ‹ ΠΌΠΎΠΆΠ΅ΠΌ Π»Π΅Π³ΠΊΠΎ Π·Π°ΠΏΠΈΡΠ°Ρ‚ΡŒ ΡΠ»Π΅Π΄ΡƒΡŽΡ‰ΠΈΠ΅ уравнСния для ΠΊΠ°ΠΆΠ΄ΠΎΠ³ΠΎ пространства элСмСнтов:

Π£Ρ€Π°Π²Π½Π΅Π½ΠΈΠ΅ 25: Набор ΡƒΡ€Π°Π²Π½Π΅Π½ΠΈΠΉ для ΠΊΠ°ΠΆΠ΄ΠΎΠΉ ΠΏΠ΅Ρ€Π΅ΠΌΠ΅Π½Π½ΠΎΠΉ, ΠΊΠΎΡ‚ΠΎΡ€ΡƒΡŽ Π½Π΅ΠΎΠ±Ρ…ΠΎΠ΄ΠΈΠΌΠΎ Ρ€Π΅ΡˆΠΈΡ‚ΡŒ

И ΠΈΠ· этого ΠΌΡ‹ ΠΌΠΎΠΆΠ΅ΠΌ Ρ€Π΅ΡˆΠΈΡ‚ΡŒ Π·Π½Π°Ρ‡Π΅Π½ΠΈΠ΅ ΠΊΠ°ΠΆΠ΄ΠΎΠΉ ΠΏΠ΅Ρ€Π΅ΠΌΠ΅Π½Π½ΠΎΠΉ:

Π£Ρ€Π°Π²Π½Π΅Π½ΠΈΠ΅ 25: РСшСниС ΠΏΠ΅Ρ€Π΅ΠΌΠ΅Π½Π½Ρ‹Ρ… ΠΎΡ‚ a Π΄ΠΎ f

ПослС Ρ‚ΠΎΠ³ΠΎ, ΠΊΠ°ΠΊ ΠΌΡ‹ научимся ΡΠΊΠ»Π°Π΄Ρ‹Π²Π°Ρ‚ΡŒ ΠΈ Π²Ρ‹Ρ‡ΠΈΡ‚Π°Ρ‚ΡŒ ΠΌΠ°Ρ‚Ρ€ΠΈΡ†Ρ‹, ΠΏΡ€ΠΈΡˆΠ»ΠΎ врСмя ΠΈΠ·ΡƒΡ‡ΠΈΡ‚ΡŒ Π΄Ρ€ΡƒΠ³ΠΈΠ΅ ΠΎΠΏΠ΅Ρ€Π°Ρ†ΠΈΠΈ: скалярноС ΡƒΠΌΠ½ΠΎΠΆΠ΅Π½ΠΈΠ΅ ΠΌΠ°Ρ‚Ρ€ΠΈΡ† ΠΈ ΡƒΠΌΠ½ΠΎΠΆΠ΅Π½ΠΈΠ΅ Π΄Π²ΡƒΡ… ΠΈΠ»ΠΈ Π±ΠΎΠ»Π΅Π΅ ΠΌΠ°Ρ‚Ρ€ΠΈΡ† Π±ΡƒΠ΄ΡƒΡ‚ ΠΎΠ±ΡŠΡΡΠ½Π΅Π½Ρ‹ Π² ΡΠ»Π΅Π΄ΡƒΡŽΡ‰ΠΈΡ… Π΄Π²ΡƒΡ… ΡƒΡ€ΠΎΠΊΠ°Ρ….

ΠœΡ‹ Π·Π°Π²Π΅Ρ€ΡˆΠ°Π΅ΠΌ наш ΡƒΡ€ΠΎΠΊ Π½Π° сСгодня нСсколькими ΠΏΠΎΠ»Π΅Π·Π½Ρ‹ΠΌΠΈ ссылками: пСрвая содСрТит ΠΊΡ€Π°Ρ‚ΠΊΠΈΠΉ список основных ΠΏΡ€Π°Π²ΠΈΠ» слоТСния ΠΌΠ°Ρ‚Ρ€ΠΈΡ† ΠΈ Π΄ΠΎΠΏΠΎΠ»Π½ΠΈΡ‚Π΅Π»ΡŒΠ½ΡƒΡŽ Π»Π΅ΠΊΡ†ΠΈΡŽ ΠΏΠΎ ΠΌΠ°Ρ‚Ρ€ΠΈΡ‡Π½Ρ‹ΠΌ опСрациям.

И это всС, Π΄ΠΎ встрСчи Π½Π° нашСм ΡΠ»Π΅Π΄ΡƒΡŽΡ‰Π΅ΠΌ ΡƒΡ€ΠΎΠΊΠ΅!

Как ΡΠΊΠ»Π°Π΄Ρ‹Π²Π°Ρ‚ΡŒ ΠΈ Π²Ρ‹Ρ‡ΠΈΡ‚Π°Ρ‚ΡŒ ΠΌΠ°Ρ‚Ρ€ΠΈΡ†Ρ‹

ΠœΠ°Ρ‚Ρ€ΠΈΡ†Π° (мноТСствСнноС число ΠΌΠ°Ρ‚Ρ€ΠΈΡ† ) – это просто массив ΠΈΠ»ΠΈ Ρ‚Π°Π±Π»ΠΈΡ†Π° чисСл. Π§Ρ‚ΠΎ Π±Ρ‹ Π·Π½Π°Ρ‡ΠΈΠ»ΠΎ ΡΠ»ΠΎΠΆΠΈΡ‚ΡŒ Π΄Π²Π΅ Ρ‚Π°Π±Π»ΠΈΡ†Ρ‹ вмСстС? Π§Ρ‚ΠΎ ΠΆ, Ρ‚Ρ‹ скоро ΡƒΠ·Π½Π°Π΅ΡˆΡŒ! Π’ этой ΠΊΠΎΡ€ΠΎΡ‚ΠΊΠΎΠΉ ΡΡ‚Π°Ρ‚ΡŒΠ΅ ΠΌΡ‹ обсудим, ΠΊΠ°ΠΊ ΡΠΊΠ»Π°Π΄Ρ‹Π²Π°Ρ‚ΡŒ ΠΈ Π²Ρ‹Ρ‡ΠΈΡ‚Π°Ρ‚ΡŒ ΠΌΠ°Ρ‚Ρ€ΠΈΡ†Ρ‹.

Π˜Π·ΠΎΠ±Ρ€Π°ΠΆΠ΅Π½ΠΈΠ΅ АндрСя Попова

Π§Ρ‚ΠΎ Ρ‚Π°ΠΊΠΎΠ΅ ΠΌΠ°Ρ‚Ρ€ΠΈΡ†Π°?

Π’ΠΎ-ΠΏΠ΅Ρ€Π²Ρ‹Ρ…, Π΄Π°Π²Π°ΠΉΡ‚Π΅ ΠΏΠΎΠ³ΠΎΠ²ΠΎΡ€ΠΈΠΌ ΠΎ Ρ‚ΠΎΠΌ, Ρ‡Ρ‚ΠΎ Ρ‚Π°ΠΊΠΎΠ΅ ΠΌΠ°Ρ‚Ρ€ΠΈΡ†Π°.

ΠœΠ°Ρ‚Ρ€ΠΈΡ†Π° прСдставляСт собой ΠΏΡ€ΡΠΌΠΎΡƒΠ³ΠΎΠ»ΡŒΠ½Ρ‹ΠΉ массив чисСл.

КаТдая Π³ΠΎΡ€ΠΈΠ·ΠΎΠ½Ρ‚Π°Π»ΡŒΠ½Π°Ρ строка чисСл Π² ΠΌΠ°Ρ‚Ρ€ΠΈΡ†Π΅ являСтся строкой . КаТдая Π²Π΅Ρ€Ρ‚ΠΈΠΊΠ°Π»ΡŒΠ½Π°Ρ линия прСдставляСт собой столбСц . ΠœΡ‹ часто Π½Π°Π·Ρ‹Π²Π°Π΅ΠΌ ΠΎΡ‚Π΄Π΅Π»ΡŒΠ½Ρ‹Π΅ числа элСмСнтами ΠΌΠ°Ρ‚Ρ€ΠΈΡ†Ρ‹ .

Если Π² массивС m строк ΠΈ n столбцов, Ρ‚ΠΎ ΠΌΡ‹ Π³ΠΎΠ²ΠΎΡ€ΠΈΠΌ, Ρ‡Ρ‚ΠΎ Ρ€Π°Π·ΠΌΠ΅Ρ€ ΠΌΠ°Ρ‚Ρ€ΠΈΡ†Ρ‹ Ρ€Π°Π²Π΅Π½ m Γ— n .Π­Ρ‚ΠΎ станСт Π²Π°ΠΆΠ½Ρ‹ΠΌ, ΠΏΠΎΡ‚ΠΎΠΌΡƒ Ρ‡Ρ‚ΠΎ ΠΌΠΎΠΆΠ½ΠΎ ΡΠΊΠ»Π°Π΄Ρ‹Π²Π°Ρ‚ΡŒ ΠΈΠ»ΠΈ Π²Ρ‹Ρ‡ΠΈΡ‚Π°Ρ‚ΡŒ Ρ‚ΠΎΠ»ΡŒΠΊΠΎ ΠΌΠ°Ρ‚Ρ€ΠΈΡ†Ρ‹ с ΠΎΠ΄ΠΈΠ½Π°ΠΊΠΎΠ²Ρ‹ΠΌΠΈ Ρ€Π°Π·ΠΌΠ΅Ρ€Π°ΠΌΠΈ .

Π­Ρ‚Π° ΠΌΠ°Ρ‚Ρ€ΠΈΡ†Π° ΠΈΠΌΠ΅Π΅Ρ‚ 2 строки ΠΈ 3 столбца, поэтому Π΅Π΅ Ρ€Π°Π·ΠΌΠ΅Ρ€Ρ‹ ΡΠΎΡΡ‚Π°Π²Π»ΡΡŽΡ‚ 2 Γ— 3.

Как ΡΠΊΠ»Π°Π΄Ρ‹Π²Π°Ρ‚ΡŒ ΠΈ Π²Ρ‹Ρ‡ΠΈΡ‚Π°Ρ‚ΡŒ ΠΌΠ°Ρ‚Ρ€ΠΈΡ†Ρ‹

Π•ΡΡ‚ΡŒ Π΄Π²Π° основных ΠΏΡ€Π°Π²ΠΈΠ»Π° для слоТСния ΠΌΠ°Ρ‚Ρ€ΠΈΡ† вмСстС.

  1. МоТно Π΄ΠΎΠ±Π°Π²ΠΈΡ‚ΡŒ Ρ‚ΠΎΠ»ΡŒΠΊΠΎ Π΄Π²Π΅ ΠΌΠ°Ρ‚Ρ€ΠΈΡ†Ρ‹ с ΠΎΠ΄ΠΈΠ½Π°ΠΊΠΎΠ²Ρ‹ΠΌΠΈ Ρ‚ΠΎΡ‡Π½Ρ‹ΠΌΠΈ Ρ€Π°Π·ΠΌΠ΅Ρ€Π°ΠΌΠΈ . Π˜Ρ‚Π°ΠΊ, Π²Ρ‹ ΠΌΠΎΠΆΠ΅Ρ‚Π΅ ΡΠ»ΠΎΠΆΠΈΡ‚ΡŒ Π΄Π²Π΅ ΠΌΠ°Ρ‚Ρ€ΠΈΡ†Ρ‹ 3 Γ— 4 вмСстС. Но Π²Ρ‹ Π½Π΅ ΠΌΠΎΠΆΠ΅Ρ‚Π΅ Π΄ΠΎΠ±Π°Π²ΠΈΡ‚ΡŒ ΠΌΠ°Ρ‚Ρ€ΠΈΡ†Ρƒ 3 Γ— 4 ΠΊ ΠΌΠ°Ρ‚Ρ€ΠΈΡ†Π΅ 4 Γ— 3.(ΠŸΠΎΡ€ΡΠ΄ΠΎΠΊ Ρ€Π°Π·ΠΌΠ΅Ρ€ΠΎΠ² Ρ‚ΠΎΠΆΠ΅ ΠΈΠΌΠ΅Π΅Ρ‚ Π·Π½Π°Ρ‡Π΅Π½ΠΈΠ΅!)
  2. Π”ΠΎΠ±Π°Π²ΡŒΡ‚Π΅ ΡΠΎΠΎΡ‚Π²Π΅Ρ‚ΡΡ‚Π²ΡƒΡŽΡ‰ΠΈΠ΅ записи. Π’ΠΎ Π΅ΡΡ‚ΡŒ Π»Π΅Π²Ρ‹ΠΉ Π²Π΅Ρ€Ρ…Π½ΠΈΠΉ элСмСнт ΠΏΠ΅Ρ€Π²ΠΎΠΉ ΠΌΠ°Ρ‚Ρ€ΠΈΡ†Ρ‹ Π΄ΠΎΠ»ΠΆΠ΅Π½ Π±Ρ‹Ρ‚ΡŒ Π΄ΠΎΠ±Π°Π²Π»Π΅Π½ ΠΊ Π»Π΅Π²ΠΎΠΌΡƒ Π²Π΅Ρ€Ρ…Π½Π΅ΠΌΡƒ элСмСнту Π²Ρ‚ΠΎΡ€ΠΎΠΉ ΠΌΠ°Ρ‚Ρ€ΠΈΡ†Ρ‹ ΠΈ Ρ‚Π°ΠΊ Π΄Π°Π»Π΅Π΅.

Π§Ρ‚ΠΎΠ±Ρ‹ Π²Ρ‹Ρ‡Π΅ΡΡ‚ΡŒ Π΄Π²ΡƒΡ… ΠΌΠ°Ρ‚Ρ€ΠΈΡ†, A B , Π²Ρ‹ΠΏΠΎΠ»Π½ΠΈΡ‚Π΅ ΡΠ»Π΅Π΄ΡƒΡŽΡ‰ΠΈΠ΅ дСйствия:

  1. Π˜Π·ΠΌΠ΅Π½ΠΈΡ‚Π΅ ΠΎΠΏΠ΅Ρ€Π°Ρ†ΠΈΡŽ Π½Π° слоТСниС, A + (- B ), ΠΏΡ€Π΅ΠΎΠ±Ρ€Π°Π·ΠΎΠ²Π°Π² ΠΊΠ°ΠΆΠ΄ΡƒΡŽ запись B Π² ΠΏΡ€ΠΎΡ‚ΠΈΠ²ΠΎΠΏΠΎΠ»ΠΎΠΆΠ½ΡƒΡŽ (ΠΏΠ΅Ρ€Π΅Π²Π΅Ρ€Π½ΠΈΡ‚Π΅ Π·Π½Π°ΠΊΠΈ).
  2. Π”ΠΎΠ±Π°Π²ΡŒΡ‚Π΅ ΠΌΠ°Ρ‚Ρ€ΠΈΡ†Ρ‹, ΠΈΡΠΏΠΎΠ»ΡŒΠ·ΡƒΡ описанный Π²Ρ‹ΡˆΠ΅ ΠΌΠ΅Ρ‚ΠΎΠ΄.

(ВмСсто этого Π²Ρ‹ ΠΌΠΎΠΆΠ΅Ρ‚Π΅ просто Π²Ρ‹Ρ‡Π΅ΡΡ‚ΡŒ ΡΠΎΠΎΡ‚Π²Π΅Ρ‚ΡΡ‚Π²ΡƒΡŽΡ‰ΠΈΡ… записСй, Ρ‡Ρ‚ΠΎΠ±Ρ‹ Π²Ρ‹Ρ‡ΠΈΡΠ»ΠΈΡ‚ΡŒ A B , Π½ΠΎ Ρ‚ΠΎΠ³Π΄Π° Π²Π°ΠΌ ΠΈΠ½ΠΎΠ³Π΄Π° придСтся ΠΎΡ‚ΡΠ»Π΅ΠΆΠΈΠ²Π°Ρ‚ΡŒ Π²Ρ‹Ρ‡ΠΈΡ‚Π°Π½ΠΈΠ΅ ΠΎΡ‚Ρ€ΠΈΡ†Π°Ρ‚Π΅Π»ΡŒΠ½Ρ‹Ρ… сумм. МнС просто Π»Π΅Π³Ρ‡Π΅ сначала ΠΏΠ΅Ρ€Π΅Π²Π΅Ρ€Π½ΡƒΡ‚ΡŒ ΠΊΠ°ΠΆΠ΄Ρ‹ΠΉ Π·Π½Π°ΠΊ, Π° Π·Π°Ρ‚Π΅ΠΌ ΠΏΡ€ΠΈΠ±Π°Π²ΠΈΡ‚ΡŒ. Π‘ алгСбраичСской Ρ‚ΠΎΡ‡ΠΊΠΈ зрСния это Ρ‚ΠΎ ΠΆΠ΅ самоС.)

ΠŸΡ€ΠΈΠΌΠ΅Ρ€Ρ‹

Рассмотрим ΡΠ»Π΅Π΄ΡƒΡŽΡ‰ΠΈΠ΅ Ρ‚Ρ€ΠΈ ΠΌΠ°Ρ‚Ρ€ΠΈΡ†Ρ‹:

Находят:

Π°) А + Π’

б) А + Б

Π²) Π’ А

Π³) Π‘ Π‘

РСшСния

a) Π‘Π½Π°Ρ‡Π°Π»Π° ΠΏΡ€ΠΎΠ²Π΅Ρ€ΡŒΡ‚Π΅ Ρ€Π°Π·ΠΌΠ΅Ρ€Ρ‹.И A , ΠΈ B ΠΈΠΌΠ΅ΡŽΡ‚ Π΄Π²Π΅ строки ΠΈ Π΄Π²Π° столбца. ΠŸΠΎΡΠΊΠΎΠ»ΡŒΠΊΡƒ ΠΎΠ±Π΅ ΠΎΠ½ΠΈ ΠΏΡ€Π΅Π΄ΡΡ‚Π°Π²Π»ΡΡŽΡ‚ собой ΠΌΠ°Ρ‚Ρ€ΠΈΡ†Ρ‹ 2 Γ— 2, ΠΈΡ… ΠΌΠΎΠΆΠ½ΠΎ ΡΠΊΠ»Π°Π΄Ρ‹Π²Π°Ρ‚ΡŒ вмСстС. НС Π·Π°Π±ΡƒΠ΄ΡŒΡ‚Π΅ Π΄ΠΎΠ±Π°Π²ΠΈΡ‚ΡŒ ΡΠΎΠΎΡ‚Π²Π΅Ρ‚ΡΡ‚Π²ΡƒΡŽΡ‰ΠΈΠ΅ записи.

Π±) На этот Ρ€Π°Π· ΠΏΡ€ΠΈ ΠΏΡ€ΠΎΠ²Π΅Ρ€ΠΊΠ΅ Ρ€Π°Π·ΠΌΠ΅Ρ€ΠΎΠ² ΠΎΠ±Π½Π°Ρ€ΡƒΠΆΠΈΠ²Π°Π΅ΠΌ нСсоотвСтствиС. A – это 2 Γ— 2, Π½ΠΎ C – 3 Γ— 2. Π‘Π»Π΅Π΄ΠΎΠ²Π°Ρ‚Π΅Π»ΡŒΠ½ΠΎ, сумма Π½Π΅ сущСствуСт. Π”Ρ€ΡƒΠ³ΠΈΠΌΠΈ словами, Π½Π΅Ρ‚ ΠΎΡ‚Π²Π΅Ρ‚Π° .

c) Как ΠΈ Π² части a , ΠΎΠ±Π΅ ΠΌΠ°Ρ‚Ρ€ΠΈΡ†Ρ‹ ΠΈΠΌΠ΅ΡŽΡ‚ ΠΎΠ΄ΠΈΠ½Π°ΠΊΠΎΠ²Ρ‹Π΅ Ρ€Π°Π·ΠΌΠ΅Ρ€Ρ‹.Π’Π°ΠΊ Ρ‡Ρ‚ΠΎ Π½Π°ΠΌ Ρ€Π°Π·Ρ€Π΅ΡˆΠ΅Π½ΠΎ ΠΈΡ… Π²Ρ‹Ρ‡Π΅ΡΡ‚ΡŒ. НС Π·Π°Π±Ρ‹Π²Π°ΠΉΡ‚Π΅ ΠΏΠ΅Ρ€Π΅Π²ΠΎΡ€Π°Ρ‡ΠΈΠ²Π°Ρ‚ΡŒ Π·Π½Π°ΠΊΠΈ Π²Ρ‚ΠΎΡ€ΠΎΠ³ΠΎ!

d) ΠžΡ‡Π΅Π²ΠΈΠ΄Π½ΠΎ, Ρ‡Ρ‚ΠΎ C ΠΈΠΌΠ΅Π΅Ρ‚ Ρ‚Π΅ ΠΆΠ΅ Ρ€Π°Π·ΠΌΠ΅Ρ€Ρ‹, Ρ‡Ρ‚ΠΎ ΠΈ ΠΎΠ½ сам, поэтому ΠΏΡ€Π°Π²ΠΈΠ»Π° говорят, Ρ‡Ρ‚ΠΎ Π²Ρ‹ ΠΌΠΎΠΆΠ΅Ρ‚Π΅ Π²Ρ‹Ρ‡Π΅ΡΡ‚ΡŒ.

ΠŸΠΎΡΠΊΠΎΠ»ΡŒΠΊΡƒ всС записи ΠΎΠ΄ΠΈΠ½Π°ΠΊΠΎΠ²Ρ‹, ΠΏΡ€ΠΈ Π²Ρ‹Ρ‡ΠΈΡ‚Π°Π½ΠΈΠΈ C C Π²Ρ‹ ΠΏΠΎΠ»ΡƒΡ‡ΠΈΡ‚Π΅ ноль (0) Π² ΠΊΠ°ΠΆΠ΄ΠΎΠΉ Ρ‚ΠΎΡ‡ΠΊΠ΅.

(ΠšΡΡ‚Π°Ρ‚ΠΈ, любая ΠΌΠ°Ρ‚Ρ€ΠΈΡ†Π°, Π² ΠΊΠ°ΠΆΠ΄ΠΎΠΉ записи ΠΊΠΎΡ‚ΠΎΡ€ΠΎΠΉ Π½Π΅Ρ‚ Π½ΠΈΡ‡Π΅Π³ΠΎ, ΠΊΡ€ΠΎΠΌΠ΅ 0, называСтся Π½ΡƒΠ»Π΅Π²ΠΎΠΉ ΠΌΠ°Ρ‚Ρ€ΠΈΡ†Π΅ΠΉ ΠΈΠ»ΠΈ Π½ΡƒΠ»Π΅Π²ΠΎΠΉ ΠΌΠ°Ρ‚Ρ€ΠΈΡ†Π΅ΠΉ .)

  • Π¨ΠΎΠ½ ΠΏΠΎΠ»ΡƒΡ‡ΠΈΠ» Π΄ΠΎΠΊΡ‚ΠΎΡ€ΡΠΊΡƒΡŽ ΡΡ‚Π΅ΠΏΠ΅Π½ΡŒ ΠΏΠΎ ΠΌΠ°Ρ‚Π΅ΠΌΠ°Ρ‚ΠΈΠΊΠ΅ Π² УнивСрситСтС ΡˆΡ‚Π°Ρ‚Π° Огайо Π² 2008 Π³ΠΎΠ΄Ρƒ (Go Bucks !!). Π’ 2002 Π³ΠΎΠ΄Ρƒ ΠΎΠ½ ΠΏΠΎΠ»ΡƒΡ‡ΠΈΠ» ΡΡ‚Π΅ΠΏΠ΅Π½ΡŒ Π±Π°ΠΊΠ°Π»Π°Π²Ρ€Π° ΠΌΠ°Ρ‚Π΅ΠΌΠ°Ρ‚ΠΈΠΊΠΈ ΠΈ ΠΈΠ½Ρ„ΠΎΡ€ΠΌΠ°Ρ‚ΠΈΠΊΡƒ Π² ΠžΠ±Π΅Ρ€Π»ΠΈΠ½ΡΠΊΠΎΠΌ ΠΊΠΎΠ»Π»Π΅Π΄ΠΆΠ΅. ΠšΡ€ΠΎΠΌΠ΅ Ρ‚ΠΎΠ³ΠΎ, Π¨ΠΎΠ½ ΠΏΠΎΠ»ΡƒΡ‡ΠΈΠ» ΡΡ‚Π΅ΠΏΠ΅Π½ΡŒ Π±Π°ΠΊΠ°Π»Π°Π²Ρ€Π° искусств. ΠΈΠ· ΠšΠΎΠ½ΡΠ΅Ρ€Π²Π°Ρ‚ΠΎΡ€ΠΈΠΈ ΠžΠ±Π΅Ρ€Π»ΠΈΠ½Π° Π² Ρ‚ΠΎΠΌ ΠΆΠ΅ Π³ΠΎΠ΄Ρƒ ΠΏΠΎ ΡΠΏΠ΅Ρ†ΠΈΠ°Π»ΡŒΠ½ΠΎΡΡ‚ΠΈ “ΠΌΡƒΠ·Ρ‹ΠΊΠ°Π»ΡŒΠ½Π°Ρ композиция”.Π¨ΠΎΠ½ ΠΏΠΎ-ΠΏΡ€Π΅ΠΆΠ½Π΅ΠΌΡƒ Π»ΡŽΠ±ΠΈΡ‚ ΠΌΡƒΠ·Ρ‹ΠΊΡƒ – ΠΏΠΎΡ‡Ρ‚ΠΈ Ρ‚Π°ΠΊ ΠΆΠ΅, ΠΊΠ°ΠΊ ΠΌΠ°Ρ‚Π΅ΠΌΠ°Ρ‚ΠΈΠΊΡƒ! – ΠΈ ΠΎΠ½ (Π΄ΡƒΠΌΠ°Π΅Ρ‚, Ρ‡Ρ‚ΠΎ ΠΎΠ½) ΠΌΠΎΠΆΠ΅Ρ‚ ΠΈΠ³Ρ€Π°Ρ‚ΡŒ Π½Π° ΠΏΠΈΠ°Π½ΠΈΠ½ΠΎ, Π³ΠΈΡ‚Π°Ρ€Π΅ ΠΈ басу. Π¨ΠΎΠ½ ΠΎΠ±ΡƒΡ‡Π°Π» ΠΈ ΠΎΠ±ΡƒΡ‡Π°Π» студСнтов ΠΌΠ°Ρ‚Π΅ΠΌΠ°Ρ‚ΠΈΠΊΠ΅ ΠΎΠΊΠΎΠ»ΠΎ дСсяти Π»Π΅Ρ‚ ΠΈ надССтся, Ρ‡Ρ‚ΠΎ Π΅Π³ΠΎ ΠΎΠΏΡ‹Ρ‚ ΠΏΠΎΠΌΠΎΠΆΠ΅Ρ‚ Π²Π°ΠΌ Π΄ΠΎΠ±ΠΈΡ‚ΡŒΡΡ успСха!

    ΠŸΡ€ΠΎΡΠΌΠΎΡ‚Ρ€Π΅Ρ‚ΡŒ всС сообщСния

Π’Ρ‹Ρ‡ΠΈΡ‚Π°Π½ΠΈΠ΅ ΠΌΠ°Ρ‚Ρ€ΠΈΡ† – Бвойства

Π’Ρ‹Ρ‡ΠΈΡ‚Π°Π½ΠΈΠ΅ ΠΌΠ°Ρ‚Ρ€ΠΈΡ† относится ΠΊ Π²Ρ‹Ρ‡ΠΈΡ‚Π°Π½ΠΈΡŽ ΡΠΎΠΎΡ‚Π²Π΅Ρ‚ΡΡ‚Π²ΡƒΡŽΡ‰ΠΈΡ… элСмСнтов Π΄Π²ΡƒΡ… ΠΈΠ»ΠΈ Π±ΠΎΠ»Π΅Π΅ ΠΌΠ°Ρ‚Ρ€ΠΈΡ†.ΠœΠ°Ρ‚Ρ€ΠΈΡ†Π° – это матСматичСский Ρ„ΠΎΡ€ΠΌΠ°Ρ‚ для размСщСния Π΄Π°Π½Π½Ρ‹Ρ… Π² Π²ΠΈΠ΄Π΅ строк ΠΈ столбцов. Π’Ρ‹Ρ‡ΠΈΡ‚Π°Π½ΠΈΠ΅ ΠΌΠ°Ρ‚Ρ€ΠΈΡ† ΠΌΠΎΠΆΠ½ΠΎ ΠΏΡ€ΠΎΠΈΠ·Π²ΠΎΠ΄ΠΈΡ‚ΡŒ посрСдством поэлСмСнтного вычитания ΠΌΠ°Ρ‚Ρ€ΠΈΡ†. К ΠΌΠ°Ρ‚Ρ€ΠΈΡ†Π°ΠΌ ΠΌΠΎΠ³ΡƒΡ‚ ΠΏΡ€ΠΈΠΌΠ΅Π½ΡΡ‚ΡŒΡΡ Ρ€Π°Π·Π»ΠΈΡ‡Π½Ρ‹Π΅ ΠΎΠΏΠ΅Ρ€Π°Ρ†ΠΈΠΈ, Ρ‚Π°ΠΊΠΈΠ΅ ΠΊΠ°ΠΊ слоТСниС, Π²Ρ‹Ρ‡ΠΈΡ‚Π°Π½ΠΈΠ΅ ΠΈ ΡƒΠΌΠ½ΠΎΠΆΠ΅Π½ΠΈΠ΅. Π’ этой ΡΡ‚Π°Ρ‚ΡŒΠ΅ ΠΌΡ‹ Π² основном сосрСдоточимся Π½Π° ΠΎΠΏΠ΅Ρ€Π°Ρ†ΠΈΠΈ вычитания ΠΌΠ°Ρ‚Ρ€ΠΈΡ†. Π’Ρ‹Ρ‡ΠΈΡ‚Π°Π½ΠΈΠ΅ ΠΌΠ°Ρ‚Ρ€ΠΈΡ† – это процСсс вычитания ΡΠΎΠΎΡ‚Π²Π΅Ρ‚ΡΡ‚Π²ΡƒΡŽΡ‰ΠΈΡ… элСмСнтов ΠΌΠ°Ρ‚Ρ€ΠΈΡ†.

Π’Ρ‹Ρ‡ΠΈΡ‚Π°Π½ΠΈΠ΅ ΠΌΠ°Ρ‚Ρ€ΠΈΡ† выполняСтся Ρ‚Π°ΠΊ ΠΆΠ΅, ΠΊΠ°ΠΊ ΠΈ слоТСниС ΠΌΠ°Ρ‚Ρ€ΠΈΡ†.ΠžΠ³Ρ€Π°Π½ΠΈΡ‡Π΅Π½ΠΈΡ слоТСния ΠΌΠ°Ρ‚Ρ€ΠΈΡ† ΠΏΡ€ΠΈΠΌΠ΅Π½ΠΈΠΌΡ‹ ΠΈ ΠΊ Π²Ρ‹Ρ‡ΠΈΡ‚Π°Π½ΠΈΡŽ ΠΌΠ°Ρ‚Ρ€ΠΈΡ†. Π’Ρ‹Ρ‡ΠΈΡ‚Π°Π½ΠΈΠ΅ ΠΌΠ°Ρ‚Ρ€ΠΈΡ† опрСдСляСтся Ρ‚ΠΎΠ»ΡŒΠΊΠΎ для ΠΌΠ°Ρ‚Ρ€ΠΈΡ† ΠΎΠ΄ΠΈΠ½Π°ΠΊΠΎΠ²ΠΎΠ³ΠΎ Ρ€Π°Π·ΠΌΠ΅Ρ€Π°. Π Π°Π·Π±Π΅Ρ€Π΅ΠΌ ΠΊΠΎΠ½Ρ†Π΅ΠΏΡ†ΠΈΡŽ ΠΏΠΎΠ΄Ρ€ΠΎΠ±Π½ΠΎ Π½Π° ΠΏΡ€ΠΈΠΌΠ΅Ρ€Π°Ρ….

Π§Ρ‚ΠΎ Ρ‚Π°ΠΊΠΎΠ΅ Π²Ρ‹Ρ‡ΠΈΡ‚Π°Π½ΠΈΠ΅ ΠΌΠ°Ρ‚Ρ€ΠΈΡ†?

Π’Ρ‹Ρ‡ΠΈΡ‚Π°Π½ΠΈΠ΅ ΠΌΠ°Ρ‚Ρ€ΠΈΡ† – это опСрация поэлСмСнтного вычитания ΠΌΠ°Ρ‚Ρ€ΠΈΡ† ΠΎΠ΄Π½ΠΎΠ³ΠΎ порядка, Ρ‚ΠΎ Π΅ΡΡ‚ΡŒ ΠΌΠ°Ρ‚Ρ€ΠΈΡ†, ΠΈΠΌΠ΅ΡŽΡ‰ΠΈΡ… ΠΎΠ΄ΠΈΠ½Π°ΠΊΠΎΠ²ΠΎΠ΅ количСство строк ΠΈ столбцов. Если количСство Π³ΠΎΡ€ΠΈΠ·ΠΎΠ½Ρ‚Π°Π»ΡŒΠ½Ρ‹Ρ… строк Π² ΠΌΠ°Ρ‚Ρ€ΠΈΡ†Π΅ Ρ€Π°Π²Π½ΠΎ Β«mΒ», Π° количСство Π²Π΅Ρ€Ρ‚ΠΈΠΊΠ°Π»ΡŒΠ½Ρ‹Ρ… столбцов – Β«nΒ», Ρ‚ΠΎ говорят, Ρ‡Ρ‚ΠΎ ΠΌΠ°Ρ‚Ρ€ΠΈΡ†Π° ΠΈΠΌΠ΅Π΅Ρ‚ Ρ€Π°Π·ΠΌΠ΅Ρ€Π½ΠΎΡΡ‚ΡŒ Β«m Γ— nΒ».Для вычитания ΠΌΠ°Ρ‚Ρ€ΠΈΡ† Π½Π΅ΠΎΠ±Ρ…ΠΎΠ΄ΠΈΠΌΠΎ, Ρ‡Ρ‚ΠΎΠ±Ρ‹ Π²Ρ‹Ρ‡ΠΈΡ‚Π°Π΅ΠΌΡ‹Π΅ ΠΌΠ°Ρ‚Ρ€ΠΈΡ†Ρ‹ ΠΈΠΌΠ΅Π»ΠΈ Ρ‚Ρƒ ΠΆΠ΅ Ρ€Π°Π·ΠΌΠ΅Ρ€Π½ΠΎΡΡ‚ΡŒ, Ρ‡Ρ‚ΠΎ ΠΈ Π²Ρ‹Ρ‡ΠΈΡ‚Π°Π΅ΠΌΡ‹Π΅ ΡΠΎΠΎΡ‚Π²Π΅Ρ‚ΡΡ‚Π²ΡƒΡŽΡ‰ΠΈΠ΅ элСмСнты ΠΌΠ°Ρ‚Ρ€ΠΈΡ†.

Π’Ρ‹Ρ‡ΠΈΡ‚Π°Π½ΠΈΠ΅ ΠΌΠ°Ρ‚Ρ€ΠΈΡ†, ΠΎΠ·Π½Π°Ρ‡Π°ΡŽΡ‰ΠΈΡ…

Π’Ρ‹Ρ‡ΠΈΡ‚Π°Π½ΠΈΠ΅ ΠΌΠ°Ρ‚Ρ€ΠΈΡ† ΠΈΠ»ΠΈ Π²Ρ‹Ρ‡ΠΈΡ‚Π°Π½ΠΈΠ΅ ΠΌΠ°Ρ‚Ρ€ΠΈΡ† Π²ΠΎΠ·ΠΌΠΎΠΆΠ½ΠΎ Ρ‚ΠΎΠ»ΡŒΠΊΠΎ Π² Ρ‚ΠΎΠΌ случаС, Ссли количСство строк ΠΈ столбцов ΠΎΠ±Π΅ΠΈΡ… ΠΌΠ°Ρ‚Ρ€ΠΈΡ† ΠΎΠ΄ΠΈΠ½Π°ΠΊΠΎΠ²ΠΎ. Вычитая Π΄Π²Π΅ ΠΌΠ°Ρ‚Ρ€ΠΈΡ†Ρ‹, ΠΌΡ‹ Π²Ρ‹Ρ‡ΠΈΡ‚Π°Π΅ΠΌ элСмСнты Π² ΠΊΠ°ΠΆΠ΄ΠΎΠΉ строкС ΠΈ столбцС ΠΈΠ· ΡΠΎΠΎΡ‚Π²Π΅Ρ‚ΡΡ‚Π²ΡƒΡŽΡ‰ΠΈΡ… элСмСнтов Π² строкС ΠΈ столбцС Π΄Ρ€ΡƒΠ³ΠΎΠΉ ΠΌΠ°Ρ‚Ρ€ΠΈΡ†Ρ‹.Рассмотрим Π΄Π²Π΅ ΠΌΠ°Ρ‚Ρ€ΠΈΡ†Ρ‹ A ΠΈ B ΠΎΠ΄Π½ΠΎΠ³ΠΎ порядка ‘m Γ— n’, Π³Π΄Π΅ m – количСство строк, Π° n – количСство столбцов Π΄Π²ΡƒΡ… ΠΌΠ°Ρ‚Ρ€ΠΈΡ†, ΠΎΠ±ΠΎΠ·Π½Π°Ρ‡Π΅Π½Π½Ρ‹Ρ… ΠΊΠ°ΠΊ A = [a ij ] ΠΈ B = [b ij ]. Π’Π΅ΠΏΠ΅Ρ€ΡŒ Ρ€Π°Π·Π½ΠΎΡΡ‚ΡŒ Π΄Π²ΡƒΡ… ΠΌΠ°Ρ‚Ρ€ΠΈΡ† A ΠΈ B опрСдСляСтся ΠΊΠ°ΠΊ: A – B = [a ij ] – [b ij ] = [a ij – b ij ], Π³Π΄Π΅ ij ΠΎΠ±ΠΎΠ·Π½Π°Ρ‡Π°Π΅Ρ‚ ΠΏΠΎΠ»ΠΎΠΆΠ΅Π½ΠΈΠ΅ ΠΊΠ°ΠΆΠ΄ΠΎΠ³ΠΎ элСмСнта Π² i -ΠΉ строкС ΠΈ j -ΠΉ столбцС . Π Π°Π·ΠΌΠ΅Ρ€Π½ΠΎΡΡ‚ΡŒ ΠΌΠ°Ρ‚Ρ€ΠΈΡ†Ρ‹ разностСй, Ρ‚ΠΎ Π΅ΡΡ‚ΡŒ A – B, Ρ‚Π°ΠΊΠΆΠ΅ Ρ€Π°Π²Π½Π° m Γ— n.

Π’Ρ‹Ρ‡ΠΈΡ‚Π°Π½ΠΈΠ΅ ΠΌΠ°Ρ‚Ρ€ΠΈΡ† порядка 2 Γ— 2

Как ΠΌΡ‹ Π·Π½Π°Π΅ΠΌ, Π²Ρ‹Ρ‡ΠΈΡ‚Π°Π½ΠΈΠ΅ ΠΌΠ°Ρ‚Ρ€ΠΈΡ† Π²ΠΎΠ·ΠΌΠΎΠΆΠ½ΠΎ Ρ‚ΠΎΠ»ΡŒΠΊΠΎ Π² Ρ‚ΠΎΠΌ случаС, Ссли ΠΌΠ°Ρ‚Ρ€ΠΈΡ†Ρ‹ ΠΈΠΌΠ΅ΡŽΡ‚ Ρ€Π°Π²Π½ΠΎΠ΅ количСство строк ΠΈ столбцов, поэтому для вычитания ΠΌΠ°Ρ‚Ρ€ΠΈΡ† порядка 2 Γ— 2 ΠΌΠ°Ρ‚Ρ€ΠΈΡ†Ρ‹ Π΄ΠΎΠ»ΠΆΠ½Ρ‹ ΠΈΠΌΠ΅Ρ‚ΡŒ 2 строки ΠΈ 2 столбца. Π’Π΅ΠΏΠ΅Ρ€ΡŒ рассмотрим Π΄Π²Π΅ ΠΌΠ°Ρ‚Ρ€ΠΈΡ†Ρ‹ A ΠΈ B с Ρ€Π°Π·ΠΌΠ΅Ρ€Π°ΠΌΠΈ 2 Γ— 2. Π§Ρ‚ΠΎΠ±Ρ‹ Π²Ρ‹Ρ‡Π΅ΡΡ‚ΡŒ B ΠΈΠ· A, ΠΌΡ‹ Π²Ρ‹Ρ‡Ρ‚Π΅ΠΌ элСмСнты B ΠΈΠ· ΡΠΎΠΎΡ‚Π²Π΅Ρ‚ΡΡ‚Π²ΡƒΡŽΡ‰ΠΈΡ… элСмСнтов A. ΠžΠ±Ρ‰Π°Ρ Ρ„ΠΎΡ€ΠΌΠ° вычитания B ΠΈΠ· A (порядок 2 Γ— 2): :

Π§Ρ‚ΠΎΠ±Ρ‹ Π»ΡƒΡ‡ΡˆΠ΅ ΠΏΠΎΠ½ΡΡ‚ΡŒ ΠΊΠΎΠ½Ρ†Π΅ΠΏΡ†ΠΈΡŽ ΠΌΠ°Ρ‚Ρ€ΠΈΡ‡Π½ΠΎΠ³ΠΎ вычитания размСрности 2 Γ— 2, Π΄Π°Π²Π°ΠΉΡ‚Π΅ рассмотрим ΠΏΡ€ΠΈΠΌΠ΅Ρ€ Π΄Π²ΡƒΡ… ΠΌΠ°Ρ‚Ρ€ΠΈΡ† A ΠΈ B ΠΈ Π²Ρ‹Ρ‡Ρ‚Π΅ΠΌ B ΠΈΠ· A.

Π’Ρ‹Ρ‡ΠΈΡ‚Π°Π½ΠΈΠ΅ ΠΌΠ°Ρ‚Ρ€ΠΈΡ† порядка 3 Γ— 3

Π’Ρ‹Ρ‡ΠΈΡ‚Π°Π½ΠΈΠ΅ ΠΌΠ°Ρ‚Ρ€ΠΈΡ† 3 Γ— 3 ΠΎΠ·Π½Π°Ρ‡Π°Π΅Ρ‚, Ρ‡Ρ‚ΠΎ Π²Ρ‹Ρ‡ΠΈΡ‚Π°Π΅ΠΌΡ‹Π΅ Π΄Ρ€ΡƒΠ³ ΠΈΠ· Π΄Ρ€ΡƒΠ³Π° ΠΌΠ°Ρ‚Ρ€ΠΈΡ†Ρ‹ ΠΈΠΌΠ΅ΡŽΡ‚ 3 строки ΠΈ 3 столбца. ΠŸΡ€ΠΈ Π²Ρ‹Ρ‡ΠΈΡ‚Π°Π½ΠΈΠΈ ΠΌΠ°Ρ‚Ρ€ΠΈΡ† ΠΌΡ‹ Π²Ρ‹Ρ‡ΠΈΡ‚Π°Π΅ΠΌ элСмСнты ΠΎΠ΄Π½ΠΎΠΉ ΠΌΠ°Ρ‚Ρ€ΠΈΡ†Ρ‹ ΠΈΠ· ΡΠΎΠΎΡ‚Π²Π΅Ρ‚ΡΡ‚Π²ΡƒΡŽΡ‰ΠΈΡ… элСмСнтов Π΄Ρ€ΡƒΠ³ΠΎΠΉ ΠΌΠ°Ρ‚Ρ€ΠΈΡ†Ρ‹. ΠžΠ±Ρ‰Π°Ρ Ρ„ΠΎΡ€ΠΌΠ° вычитания ΠΌΠ°Ρ‚Ρ€ΠΈΡ† A ΠΈ B порядка 3 Γ— 3:

ΠžΠ±Ρ€Π°Ρ‚ΠΈΡ‚Π΅ Π²Π½ΠΈΠΌΠ°Π½ΠΈΠ΅, Ρ‡Ρ‚ΠΎ для вычитания ΠΌΠ°Ρ‚Ρ€ΠΈΡ† ΠΌΠ°Ρ‚Ρ€ΠΈΡ†Ρ‹ Π½Π΅ ΠΎΠ±ΡΠ·Π°Ρ‚Π΅Π»ΡŒΠ½ΠΎ Π΄ΠΎΠ»ΠΆΠ½Ρ‹ Π±Ρ‹Ρ‚ΡŒ ΠΊΠ²Π°Π΄Ρ€Π°Ρ‚Π½Ρ‹ΠΌΠΈ.ΠœΠ°Ρ‚Ρ€ΠΈΡ‡Π½ΠΎΠ΅ Π²Ρ‹Ρ‡ΠΈΡ‚Π°Π½ΠΈΠ΅ ΠΏΡ€ΡΠΌΠΎΡƒΠ³ΠΎΠ»ΡŒΠ½Ρ‹Ρ… ΠΌΠ°Ρ‚Ρ€ΠΈΡ† Ρ‚Π°ΠΊΠΆΠ΅ опрСдСляСтся, Ссли порядок ΠΌΠ°Ρ‚Ρ€ΠΈΡ† ΠΎΠ΄ΠΈΠ½ ΠΈ Ρ‚ΠΎΡ‚ ΠΆΠ΅.

Бвойства ΠΌΠ°Ρ‚Ρ€ΠΈΡ‡Π½ΠΎΠ³ΠΎ вычитания

ВсС ограничСния Π½Π° Π΄ΠΎΠ±Π°Π²Π»Π΅Π½ΠΈΠ΅ ΠΌΠ°Ρ‚Ρ€ΠΈΡ† ΠΏΡ€ΠΈΠΌΠ΅Π½ΡΡŽΡ‚ΡΡ Ρ‚Π°ΠΊΠΆΠ΅ ΠΈ ΠΊ Π²Ρ‹Ρ‡ΠΈΡ‚Π°Π½ΠΈΡŽ ΠΌΠ°Ρ‚Ρ€ΠΈΡ†. Но Π΅ΡΡ‚ΡŒ ΠΎΠΏΡ€Π΅Π΄Π΅Π»Π΅Π½Π½Ρ‹Π΅ Π·Π°ΠΊΠΎΠ½Ρ‹, ΠΊΠΎΡ‚ΠΎΡ€Ρ‹ΠΌ Π²Ρ‹Ρ‡ΠΈΡ‚Π°Π½ΠΈΠ΅ ΠΌΠ°Ρ‚Ρ€ΠΈΡ† Π½Π΅ слСдуСт Ρ‚Π°ΠΊ ΠΆΠ΅, ΠΊΠ°ΠΊ Π²Ρ‹Ρ‡ΠΈΡ‚Π°Π½ΠΈΠ΅ чисСл. Бамая ваТная Π½Π΅ΠΎΠ±Ρ…ΠΎΠ΄ΠΈΠΌΠΎΡΡ‚ΡŒ для вычитания ΠΌΠ°Ρ‚Ρ€ΠΈΡ† для сохранСния всСх этих свойств состоит Π² Ρ‚ΠΎΠΌ, Ρ‡Ρ‚ΠΎ Π²Ρ‹Ρ‡ΠΈΡ‚Π°Π½ΠΈΠ΅ ΠΌΠ°Ρ‚Ρ€ΠΈΡ† опрСдСляСтся Ρ‚ΠΎΠ»ΡŒΠΊΠΎ Π² Ρ‚ΠΎΠΌ случаС, Ссли порядок ΠΌΠ°Ρ‚Ρ€ΠΈΡ† ΠΎΠ΄ΠΈΠ½ ΠΈ Ρ‚ΠΎΡ‚ ΠΆΠ΅.

  • ΠšΠΎΠ»ΠΈΡ‡Π΅ΡΡ‚Π²ΠΎ строк ΠΈ столбцов ΠΏΡ€ΠΈ Π²Ρ‹Ρ‡ΠΈΡ‚Π°Π½ΠΈΠΈ ΠΌΠ°Ρ‚Ρ€ΠΈΡ†Ρ‹ Π΄ΠΎΠ»ΠΆΠ½ΠΎ Π±Ρ‹Ρ‚ΡŒ ΠΎΠ΄ΠΈΠ½Π°ΠΊΠΎΠ²Ρ‹ΠΌ.
  • Π’Ρ‹Ρ‡ΠΈΡ‚Π°Π½ΠΈΠ΅ ΠΌΠ°Ρ‚Ρ€ΠΈΡ† Π½Π΅ ΠΊΠΎΠΌΠΌΡƒΡ‚Π°Ρ‚ΠΈΠ²Π½ΠΎ, Ρ‚ΠΎ Π΅ΡΡ‚ΡŒ A – B β‰  B – A
  • Π’Ρ‹Ρ‡ΠΈΡ‚Π°Π½ΠΈΠ΅ ΠΌΠ°Ρ‚Ρ€ΠΈΡ† Π½Π΅ ассоциативно, Ρ‚ΠΎ Π΅ΡΡ‚ΡŒ (A – B) – C β‰  A – (B – C)
  • Π’Ρ‹Ρ‡ΠΈΡ‚Π°Π½ΠΈΠ΅ ΠΌΠ°Ρ‚Ρ€ΠΈΡ†Ρ‹ ΠΈΠ· самой сСбя ΠΏΡ€ΠΈΠ²ΠΎΠ΄ΠΈΡ‚ ΠΊ Π½ΡƒΠ»Π΅Π²ΠΎΠΉ ΠΌΠ°Ρ‚Ρ€ΠΈΡ†Π΅, Ρ‚ΠΎ Π΅ΡΡ‚ΡŒ A – A = O.
  • Π’Ρ‹Ρ‡ΠΈΡ‚Π°Π½ΠΈΠ΅ ΠΌΠ°Ρ‚Ρ€ΠΈΡ† – это Π΄ΠΎΠ±Π°Π²Π»Π΅Π½ΠΈΠ΅ ΠΎΡ‚Ρ€ΠΈΡ†Π°Ρ‚Π΅Π»ΡŒΠ½ΠΎΠ³ΠΎ значСния ΠΎΠ΄Π½ΠΎΠΉ ΠΌΠ°Ρ‚Ρ€ΠΈΡ†Ρ‹ ΠΊ Π΄Ρ€ΡƒΠ³ΠΎΠΉ ΠΌΠ°Ρ‚Ρ€ΠΈΡ†Π΅, Ρ‚ΠΎ Π΅ΡΡ‚ΡŒ A – B = A + (-B).

Π’Π°ΠΆΠ½Ρ‹Π΅ примСчания ΠΏΠΎ Π²Ρ‹Ρ‡ΠΈΡ‚Π°Π½ΠΈΡŽ ΠΌΠ°Ρ‚Ρ€ΠΈΡ†

  • Π’Ρ‹Ρ‡ΠΈΡ‚Π°Π½ΠΈΠ΅ ΠΌΠ°Ρ‚Ρ€ΠΈΡ† Π²ΠΎΠ·ΠΌΠΎΠΆΠ½ΠΎ Ρ‚ΠΎΠ»ΡŒΠΊΠΎ Π² Ρ‚ΠΎΠΌ случаС, Ссли ΠΌΠ°Ρ‚Ρ€ΠΈΡ†Ρ‹ ΠΈΠΌΠ΅ΡŽΡ‚ ΠΎΠ΄ΠΈΠ½Π°ΠΊΠΎΠ²ΡƒΡŽ Ρ€Π°Π·ΠΌΠ΅Ρ€Π½ΠΎΡΡ‚ΡŒ.
  • Π’Ρ‹Ρ‡ΠΈΡ‚Π°Π½ΠΈΠ΅ ΠΌΠ°Ρ‚Ρ€ΠΈΡ† Π½Π΅ ΠΊΠΎΠΌΠΌΡƒΡ‚Π°Ρ‚ΠΈΠ²Π½ΠΎ ΠΈ ассоциативно.
  • Π’Ρ‹Ρ‡ΠΈΡ‚Π°Π΅ΠΌ ΡΠΎΠΎΡ‚Π²Π΅Ρ‚ΡΡ‚Π²ΡƒΡŽΡ‰ΠΈΠ΅ элСмСнты ΠΌΠ°Ρ‚Ρ€ΠΈΡ† для ΠΌΠ°Ρ‚Ρ€ΠΈΡ‡Π½ΠΎΠ³ΠΎ вычитания.

БвязанныС Ρ‚Π΅ΠΌΡ‹ с ΠΌΠ°Ρ‚Ρ€ΠΈΡ‡Π½Ρ‹ΠΌ Π²Ρ‹Ρ‡ΠΈΡ‚Π°Π½ΠΈΠ΅ΠΌ

Часто Π·Π°Π΄Π°Π²Π°Π΅ΠΌΡ‹Π΅ вопросы ΠΎ Π²Ρ‹Ρ‡ΠΈΡ‚Π°Π½ΠΈΠΈ ΠΌΠ°Ρ‚Ρ€ΠΈΡ†

Π§Ρ‚ΠΎ Ρ‚Π°ΠΊΠΎΠ΅ Π²Ρ‹Ρ‡ΠΈΡ‚Π°Π½ΠΈΠ΅ ΠΌΠ°Ρ‚Ρ€ΠΈΡ†?

Π’Ρ‹Ρ‡ΠΈΡ‚Π°Π½ΠΈΠ΅ ΠΌΠ°Ρ‚Ρ€ΠΈΡ† – это опСрация поэлСмСнтного вычитания ΠΌΠ°Ρ‚Ρ€ΠΈΡ† ΠΎΠ΄Π½ΠΎΠ³ΠΎ порядка, Ρ‚ΠΎ Π΅ΡΡ‚ΡŒ ΠΌΠ°Ρ‚Ρ€ΠΈΡ†, ΠΈΠΌΠ΅ΡŽΡ‰ΠΈΡ… ΠΎΠ΄ΠΈΠ½Π°ΠΊΠΎΠ²ΠΎΠ΅ количСство строк ΠΈ столбцов.Вычитая Π΄Π²Π΅ ΠΌΠ°Ρ‚Ρ€ΠΈΡ†Ρ‹, ΠΌΡ‹ Π²Ρ‹Ρ‡ΠΈΡ‚Π°Π΅ΠΌ элСмСнты Π² ΠΊΠ°ΠΆΠ΄ΠΎΠΉ строкС ΠΈ столбцС ΠΈΠ· ΡΠΎΠΎΡ‚Π²Π΅Ρ‚ΡΡ‚Π²ΡƒΡŽΡ‰ΠΈΡ… элСмСнтов Π² строкС ΠΈ столбцС Π΄Ρ€ΡƒΠ³ΠΎΠΉ ΠΌΠ°Ρ‚Ρ€ΠΈΡ†Ρ‹.

Как Π²Ρ‹ΠΏΠΎΠ»Π½ΡΡ‚ΡŒ Π²Ρ‹Ρ‡ΠΈΡ‚Π°Π½ΠΈΠ΅ ΠΌΠ°Ρ‚Ρ€ΠΈΡ†?

Рассмотрим Π΄Π²Π΅ ΠΌΠ°Ρ‚Ρ€ΠΈΡ†Ρ‹ A ΠΈ B ΠΎΠ΄Π½ΠΎΠ³ΠΎ порядка ‘m Γ— n’, Π³Π΄Π΅ m – количСство строк, Π° n – количСство столбцов Π΄Π²ΡƒΡ… ΠΌΠ°Ρ‚Ρ€ΠΈΡ†, ΠΎΠ±ΠΎΠ·Π½Π°Ρ‡Π΅Π½Π½Ρ‹Ρ… ΠΊΠ°ΠΊ A = [a ij ] ΠΈ B = [b ij ]. Π’Π΅ΠΏΠ΅Ρ€ΡŒ Ρ€Π°Π·Π½ΠΎΡΡ‚ΡŒ Π΄Π²ΡƒΡ… ΠΌΠ°Ρ‚Ρ€ΠΈΡ† A ΠΈ B опрСдСляСтся ΠΊΠ°ΠΊ: A – B = [a ij ] – [b ij ] = [a ij – b ij ], Π³Π΄Π΅ ij ΠΎΠ±ΠΎΠ·Π½Π°Ρ‡Π°Π΅Ρ‚ ΠΏΠΎΠ»ΠΎΠΆΠ΅Π½ΠΈΠ΅ ΠΊΠ°ΠΆΠ΄ΠΎΠ³ΠΎ элСмСнта Π² i -ΠΉ строкС ΠΈ j -ΠΉ столбцС .Π Π°Π·ΠΌΠ΅Ρ€Π½ΠΎΡΡ‚ΡŒ ΠΌΠ°Ρ‚Ρ€ΠΈΡ†Ρ‹ разностСй, Ρ‚ΠΎ Π΅ΡΡ‚ΡŒ A – B, Ρ‚Π°ΠΊΠΆΠ΅ Ρ€Π°Π²Π½Π° m Γ— n.

ЯвляСтся Π»ΠΈ

Π²Ρ‹Ρ‡ΠΈΡ‚Π°Π½ΠΈΠ΅ ΠΌΠ°Ρ‚Ρ€ΠΈΡ† ΠΊΠΎΠΌΠΌΡƒΡ‚Π°Ρ‚ΠΈΠ²Π½Ρ‹ΠΌ?

Π’Ρ‹Ρ‡ΠΈΡ‚Π°Π½ΠΈΠ΅ ΠΌΠ°Ρ‚Ρ€ΠΈΡ† Π½Π΅ ΠΊΠΎΠΌΠΌΡƒΡ‚Π°Ρ‚ΠΈΠ²Π½ΠΎ, Ρ‚ΠΎ Π΅ΡΡ‚ΡŒ A – B β‰  B – A. Как ΠΈ Π²Ρ‹Ρ‡ΠΈΡ‚Π°Π½ΠΈΠ΅ чисСл, Π²Ρ‹Ρ‡ΠΈΡ‚Π°Π½ΠΈΠ΅ ΠΌΠ°Ρ‚Ρ€ΠΈΡ† Ρ‚Π°ΠΊΠΆΠ΅ ΠΈΠΌΠ΅Π΅Ρ‚ ΠΎΠΏΡ€Π΅Π΄Π΅Π»Π΅Π½Π½Ρ‹Π΅ ограничСния.

КакоС Π½Π΅ΠΎΠ±Ρ…ΠΎΠ΄ΠΈΠΌΠΎΠ΅ условиС для вычитания ΠΌΠ°Ρ‚Ρ€ΠΈΡ†?

Для вычитания ΠΌΠ°Ρ‚Ρ€ΠΈΡ† Π½Π΅ΠΎΠ±Ρ…ΠΎΠ΄ΠΈΠΌΠΎ, Ρ‡Ρ‚ΠΎΠ±Ρ‹ ΠΎΠ½ΠΈ ΠΈΠΌΠ΅Π»ΠΈ ΠΎΠ΄ΠΈΠ½Π°ΠΊΠΎΠ²ΠΎΠ΅ количСство строк ΠΈ столбцов.Π’Ρ‹Ρ‡ΠΈΡ‚Π°Π΅ΠΌΡ‹Π΅ ΠΌΠ°Ρ‚Ρ€ΠΈΡ†Ρ‹ Π΄ΠΎΠ»ΠΆΠ½Ρ‹ ΠΈΠΌΠ΅Ρ‚ΡŒ ΠΎΠ΄ΠΈΠ½Π°ΠΊΠΎΠ²ΡƒΡŽ Ρ€Π°Π·ΠΌΠ΅Ρ€Π½ΠΎΡΡ‚ΡŒ.

МоТно Π»ΠΈ Π²Ρ‹Ρ‡Π΅ΡΡ‚ΡŒ Π΄Π²Π΅ ΠΌΠ°Ρ‚Ρ€ΠΈΡ†Ρ‹ Ρ€Π°Π·Π½ΠΎΠ³ΠΎ порядка?

ΠœΠ°Ρ‚Ρ€ΠΈΡ†Ρ‹ Ρ€Π°Π·Π½ΠΎΠ³ΠΎ порядка Π½Π΅ ΠΌΠΎΠ³ΡƒΡ‚ Π±Ρ‹Ρ‚ΡŒ Π²Ρ‹Ρ‡Ρ‚Π΅Π½Ρ‹, Ρ‚Π°ΠΊ ΠΊΠ°ΠΊ ΠΏΡ€ΠΈ Π²Ρ‹Ρ‡ΠΈΡ‚Π°Π½ΠΈΠΈ Π»ΡŽΠ±Ρ‹Ρ… Π΄Π²ΡƒΡ… ΠΌΠ°Ρ‚Ρ€ΠΈΡ† ΠΌΡ‹ Π²Ρ‹Ρ‡ΠΈΡ‚Π°Π΅ΠΌ элСмСнты ΠΎΠ΄Π½ΠΎΠΉ ΠΌΠ°Ρ‚Ρ€ΠΈΡ†Ρ‹ ΠΈΠ· ΡΠΎΠΎΡ‚Π²Π΅Ρ‚ΡΡ‚Π²ΡƒΡŽΡ‰ΠΈΡ… элСмСнтов Π΄Ρ€ΡƒΠ³ΠΎΠΉ ΠΌΠ°Ρ‚Ρ€ΠΈΡ†Ρ‹. Если порядки Ρ€Π°Π·Π½Ρ‹Π΅, Ρ‚ΠΎ Π² ΠΎΠ΄Π½ΠΎΠΉ ΠΈΠ· ΠΌΠ°Ρ‚Ρ€ΠΈΡ† ΠΎΡ‚ΡΡƒΡ‚ΡΡ‚Π²ΡƒΡŽΡ‚ ΡΠΎΠΎΡ‚Π²Π΅Ρ‚ΡΡ‚Π²ΡƒΡŽΡ‰ΠΈΠ΅ элСмСнты.

ЯвляСтся Π»ΠΈ ассоциативноС Π²Ρ‹Ρ‡ΠΈΡ‚Π°Π½ΠΈΠ΅ ΠΌΠ°Ρ‚Ρ€ΠΈΡ†Ρ‹?

Π’Ρ‹Ρ‡ΠΈΡ‚Π°Π½ΠΈΠ΅ ΠΌΠ°Ρ‚Ρ€ΠΈΡ† Π½Π΅ ассоциативно, Ρ‚ΠΎ Π΅ΡΡ‚ΡŒ (A – B) – C β‰  A – (B – C).Как ΠΈ Π²Ρ‹Ρ‡ΠΈΡ‚Π°Π½ΠΈΠ΅ чисСл, Π²Ρ‹Ρ‡ΠΈΡ‚Π°Π½ΠΈΠ΅ ΠΌΠ°Ρ‚Ρ€ΠΈΡ† Ρ‚Π°ΠΊΠΆΠ΅ ΠΈΠΌΠ΅Π΅Ρ‚ ΠΎΠΏΡ€Π΅Π΄Π΅Π»Π΅Π½Π½Ρ‹Π΅ ограничСния.

Π‘Π»ΠΎΠΆΠ΅Π½ΠΈΠ΅ ΠΈ Π²Ρ‹Ρ‡ΠΈΡ‚Π°Π½ΠΈΠ΅ ΠΌΠ°Ρ‚Ρ€ΠΈΡ† ΠΈ ΡƒΠΌΠ½ΠΎΠΆΠ΅Π½ΠΈΠ΅ ΠΌΠ°Ρ‚Ρ€ΠΈΡ†Ρ‹ Π½Π° константу

ΠŸΡ€ΠΈ ΡƒΠΌΠ½ΠΎΠΆΠ΅Π½ΠΈΠΈ ΠΌΠ°Ρ‚Ρ€ΠΈΡ†Ρ‹ Π½Π° скаляр (константа ΠΈΠ»ΠΈ число) ΠΈΠ»ΠΈ слоТСнии ΠΈ Π²Ρ‹Ρ‡ΠΈΡ‚Π°Π½ΠΈΠΈ ΠΌΠ°Ρ‚Ρ€ΠΈΡ† ΠΎΠΏΠ΅Ρ€Π°Ρ†ΠΈΠΈ Π²Ρ‹ΠΏΠΎΠ»Π½ΡΡŽΡ‚ΡΡ построчно. Π”Π°Π²Π°ΠΉΡ‚Π΅ рассмотрим ΠΊΠ°ΠΆΠ΄ΡƒΡŽ ΠΎΠΏΠ΅Ρ€Π°Ρ†ΠΈΡŽ ΠΎΡ‚Π΄Π΅Π»ΡŒΠ½ΠΎ, Ρ‡Ρ‚ΠΎΠ±Ρ‹ ΡƒΠ²ΠΈΠ΄Π΅Ρ‚ΡŒ, ΠΊΠ°ΠΊ это Ρ€Π°Π±ΠΎΡ‚Π°Π΅Ρ‚.

Π‘ΠΎΠ΄Π΅Ρ€ΠΆΠ°Π½ΠΈΠ΅

  1. Π”ΠΎΠ±Π°Π²Π»Π΅Π½ΠΈΠ΅ ΠΌΠ°Ρ‚Ρ€ΠΈΡ†
  2. Π’Ρ‹Ρ‡ΠΈΡ‚Π°Π½ΠΈΠ΅ ΠΌΠ°Ρ‚Ρ€ΠΈΡ†
  3. Π£ΠΌΠ½ΠΎΠΆΠ΅Π½ΠΈΠ΅ ΠΌΠ°Ρ‚Ρ€ΠΈΡ†Ρ‹ Π½Π° константу (скалярноС ΡƒΠΌΠ½ΠΎΠΆΠ΅Π½ΠΈΠ΅)
  4. Π‘ΠΎΡ‡Π΅Ρ‚Π°Π½ΠΈΠ΅ слоТСния, вычитания ΠΈ скалярного умноТСния

Ρ€Π΅ΠΊΠ»Π°ΠΌΠ°

Π”ΠΎΠ±Π°Π²Π»Π΅Π½ΠΈΠ΅ ΠΌΠ°Ρ‚Ρ€ΠΈΡ†

Π§Ρ‚ΠΎΠ±Ρ‹ Π΄ΠΎΠ±Π°Π²ΠΈΡ‚ΡŒ Π΄Π²Π΅ ΠΌΠ°Ρ‚Ρ€ΠΈΡ†Ρ‹, Π΄ΠΎΠ±Π°Π²ΡŒΡ‚Π΅ ΡΠΎΠΎΡ‚Π²Π΅Ρ‚ΡΡ‚Π²ΡƒΡŽΡ‰ΠΈΠ΅ записи, ΠΊΠ°ΠΊ ΠΏΠΎΠΊΠ°Π·Π°Π½ΠΎ Π½ΠΈΠΆΠ΅.ΠžΠ±Ρ€Π°Ρ‚ΠΈΡ‚Π΅ Π²Π½ΠΈΠΌΠ°Π½ΠΈΠ΅, Ρ‡Ρ‚ΠΎ Π²Π°ΠΌ Π½ΡƒΠΆΠ½ΠΎ, Ρ‡Ρ‚ΠΎΠ±Ρ‹ ΠΌΠ°Ρ‚Ρ€ΠΈΡ†Ρ‹ Π±Ρ‹Π»ΠΈ ΠΎΠ΄ΠΈΠ½Π°ΠΊΠΎΠ²ΠΎΠ³ΠΎ Ρ€Π°Π·ΠΌΠ΅Ρ€Π°, Ρ‡Ρ‚ΠΎΠ±Ρ‹ это ΠΈΠΌΠ΅Π»ΠΎ смысл.

Если ΠΌΠ°Ρ‚Ρ€ΠΈΡ†Ρ‹ Ρ€Π°Π·Π½ΠΎΠ³ΠΎ Ρ€Π°Π·ΠΌΠ΅Ρ€Π°, слоТСниС Π½Π΅ ΠΎΠΏΡ€Π΅Π΄Π΅Π»Π΅Π½ΠΎ.

ΠœΠ°Ρ‚Ρ€ΠΈΡ†Ρ‹ вычитания

Π’Ρ‹Ρ‡ΠΈΡ‚Π°Π½ΠΈΠ΅ ΠΌΠ°Ρ‚Ρ€ΠΈΡ† Ρ€Π°Π±ΠΎΡ‚Π°Π΅Ρ‚ Ρ‚ΠΎΡ‡Π½ΠΎ Ρ‚Π°ΠΊ ΠΆΠ΅. Π’Ρ‹ ΠΌΠΎΠΆΠ΅Ρ‚Π΅ Π²Ρ‹Ρ‡Π΅ΡΡ‚ΡŒ запись Π·Π° записью.

Как ΠΈ Π² случаС слоТСния, это Π½Π΅ Π±Ρ‹Π»ΠΎ Π±Ρ‹ ΠΎΠΏΡ€Π΅Π΄Π΅Π»Π΅Π½ΠΎ, Ссли Π±Ρ‹ ΠΌΠ°Ρ‚Ρ€ΠΈΡ†Ρ‹ Π±Ρ‹Π»ΠΈ Ρ€Π°Π·Π½Ρ‹Ρ… Ρ€Π°Π·ΠΌΠ΅Ρ€ΠΎΠ². Π’ этой ситуации ваш ΠΎΡ‚Π²Π΅Ρ‚ Π±ΡƒΠ΄Π΅Ρ‚ просто Β«Π½Π΅ ΠΎΠΏΡ€Π΅Π΄Π΅Π»Π΅Π½Β».

Π£ΠΌΠ½ΠΎΠΆΠ΅Π½ΠΈΠ΅ ΠΌΠ°Ρ‚Ρ€ΠΈΡ†Ρ‹ Π½Π° константу (скалярноС ΡƒΠΌΠ½ΠΎΠΆΠ΅Π½ΠΈΠ΅)

Π£ΠΌΠ½ΠΎΠΆΠ΅Π½ΠΈΠ΅ ΠΌΠ°Ρ‚Ρ€ΠΈΡ†Ρ‹ Π½Π° константу ΠΈΠ»ΠΈ число (ΠΈΠ½ΠΎΠ³Π΄Π° Π½Π°Π·Ρ‹Π²Π°Π΅ΠΌΠΎΠ΅ скаляром) всСгда опрСдСляСтся нСзависимо ΠΎΡ‚ Ρ€Π°Π·ΠΌΠ΅Ρ€Π° ΠΌΠ°Ρ‚Ρ€ΠΈΡ†Ρ‹.Π’Π°ΠΌ просто Π½ΡƒΠΆΠ½ΠΎ ΡƒΠ±Π΅Π΄ΠΈΡ‚ΡŒΡΡ, Ρ‡Ρ‚ΠΎ каТдая запись Π² ΠΌΠ°Ρ‚Ρ€ΠΈΡ†Π΅ ΡƒΠΌΠ½ΠΎΠΆΠ΅Π½Π° Π½Π° число.

ОбъСдинСниС ΠΎΠΏΠ΅Ρ€Π°Ρ†ΠΈΠΉ

Π’ Π½Π΅ΠΊΠΎΡ‚ΠΎΡ€Ρ‹Ρ… вопросах вас ΠΌΠΎΠ³ΡƒΡ‚ ΠΏΠΎΠΏΡ€ΠΎΡΠΈΡ‚ΡŒ ΠΎΠ±ΡŠΠ΅Π΄ΠΈΠ½ΠΈΡ‚ΡŒ слоТСниС, Π²Ρ‹Ρ‡ΠΈΡ‚Π°Π½ΠΈΠ΅ ΠΈ ΡƒΠΌΠ½ΠΎΠΆΠ΅Π½ΠΈΠ΅ Π½Π° константу. Π—Π΄Π΅ΡΡŒ ΠΌΡ‹ рассмотрим нСсколько ΠΏΡ€ΠΈΠΌΠ΅Ρ€ΠΎΠ², Ρ‡Ρ‚ΠΎΠ±Ρ‹ ΡƒΠ±Π΅Π΄ΠΈΡ‚ΡŒΡΡ, Ρ‡Ρ‚ΠΎ Π²Ρ‹ Π·Π½Π°Π΅Ρ‚Π΅, ΠΊΠ°ΠΊ это ΡΠ΄Π΅Π»Π°Ρ‚ΡŒ.

ΠŸΡ€ΠΈΠΌΠ΅Ρ€

НайдитС \ (- 2A + B \) для:

\ (A = \ left [\ begin {array} {cc} -4 & 1 \\ 2 & -2 \\ \ end {array} \ right] \) ΠΈ \ (B = \ left [\ begin {array} {cc} 9 & -4 \\ 0 & 8 \\ \ end {array} \ right] \)

НС Π·Π°Π±ΡƒΠ΄ΡŒΡ‚Π΅ ΡƒΠΌΠ½ΠΎΠΆΠ°Ρ‚ΡŒ ΠΊΠ°ΠΆΠ΄ΡƒΡŽ запись Π½Π° константу ΠΈ Ρ€Π°Π±ΠΎΡ‚Π°Ρ‚ΡŒ с записью ΠΏΡ€ΠΈ слоТСнии.

\ (\ begin {align} -2A + B & = 2 \ left [\ begin {array} {cc} -4 & 1 \\ 2 & -2 \\ \ end {array} \ right] + \ left [\ begin {array} {cc} 9 & -4 \\ 0 & 8 \\ \ end {array} \ right] \\ & = \ left [\ begin {array} {cc} -2 \ times -4 & -2 \ times 1 \\ -2 \ times 2 & -2 \ times -2 \\ \ end {array} \ right] + \ left [\ begin {array} {cc} 9 & -4 \\ 0 & 8 \\ \ end {array} \ right] \\ & = \ left [\ begin {array} {cc} 8 & -2 \\ -4 & 4 \\ \ end {array} \ right] + \ left [\ begin { массив} {cc} 9 & -4 \\ 0 & 8 \\ \ end {array} \ right] \\ & = \ left [\ begin {array} {cc} 8 + 9 & -2 + (-4) \\ -4 + 0 & 4 + 8 \\ \ end {array} \ right] \\ & = \ boxed {\ left [\ begin {array} {cc} 17 & -6 \\ -4 & 12 \\ \ end {array} \ right]} \ end {align} \)

Π­Ρ‚ΠΎ Ρ‚Π°ΠΊΠΆΠ΅ Ρ€Π°Π±ΠΎΡ‚Π°Π΅Ρ‚, ΠΊΠΎΠ³Π΄Π° Ρƒ вас Π±ΠΎΠ»Π΅Π΅ Π΄Π²ΡƒΡ… ΠΌΠ°Ρ‚Ρ€ΠΈΡ†, ΠΊΠ°ΠΊ ΠΏΠΎΠΊΠ°Π·Π°Π½ΠΎ Π² ΡΠ»Π΅Π΄ΡƒΡŽΡ‰Π΅ΠΌ ΠΏΡ€ΠΈΠΌΠ΅Ρ€Π΅.

ΠŸΡ€ΠΈΠΌΠ΅Ρ€

НайдитС \ (A – 3B + 2C \) для:

\ (A = \ left [\ begin {array} {cc} 1 & 1 \\ 0 & 0 \\ \ end {array} \ right] \), \ (B = \ left [\ begin {array} {cc} 2 & 1 \\ 1 & 4 \\ \ end {array} \ right] \) ΠΈ \ (C = \ left [\ begin {array} { cc} 5 & 2 \\ 3 & 0 \\ \ end {array} \ right] \)

\ (\ begin {align} A – 3B + 2C & = \ left [\ begin {array} {cc} 1 & 1 \\ 0 & 0 \\ \ end {array} \ right] – 3 \ left [\ begin {массив} {cc} 2 & 1 \\ 1 & 4 \\ \ end {array} \ right] + 2 \ left [\ begin {array} {cc} 5 & 2 \\ 3 & 0 \\ \ end { массив} \ right] \\ & = \ left [\ begin {array} {cc} 1 & 1 \\ 0 & 0 \\ \ end {array} \ right] – \ left [\ begin {array} {cc} 3 \ times2 ΠΈ 3 \ times1 \\ 3 \ times1 & 3 \ times4 \\ \ end {array} \ right] + \ left [\ begin {array} {cc} 2 \ times5 & 2 \ times2 \\ 2 \ times3 & 2 \ times0 \\ \ end {массив} \ right] \\ & = \ left [\ begin {array} {cc} 1 & 1 \\ 0 & 0 \\ \ end {array} \ right] – \ left [\ begin {array} {cc} 6 & 3 \\ 3 & 12 \\ \ end {array} \ right] + \ left [\ begin {array} {cc} 10 & 4 \\ 6 & 0 \\ \ ΠΊΠΎΠ½Π΅Ρ† {массив} \ right] \\ & = \ left [\ begin {array} {cc} 1 – 6 + 10 & 1 – 3 + 4 \\ 0 – 3 + 6 & 0 – 12 + 0 \\ \ end {массив} \ right] \\ & = \ boxed {\ left [\ begin {array} {cc} 5 & 2 \\ 3 & – 12 \\ \ end {array} \ right]} \ end {align} \ )

РСзюмС

Π—Π°ΠΏΠΎΠΌΠ½ΠΈΡ‚Π΅ ΡΠ»Π΅Π΄ΡƒΡŽΡ‰Π΅Π΅ для ΠΎΠΏΠ΅Ρ€Π°Ρ†ΠΈΠΉ с ΠΌΠ°Ρ‚Ρ€ΠΈΡ†Π°ΠΌΠΈ:

  • Π§Ρ‚ΠΎΠ±Ρ‹ Π΄ΠΎΠ±Π°Π²ΠΈΡ‚ΡŒ ΠΈΠ»ΠΈ Π²Ρ‹Ρ‡Π΅ΡΡ‚ΡŒ, ΠΏΠ΅Ρ€Π΅Ρ…ΠΎΠ΄ΠΈΡ‚Π΅ Π²Π²ΠΎΠ΄ Π·Π° Π²Π²ΠΎΠ΄ΠΎΠΌ.
  • Π‘Π»ΠΎΠΆΠ΅Π½ΠΈΠ΅ ΠΈ Π²Ρ‹Ρ‡ΠΈΡ‚Π°Π½ΠΈΠ΅ ΠΎΠΏΡ€Π΅Π΄Π΅Π»ΡΡŽΡ‚ΡΡ Ρ‚ΠΎΠ»ΡŒΠΊΠΎ Π² Ρ‚ΠΎΠΌ случаС, Ссли ΠΌΠ°Ρ‚Ρ€ΠΈΡ†Ρ‹ ΠΎΠ΄ΠΈΠ½Π°ΠΊΠΎΠ²ΠΎΠ³ΠΎ Ρ€Π°Π·ΠΌΠ΅Ρ€Π°.
  • БкалярноС ΡƒΠΌΠ½ΠΎΠΆΠ΅Π½ΠΈΠ΅ всСгда опрСдСляСтся – просто ΡƒΠΌΠ½ΠΎΠΆΡŒΡ‚Π΅ ΠΊΠ°ΠΆΠ΄Ρ‹ΠΉ элСмСнт ΠΌΠ°Ρ‚Ρ€ΠΈΡ†Ρ‹ Π½Π° скаляр.

Ρ€Π΅ΠΊΠ»Π°ΠΌΠ°

ΠŸΡ€ΠΎΠ΄ΠΎΠ»ΠΆΠ°ΠΉΡ‚Π΅ ΠΈΠ·ΡƒΡ‡Π΅Π½ΠΈΠ΅ ΠΌΠ°Ρ‚Ρ€ΠΈΡ‡Π½Ρ‹Ρ… ΠΎΠΏΠ΅Ρ€Π°Ρ†ΠΈΠΉ

Π”Π°Π»Π΅Π΅: Π£ΠΌΠ½ΠΎΠΆΠ΅Π½ΠΈΠ΅ ΠΌΠ°Ρ‚Ρ€ΠΈΡ†

ΠŸΠΎΠ΄ΠΏΠΈΡˆΠΈΡ‚Π΅ΡΡŒ Π½Π° Π½Π°ΡˆΡƒ рассылку новостСй!

ΠœΡ‹ всСгда ΠΏΡƒΠ±Π»ΠΈΠΊΡƒΠ΅ΠΌ Π½ΠΎΠ²Ρ‹Π΅ бСсплатныС ΡƒΡ€ΠΎΠΊΠΈ ΠΈ добавляСм Π½ΠΎΠ²Ρ‹Π΅ ΡƒΡ‡Π΅Π±Π½Ρ‹Π΅ пособия, руководства ΠΏΠΎ ΠΊΠ°Π»ΡŒΠΊΡƒΠ»ΡΡ‚ΠΎΡ€Ρƒ ΠΈ ΠΏΠ°ΠΊΠ΅Ρ‚Ρ‹ Π·Π°Π΄Π°Ρ‡.

ΠŸΠΎΠ΄ΠΏΠΈΡˆΠΈΡ‚Π΅ΡΡŒ, Ρ‡Ρ‚ΠΎΠ±Ρ‹ ΠΏΠΎΠ»ΡƒΡ‡Π°Ρ‚ΡŒ элСктронныС письма (Ρ€Π°Π· Π² ΠΏΠ°Ρ€Ρƒ ΠΈΠ»ΠΈ Ρ‚Ρ€ΠΈ Π½Π΅Π΄Π΅Π»ΠΈ) с ΠΈΠ½Ρ„ΠΎΡ€ΠΌΠ°Ρ†ΠΈΠ΅ΠΉ ΠΎ Π½ΠΎΠ²ΠΈΠ½ΠΊΠ°Ρ…!

БвязанныС

Π’Ρ‹Ρ‡ΠΈΡ‚Π°Π½ΠΈΠ΅ ΠΌΠ°Ρ‚Ρ€ΠΈΡ†Ρ‹ – объяснСниС ΠΈ ΠΏΡ€ΠΈΠΌΠ΅Ρ€Ρ‹

Π•ΡΡ‚ΡŒ Π±Π°Π·ΠΎΠ²Ρ‹Π΅ ΠΎΠΏΠ΅Ρ€Π°Ρ†ΠΈΠΈ ΡΡ‚ΠΎΠΈΠΌΠΎΡΡ‚ΡŒΡŽ 4 Π΄ΠΎΠ»Π»Π°Ρ€Π°, ΠΊΠΎΡ‚ΠΎΡ€Ρ‹Π΅ ΠΌΡ‹ ΠΌΠΎΠΆΠ΅ΠΌ Π²Ρ‹ΠΏΠΎΠ»Π½ΡΡ‚ΡŒ с ΠΌΠ°Ρ‚Ρ€ΠΈΡ†Π°ΠΌΠΈ. Π­Ρ‚ΠΎ слоТСниС, Π²Ρ‹Ρ‡ΠΈΡ‚Π°Π½ΠΈΠ΅, скалярноС ΡƒΠΌΠ½ΠΎΠΆΠ΅Π½ΠΈΠ΅ ΠΈ ΡƒΠΌΠ½ΠΎΠΆΠ΅Π½ΠΈΠ΅ ΠΌΠ°Ρ‚Ρ€ΠΈΡ†. ΠœΠ°Ρ‚Ρ€ΠΈΡ‡Π½ΠΎΠ΅ Π²Ρ‹Ρ‡ΠΈΡ‚Π°Π½ΠΈΠ΅ являСтся ΠΎΠ΄Π½ΠΈΠΌ ΠΈΠ· Π½ΠΈΡ…, ΠΈ ΠΌΡ‹ рассмотрим Π΅Π³ΠΎ Π² этой ΡΡ‚Π°Ρ‚ΡŒΠ΅.

ΠŸΡ€ΠΈΠΌΠ΅Ρ‡Π°Π½ΠΈΠ΅. Π”Π΅Π»Π΅Π½ΠΈΠ΅ ΠΌΠ°Ρ‚Ρ€ΠΈΡ†Ρ‹ Π½Π΅ являСтся ΠΎΠΏΡ€Π΅Π΄Π΅Π»Π΅Π½Π½ΠΎΠΉ ΠΎΠΏΠ΅Ρ€Π°Ρ†ΠΈΠ΅ΠΉ.ΠœΡ‹ ΠΌΠΎΠΆΠ΅ΠΌ Ρ€Π°Π·Π΄Π΅Π»ΠΈΡ‚ΡŒ ΠΌΠ°Ρ‚Ρ€ΠΈΡ†Ρƒ Ρ‚ΠΎΠ»ΡŒΠΊΠΎ Π½Π° скалярноС число. Π§Ρ‚ΠΎΠ±Ρ‹ ΡƒΠ·Π½Π°Ρ‚ΡŒ большС ΠΎ ΠΌΠ°Ρ‚Ρ€ΠΈΡ‡Π½ΠΎΠΌ Π΄Π΅Π»Π΅Π½ΠΈΠΈ, Ρ‰Π΅Π»ΠΊΠ½ΠΈΡ‚Π΅ здСсь.

ΠžΠΏΡ€Π΅Π΄Π΅Π»ΠΈΠΌ ΠΌΠ°Ρ‚Ρ€ΠΈΡ‡Π½ΠΎΠ΅ Π²Ρ‹Ρ‡ΠΈΡ‚Π°Π½ΠΈΠ΅:

ΠœΠ°Ρ‚Ρ€ΠΈΡ‡Π½ΠΎΠ΅ Π²Ρ‹Ρ‡ΠΈΡ‚Π°Π½ΠΈΠ΅ – это опСрация вычитания Π΄Π²ΡƒΡ… ΠΌΠ°Ρ‚Ρ€ΠΈΡ† ΠΎΠ΄ΠΈΠ½Π°ΠΊΠΎΠ²ΠΎΠΉ размСрности ΠΏΡƒΡ‚Π΅ΠΌ вычитания ΡΠΎΠΎΡ‚Π²Π΅Ρ‚ΡΡ‚Π²ΡƒΡŽΡ‰ΠΈΡ… элСмСнтов.

Π’ этой ΡΡ‚Π°Ρ‚ΡŒΠ΅ Π±ΡƒΠ΄ΡƒΡ‚ ΠΏΠΎΠΊΠ°Π·Π°Π½Ρ‹ условия, ΠΏΡ€ΠΈ ΠΊΠΎΡ‚ΠΎΡ€Ρ‹Ρ… Π²ΠΎΠ·ΠΌΠΎΠΆΠ½ΠΎ Π²Ρ‹Ρ‡ΠΈΡ‚Π°Π½ΠΈΠ΅ ΠΌΠ°Ρ‚Ρ€ΠΈΡ†, Π²Ρ‹Ρ‡ΠΈΡ‚Π°Π½ΠΈΠ΅ ΠΌΠ°Ρ‚Ρ€ΠΈΡ† ΠΈ нСсколько ΠΏΡ€ΠΈΠΌΠ΅Ρ€ΠΎΠ², Π΄Π΅ΠΌΠΎΠ½ΡΡ‚Ρ€ΠΈΡ€ΡƒΡŽΡ‰ΠΈΡ… этот процСсс.

Когда ΠΌΠΎΠΆΠ½ΠΎ Π²Ρ‹Ρ‡ΠΈΡ‚Π°Ρ‚ΡŒ ΠΌΠ°Ρ‚Ρ€ΠΈΡ†Ρ‹?

МоТно Π»ΠΈ Π²Ρ‹Ρ‡Π΅ΡΡ‚ΡŒ ΠΊΠ°ΠΊΠΈΠ΅-Π»ΠΈΠ±ΠΎ ΠΌΠ°Ρ‚Ρ€ΠΈΡ†Ρ‹?

НСт!

ΠœΡ‹ ΠΌΠΎΠΆΠ΅ΠΌ Π²Ρ‹Ρ‡Π΅ΡΡ‚ΡŒ ΠΌΠ°Ρ‚Ρ€ΠΈΡ†Ρ‹ ΠΏΠΎ 2 Π΄ΠΎΠ»Π»Π°Ρ€Π°, Ρ‚ΠΎΠ»ΡŒΠΊΠΎ Ссли ΠΎΠ½ΠΈ ΠΈΠΌΠ΅ΡŽΡ‚ ΠΎΠ΄ΠΈΠ½Π°ΠΊΠΎΠ²ΡƒΡŽ Ρ€Π°Π·ΠΌΠ΅Ρ€Π½ΠΎΡΡ‚ΡŒ. Π’ΠΎ Π΅ΡΡ‚ΡŒ, Ссли ΠΌΠ°Ρ‚Ρ€ΠΈΡ†Ρ‹ $ 2 $ ΠΈΠΌΠ΅ΡŽΡ‚ ΠΎΠ΄ΠΈΠ½Π°ΠΊΠΎΠ²ΠΎΠ΅ количСство строк ΠΈ столбцов, Ρ‚ΠΎΠ³Π΄Π° ΠΈ Ρ‚ΠΎΠ»ΡŒΠΊΠΎ Ρ‚ΠΎΠ³Π΄Π° ΠΌΡ‹ ΠΌΠΎΠΆΠ΅ΠΌ Π²Ρ‹Ρ‡Π΅ΡΡ‚ΡŒ ΠΌΠ°Ρ‚Ρ€ΠΈΡ†Ρ‹ $ 2 $ вмСстС.

Напомним, Ρ‡Ρ‚ΠΎ Ρ€Π°Π·ΠΌΠ΅Ρ€Π½ΠΎΡΡ‚ΡŒ ΠΌΠ°Ρ‚Ρ€ΠΈΡ†Ρ‹ – это количСство строк ΠΈ столбцов. Если Π² ΠΌΠ°Ρ‚Ρ€ΠΈΡ†Π΅ Π΅ΡΡ‚ΡŒ $ a $ строк ΠΈ $ b $ столбцов, ΠΌΡ‹ ΠΌΠΎΠΆΠ΅ΠΌ ΡΠΊΠ°Π·Π°Ρ‚ΡŒ, Ρ‡Ρ‚ΠΎ ΠΌΠ°Ρ‚Ρ€ΠΈΡ†Π° ΠΈΠΌΠ΅Π΅Ρ‚ Ρ€Π°Π·ΠΌΠ΅Ρ€Ρ‹ $ a \ times b $ (произносится ΠΊΠ°ΠΊ $ a $ -by- $ b $). Π§Ρ‚ΠΎΠ±Ρ‹ ΡƒΠ·Π½Π°Ρ‚ΡŒ большС ΠΎ Ρ€Π°Π·ΠΌΠ΅Ρ€Π°Ρ… ΠΌΠ°Ρ‚Ρ€ΠΈΡ†Ρ‹, ΠΏΡ€ΠΎΡ‡Ρ‚ΠΈΡ‚Π΅ эту ΡΡ‚Π°Ρ‚ΡŒΡŽ!

ΠŸΠΎΠΌΠ½ΠΈΡ‚Π΅, Ρ‡Ρ‚ΠΎ ΠΌΠ°Ρ‚Ρ€ΠΈΡ†Π°, получСнная Π² Ρ€Π΅Π·ΡƒΠ»ΡŒΡ‚Π°Ρ‚Π΅ ΠΎΠΏΠ΅Ρ€Π°Ρ†ΠΈΠΈ вычитания ΠΌΠ°Ρ‚Ρ€ΠΈΡ†, Π±ΡƒΠ΄Π΅Ρ‚ Ρ‚ΠΎΠΉ ΠΆΠ΅ размСрности, Ρ‡Ρ‚ΠΎ ΠΈ ΠΌΠ°Ρ‚Ρ€ΠΈΡ†Ρ‹, ΠΊΠΎΡ‚ΠΎΡ€Ρ‹Π΅ Π΅Π΅ создали.

Как Π²Ρ‹Ρ‡Π΅ΡΡ‚ΡŒ ΠΌΠ°Ρ‚Ρ€ΠΈΡ†Ρ‹?

ПослС Ρ‚ΠΎΠ³ΠΎ, ΠΊΠ°ΠΊ ΠΌΡ‹ ΠΏΡ€ΠΈΡˆΠ»ΠΈ ΠΊ Π²Ρ‹Π²ΠΎΠ΄Ρƒ, Ρ‡Ρ‚ΠΎ ΠΌΠ°Ρ‚Ρ€ΠΈΡ†Ρ‹ ΠΈΠΌΠ΅ΡŽΡ‚ ΠΎΠ΄ΠΈΠ½Π°ΠΊΠΎΠ²Ρ‹Π΅ Ρ€Π°Π·ΠΌΠ΅Ρ€Ρ‹, ΠΌΡ‹ Π²Ρ‹Ρ‡ΠΈΡ‚Π°Π΅ΠΌ ΠΌΠ°Ρ‚Ρ€ΠΈΡ†Ρ‹ Π½Π° 2 Π΄ΠΎΠ»Π»Π°Ρ€Π°, вычитая ΡΠΎΠΎΡ‚Π²Π΅Ρ‚ΡΡ‚Π²ΡƒΡŽΡ‰ΠΈΠ΅ Π΄Ρ€ΡƒΠ³ Π΄Ρ€ΡƒΠ³Ρƒ элСмСнты . Рассмотрим ΠΌΠ°Ρ‚Ρ€ΠΈΡ†Ρƒ $ A $ ΠΈ ΠΌΠ°Ρ‚Ρ€ΠΈΡ†Ρƒ $ B $, ΠΏΠΎΠΊΠ°Π·Π°Π½Π½Ρ‹Π΅ Π½ΠΈΠΆΠ΅:

$ A = \ begin {bmatrix} {0} & {4} \\ {- 3} & {- 3} \ end {bmatrix} $

$ B = \ begin {bmatrix} {- 9} & 0 \\ 2 & {- 2} \ end {bmatrix} $

И ΠΌΠ°Ρ‚Ρ€ΠΈΡ†Π° $ A $, ΠΈ $ B $ ΠΈΠΌΠ΅ΡŽΡ‚ $ 2 $ строк ΠΈ $ 2 $ столбцов.Π‘Π»Π΅Π΄ΠΎΠ²Π°Ρ‚Π΅Π»ΡŒΠ½ΠΎ, Ρ€Π°Π·ΠΌΠ΅Ρ€Ρ‹ ΠΌΠ°Ρ‚Ρ€ΠΈΡ† Ρ€Π°Π²Π½Ρ‹ $ 2 \ times 2 $, Ρ‡Ρ‚ΠΎ Π΄Π΅Π»Π°Π΅Ρ‚ ΠΈΡ… Ρ€Π°Π²Π½Ρ‹ΠΌΠΈ. Π­Ρ‚ΠΎ ΠΎΠ·Π½Π°Ρ‡Π°Π΅Ρ‚, Ρ‡Ρ‚ΠΎ ΠΌΡ‹ ΠΌΠΎΠΆΠ΅ΠΌ Π²Ρ‹Ρ‡Π΅ΡΡ‚ΡŒ ΠΌΠ°Ρ‚Ρ€ΠΈΡ†Ρ‹ ΠΏΠΎ $ 2 $, вычитая ΠΊΠ°ΠΆΠ΄ΡƒΡŽ ΡΠΎΠΎΡ‚Π²Π΅Ρ‚ΡΡ‚Π²ΡƒΡŽΡ‰ΡƒΡŽ запись Π΄Ρ€ΡƒΠ³ ΠΈΠ· Π΄Ρ€ΡƒΠ³Π°. ΠŸΡ€ΠΎΡ†Π΅ΡΡ ΠΏΠΎΠΊΠ°Π·Π°Π½ Π½ΠΈΠΆΠ΅:

$ A – B = \ begin {bmatrix} {0 – – 9} & {4 – 0} \\ {- 3 – 2} & {- 3 – – 2} \ end {bmatrix } $

$ A – B = \ begin {bmatrix} {0 + 9} & {4 – 0} \\ {- 3-2} & {- 3 + 2} \ end {bmatrix} $

$ A – B = \ begin {bmatrix} {9} & {4} \\ {- 5} & {- 1} \ end {bmatrix} $

Π’Π΅ΠΏΠ΅Ρ€ΡŒ Π΄Π°Π²Π°ΠΉΡ‚Π΅ рассмотрим Π΄Π²Π΅ ΠΌΠ°Ρ‚Ρ€ΠΈΡ†Ρ‹, ΠΏΠΎΠΊΠ°Π·Π°Π½Π½Ρ‹Π΅ Π½ΠΈΠΆΠ΅:

$ A = \ begin {bmatrix} {0} & {- 1} \\ 10 & 2 \ end {bmatrix} $

$ B = \ begin {bmatrix} {- 5} & 0 \\ 6 & 0 \\ 3 & 7 \ \ 4 & {- 7} \ end {bmatrix} $

ΠžΠΏΡ€Π΅Π΄Π΅Π»ΡΠ΅Ρ‚ΡΡ Π»ΠΈ $ A – B $?

β„–
ΠœΠ°Ρ‚Ρ€ΠΈΡ†Π° $ A $ ΠΈΠΌΠ΅Π΅Ρ‚ Ρ€Π°Π·ΠΌΠ΅Ρ€Π½ΠΎΡΡ‚ΡŒ $ 2 \ times 2 $. ΠœΠ°Ρ‚Ρ€ΠΈΡ†Π° $ B $ ΠΈΠΌΠ΅Π΅Ρ‚ Ρ€Π°Π·ΠΌΠ΅Ρ€Π½ΠΎΡΡ‚ΡŒ $ 4 \ times 2 $. ΠŸΠΎΡΠΊΠΎΠ»ΡŒΠΊΡƒ Ρ€Π°Π·ΠΌΠ΅Ρ€Ρ‹ ΠΎΠ±Π΅ΠΈΡ… ΠΌΠ°Ρ‚Ρ€ΠΈΡ† Π½Π΅ Ρ€Π°Π²Π½Ρ‹, Π²Ρ‹Ρ‡ΠΈΡ‚Π°Π½ΠΈΠ΅ ΠΌΠ°Ρ‚Ρ€ΠΈΡ† Π½Π΅ ΠΎΠΏΡ€Π΅Π΄Π΅Π»Π΅Π½ΠΎ! ΠœΡ‹ Π½Π΅ ΠΌΠΎΠΆΠ΅ΠΌ Π²Ρ‹Ρ‡Π΅ΡΡ‚ΡŒ ΠΌΠ°Ρ‚Ρ€ΠΈΡ†Ρƒ $ A $ ΠΈ $ B $, ΠΏΠΎΡ‚ΠΎΠΌΡƒ Ρ‡Ρ‚ΠΎ Π½Π΅ Π±ΡƒΠ΄Π΅Ρ‚ ΡΠΎΠΎΡ‚Π²Π΅Ρ‚ΡΡ‚Π²ΡƒΡŽΡ‰Π΅ΠΉ записи для ΠΊΠ°ΠΆΠ΄ΠΎΠ³ΠΎ элСмСнта ΠΌΠ°Ρ‚Ρ€ΠΈΡ†Ρ‹ $ A $ ΠΈ ΠΌΠ°Ρ‚Ρ€ΠΈΡ†Ρ‹ $ B $.

ΠŸΡ€Π°Π²ΠΈΠ»Π° вычитания ΠΌΠ°Ρ‚Ρ€ΠΈΡ†

ΠœΡ‹ ΠΌΠΎΠΆΠ΅ΠΌ Π½Π°ΠΏΠΈΡΠ°Ρ‚ΡŒ ΠΏΡ€Π°Π²ΠΈΠ»Π° вычитания ΠΌΠ°Ρ‚Ρ€ΠΈΡ† ΡΡ‚ΠΎΠΈΠΌΠΎΡΡ‚ΡŒΡŽ 4 Π΄ΠΎΠ»Π»Π°Ρ€Π°. Показано Π½ΠΈΠΆΠ΅:

  1. Π’Ρ‹Ρ‡ΠΈΡ‚Π°Ρ‚ΡŒ ΠΌΠΎΠΆΠ½ΠΎ Ρ‚ΠΎΠ»ΡŒΠΊΠΎ ΠΌΠ°Ρ‚Ρ€ΠΈΡ†Ρ‹ с ΠΎΠ΄ΠΈΠ½Π°ΠΊΠΎΠ²Ρ‹ΠΌΠΈ Ρ€Π°Π·ΠΌΠ΅Ρ€Π°ΠΌΠΈ (Ρ‚.Π΅.Π΅. количСство строк ΠΏΠ΅Ρ€Π²ΠΎΠΉ ΠΌΠ°Ρ‚Ρ€ΠΈΡ†Ρ‹ Π΄ΠΎΠ»ΠΆΠ½ΠΎ Ρ€Π°Π²Π½ΡΡ‚ΡŒΡΡ количСству строк Π²Ρ‚ΠΎΡ€ΠΎΠΉ ΠΌΠ°Ρ‚Ρ€ΠΈΡ†Ρ‹. ΠšΡ€ΠΎΠΌΠ΅ Ρ‚ΠΎΠ³ΠΎ, количСство столбцов ΠΏΠ΅Ρ€Π²ΠΎΠΉ ΠΌΠ°Ρ‚Ρ€ΠΈΡ†Ρ‹ Π΄ΠΎΠ»ΠΆΠ½ΠΎ Π±Ρ‹Ρ‚ΡŒ Ρ€Π°Π²Π½ΠΎ количСству столбцов Π²Ρ‚ΠΎΡ€ΠΎΠΉ ΠΌΠ°Ρ‚Ρ€ΠΈΡ†Ρ‹.)
  2. Π Π΅Π·ΡƒΠ»ΡŒΡ‚ΠΈΡ€ΡƒΡŽΡ‰Π°Ρ ΠΌΠ°Ρ‚Ρ€ΠΈΡ†Π° вычитания ΠΌΠ°Ρ‚Ρ€ΠΈΡ†Ρ‹ Π±ΡƒΠ΄Π΅Ρ‚ ΠΈΠΌΠ΅Ρ‚ΡŒ Ρ‚ΠΎΡ‚ ΠΆΠ΅ Ρ€Π°Π·ΠΌΠ΅Ρ€, Ρ‡Ρ‚ΠΎ ΠΈ исходныС ΠΌΠ°Ρ‚Ρ€ΠΈΡ†Ρ‹, ΠΊΠΎΡ‚ΠΎΡ€Ρ‹Π΅ Π΅Π΅ создали.
  3. ΠœΠ°Ρ‚Ρ€ΠΈΡ‡Π½ΠΎΠ΅ Π²Ρ‹Ρ‡ΠΈΡ‚Π°Π½ΠΈΠ΅ НЕ ΠΊΠΎΠΌΠΌΡƒΡ‚Π°Ρ‚ΠΈΠ²Π½ΠΎ (Ρ‚.Π΅. $ A – B \ neq B – A $). Когда Π²Ρ‹ мСняСтС порядок ΠΌΠ°Ρ‚Ρ€ΠΈΡ† ΠΈ Π²Ρ‹Ρ‡ΠΈΡ‚Π°Π΅Ρ‚Π΅, ΠΎΠ½ Π΄Π°Π΅Ρ‚ Π΄Ρ€ΡƒΠ³ΠΎΠΉ ΠΎΡ‚Π²Π΅Ρ‚ (ΠΊΠ°ΠΊ ΠΈ Π΄Π΅ΠΉΡΡ‚Π²ΠΈΡ‚Π΅Π»ΡŒΠ½Ρ‹Π΅ числа).
  4. ΠœΠ°Ρ‚Ρ€ΠΈΡ‡Π½ΠΎΠ΅ Π²Ρ‹Ρ‡ΠΈΡ‚Π°Π½ΠΈΠ΅ НЕ ассоциативно (Ρ‚.Π΅. $ (A – B) – C \ neq A – (B – C) $).

Π”Π°Π²Π°ΠΉΡ‚Π΅ посмотрим Π½Π° нСсколько ΠΏΡ€ΠΈΠΌΠ΅Ρ€ΠΎΠ² вычитания ΠΌΠ°Ρ‚Ρ€ΠΈΡ† .

ΠŸΡ€ΠΈΠΌΠ΅Ρ€ 1

ΠŸΡ€ΠΎΠ²Π΅Ρ€ΡŒΡ‚Π΅, ΠΎΠΏΡ€Π΅Π΄Π΅Π»Π΅Π½ΠΎ Π»ΠΈ Π²Ρ‹Ρ‡ΠΈΡ‚Π°Π½ΠΈΠ΅ ΠΌΠ°Ρ‚Ρ€ΠΈΡ†Ρ‹ ΠΌΠ΅ΠΆΠ΄Ρƒ ΠœΠ°Ρ‚Ρ€ΠΈΡ†Π΅ΠΉ $ A $ ΠΈ ΠœΠ°Ρ‚Ρ€ΠΈΡ†Π΅ΠΉ $ B $. Если Π΄Π°, Π²Ρ‹Ρ‡Ρ‚ΠΈΡ‚Π΅ ΠΈΡ….

$ A = \ begin {bmatrix} {0} & {- 5} \\ {- 5} & {0} \ end {bmatrix} $

$ B = \ begin {bmatrix} {3} & {- 4} \\ {0} & {- 5} \ end {bmatrix} $

РСшСниС

Для опрСдСлСния вычитания ΠΌΠ°Ρ‚Ρ€ΠΈΡ†Ρ‹ Ρ€Π°Π·ΠΌΠ΅Ρ€ ΠΊΠ°ΠΆΠ΄ΠΎΠΉ ΠΌΠ°Ρ‚Ρ€ΠΈΡ†Ρ‹ Π΄ΠΎΠ»ΠΆΠ΅Π½ Π±Ρ‹Ρ‚ΡŒ ΠΎΠ΄ΠΈΠ½Π°ΠΊΠΎΠ²Ρ‹ΠΌ.
ΠœΠ°Ρ‚Ρ€ΠΈΡ†Π° $ A $ – это ΠΌΠ°Ρ‚Ρ€ΠΈΡ†Π° Ρ€Π°Π·ΠΌΠ΅Ρ€ΠΎΠΌ $ 2 \ times 2 $. ΠœΠ°Ρ‚Ρ€ΠΈΡ†Π° $ B $ Ρ‚Π°ΠΊΠΆΠ΅ являСтся ΠΌΠ°Ρ‚Ρ€ΠΈΡ†Π΅ΠΉ $ 2 \ times 2 $. Π’Π°ΠΊΠΈΠΌ ΠΎΠ±Ρ€Π°Π·ΠΎΠΌ, опрСдСляСтся Π²Ρ‹Ρ‡ΠΈΡ‚Π°Π½ΠΈΠ΅ ΠΌΠ°Ρ‚Ρ€ΠΈΡ† ΠΌΠ΅ΠΆΠ΄Ρƒ ΠΌΠ°Ρ‚Ρ€ΠΈΡ†Π°ΠΌΠΈ $ A $ ΠΈ $ B $.

Π’Π΅ΠΏΠ΅Ρ€ΡŒ Π΄Π°Π²Π°ΠΉΡ‚Π΅ Π²Ρ‹Ρ‡Ρ‚Π΅ΠΌ ΠΌΠ°Ρ‚Ρ€ΠΈΡ†Ρ‹ ΠΏΠΎ $ 2 $ вмСстС, вычтя ΠΊΠ°ΠΆΠ΄ΡƒΡŽ ΡΠΎΠΎΡ‚Π²Π΅Ρ‚ΡΡ‚Π²ΡƒΡŽΡ‰ΡƒΡŽ запись ΠΌΠ°Ρ‚Ρ€ΠΈΡ†Ρ‹ $ B $ ΠΈΠ· ΠΊΠ°ΠΆΠ΄ΠΎΠΉ ΡΠΎΠΎΡ‚Π²Π΅Ρ‚ΡΡ‚Π²ΡƒΡŽΡ‰Π΅ΠΉ записи ΠΌΠ°Ρ‚Ρ€ΠΈΡ†Ρ‹ $ A $. Показано Π½ΠΈΠΆΠ΅:

$ A – B = \ begin {bmatrix} {0 – 3} & {- 5 – – 4} \\ {- 5 – 0} & {0 – – 5} \ end {bmatrix} $

$ A – B = \ begin {bmatrix} {- 3} & {- 5 + 4} \\ {- 5} & {0 + 5} \ end {bmatrix} $

$ A – B = \ begin { bmatrix} {- 3} & {- 1} \\ {- 5} & {5} \ end {bmatrix} $

ΠŸΡ€ΠΈΠΌΠ΅Ρ€ 2

Для прСдставлСнных Π½ΠΈΠΆΠ΅ ΠΌΠ°Ρ‚Ρ€ΠΈΡ† $ 2 $ Π½Π°ΠΉΠ΄ΠΈΡ‚Π΅ $ C – D $.

$ C = \ begin {bmatrix} {1} & {- 1} \\ 1 & {- 1} \\ 7 & {- 7} \ end {bmatrix} $

$ D = \ begin {bmatrix} {1} & {- 2} & {- 1} \\ {- 1} & 0 & {- 1} \ end {bmatrix} $

РСшСниС

Π Π°Π·ΠΌΠ΅Ρ€Π½ΠΎΡΡ‚ΡŒ ΠΌΠ°Ρ‚Ρ€ΠΈΡ†Ρ‹ $ C $ Ρ€Π°Π²Π½Π° $ 3 \ times 2 $. Π Π°Π·ΠΌΠ΅Ρ€Π½ΠΎΡΡ‚ΡŒ ΠœΠ°Ρ‚Ρ€ΠΈΡ†Ρ‹ $ D $ Ρ€Π°Π²Π½Π° $ 2 \ times 3 $. НСсмотря Π½Π° Ρ‚ΠΎ, Ρ‡Ρ‚ΠΎ Ρ€Π°Π·ΠΌΠ΅Ρ€Ρ‹ эквивалСнтны, ΠΌΡ‹, , Π½Π΅ ΠΌΠΎΠΆΠ΅ΠΌ Π²Ρ‹Ρ‡Π΅ΡΡ‚ΡŒ Matrix $ D $ ΠΈΠ· Matrix $ C $ . ΠŸΠΎΡ€ΡΠ΄ΠΎΠΊ Ρ€Π°Π·ΠΌΠ΅Ρ€ΠΎΠ² Ρ‚Π°ΠΊΠΆΠ΅ Π²Π°ΠΆΠ΅Π½.ΠœΠ°Ρ‚Ρ€ΠΈΡ†Π° $ 3 \ times 2 $ Π½Π΅ Ρ€Π°Π²Π½Π° ΠΌΠ°Ρ‚Ρ€ΠΈΡ†Π΅ $ 2 \ times 3 $. Π’Π°ΠΊΠΈΠΌ ΠΎΠ±Ρ€Π°Π·ΠΎΠΌ, ΠΌΡ‹ Π½Π΅ ΠΌΠΎΠΆΠ΅ΠΌ Π²Ρ‹Ρ‡Π΅ΡΡ‚ΡŒ ΠœΠ°Ρ‚Ρ€ΠΈΡ†Ρƒ $ D $ ΠΈΠ· ΠœΠ°Ρ‚Ρ€ΠΈΡ†Ρ‹ $ C $.

ΠœΡ‹ Ρ‚Π°ΠΊΠΆΠ΅ ΠΌΠΎΠΆΠ΅ΠΌ Ρ€Π΅ΡˆΠ°Ρ‚ΡŒ основныС алгСбраичСскиС уравнСния с Π²Ρ‹Ρ‡ΠΈΡ‚Π°Π½ΠΈΠ΅ΠΌ ΠΌΠ°Ρ‚Ρ€ΠΈΡ†. Рассмотрим ΠΏΡ€ΠΈΠΌΠ΅Ρ€, ΠΏΠΎΠΊΠ°Π·Π°Π½Π½Ρ‹ΠΉ Π½ΠΈΠΆΠ΅.

ΠŸΡ€ΠΈΠΌΠ΅Ρ€ 3

НайдитС значСния $ a $ ΠΈ $ b $ ΠΏΠΎ ΡΠ»Π΅Π΄ΡƒΡŽΡ‰Π΅ΠΌΡƒ ΡƒΡ€Π°Π²Π½Π΅Π½ΠΈΡŽ:

$ \ begin {pmatrix} {1} & {2} \\ b & {- 6} \ ΠΊΠΎΠ½Π΅Ρ† {pmatrix} – \ begin {pmatrix} {a} & {- 1} \\ 0 & {- 3} \ end {pmatrix} = \ begin {pmatrix} {7} & {3} \\ {- 2} & {- 3} \ end {pmatrix} $

РСшСниС

ΠœΡ‹ ΠΌΠΎΠΆΠ΅ΠΌ Π²Ρ‹Ρ‡Π΅ΡΡ‚ΡŒ ΡΠΎΠΎΡ‚Π²Π΅Ρ‚ΡΡ‚Π²ΡƒΡŽΡ‰ΠΈΠ΅ записи, Ρ‡Ρ‚ΠΎΠ±Ρ‹ Π½Π°ΠΉΡ‚ΠΈ $ a $ ΠΈ $ b $.Π‘Π½Π°Ρ‡Π°Π»Π° Π²Ρ‹Ρ‡Ρ‚Π΅ΠΌ ΡΠΎΠΎΡ‚Π²Π΅Ρ‚ΡΡ‚Π²ΡƒΡŽΡ‰ΠΈΠ΅ элСмСнты для $ a $ ΠΈ Ρ€Π΅ΡˆΠΈΠΌ для ΠΏΠ΅Ρ€Π΅ΠΌΠ΅Π½Π½ΠΎΠΉ:

$ 1 – a = 7 $
$ a = 1-7 $
$ a = – 6 $

Π’Π΅ΠΏΠ΅Ρ€ΡŒ Π΄Π°Π²Π°ΠΉΡ‚Π΅ Π½Π°ΠΉΠ΄Π΅ΠΌ Π·Π½Π°Ρ‡Π΅Π½ΠΈΠ΅ $ b $ ΠΏΡƒΡ‚Π΅ΠΌ вычитания ΡΠΎΠΎΡ‚Π²Π΅Ρ‚ΡΡ‚Π²ΡƒΡŽΡ‰ΠΈΡ… записСй для $ b $:

$ b – 0 = – 2 $
$ b = – 2 $

Π’Π΅ΠΏΠ΅Ρ€ΡŒ Π²Ρ‹ ΠΌΠΎΠΆΠ΅Ρ‚Π΅ ΠΏΠΎΠΏΡ€ΠΎΠ±ΠΎΠ²Π°Ρ‚ΡŒ нСсколько практичСских вопросов ΠΈ ΠΏΠΎΡΠΌΠΎΡ‚Ρ€Π΅Ρ‚ΡŒ, ΠΏΡ€Π°Π²ΠΈΠ»ΡŒΠ½ΠΎ Π»ΠΈ Π²Ρ‹ ΠΈΡ… ΠΎΡ‚Π²Π΅Ρ‚ΠΈΠ»ΠΈ. ΠžΡ‚Π²Π΅Ρ‚Ρ‹ Π½ΠΈΠΆΠ΅.

ΠŸΡ€Π°ΠΊΡ‚ΠΈΡ‡Π΅ΡΠΊΠΈΠ΅ вопросы
  1. Рассмотрим ΡΠ»Π΅Π΄ΡƒΡŽΡ‰ΠΈΠ΅ 3 $ ΠΌΠ°Ρ‚Ρ€ΠΈΡ†Ρ‹:
    $ P = \ begin {pmatrix} {- 2} & {- 2} \\ 1 & {1} \ end {pmatrix} $
    $ Q = \ begin {pmatrix} {- 3} & {- 3} \\ {- 3} & {- 3} \ end {pmatrix} $
    $ R = \ begin {pmatrix} 1 & 1 \\ 7 & {- 2} \\ 3 & {- 7} \ end {pmatrix}
    $ Найти:
    1. $ P – Q $
    2. $ Q –
    3. R $
    4. $ Q – P $
  2. Найти значСния $ a $, $ b $ ΠΈ $ c $ Π·Π°Π΄Π°Π½Ρ‹ ΡΠ»Π΅Π΄ΡƒΡŽΡ‰ΠΈΠΌ ΡƒΡ€Π°Π²Π½Π΅Π½ΠΈΠ΅ΠΌ:

    $ \ begin {pmatrix} {3} & {2} & 0 \\ b & {- 2} & 3 \\ 11 & a & -2 \ end {pmatrix} – \ begin {pmatrix} {3} & {2} & c \\ 5 & {- 5} & 3 \\ {- 4} & {6} & 10 \ end {pmatrix } = \ begin {pmatrix} {0} & {0} & -9 \\ 12 & {3} & 0 \\ 15 & -3 & {- 12} \ end {pmatrix} $


ΠžΡ‚Π²Π΅Ρ‚Ρ‹

  1. ВсС Π·Π°Π΄Π°Ρ‡ΠΈ $ 3 $ ΠΏΡ€Π΅Π΄ΡΡ‚Π°Π²Π»ΡΡŽΡ‚ собой ΠΌΠ°Ρ‚Ρ€ΠΈΡ‡Π½ΠΎΠ΅ Π²Ρ‹Ρ‡ΠΈΡ‚Π°Π½ΠΈΠ΅.ΠžΠ±Ρ€Π°Ρ‚ΠΈΡ‚Π΅ Π²Π½ΠΈΠΌΠ°Π½ΠΈΠ΅, Ρ‡Ρ‚ΠΎ, ΠΏΠΎΡΠΊΠΎΠ»ΡŒΠΊΡƒ ΠΌΠ°Ρ‚Ρ€ΠΈΡ‡Π½ΠΎΠ΅ Π²Ρ‹Ρ‡ΠΈΡ‚Π°Π½ΠΈΠ΅ Π½Π΅ ΠΊΠΎΠΌΠΌΡƒΡ‚Π°Ρ‚ΠΈΠ²Π½ΠΎ , ΠΎΡ‚Π²Π΅Ρ‚ Π½Π° Part $ A $ ΠΈ Part $ C $ Π±ΡƒΠ΄Π΅Ρ‚ Π΄Ρ€ΡƒΠ³ΠΈΠΌ. ΠžΡ‚Π²Π΅Ρ‚Ρ‹ ΠΏΡ€ΠΈΠ²Π΅Π΄Π΅Π½Ρ‹ Π½ΠΈΠΆΠ΅.
    1. ОбС ΠΌΠ°Ρ‚Ρ€ΠΈΡ†Ρ‹ $ P $ ΠΈ $ Q $ ΡΠ²Π»ΡΡŽΡ‚ΡΡ ΠΌΠ°Ρ‚Ρ€ΠΈΡ†Π°ΠΌΠΈ Ρ€Π°Π·ΠΌΠ΅Ρ€ΠΎΠΌ $ 2 \ times 2 $. Π’Π°ΠΊΠΈΠΌ ΠΎΠ±Ρ€Π°Π·ΠΎΠΌ, ΠΌΡ‹ Π²Ρ‹Ρ‡ΠΈΡ‚Π°Π΅ΠΌ ΠΌΠ°Ρ‚Ρ€ΠΈΡ†Ρ‹ Ρ€Π°Π·ΠΌΠ΅Ρ€ΠΎΠΌ $ 2 $, вычитая ΡΠΎΠΎΡ‚Π²Π΅Ρ‚ΡΡ‚Π²ΡƒΡŽΡ‰ΠΈΠ΅ элСмСнты. Показано Π½ΠΈΠΆΠ΅:
      $ P – Q = \ begin {pmatrix} {- 2 – – 3} & {- 2 – – 3} \\ {1 – – 3} & {1 – – 3} \ end {pmatrix} $
      $ P – Q = \ begin {pmatrix} {- 2 + 3} & {- 2 + 3} \\ {1 + 3} & {1 + 3} \ end {pmatrix} $
      $ P – Q = \ begin {pmatrix} {1} & {1} \\ {4} & {4} \ end {pmatrix} $
    2. ΠœΡ‹ Π½Π΅ ΠΌΠΎΠΆΠ΅ΠΌ Π²Ρ‹Ρ‡Π΅ΡΡ‚ΡŒ ΠΌΠ°Ρ‚Ρ€ΠΈΡ†Ρƒ $ R $ ΠΈΠ· ΠΌΠ°Ρ‚Ρ€ΠΈΡ†Ρ‹ $ Q $, ΠΏΠΎΡ‚ΠΎΠΌΡƒ Ρ‡Ρ‚ΠΎ ΠΈΡ… Ρ€Π°Π·ΠΌΠ΅Ρ€Ρ‹ Π½Π΅ ΡΠΎΠ²ΠΏΠ°Π΄Π°ΡŽΡ‚ .ΠœΠ°Ρ‚Ρ€ΠΈΡ†Π° $ Q $ – это ΠΌΠ°Ρ‚Ρ€ΠΈΡ†Π° $ 2 \ times 2 $, Π° ΠœΠ°Ρ‚Ρ€ΠΈΡ†Π° $ R $ – ΠΌΠ°Ρ‚Ρ€ΠΈΡ†Π° $ 3 \ times 2 $.
    3. ΠžΡ‚Π²Π΅Ρ‚ Π½Π° эту Ρ‡Π°ΡΡ‚ΡŒ вопроса Π±ΡƒΠ΄Π΅Ρ‚ ΠΎΡ‚Π»ΠΈΡ‡Π°Ρ‚ΡŒΡΡ ΠΎΡ‚ ΠΎΡ‚Π²Π΅Ρ‚Π° Π² части $ A $, ΠΏΠΎΡΠΊΠΎΠ»ΡŒΠΊΡƒ Π²Ρ‹Ρ‡ΠΈΡ‚Π°Π½ΠΈΠ΅ ΠΌΠ°Ρ‚Ρ€ΠΈΡ† Π½Π΅ ΠΊΠΎΠΌΠΌΡƒΡ‚Π°Ρ‚ΠΈΠ²Π½ΠΎ. Π’Ρ‹Ρ‡Ρ‚Π΅ΠΌ ΠœΠ°Ρ‚Ρ€ΠΈΡ†Ρƒ $ P $ ΠΈΠ· ΠœΠ°Ρ‚Ρ€ΠΈΡ†Ρ‹ $ Q $.
      $ Q – P = \ begin {pmatrix} {- 3 – – 2} & {- 3 – – 2} \\ {- 3 – 1} & {- 3 – 1} \ end {pmatrix} $
      $ Q – P = \ begin {pmatrix} {- 3 + 2} & {- 3 + 2} \\ {- 4} & {- 4} \ end {pmatrix} $
      $ Q – P = \ begin {pmatrix} { – 1} & {- 1} \\ {- 4} & {- 4} \ end {pmatrix} $
  2. ΠœΡ‹ ΠΌΠΎΠΆΠ΅ΠΌ Π²Ρ‹Ρ‡Π΅ΡΡ‚ΡŒ ΡΠΎΠΎΡ‚Π²Π΅Ρ‚ΡΡ‚Π²ΡƒΡŽΡ‰ΠΈΠ΅ записи, Ρ‡Ρ‚ΠΎΠ±Ρ‹ Π½Π°ΠΉΡ‚ΠΈ $ a $, $ b $ ΠΈ $ c $.Π‘Π½Π°Ρ‡Π°Π»Π° Π²Ρ‹Ρ‡Ρ‚Π΅ΠΌ ΡΠΎΠΎΡ‚Π²Π΅Ρ‚ΡΡ‚Π²ΡƒΡŽΡ‰ΠΈΠ΅ элСмСнты для $ a $ ΠΈ Ρ€Π΅ΡˆΠΈΠΌ для ΠΏΠ΅Ρ€Π΅ΠΌΠ΅Π½Π½ΠΎΠΉ:

    $ a – 6 = -3 $
    $ a = – 3 + 6 $
    $ a = 3 $

    Π’Π΅ΠΏΠ΅Ρ€ΡŒ Π΄Π°Π²Π°ΠΉΡ‚Π΅ Π½Π°ΠΉΠ΄Π΅ΠΌ Π·Π½Π°Ρ‡Π΅Π½ΠΈΠ΅ $ b $ ΠΏΡƒΡ‚Π΅ΠΌ вычитания ΡΠΎΠΎΡ‚Π²Π΅Ρ‚ΡΡ‚Π²ΡƒΡŽΡ‰ΠΈΡ… записСй для $ b $:

    $ b – 5 = 12 $
    $ b = 12 + 5 $
    $ b = 17 $

    НаконСц, Π΄Π°Π²Π°ΠΉΡ‚Π΅ Π½Π°ΠΉΠ΄Π΅ΠΌ Π·Π½Π°Ρ‡Π΅Π½ΠΈΠ΅ $ c $ ΠΏΡƒΡ‚Π΅ΠΌ вычитания ΡΠΎΠΎΡ‚Π²Π΅Ρ‚ΡΡ‚Π²ΡƒΡŽΡ‰ΠΈΠ΅ записи для $ c $:

    $ 0 – c = -9 $
    $ c = 0 – – 9 $
    $ c = 0 + 9 $
    $ c = 9 $

ΠŸΡ€Π΅Π΄Ρ‹Π΄ΡƒΡ‰ΠΈΠΉ ΡƒΡ€ΠΎΠΊ | Главная страница | Π‘Π»Π΅Π΄ΡƒΡŽΡ‰ΠΈΠΉ ΡƒΡ€ΠΎΠΊ

Как ΡΠΊΠ»Π°Π΄Ρ‹Π²Π°Ρ‚ΡŒ ΠΈ Π²Ρ‹Ρ‡ΠΈΡ‚Π°Ρ‚ΡŒ ΠΌΠ°Ρ‚Ρ€ΠΈΡ†Ρ‹ | Precalculus

Π‘Π»ΠΎΠΆΠ΅Π½ΠΈΠ΅ ΠΈ Π²Ρ‹Ρ‡ΠΈΡ‚Π°Π½ΠΈΠ΅ ΠΌΠ°Ρ‚Ρ€ΠΈΡ†

Π¨Π°Π³ 1: НайдитС Ρ€Π°Π·ΠΌΠ΅Ρ€Ρ‹ Π·Π°Π΄Π°Π½Π½Ρ‹Ρ… ΠΌΠ°Ρ‚Ρ€ΠΈΡ†.ΠœΠ°Ρ‚Ρ€ΠΈΡ†Ρ‹ ΠΌΠΎΠΆΠ½ΠΎ ΡΠΊΠ»Π°Π΄Ρ‹Π²Π°Ρ‚ΡŒ ΠΈΠ»ΠΈ Π²Ρ‹Ρ‡ΠΈΡ‚Π°Ρ‚ΡŒ Ρ‚ΠΎΠ»ΡŒΠΊΠΎ Π² Ρ‚ΠΎΠΌ случаС, Ссли ΠΎΠ½ΠΈ ΠΈΠΌΠ΅ΡŽΡ‚ ΠΎΠ΄ΠΈΠ½Π°ΠΊΠΎΠ²Ρ‹Π΅ Ρ€Π°Π·ΠΌΠ΅Ρ€Ρ‹.

Π¨Π°Π³ 2: Если Π΄Π°Π½Π½Ρ‹Π΅ ΠΌΠ°Ρ‚Ρ€ΠΈΡ†Ρ‹ ΠΈΠΌΠ΅ΡŽΡ‚ ΠΎΠ΄ΠΈΠ½Π°ΠΊΠΎΠ²Ρ‹Π΅ Ρ€Π°Π·ΠΌΠ΅Ρ€Ρ‹, Ρ‡Ρ‚ΠΎΠ±Ρ‹ Π΄ΠΎΠ±Π°Π²ΠΈΡ‚ΡŒ ΠΌΠ°Ρ‚Ρ€ΠΈΡ†Ρ‹, Π΄ΠΎΠ±Π°Π²ΡŒΡ‚Π΅ ΡΠΎΠΎΡ‚Π²Π΅Ρ‚ΡΡ‚Π²ΡƒΡŽΡ‰ΠΈΠ΅ элСмСнты ΠΊΠ°ΠΆΠ΄ΠΎΠΉ ΠΌΠ°Ρ‚Ρ€ΠΈΡ†Ρ‹. Π—Π°ΠΏΠΈΡΡŒ ΠΏΠ΅Ρ€Π²ΠΎΠ³ΠΎ столбца ΠΈ строки ΠΏΠ΅Ρ€Π²ΠΎΠΉ ΠΌΠ°Ρ‚Ρ€ΠΈΡ†Ρ‹ Π±ΡƒΠ΄Π΅Ρ‚ Π΄ΠΎΠ±Π°Π²Π»Π΅Π½Π° ​​к записи ΠΏΠ΅Ρ€Π²ΠΎΠΉ строки ΠΈ столбца Π²Ρ‚ΠΎΡ€ΠΎΠΉ ΠΌΠ°Ρ‚Ρ€ΠΈΡ†Ρ‹. ΠŸΠΎΠ»ΡƒΡ‡Π΅Π½Π½Π°Ρ сумма Π±ΡƒΠ΄Π΅Ρ‚ ΠΏΠΎΠΌΠ΅Ρ‰Π΅Π½Π° Π² ΡΠΎΠΎΡ‚Π²Π΅Ρ‚ΡΡ‚Π²ΡƒΡŽΡ‰ΡƒΡŽ ΠΏΠΎΠ·ΠΈΡ†ΠΈΡŽ; Ρ‚ΠΎ Π΅ΡΡ‚ΡŒ сумма записСй Π² ΠΏΠ΅Ρ€Π²ΠΎΠΉ строкС-столбцС ΠΎΠ±Π΅ΠΈΡ… ΠΌΠ°Ρ‚Ρ€ΠΈΡ† Π±ΡƒΠ΄Π΅Ρ‚ ΠΏΠΎΠΌΠ΅Ρ‰Π΅Π½Π° Π² ΠΏΠ΅Ρ€Π²Ρ‹ΠΉ столбСц ΠΏΠ΅Ρ€Π²ΠΎΠΉ строки Ρ€Π΅Π·ΡƒΠ»ΡŒΡ‚ΠΈΡ€ΡƒΡŽΡ‰Π΅ΠΉ ΠΌΠ°Ρ‚Ρ€ΠΈΡ†Ρ‹.

Аналогичным ΠΎΠ±Ρ€Π°Π·ΠΎΠΌ ΠΌΡ‹ Π²Ρ‹Ρ‡Ρ‚Π΅ΠΌ ΡΠΎΠΎΡ‚Π²Π΅Ρ‚ΡΡ‚Π²ΡƒΡŽΡ‰ΠΈΠ΅ элСмСнты Π² Π΄Π²ΡƒΡ… ΠΌΠ°Ρ‚Ρ€ΠΈΡ†Π°Ρ…, Ссли ΠΌΡ‹ выполняСм Π²Ρ‹Ρ‡ΠΈΡ‚Π°Π½ΠΈΠ΅ ΠΌΠ°Ρ‚Ρ€ΠΈΡ†.

ΠœΡ‹ ΠΌΠΎΠΆΠ΅ΠΌ Ρ€Π°Π·ΠΌΠ΅ΡΡ‚ΠΈΡ‚ΡŒ ΠΌΠ°Ρ‚Ρ€ΠΈΡ†Ρ‹ Π² любом порядкС, ΠΊΠΎΠ³Π΄Π° нас попросят ΠΈΡ… Π΄ΠΎΠ±Π°Π²ΠΈΡ‚ΡŒ; Ρ‚ΠΎ Π΅ΡΡ‚ΡŒ, Ссли нас попросят Π΄ΠΎΠ±Π°Π²ΠΈΡ‚ΡŒ ΠΌΠ°Ρ‚Ρ€ΠΈΡ†Ρƒ {eq} A {/ eq} ΠΈ ΠΌΠ°Ρ‚Ρ€ΠΈΡ†Π° {eq} B {/ eq}, ΠΌΡ‹ ΠΌΠΎΠΆΠ΅ΠΌ Ρ€Π°Π·ΠΌΠ΅ΡΡ‚ΠΈΡ‚ΡŒ ΠΈΡ… Π² любом порядкС ΠΈ Π½Π°Ρ‡Π°Ρ‚ΡŒ Π΄ΠΎΠ±Π°Π²Π»ΡΡ‚ΡŒ. Когда нас просят Π²Ρ‹Ρ‡Π΅ΡΡ‚ΡŒ ΠΌΠ°Ρ‚Ρ€ΠΈΡ†Ρ‹, ΠΌΡ‹ Π΄ΠΎΠ»ΠΆΠ½Ρ‹ Ρ€Π°ΡΠΏΠΎΠ»ΠΎΠΆΠΈΡ‚ΡŒ ΠΈΡ… Π² Ρ‚ΠΎΠΌ ΠΆΠ΅ порядкС, Π² ΠΊΠ°ΠΊΠΎΠΌ Π½Π°ΠΌ даСтся Π²Ρ‹Ρ‡ΠΈΡ‚Π°Π½ΠΈΠ΅; Ρ‚ΠΎ Π΅ΡΡ‚ΡŒ, Ссли нас попросят Π²Ρ‹Ρ‡Π΅ΡΡ‚ΡŒ ΠΌΠ°Ρ‚Ρ€ΠΈΡ†Ρ‹ {eq} A – B {/ eq}, Ρ‚ΠΎΠ³Π΄Π° ΠΌΡ‹ Π΄ΠΎΠ»ΠΆΠ½Ρ‹ ΠΏΠΎΠΌΠ΅ΡΡ‚ΠΈΡ‚ΡŒ ΠΌΠ°Ρ‚Ρ€ΠΈΡ†Ρƒ {eq} A {/ eq} сначала, Π° Π·Π°Ρ‚Π΅ΠΌ Π²Ρ‹Ρ‡Ρ‚ΠΈΡ‚Π΅ ΠΌΠ°Ρ‚Ρ€ΠΈΡ†Ρƒ {eq} B {/ eq} ΠΎΡ‚ Π½Π΅Π³ΠΎ.И Π½Π°ΠΎΠ±ΠΎΡ€ΠΎΡ‚, Π±Ρ‹Ρ‚ΡŒ Π½Π΅ ΠΌΠΎΠΆΠ΅Ρ‚.

Π‘Π»ΠΎΠΆΠ΅Π½ΠΈΠ΅ ΠΈ Π²Ρ‹Ρ‡ΠΈΡ‚Π°Π½ΠΈΠ΅ ΠΌΠ°Ρ‚Ρ€ΠΈΡ† Π‘Π»ΠΎΠ²Π°Ρ€ΡŒ

ΠœΠ°Ρ‚Ρ€ΠΈΡ†Ρ‹: ΠœΠ°Ρ‚Ρ€ΠΈΡ†Ρ‹ ΠΏΡ€Π΅Π΄ΡΡ‚Π°Π²Π»ΡΡŽΡ‚ собой ΠΏΡ€ΡΠΌΠΎΡƒΠ³ΠΎΠ»ΡŒΠ½ΠΎΠ΅ располоТСниС чисСл Π² строках ΠΈ столбцах. Β«ΠœΠ°Ρ‚Ρ€ΠΈΡ†Π°Β» – это СдинствСнноС число Β«ΠΌΠ°Ρ‚Ρ€ΠΈΡ†Β». Числа Π² ΠΌΠ°Ρ‚Ρ€ΠΈΡ†Π΅ Π·Π°ΠΊΠ»ΡŽΡ‡Π΅Π½Ρ‹ Π² скобки. ΠœΠ°Ρ‚Ρ€ΠΈΡ†Ρƒ ΠΌΠΎΠΆΠ½ΠΎ ΠΎΠ±ΠΎΠ·Π½Π°Ρ‡ΠΈΡ‚ΡŒ прописной Π±ΡƒΠΊΠ²ΠΎΠΉ Π°Π»Ρ„Π°Π²ΠΈΡ‚Π°. Π“ΠΎΡ€ΠΈΠ·ΠΎΠ½Ρ‚Π°Π»ΡŒΠ½Π°Ρ линия – это строка Π² ΠΌΠ°Ρ‚Ρ€ΠΈΡ†Π΅, Π° Π²Π΅Ρ€Ρ‚ΠΈΠΊΠ°Π»ΡŒΠ½Π°Ρ линия – это столбСц Π² ΠΌΠ°Ρ‚Ρ€ΠΈΡ†Π΅. КаТдоС число ΠΈΠ»ΠΈ Π²Ρ‹Ρ€Π°ΠΆΠ΅Π½ΠΈΠ΅ Π² ΠΌΠ°Ρ‚Ρ€ΠΈΡ†Π΅ – это запись Π² ΠΌΠ°Ρ‚Ρ€ΠΈΡ†Π΅.

Π Π°Π·ΠΌΠ΅Ρ€Π½ΠΎΡΡ‚ΡŒ ΠΌΠ°Ρ‚Ρ€ΠΈΡ†Ρ‹: Π Π°Π·ΠΌΠ΅Ρ€Π½ΠΎΡΡ‚ΡŒ ΠΌΠ°Ρ‚Ρ€ΠΈΡ†Ρ‹ сообщаСт Π½Π°ΠΌ количСство строк ΠΈ количСство столбцов Π² ΠΌΠ°Ρ‚Ρ€ΠΈΡ†Π΅. Если данная ΠΌΠ°Ρ‚Ρ€ΠΈΡ†Π° ΠΈΠΌΠ΅Π΅Ρ‚ {eq} x {/ eq} строк ΠΈ {eq} y {/ eq} столбцов, Ρ€Π°Π·ΠΌΠ΅Ρ€ ΠΌΠ°Ρ‚Ρ€ΠΈΡ†Ρ‹ Π±ΡƒΠ΄Π΅Ρ‚ записан ΠΊΠ°ΠΊ {eq} x \ times y {/ экв}. Π­Ρ‚ΠΎ Ρ‚Π°ΠΊΠΆΠ΅ называСтся {eq} x \ text {by} y {/ экв}. ΠŸΠΎΠΌΠ½ΠΈΡ‚Π΅, Ρ‡Ρ‚ΠΎ {eq} \ bf {(\ times)} Π—Π½Π°ΠΊ {/ eq} Π½Π΅ ΠΎΠ·Π½Π°Ρ‡Π°Π΅Ρ‚ ΡƒΠΌΠ½ΠΎΠΆΠ΅Π½ΠΈΠ΅.

ΠœΡ‹ рассмотрим Π΄Π²Π° ΠΏΡ€ΠΈΠΌΠ΅Ρ€Π° ΠΈ Ρ€Π΅ΡˆΠΈΠΌ ΠΈΡ… вмСстС, ΠΈΡΠΏΠΎΠ»ΡŒΠ·ΡƒΡ шаги, упомянутыС Π²Ρ‹ΡˆΠ΅, Ρ‡Ρ‚ΠΎΠ±Ρ‹ Π»ΡƒΡ‡ΡˆΠ΅ ΠΏΠΎΠ½ΡΡ‚ΡŒ слоТСниС ΠΈ Π²Ρ‹Ρ‡ΠΈΡ‚Π°Π½ΠΈΠ΅ ΠΌΠ°Ρ‚Ρ€ΠΈΡ†.ΠŸΠ΅Ρ€Π²Ρ‹ΠΉ ΠΏΡ€ΠΈΠΌΠ΅Ρ€ ΠΏΠΎΠΊΠ°ΠΆΠ΅Ρ‚ слоТСниС ΠΌΠ°Ρ‚Ρ€ΠΈΡ†, Π° Π²Ρ‚ΠΎΡ€ΠΎΠΉ ΠΏΡ€ΠΈΠΌΠ΅Ρ€ ΠΏΠΎΠΊΠ°ΠΆΠ΅Ρ‚ Π²Ρ‹Ρ‡ΠΈΡ‚Π°Π½ΠΈΠ΅ ΠΌΠ°Ρ‚Ρ€ΠΈΡ†.

Π‘Π»ΠΎΠΆΠ΅Π½ΠΈΠ΅ ΠΈ Π²Ρ‹Ρ‡ΠΈΡ‚Π°Π½ΠΈΠ΅ ΠΌΠ°Ρ‚Ρ€ΠΈΡ†: ΠΏΡ€ΠΈΠΌΠ΅Ρ€ слоТСния ΠΌΠ°Ρ‚Ρ€ΠΈΡ†Ρ‹

Π‘Π»ΠΎΠΆΠΈΡ‚Π΅ Π΄Π²Π΅ Π·Π°Π΄Π°Π½Π½Ρ‹Π΅ ΠΌΠ°Ρ‚Ρ€ΠΈΡ†Ρ‹ {eq} A + B {/ eq}

{eq} A = \ begin {bmatrix} 3 ΠΈ -6 ΠΈ 5 \\ 4 ΠΈ 5 ΠΈ -2 \ end {bmatrix}, B = \ begin {bmatrix} 5 ΠΈ 3 ΠΈ -8 \\ 2 ΠΈ 8 ΠΈ -4 \ end {bmatrix} \\ {/ eq}

Π¨Π°Π³ 1: НайдитС Ρ€Π°Π·ΠΌΠ΅Ρ€Ρ‹ Π·Π°Π΄Π°Π½Π½Ρ‹Ρ… ΠΌΠ°Ρ‚Ρ€ΠΈΡ†. ΠœΠ°Ρ‚Ρ€ΠΈΡ†Ρ‹ ΠΌΠΎΠΆΠ½ΠΎ ΡΠΊΠ»Π°Π΄Ρ‹Π²Π°Ρ‚ΡŒ ΠΈΠ»ΠΈ Π²Ρ‹Ρ‡ΠΈΡ‚Π°Ρ‚ΡŒ Ρ‚ΠΎΠ»ΡŒΠΊΠΎ Π² Ρ‚ΠΎΠΌ случаС, Ссли ΠΎΠ½ΠΈ ΠΈΠΌΠ΅ΡŽΡ‚ ΠΎΠ΄ΠΈΠ½Π°ΠΊΠΎΠ²Ρ‹Π΅ Ρ€Π°Π·ΠΌΠ΅Ρ€Ρ‹.

  • ΠšΠΎΠ»ΠΈΡ‡Π΅ΡΡ‚Π²ΠΎ строк ΠΈΠ»ΠΈ Π³ΠΎΡ€ΠΈΠ·ΠΎΠ½Ρ‚Π°Π»ΡŒΠ½Ρ‹Ρ… Π»ΠΈΠ½ΠΈΠΉ Π² ΠΌΠ°Ρ‚Ρ€ΠΈΡ†Π΅ {eq} A {/ eq} Ρ€Π°Π²Π½ΠΎ Π΄Π²ΡƒΠΌ.
  • ΠšΠΎΠ»ΠΈΡ‡Π΅ΡΡ‚Π²ΠΎ столбцов Π² ΠΌΠ°Ρ‚Ρ€ΠΈΡ†Π΅ {eq} A {/ eq} Ρ€Π°Π²Π½ΠΎ Ρ‚Ρ€Π΅ΠΌ.

Π’Π°ΠΊΠΈΠΌ ΠΎΠ±Ρ€Π°Π·ΠΎΠΌ, Ρ€Π°Π·ΠΌΠ΅Ρ€Π½ΠΎΡΡ‚ΡŒ ΠΌΠ°Ρ‚Ρ€ΠΈΡ†Ρ‹ {eq} A {/ eq} Ρ€Π°Π²Π½ΠΎ {eq} 2 \ times3 {/ eq}

  • ΠšΠΎΠ»ΠΈΡ‡Π΅ΡΡ‚Π²ΠΎ строк ΠΈΠ»ΠΈ Π³ΠΎΡ€ΠΈΠ·ΠΎΠ½Ρ‚Π°Π»ΡŒΠ½Ρ‹Ρ… Π»ΠΈΠ½ΠΈΠΉ Π² ΠΌΠ°Ρ‚Ρ€ΠΈΡ†Π΅ {eq} B {/ eq} Ρ€Π°Π²Π½ΠΎ Π΄Π²ΡƒΠΌ.
  • ΠšΠΎΠ»ΠΈΡ‡Π΅ΡΡ‚Π²ΠΎ столбцов Π² ΠΌΠ°Ρ‚Ρ€ΠΈΡ†Π΅ {eq} B {/ eq} Ρ€Π°Π²Π½ΠΎ Ρ‚Ρ€Π΅ΠΌ.

Π’Π°ΠΊΠΈΠΌ ΠΎΠ±Ρ€Π°Π·ΠΎΠΌ, Ρ€Π°Π·ΠΌΠ΅Ρ€Π½ΠΎΡΡ‚ΡŒ ΠΌΠ°Ρ‚Ρ€ΠΈΡ†Ρ‹ {eq} B {/ eq} Ρ€Π°Π²Π½ΠΎ {eq} 2 \ times3 {/ eq}

Π’Π°ΠΊΠΈΠΌ ΠΎΠ±Ρ€Π°Π·ΠΎΠΌ, ΠΎΠ½ΠΈ ΠΈΠΌΠ΅ΡŽΡ‚ ΠΎΠ΄ΠΈΠ½Π°ΠΊΠΎΠ²Ρ‹Π΅ Ρ€Π°Π·ΠΌΠ΅Ρ€Ρ‹.ΠœΡ‹ ΠΌΠΎΠΆΠ΅ΠΌ Π΄ΠΎΠ±Π°Π²ΠΈΡ‚ΡŒ ΠΌΠ°Ρ‚Ρ€ΠΈΡ†Ρ‹.

Π¨Π°Π³ 2: Если Π΄Π°Π½Π½Ρ‹Π΅ ΠΌΠ°Ρ‚Ρ€ΠΈΡ†Ρ‹ ΠΈΠΌΠ΅ΡŽΡ‚ ΠΎΠ΄ΠΈΠ½Π°ΠΊΠΎΠ²Ρ‹Π΅ Ρ€Π°Π·ΠΌΠ΅Ρ€Ρ‹, Ρ‡Ρ‚ΠΎΠ±Ρ‹ Π΄ΠΎΠ±Π°Π²ΠΈΡ‚ΡŒ ΠΌΠ°Ρ‚Ρ€ΠΈΡ†Ρ‹, Π΄ΠΎΠ±Π°Π²ΡŒΡ‚Π΅ ΡΠΎΠΎΡ‚Π²Π΅Ρ‚ΡΡ‚Π²ΡƒΡŽΡ‰ΠΈΠ΅ записи; Ρ‚ΠΎ Π΅ΡΡ‚ΡŒ столбСц ΠΏΠ΅Ρ€Π²ΠΎΠΉ строки, ΠΎΠ΄Π½Π° запись ΠΏΠ΅Ρ€Π²ΠΎΠΉ ΠΌΠ°Ρ‚Ρ€ΠΈΡ†Ρ‹ Π±ΡƒΠ΄Π΅Ρ‚ Π΄ΠΎΠ±Π°Π²Π»Π΅Π½ ΠΊ столбцу ΠΏΠ΅Ρ€Π²ΠΎΠΉ строки, ΠΎΠ΄Π½Π° запись Π²Ρ‚ΠΎΡ€ΠΎΠΉ ΠΌΠ°Ρ‚Ρ€ΠΈΡ†Ρ‹. ΠŸΠΎΠ»ΡƒΡ‡Π΅Π½Π½Π°Ρ сумма Π±ΡƒΠ΄Π΅Ρ‚ ΠΏΠΎΠΌΠ΅Ρ‰Π΅Π½Π° Π² ΡΠΎΠΎΡ‚Π²Π΅Ρ‚ΡΡ‚Π²ΡƒΡŽΡ‰ΡƒΡŽ ΠΏΠΎΠ·ΠΈΡ†ΠΈΡŽ Ρ€Π΅Π·ΡƒΠ»ΡŒΡ‚ΠΈΡ€ΡƒΡŽΡ‰Π΅ΠΉ ΠΌΠ°Ρ‚Ρ€ΠΈΡ†Ρ‹.

$$ A + B = \ begin {bmatrix} 3 ΠΈ -6 ΠΈ 5 \\ 4 ΠΈ 5 ΠΈ -2 \ end {bmatrix} + \ begin {bmatrix} 5 ΠΈ 3 ΠΈ -8 \\ 2 ΠΈ 8 ΠΈ -4 \ end {bmatrix} \\ A + B = \ begin {bmatrix} 3 + 5 ΠΈ -6 + 3 ΠΈ 5 + (- 8) \\ 4 + 2 ΠΈ 5 + 8 ΠΈ -2 + (- 4) \ end {bmatrix} \\ \ boxed {A + B = \ begin {bmatrix} 8 & -3 & -3 \\ 6 ΠΈ 13 ΠΈ -6 \ end {bmatrix}} $$

Π‘Π»ΠΎΠΆΠ΅Π½ΠΈΠ΅ ΠΈ Π²Ρ‹Ρ‡ΠΈΡ‚Π°Π½ΠΈΠ΅ ΠΌΠ°Ρ‚Ρ€ΠΈΡ†: ΠΏΡ€ΠΈΠΌΠ΅Ρ€ ΠΌΠ°Ρ‚Ρ€ΠΈΡ†Ρ‹ вычитания

Π’Ρ‹Ρ‡Ρ‚ΠΈΡ‚Π΅ Π΄Π²Π΅ Π·Π°Π΄Π°Π½Π½Ρ‹Π΅ ΠΌΠ°Ρ‚Ρ€ΠΈΡ†Ρ‹ {eq} A – B. {/ eq}

{eq} A = \ begin {bmatrix} 3 ΠΈ -6 ΠΈ 5 \\ 4 ΠΈ 5 ΠΈ -2 \ end {bmatrix}, B = \ begin {bmatrix} 5 ΠΈ 3 ΠΈ -8 \\ 2 ΠΈ 8 ΠΈ -4 \ end {bmatrix} \\ {/ eq}

Π¨Π°Π³ 1: НайдитС Ρ€Π°Π·ΠΌΠ΅Ρ€Ρ‹ Π·Π°Π΄Π°Π½Π½Ρ‹Ρ… ΠΌΠ°Ρ‚Ρ€ΠΈΡ†.ΠœΠ°Ρ‚Ρ€ΠΈΡ†Ρ‹ ΠΌΠΎΠΆΠ½ΠΎ ΡΠΊΠ»Π°Π΄Ρ‹Π²Π°Ρ‚ΡŒ ΠΈΠ»ΠΈ Π²Ρ‹Ρ‡ΠΈΡ‚Π°Ρ‚ΡŒ Ρ‚ΠΎΠ»ΡŒΠΊΠΎ Π² Ρ‚ΠΎΠΌ случаС, Ссли ΠΎΠ½ΠΈ ΠΈΠΌΠ΅ΡŽΡ‚ ΠΎΠ΄ΠΈΠ½Π°ΠΊΠΎΠ²Ρ‹Π΅ Ρ€Π°Π·ΠΌΠ΅Ρ€Ρ‹.

Π Π°Π·ΠΌΠ΅Ρ€Π½ΠΎΡΡ‚ΡŒ ΠΌΠ°Ρ‚Ρ€ΠΈΡ†Ρ‹ {eq} A \ text {and} B {/ eq} Ρ€Π°Π²Π½ΠΎ {eq} 2 \ times3 {/ eq}

ΠœΡ‹ ΠΌΠΎΠΆΠ΅ΠΌ Π²Ρ‹Ρ‡Π΅ΡΡ‚ΡŒ ΠΌΠ°Ρ‚Ρ€ΠΈΡ†Ρ‹.

Π¨Π°Π³ 2: Если Π΄Π°Π½Π½Ρ‹Π΅ ΠΌΠ°Ρ‚Ρ€ΠΈΡ†Ρ‹ ΠΈΠΌΠ΅ΡŽΡ‚ ΠΎΠ΄ΠΈΠ½Π°ΠΊΠΎΠ²Ρ‹Π΅ Ρ€Π°Π·ΠΌΠ΅Ρ€Ρ‹ для вычитания ΠΌΠ°Ρ‚Ρ€ΠΈΡ†, Π²Ρ‹Ρ‡Ρ‚ΠΈΡ‚Π΅ ΡΠΎΠΎΡ‚Π²Π΅Ρ‚ΡΡ‚Π²ΡƒΡŽΡ‰ΠΈΠ΅ записи ΠΈ помСститС Ρ€Π΅Π·ΡƒΠ»ΡŒΡ‚Π°Ρ‚ послС вычитания Π² ΡΠΎΠΎΡ‚Π²Π΅Ρ‚ΡΡ‚Π²ΡƒΡŽΡ‰ΡƒΡŽ ΠΏΠΎΠ·ΠΈΡ†ΠΈΡŽ.

$$ A – B = \ begin {bmatrix} 3 ΠΈ -6 ΠΈ 5 \\ 4 ΠΈ 5 ΠΈ -2 \ end {bmatrix} – \ begin {bmatrix} 5 ΠΈ 3 ΠΈ -8 \\ 2 ΠΈ 8 ΠΈ -4 \ end {bmatrix} \\ А – Π’ = \ begin {bmatrix} 3-5 ΠΈ -6-3 ΠΈ 5 – (- 8) \\ 4-2 ΠΈ 5-8 ΠΈ -2 – (- 4) \ end {bmatrix} \\ \ boxed {A – B = \ begin {bmatrix} -2 ΠΈ -9 ΠΈ 13 \\ 2 ΠΈ -3 ΠΈ 2 \ end {bmatrix}}

$ ΠŸΠΎΠ»ΡƒΡ‡ΠΈΡ‚Π΅ доступ ΠΊ тысячам практичСских вопросов ΠΈ объяснСний!

Онлайн-справка – Быстрая справка

Π’Ρ‹Ρ‡Π΅ΡΡ‚ΡŒ ΠΎΠ΄Π½ΠΎ ΠΈΠ·ΠΎΠ±Ρ€Π°ΠΆΠ΅Π½ΠΈΠ΅ ΠΈΠ· Π΄Ρ€ΡƒΠ³ΠΎΠ³ΠΎ

ПослСднСС обновлСниС: 05.03.2020

Π”Π°, хотя, Ссли изобраТСния Ρ€Π°Π·Π»ΠΈΡ‡Π°ΡŽΡ‚ΡΡ ΠΏΠΎ Ρ€Π°Π·ΠΌΠ΅Ρ€Ρƒ, опСрация Π½Π΅ΠΌΠ½ΠΎΠ³ΠΎ слоТнСС.

Π˜ΠΌΠΏΠΎΡ€Ρ‚ ΠΈΠ·ΠΎΠ±Ρ€Π°ΠΆΠ΅Π½ΠΈΠΉ Π² ΠΌΠ°Ρ‚Ρ€ΠΈΡ†Ρƒ

  1. НаТмитС ΠΊΠ½ΠΎΠΏΠΊΡƒ Новая ΠΌΠ°Ρ‚Ρ€ΠΈΡ†Π° Π½Π° стандартной ΠΏΠ°Π½Π΅Π»ΠΈ инструмСнтов.
  2. Π’ ΠΏΡ€ΠΎΠ²ΠΎΠ΄Π½ΠΈΠΊΠ΅ Windows, CTRL + Π²Ρ‹Π±Π΅Ρ€ΠΈΡ‚Π΅ Ρ„Π°ΠΉΠ»Ρ‹ ΠΈΠ·ΠΎΠ±Ρ€Π°ΠΆΠ΅Π½ΠΈΠΉ ΠΈ ΠΏΠ΅Ρ€Π΅Ρ‚Π°Ρ‰ΠΈΡ‚Π΅ ΠΈΡ… Π² ΠΎΠΊΠ½ΠΎ ΠΌΠ°Ρ‚Ρ€ΠΈΡ†Ρ‹.
  • Если изобраТСния ΠΈΠΌΠ΅ΡŽΡ‚ ΠΎΠ΄ΠΈΠ½Π°ΠΊΠΎΠ²Ρ‹Π΅ Ρ€Π°Π·ΠΌΠ΅Ρ€Ρ‹, ΠΎΠ½ΠΈ Π±ΡƒΠ΄ΡƒΡ‚ ΠΈΠΌΠΏΠΎΡ€Ρ‚ΠΈΡ€ΠΎΠ²Π°Π½Ρ‹ ΠΊΠ°ΠΊ ΠΎΡ‚Π΄Π΅Π»ΡŒΠ½Ρ‹Π΅ ΠΎΠ±ΡŠΠ΅ΠΊΡ‚Ρ‹ Π² ΠΎΠΊΠ½ΠΎ ΠΌΠ°Ρ‚Ρ€ΠΈΡ†Ρ‹.
  • Если изобраТСния ΠΈΠΌΠ΅ΡŽΡ‚ Ρ€Π°Π·Π½Ρ‹Π΅ Ρ€Π°Π·ΠΌΠ΅Ρ€Ρ‹, ΠΊΠ°ΠΆΠ΄ΠΎΠ΅ Π±ΡƒΠ΄Π΅Ρ‚ ΠΈΠΌΠΏΠΎΡ€Ρ‚ΠΈΡ€ΠΎΠ²Π°Π½ΠΎ Π½Π° ΠΎΡ‚Π΄Π΅Π»ΡŒΠ½Ρ‹ΠΉ лист Π² ΠΎΠΊΠ½Π΅ ΠΌΠ°Ρ‚Ρ€ΠΈΡ†Ρ‹.


ΠŸΠ Π˜ΠœΠ•Π§ΠΠΠ˜Π•. Π’Ρ‹ Ρ‚Π°ΠΊΠΆΠ΅ ΠΌΠΎΠΆΠ΅Ρ‚Π΅ ΠΏΠ΅Ρ€Π΅Ρ‚Π°Ρ‰ΠΈΡ‚ΡŒ изобраТСния Π² Origin Π²Π½Π΅ любого ΠΎΠΊΠ½Π°. Π‘ΡƒΠ΄Π΅Ρ‚ создано Π½ΠΎΠ²ΠΎΠ΅ ΠΎΠΊΠ½ΠΎ ΠΌΠ°Ρ‚Ρ€ΠΈΡ†Ρ‹.


Π’Ρ‹Ρ‡ΠΈΡ‚Π°Π½ΠΈΠ΅ ΠΈΠ·ΠΎΠ±Ρ€Π°ΠΆΠ΅Π½ΠΈΠΉ ΠΈΠ»ΠΈ ΠΌΠ°Ρ‚Ρ€ΠΈΡ† Π½Π° ΠΎΠ΄Π½ΠΎΠΌ листС
НиТС прСдполагаСтся, Ρ‡Ρ‚ΠΎ ваши изобраТСния ΠΈΠΌΠ΅ΡŽΡ‚ ΠΎΠ΄ΠΈΠ½Π°ΠΊΠΎΠ²Ρ‹ΠΉ Ρ€Π°Π·ΠΌΠ΅Ρ€:

  1. Если Π²Ρ‹ Ρ…ΠΎΡ‚ΠΈΡ‚Π΅ ΠΎΡ‚ΠΏΡ€Π°Π²ΠΈΡ‚ΡŒ Π²Ρ‹Π²ΠΎΠ΄ Π² Π½ΠΎΠ²Ρ‹ΠΉ ΠΌΠ°Ρ‚Ρ€ΠΈΡ‡Π½Ρ‹ΠΉ ΠΎΠ±ΡŠΠ΅ΠΊΡ‚ Π½Π° Ρ‚ΠΎΠΌ ΠΆΠ΅ листС, Π²Ρ‹ Π΄ΠΎΠ»ΠΆΠ½Ρ‹ сначала ΡΠΎΠ·Π΄Π°Ρ‚ΡŒ пустой ΠΎΠ±ΡŠΠ΅ΠΊΡ‚, Ρ‰Π΅Π»ΠΊΠ½ΡƒΠ² Π·Π½Π°Ρ‡ΠΎΠΊ изобраТСния, Π° Π·Π°Ρ‚Π΅ΠΌ Π²Ρ‹Π±Ρ€Π°Π² Π”ΠΎΠ±Π°Π²ΠΈΡ‚ΡŒ .
  2. Π’ мСню Π²Ρ‹Π±Π΅Ρ€ΠΈΡ‚Π΅ Π˜Π·ΠΎΠ±Ρ€Π°ΠΆΠ΅Π½ΠΈΠ΅ : АрифмСтичСскоС ΠΏΡ€Π΅ΠΎΠ±Ρ€Π°Π·ΠΎΠ²Π°Π½ΠΈΠ΅ : ΠŸΡ€ΠΎΡΡ‚Π°Ρ ΠΌΠ°Ρ‚Π΅ΠΌΠ°Ρ‚ΠΈΠΊΠ° : ΠžΡ‚ΠΊΡ€Ρ‹Ρ‚ΡŒ Π΄ΠΈΠ°Π»ΠΎΠ³ΠΎΠ²ΠΎΠ΅ ΠΎΠΊΠ½ΠΎ .
  3. Π£ΠΊΠ°ΠΆΠΈΡ‚Π΅ Π²Ρ…ΠΎΠ΄Π½Ρ‹Π΅ ΠΈ Π²Ρ‹Ρ…ΠΎΠ΄Π½Ρ‹Π΅ Π΄Π°Π½Π½Ρ‹Π΅ с ΠΏΠΎΠΌΠΎΡ‰ΡŒΡŽ ΠΊΠ½ΠΎΠΏΠΎΠΊ со стрСлками рядом с Π’Ρ…ΠΎΠ΄Π½ΠΎΠ΅ ΠΈΠ·ΠΎΠ±Ρ€Π°ΠΆΠ΅Π½ΠΈΠ΅1 , Π’Ρ…ΠΎΠ΄Π½ΠΎΠ΅ ΠΈΠ·ΠΎΠ±Ρ€Π°ΠΆΠ΅Π½ΠΈΠ΅2 ΠΈ Π’Ρ‹Ρ…ΠΎΠ΄Π½ΠΎΠ΅ ΠΈΠ·ΠΎΠ±Ρ€Π°ΠΆΠ΅Π½ΠΈΠ΅ , Π²Ρ‹Π±Ρ€Π°Π² вновь созданный (пустой) ΠΌΠ°Ρ‚Ρ€ΠΈΡ‡Π½Ρ‹ΠΉ ΠΎΠ±ΡŠΠ΅ΠΊΡ‚ для Π²Ρ‹Ρ…ΠΎΠ΄Π½ΠΎΠ³ΠΎ изобраТСния .
  4. Π©Π΅Π»ΠΊΠ½ΠΈΡ‚Π΅ Ρ€Π°ΡΠΊΡ€Ρ‹Π²Π°ΡŽΡ‰ΠΈΠΉΡΡ список ΠœΠ°Ρ‚Π΅ΠΌΠ°Ρ‚ΠΈΡ‡Π΅ΡΠΊΠ°Ρ функция ΠΈ ΡƒΠΊΠ°ΠΆΠΈΡ‚Π΅ ΠΎΠΏΠ΅Ρ€Π°Ρ†ΠΈΡŽ, ΠΊΠΎΡ‚ΠΎΡ€ΡƒΡŽ Π½Π΅ΠΎΠ±Ρ…ΠΎΠ΄ΠΈΠΌΠΎ Π²Ρ‹ΠΏΠΎΠ»Π½ΠΈΡ‚ΡŒ (Π½Π°ΠΏΡ€ΠΈΠΌΠ΅Ρ€,Π³Ρ€Π°ΠΌΠΌ. Β«Π”ΠΎΠ±Π°Π²ΠΈΡ‚ΡŒΒ», Β«img1 – img2Β» ΠΈ Ρ‚. Π”.).
  5. Π’Ρ‹Π±Π΅Ρ€ΠΈΡ‚Π΅ ΠΊΠ°Π½Π°Π», смСщСния Z ΠΈ ΠΊΠ°Π΄Ρ€ΠΈΡ€ΡƒΠΉΡ‚Π΅, Ссли Π½Π΅ΠΎΠ±Ρ…ΠΎΠ΄ΠΈΠΌΠΎ.
  6. НаТмитС ОК .


Π’Ρ‹Ρ‡ΠΈΡ‚Π°Π½ΠΈΠ΅ ΠΈΠ·ΠΎΠ±Ρ€Π°ΠΆΠ΅Π½ΠΈΠΉ ΠΈΠ»ΠΈ ΠΌΠ°Ρ‚Ρ€ΠΈΡ† Π½Π° Ρ€Π°Π·Π½Ρ‹Ρ… листах ΠΈΠ»ΠΈ ΠΊΠ½ΠΈΠ³Π°Ρ…

  1. АктивируйтС ΠΌΠ°Ρ‚Ρ€ΠΈΡ‡Π½ΡƒΡŽ ΠΊΠ½ΠΈΠ³Ρƒ, ΡΠΎΠ΄Π΅Ρ€ΠΆΠ°Ρ‰ΡƒΡŽ Π΄Π²Π° ΠΈΠ»ΠΈ Π±ΠΎΠ»Π΅Π΅ листов, Π·Π°Ρ‚Π΅ΠΌ Π²Ρ‹Π±Π΅Ρ€ΠΈΡ‚Π΅ Π˜Π·ΠΎΠ±Ρ€Π°ΠΆΠ΅Π½ΠΈΠ΅ : АрифмСтичСскоС ΠΏΡ€Π΅ΠΎΠ±Ρ€Π°Π·ΠΎΠ²Π°Π½ΠΈΠ΅ : ΠŸΡ€ΠΎΡΡ‚Π°Ρ ΠΌΠ°Ρ‚Π΅ΠΌΠ°Ρ‚ΠΈΠΊΠ° : ΠžΡ‚ΠΊΡ€Ρ‹Ρ‚ΡŒ Π΄ΠΈΠ°Π»ΠΎΠ³ΠΎΠ²ΠΎΠ΅ ΠΎΠΊΠ½ΠΎ .
  2. НаТмитС ΠΊΠ½ΠΎΠΏΠΊΡƒ со стрСлкой рядом с Π’Ρ…ΠΎΠ΄Π½ΠΎΠ΅ ΠΈΠ·ΠΎΠ±Ρ€Π°ΠΆΠ΅Π½ΠΈΠ΅1 ΠΈ Π’Ρ…ΠΎΠ΄Π½ΠΎΠ΅ ΠΈΠ·ΠΎΠ±Ρ€Π°ΠΆΠ΅Π½ΠΈΠ΅2 ΠΈ ΠΈΡΠΏΠΎΠ»ΡŒΠ·ΡƒΠΉΡ‚Π΅ ΠΎΠ±ΠΎΠ·Ρ€Π΅Π²Π°Ρ‚Π΅Π»ΡŒ ΠΈΠ·ΠΎΠ±Ρ€Π°ΠΆΠ΅Π½ΠΈΠΉ (ΠΈΡΠΏΠΎΠ»ΡŒΠ·ΡƒΠΉΡ‚Π΅ Ρ€Π°ΡΠΊΡ€Ρ‹Π²Π°ΡŽΡ‰ΠΈΠΉΡΡ список Бписок ΠΈΠ·ΠΎΠ±Ρ€Π°ΠΆΠ΅Π½ΠΈΠΉ Π²Π²Π΅Ρ€Ρ…Ρƒ для Ρ„ΠΈΠ»ΡŒΡ‚Ρ€Π°Ρ†ΠΈΠΈ) ΠΈ Π²Ρ‹Π±Π΅Ρ€ΠΈΡ‚Π΅, ΠΊΡƒΠ΄Π° ΠΎΡ‚ΠΏΡ€Π°Π²ΠΈΡ‚ΡŒ Π’Ρ‹Ρ…ΠΎΠ΄Π½ΠΎΠ΅ ΠΈΠ·ΠΎΠ±Ρ€Π°ΠΆΠ΅Π½ΠΈΠ΅ .
  3. Π©Π΅Π»ΠΊΠ½ΠΈΡ‚Π΅ Ρ€Π°ΡΠΊΡ€Ρ‹Π²Π°ΡŽΡ‰ΠΈΠΉΡΡ список ΠœΠ°Ρ‚Π΅ΠΌΠ°Ρ‚ΠΈΡ‡Π΅ΡΠΊΠ°Ρ функция ΠΈ ΡƒΠΊΠ°ΠΆΠΈΡ‚Π΅ ΠΎΠΏΠ΅Ρ€Π°Ρ†ΠΈΡŽ, ΠΊΠΎΡ‚ΠΎΡ€ΡƒΡŽ Π½ΡƒΠΆΠ½ΠΎ Π²Ρ‹ΠΏΠΎΠ»Π½ΠΈΡ‚ΡŒ (Π½Π°ΠΏΡ€ΠΈΠΌΠ΅Ρ€, Β«Π”ΠΎΠ±Π°Π²ΠΈΡ‚ΡŒΒ», Β«img1 – img2Β» ΠΈ Ρ‚.

    ΠžΡΡ‚Π°Π²ΠΈΡ‚ΡŒ ΠΊΠΎΠΌΠΌΠ΅Π½Ρ‚Π°Ρ€ΠΈΠΉ